Vcco_matlab (2).pdf

  • Uploaded by: Carlos Javier Cotrina Saavedra
  • 0
  • 0
  • March 2021
  • PDF

This document was uploaded by user and they confirmed that they have the permission to share it. If you are author or own the copyright of this book, please report to us by using this DMCA report form. Report DMCA


Overview

Download & View Vcco_matlab (2).pdf as PDF for free.

More details

  • Words: 121,746
  • Pages: 326
Loading documents preview...
Variable Compleja y Cálculo Operacional Teoría y Práctica con MATLAB William La Cruz

Variable Compleja y Ca´lculo Operacional actica con MATLAB Teor´ıa y Pr´

William La Cruz Universidad Central de Venezuela Facultad de Ingenier´ıa Escuela de Ingenier´ıa El´ectrica Departamento de Electr´onica, Computaci´on y Control

Versi´ on preliminar para el uso en el curso Variable Compleja y C´ alculo Operacional ADVERTENCIA: Esta gu´ıa se distribuye para los estudiantes de mis cursos. Por favor, cont´ acteme para otros usos. [email protected]

c 2018 W. La Cruz

Contenido Prefacio

v

I

1

Variable Compleja

1 N´ umeros Complejos 1.1 Definici´ on . . . . . . . . . . . . . 1.2 Operaciones Algebraicas . . . . . 1.3 Representaci´ on Geom´etrica . . . 1.4 Valor Absoluto y Conjugado . . . 1.5 Coordenadas Polares . . . . . . . 1.5.1 F´ ormula de Euler . . . . . 1.6 Potencias y Ra´ıces . . . . . . . . 1.7 Regiones en el Plano Complejo . 1.8 Aritm´ etica Compleja con M ATLAB 1.9 Problemas Resueltos . . . . . . . 1.10 Problemas Propuestos . . . . . .

. . . . . . . . . . .

. . . . . . . . . . .

. . . . . . . . . . .

. . . . . . . . . . .

. . . . . . . . . . .

. . . . . . . . . . .

. . . . . . . . . . .

. . . . . . . . . . .

. . . . . . . . . . .

. . . . . . . . . . .

. . . . . . . . . . .

. . . . . . . . . . .

. . . . . . . . . . .

. . . . . . . . . . .

. . . . . . . . . . .

. . . . . . . . . . .

. . . . . . . . . . .

. . . . . . . . . . .

. . . . . . . . . . .

2 2 3 5 6 9 12 13 13 17 20 22

2 Funciones de Variable Compleja 2.1 Definici´ on . . . . . . . . . . . . . . . . . . . 2.2 L´ımite y Continuidad . . . . . . . . . . . . . 2.2.1 Funciones Componentes . . . . . . . 2.2.2 L´ımite . . . . . . . . . . . . . . . . . 2.2.3 Continuidad . . . . . . . . . . . . . . 2.3 Diferenciaci´ on . . . . . . . . . . . . . . . . . 2.3.1 F´ ormulas o Reglas de Diferenciaci´ on 2.3.2 Ecuaciones de Cauchy-Riemann . . . 2.4 Funciones Anal´ıticas . . . . . . . . . . . . . 2.5 Funciones Arm´ onicas . . . . . . . . . . . . . 2.6 Funciones Elementales . . . . . . . . . . . . 2.6.1 Funci´ on Exponencial . . . . . . . . . 2.6.2 Funciones Trigonom´etricas . . . . . 2.6.3 Funciones Hiperb´ olicas . . . . . . . . 2.6.4 Funci´ on Logaritmo . . . . . . . . . . 2.6.5 Funci´ on Exponente Complejo . . . . 2.6.6 Funciones Trigonom´etricas Inversas . 2.6.7 Funciones Hiperb´ olicas Inversas . . . 2.7 Mapeos . . . . . . . . . . . . . . . . . . . . 2.7.1 Mapeo w = z + c . . . . . . . . . . . 2.7.2 Mapeo w = bz . . . . . . . . . . . . 2.7.3 Mapeo w = bz + c . . . . . . . . . . 2.7.4 Mapeo Inversi´ on . . . . . . . . . . .

. . . . . . . . . . . . . . . . . . . . . . .

. . . . . . . . . . . . . . . . . . . . . . .

. . . . . . . . . . . . . . . . . . . . . . .

. . . . . . . . . . . . . . . . . . . . . . .

. . . . . . . . . . . . . . . . . . . . . . .

. . . . . . . . . . . . . . . . . . . . . . .

. . . . . . . . . . . . . . . . . . . . . . .

. . . . . . . . . . . . . . . . . . . . . . .

. . . . . . . . . . . . . . . . . . . . . . .

. . . . . . . . . . . . . . . . . . . . . . .

. . . . . . . . . . . . . . . . . . . . . . .

. . . . . . . . . . . . . . . . . . . . . . .

. . . . . . . . . . . . . . . . . . . . . . .

. . . . . . . . . . . . . . . . . . . . . . .

. . . . . . . . . . . . . . . . . . . . . . .

. . . . . . . . . . . . . . . . . . . . . . .

. . . . . . . . . . . . . . . . . . . . . . .

. . . . . . . . . . . . . . . . . . . . . . .

24 24 25 26 26 28 31 31 32 34 36 39 40 41 43 44 46 48 49 50 51 53 55 56

. . . . . . . . . . .

. . . . . . . . . . .

i

. . . . . . . . . . .

. . . . . . . . . . .

. . . . . . . . . . .

ii

2.7.5 Mapeo Bilineal . . . . . . . . . . . . . . . . 2.8 Funciones, L´ımite, Derivada y Mapeos con M ATLAB 2.9 Problemas Resueltos . . . . . . . . . . . . . . . . . 2.10 Problemas Propuestos . . . . . . . . . . . . . . . . 3 Series de Potencias y Singularidades Aisladas 3.1 Serie de N´ umeros Complejos . . . . . . . . . . . 3.1.1 Serie de Potencias . . . . . . . . . . . . . 3.1.2 Serie de Taylor . . . . . . . . . . . . . . . 3.1.3 Serie de Laurent . . . . . . . . . . . . . . 3.1.4 Propiedades Adicionales de las Series . . . 3.2 Singularidades Aisladas . . . . . . . . . . . . . . 3.2.1 Polo de Orden m . . . . . . . . . . . . . . 3.2.2 Punto Singular Esencial . . . . . . . . . . 3.2.3 Punto Singular Removible . . . . . . . . . 3.3 Series de Potencias y Singularidades con M ATLAB 3.4 Problemas Resueltos . . . . . . . . . . . . . . . . 3.5 Problemas Propuestos . . . . . . . . . . . . . . . 4 Integraci´ on Compleja 4.1 Integral Definida . . . . . . . . . . . . . . . . . . 4.2 Integraci´ on de L´ınea . . . . . . . . . . . . . . . . 4.2.1 Contornos . . . . . . . . . . . . . . . . . . 4.2.2 Integral de L´ınea . . . . . . . . . . . . . . 4.3 Teorema de Cauchy-Goursat . . . . . . . . . . . . 4.3.1 Extensi´ on del Teorema de Cauchy-Goursat 4.4 Integral Indefinida . . . . . . . . . . . . . . . . . 4.5 F´ ormula Integral de Cauchy . . . . . . . . . . . . 4.6 Residuo . . . . . . . . . . . . . . . . . . . . . . . 4.6.1 C´ alculo del Residuo . . . . . . . . . . . . 4.6.2 Teorema de los Residuos . . . . . . . . . . 4.6.3 Expansi´ on en Fracciones Parciales . . . . . 4.7 Integraci´ on y Residuos con M ATLAB . . . . . . . 4.8 Problemas Resueltos . . . . . . . . . . . . . . . . 4.9 Problemas Propuestos . . . . . . . . . . . . . . .

II

. . . . . . . . . . . .

. . . . . . . . . . . . . . .

. . . .

. . . .

. . . .

. . . .

. . . .

. . . .

. . . .

. . . .

. . . .

. . . .

. . . .

. . . .

. . . .

. . . .

59 61 66 71

. . . . . . . . . . . .

. . . . . . . . . . . .

. . . . . . . . . . . .

. . . . . . . . . . . .

. . . . . . . . . . . .

. . . . . . . . . . . .

. . . . . . . . . . . .

. . . . . . . . . . . .

. . . . . . . . . . . .

. . . . . . . . . . . .

. . . . . . . . . . . .

. . . . . . . . . . . .

. . . . . . . . . . . .

. . . . . . . . . . . .

77 77 79 81 85 87 88 90 93 94 94 98 104

. . . . . . . . . . . . . . .

105 105 106 106 108 110 111 113 115 117 118 120 121 124 128 135

. . . . . . . . . . . . . . .

. . . . . . . . . . . . . . .

. . . . . . . . . . . . . . .

. . . . . . . . . . . . . . .

. . . . . . . . . . . . . . .

. . . . . . . . . . . . . . .

. . . . . . . . . . . . . . .

. . . . . . . . . . . . . . .

. . . . . . . . . . . . . . .

. . . . . . . . . . . . . . .

. . . . . . . . . . . . . . .

. . . . . . . . . . . . . . .

. . . . . . . . . . . . . . .

C´ alculo Operacional

5 Funciones de Dominios Continuo y Discreto 5.1 Funciones de Dominio Continuo . . . . . . 5.1.1 Impulso Unitario . . . . . . . . . . 5.1.2 Escal´ on Unitario . . . . . . . . . . 5.1.3 Pulso Rectangular . . . . . . . . . 5.1.4 Pulso Triangular . . . . . . . . . . 5.1.5 Funci´ on Signo . . . . . . . . . . . 5.1.6 Pulso Exponencial . . . . . . . . . 5.1.7 Funci´ on Rampa . . . . . . . . . . .

139 . . . . . . . .

. . . . . . . .

. . . . . . . .

. . . . . . . .

. . . . . . . .

. . . . . . . .

. . . . . . . .

. . . . . . . .

. . . . . . . .

. . . . . . . .

. . . . . . . .

. . . . . . . .

. . . . . . . .

. . . . . . . .

. . . . . . . .

. . . . . . . .

. . . . . . . .

. . . . . . . .

. . . . . . . .

140 140 141 143 144 145 145 146 147

iii

5.2

5.3 5.4 5.5

5.1.8 Funciones Sa y sinc . . . . . . . . . . . . 5.1.9 Relaci´ on entre u(t) y δ(t) . . . . . . . . . 5.1.10 Derivada Generalizada . . . . . . . . . . . 5.1.11 Convoluci´ on en el Dominio Continuo . . . Funciones de Dominio Discreto . . . . . . . . . . 5.2.1 Impulso Unitario Discreto . . . . . . . . . 5.2.2 Escal´ on Unitario Discreto . . . . . . . . . 5.2.3 Funci´ on Rampa Discreta . . . . . . . . . . 5.2.4 Funci´ on Signo Discreto . . . . . . . . . . . 5.2.5 Relaci´ on entre u(n) y δ(n) . . . . . . . . . 5.2.6 Convoluci´ on en el Dominio Discreto . . . Introducci´ on al C´ alculo Operacional con M ATLAB Problemas Resueltos . . . . . . . . . . . . . . . . Problemas Propuestos . . . . . . . . . . . . . . .

6 Transformada de Fourier 6.1 Definici´ on . . . . . . . . . . . . . . . . . . . 6.1.1 Transformada Inversa de Fourier . . 6.2 Propiedades de la Transformada de Fourier 6.3 Algunos Pares de Transformadas . . . . . . 6.4 Magnitud y Fase . . . . . . . . . . . . . . . 6.5 Transformada de Fourier con M ATLAB . . . 6.6 Problemas Resueltos . . . . . . . . . . . . . 6.7 Problemas Propuestos . . . . . . . . . . . .

. . . . . . . . . . . . . .

. . . . . . . . . . . . . .

. . . . . . . . . . . . . .

. . . . . . . . . . . . . .

. . . . . . . . . . . . . .

. . . . . . . . . . . . . .

. . . . . . . . . . . . . .

. . . . . . . . . . . . . .

. . . . . . . . . . . . . .

. . . . . . . . . . . . . .

. . . . . . . . . . . . . .

. . . . . . . . . . . . . .

. . . . . . . . . . . . . .

. . . . . . . . . . . . . .

. . . . . . . . . . . . . .

147 148 149 151 155 155 156 156 157 158 159 162 170 182

. . . . . . . .

. . . . . . . .

. . . . . . . .

. . . . . . . .

. . . . . . . .

. . . . . . . .

. . . . . . . .

. . . . . . . .

. . . . . . . .

. . . . . . . .

. . . . . . . .

. . . . . . . .

. . . . . . . .

. . . . . . . .

. . . . . . . .

. . . . . . . .

. . . . . . . .

185 185 186 187 194 200 205 207 216

7 Transformada de Laplace 7.1 Definici´ on . . . . . . . . . . . . . . . . . . . . . 7.1.1 Regi´ on de Convergencia . . . . . . . . . 7.1.2 Transformada Inversa de Laplace . . . . 7.2 Propiedades de la Transformada de Laplace . . 7.3 Algunos Pares de Transformadas . . . . . . . . 7.4 C´ alculo de la Transformada Inversa de Laplace 7.4.1 Integraci´ on de Contornos . . . . . . . . 7.4.2 Inversi´ on por Tablas . . . . . . . . . . . 7.5 Transformada de Laplace con M ATLAB . . . . . 7.6 Problemas Resueltos . . . . . . . . . . . . . . . 7.7 Problemas Propuestos . . . . . . . . . . . . . .

. . . . . . . . . . .

. . . . . . . . . . .

. . . . . . . . . . .

. . . . . . . . . . .

. . . . . . . . . . .

. . . . . . . . . . .

. . . . . . . . . . .

. . . . . . . . . . .

. . . . . . . . . . .

. . . . . . . . . . .

. . . . . . . . . . .

. . . . . . . . . . .

. . . . . . . . . . .

. . . . . . . . . . .

. . . . . . . . . . .

. . . . . . . . . . .

218 218 219 220 220 228 233 233 236 237 239 246

8 Transformada z 8.1 Definici´ on . . . . . . . . . . . . . . . . . 8.1.1 Regi´ on de Convergencia . . . . . 8.1.2 Transformada z Inversa . . . . . 8.2 Propiedades de la Transformada z . . . . 8.3 Algunos Pares de Transformadas . . . . 8.4 C´ alculo de la Transformada z Inversa . . 8.4.1 Integraci´ on Compleja . . . . . . 8.4.2 Expansi´ on en Serie de Potencias . 8.4.3 Inversi´ on por Tablas . . . . . . .

. . . . . . . . .

. . . . . . . . .

. . . . . . . . .

. . . . . . . . .

. . . . . . . . .

. . . . . . . . .

. . . . . . . . .

. . . . . . . . .

. . . . . . . . .

. . . . . . . . .

. . . . . . . . .

. . . . . . . . .

. . . . . . . . .

. . . . . . . . .

. . . . . . . . .

. . . . . . . . .

249 249 250 251 252 258 263 263 264 265

. . . . . . . . .

. . . . . . . . .

. . . . . . . .

. . . . . . . . .

. . . . . . . . .

iv

8.5 Transformada z con M ATLAB . . . . . . . . . . . . . . . . . . . . . . . . . . 266 8.6 Problemas Resueltos . . . . . . . . . . . . . . . . . . . . . . . . . . . . . . . 268 8.7 Problemas Propuestos . . . . . . . . . . . . . . . . . . . . . . . . . . . . . . 273 Respuesta a los Problemas Propuestos A Introducci´ on a M ATLAB A.1 Escritorio de M ATLAB . . . A.2 Objetos y Sintaxis . . . . . . A.3 Matrices . . . . . . . . . . . A.4 Elementos de Programaci´ on A.5 Gr´aficos . . . . . . . . . . .

. . . . .

. . . . .

275

. . . . .

. . . . .

. . . . .

. . . . .

. . . . .

. . . . .

. . . . .

. . . . .

. . . . .

. . . . .

. . . . .

. . . . .

. . . . .

. . . . .

. . . . .

. . . . .

. . . . .

. . . . .

. . . . .

. . . . .

. . . . .

. . . . .

. . . . .

. . . . .

. . . . .

282 282 284 288 295 305

Bibliograf´ıa

314

´Indice

315

Prefacio “La persona que nunca se equivoca nunca prueba algo nuevo” Albert Einstein (1879-1955)

Actualmente existe la creencia bastante com´ un que los problemas matem´aticos en su mayor´ıa se resuelven usando una computadora sin otro conocimiento previo o que las soluciones de los mismos se encuentran en Iternet, simplemente buscando en Google un link que nos dirija m´ agicamente a la respuesta. Esta visi´ on inmediatista, sin mediaci´ on alguna, sin esfuerzo, es consecuencia del gran desarrollo de la computaci´ on y la consolidaci´ on de las redes sociales, pero tal creencia no es cierta. S´ olo basta con pensar y hacerse las siguientes preguntas: ¿C´ omo se dise˜ nan las computadoras? o ¿C´ omo funciona Internet? Las respuestas a estas preguntas encierran un gran c´ umulo de conceptos, m´ etodos y t´ecnicas, todos ellos relacionados con la matem´atica y la ingenier´ıa. Por ejemplo, para dise˜ nar un procesador (CPU) de una computadora, una red de comunicaciones, un puente colgante o un tornillo, es necesario manejar ciertos conceptos tales como: microprocesadores, grafos, curva catenaria, constante de elasticidad y muchos otros m´ as, que a su vez requieren para su comprensi´ on el manejo de la suma, la multiplicaci´ on, la derivaci´ on, la integraci´ on, entre otros conceptos y operaciones matem´aticas. Por tanto, uno de los objetivos de esta gu´ıa es lograr que el estudiante se familiarice profundamente con los conceptos matem´aticos. El motivo para la elaboraci´ on de esta gu´ıa, que a su vez es otro objetivo de la misma, es brindar al estudiante el libre acceso al conocimiento, por su puesto, presentando los conceptos de una manera sencilla pero formal, usando para ello solo herramientas matem´ aticas correspondientes al cuarto semestre de Ingenier´ıa. Espec´ıficamente, el escrito presenta la teor´ıa b´asica de Variable Compleja y el C´ alculo Operacional. Para la comprensi´ on de los conceptos expuestos en el escrito, es necesario que el lector posea un nivel aceptable de conocimientos de C´ alculo (particularmente, del C´ alculo Infinitesimal Real en una y dos variables); tambi´ en debe manejar con destreza las operaciones algebraicas en los conjuntos reales, racionales y enteros. En otras palabras, el material est´ a dirigido a estudiantes cursantes del cuarto semestre de Ingenier´ıa. La gu´ıa est´ a estructurada en dos partes: I. Variable Compleja y II. C´ alculo Operacional, ya que la misma, en principio, est´ a dise˜ nada seg´ un el contenido tem´ atico del curso Variable Compleja y C´ alculo Operacional, ubicado en el cuarto semestre del Plan de Estudios de Ingenier´ıa El´ectrica. En los cap´ıtulos 1, 2, 3 y 4, se dan las propiedades del conjunto de los n´ umeros complejos; se presentan los fundamentos matem´aticos de la continuidad y diferenciaci´ on de una funci´ on de variable compleja; se estudian las series de n´ umeros complejos haciendo ´enfasis en los desarrollos de Taylor y de Laurent; y se describen los conceptos b´asicos de la integraci´ on compleja: integral definida, integral de l´ınea, primitiva y teor´ıa de residuos. En el cap´ıtulo 5 se da una introducci´ on al c´ alculo operacional, comenzando con la caracterizaci´ on de las funciones de dominio continuo, pasando luego por la convoluci´ on v

vi

PREFACIO

en el dominio continuo. En esta parte se estudia con inter´ es la funci´ on impulso unitario. Aqu´ı tambi´ en se describen las funciones de dominio discreto y se define la convoluci´ on de funciones de dominio discreto. En los cap´ıtulos 6, 7 y 8, se presentan, respectivamente, la definici´ on matem´ atica y se prueban todos los teoremas y propiedades, de la transforma de Fourier, la transformada de Laplace y la transformada z. Es de destacar que estas notas han evolucionado a medida que han transcurrido los semestres en los que se ha dictado la asignatura Variable Compleja y C´ alculo Operacional, agregando un problema resuelto o una imagen que permita la mejor visualizaci´ on de un concepto o un m´ etodo. En todo caso, siempre en mejora del aprendizaje. Con esto en mente, cada cap´ıtulo posee secciones de problemas resueltos y problemas propuestos. Los problemas resueltos fueron seleccionados de ex´ amenes parciales, adem´as, en el escrito se presenta en forma rigurosa el proceso de resoluci´ on de los mismos. Los problemas propuestos son una colecci´ on de problemas orientados al fortalecimiento y consolidaci´ on de los conceptos y m´ etodos estudiados. Todos los problemas propuestos tienen respuesta, que se muestra al final de la gu´ıa. Con el prop´ osito de visualizar de manera m´ as aplicada la signatura Variable Compleja y C´ alculo Operacional, se agreg´ o a la gu´ıa un enfoque computacional del an´alisis complejo, es decir, en el escrito tambi´ en se presenta una visi´ on computacional de los conceptos, usando para ello el software M ATLAB (R2015a), que se emplea en numerosas universidades y posee muchas facilidades gr´aficas de calidad, flexibilidad e interactividad. Al final de cada cap´ıtulo se presentan ejemplos pr´acticos codificados en M ATLAB . Tal iniciativa tiene el prop´ osito de introducir al estudiante en el uso de una herramienta computacional, pero dej´andoles ver que tal herramienta por s´ı sola no resuelve los problemas, sino que es necesario que el usuario tenga la destreza de entender y definir correctamente el problema para luego resolverlo con la computadora. Para que el estudiante tuviera a la mano un peque˜ no manual de M ATLAB , se agreg´ o como ap´ endice una breve introducci´ on a M ATLAB . Finalmente, a los lectores les pido el gran favor de apuntar mis errores, ya que citando nuevamente a Albert Einstein, “la persona que nunca se equivoca nunca prueba algo nuevo”, o como yo lo interpreto: al momento de crear se est´ a sujeto a errar, pero ello no debe ser un obst´aculo para continuar y mejorar.

William La Cruz Universidad Central de Venezuela, Caracas Junio de 2018

Parte I

Variable Compleja

1

1 N´ umeros Complejos

Trate de resolver las ecuaciones de Cardano.

La primera referencia escrita de la ra´ız cuadrada de un n´ umero negativo la encontramos en la obra Stereometr´ıa de Her´ on de Alejandr´ıa alrededor de la mitad del siglo I. La siguiente referencia sobre la ra´ız cuadrada de un n´ umero negativo data del a˜ no 275 en la obra de Diophantus, Arithmetica. En su intento del c´ alculo de los lados de un tri´angulo rect´ angulo de per´ımetro 12 y a´rea 7, Diophantus plante´ o resolver la ecuaci´ on 336x2 + 24 = 172x, ecuaci´ on cuyas soluciones no son n´ umeros reales. En 1545, Jerome Cardano (Italia, 1501-1576), un matem´atico, f´ısico y fil´ osofo italiano, publica “Ars Magn” (El Gran Arte) en el cual describe un m´ etodo para resolver ecuaciones algebraicas de grado tres y cuatro. En esta obra Cardano plantea el siguiente problema: Si alguien te pide dividir 10 en dos partes cuyo producto sea 40, es evidente que esta cuesti´on es imposible. Aparente√ mente √ este problema parece imposible, pero Cardano da las siguientes soluciones: 5+ −5 y 5 − −5. Cardano obtiene tales soluciones resolviendo las ecuaciones: x + y = 10, xy = 40. Dos siglos y medio cubrieron las dudas sobre el significado y la autenticidad de los n´ umeros complejos. No obstante, fueron estudiados por un gran n´ umero de matem´ aticos. Cabe√mencionar que fue Leonhard Euler (Suiza, 1707-1783) el primero en usar la notaci´ on i = −1. El matem´ atico alem´an Carl Friedrich Gauss (Alemania, 1777-1855), fue quien les dio nombre y los defini´ o rigurosamente. En este cap´ıtulo se introducen algunas propiedades del conjunto de los n´ umeros complejos. Tal conjunto de n´ umeros es ampliamente utilizado en el desarrollo de las ideas te´ oricas y pr´ acticas de Ingenier´ıa, en especial, de Ingenier´ıa El´ectrica.

1.1 Definici´ on Se dice que z es un n´ umero complejo si se expresa como z = x + iy o, de manera equivalente, z = x + y i, donde x e y son n´ umeros reales (recuerde que R denota el conjunto de los n´ umeros reales). El s´ımbolo i se conoce como unidad imaginaria. El conjunto de los n´ umeros complejos se denota como C, y z ∈ C indica que z pertenece al conjunto de los n´ umeros complejos. Por otra parte, un n´ umero complejo z = x + i y tambi´ en se puede definir como el par ordenado z = (x, y). Esta definici´ on equivalente de n´ umero complejo permite definir a i como el n´ umero complejo dado por i = (0, 1). M´as adelante se demostrar´a que i2 = −1. 2

3

´ MEROS COMPLEJOS CAP´ITULO 1. NU

Se denota con x = Re z la parte real del n´ umero complejo z, y con y = Im z la parte imaginaria de z. Los n´ umeros complejos de la forma x + i 0, se denominan reales puros o, simplemente, reales. Adem´ as, los n´ umeros complejos de la forma 0 + i y, se denominan imaginarios puros. Los n´ umeros reales 0 y 1, tambi´ en se pueden definir como n´ umeros complejos. El cero de los n´ umeros complejos, denotado 0, se define como 0 = 0 + i 0. El 1 de los n´ umeros complejos, denotado 1, se define por 1 = 1 + i 0. Definici´ on 1.1. Se dice que dos n´ umeros complejos z y w son iguales si, y s´ olo si sus partes reales son iguales y sus partes imaginarias son iguales. En otras palabras, si z = x + i y y w = u + i v, entonces z = w, si y s´ olo si x=u

y = v.

e

En particular, z = x + iy = 0



x = y = 0.

Observaci´ on 1.1. No existe relaci´ on de orden en los n´ umeros complejos. En los n´ umeros reales, por ejemplo, se tiene que 5 > 3, pero no tiene sentido afirmar que 1 + i < 2 + i 3.

1.2 Operaciones Algebraicas Seguidamente se definen las operaciones algebraicas de n´ umeros complejos, a saber: suma, resta, multiplicaci´on y divisi´ on. Definici´ on 1.2. Sean z1 = x1 + i y1 , y z2 = x2 + i y2 n´ umeros complejos. Se define la suma, resta, multiplicaci´on y divisi´ on de z1 y z2 como sigue. Suma: z1 + z2 = (x1 + x2 ) + i (y1 + y2 ). Resta: z1 − z2 = (x1 − x2 ) + i (y1 − y2 ). Multiplicaci´ on: z1 · z2 = z1 z2 = (x1 x2 − y1 y2 ) + i (x1 y2 + x2 y1 ). Divisi´ on:

z1 = z1 ÷ z2 = z2



x1 x2 + y 1 y 2 x22 + y22



+i



x2 y 1 − x1 y 2 x22 + y22



,

siempre que x2 6= 0 y y2 6= 0. En la siguiente proposici´ on se describen algunas propiedades de las operaciones algebraicas de los n´ umeros complejos. Esta proposici´ on permite asegurar que el conjunto de n´ umeros complejos conforma un cuerpo, esto es, un conjunto algebraico con leyes de composici´ on interna como la suma y la multiplicaci´ on.

Recuerde que los n´ umeros reales se pueden representar como un punto en una recta, que se recorre de izquierda a derecha, con lo cual se impone un orden.

4

1.2. OPERACIONES ALGEBRAICAS

Proposici´ on 1.1. Para todo z, w, s ∈ C se cumplen las siguientes propiedades: 1. Conmutativa • z+w =w+z • zw = wz

2. Asociativa

• z + (w + s) = (z + w) + s • z(ws) = (zw)s

3. Elemento Neutro • z+0=z • 1·z =z

4. Elemento Inverso • Para todo z ∈ C, existe −z ∈ C, llamado inverso aditivo, tal que z + (−z) = 0.

• Para todo z 6= 0, existe z −1 ∈ C, denominado inverso multiplicativo, tal que z z −1 = 1. 5. Distributiva • z(w + s) = zw + zs.

Demostraci´ on. Es una consecuencia inmediata de las propiedades de los n´ umeros reales, la definici´ on de n´ umero complejo y las definiciones de suma y multiplicaci´ on de n´ umeros complejos. Note que las operaciones suma, resta y multiplicaci´ on, son leyes de composici´ on interna sobre el conjunto de los n´ umeros complejos, es decir, son operaciones que asocian a cada par de n´ umeros complejos otro n´ umero complejo. La divisi´ on tambi´ en es una ley de composici´ on interna sobre el conjunto de los n´ umeros complejos distintos de cero. Adem´ as, estas operaciones pueden verse como una extensi´ on de las operaciones algebraicas de los n´ umeros reales. Ejercicio 1.1. Sean z1 = 1 − i, z2 = i − 3, y z3 = i/2. Realizar las siguientes operaciones algebraicas: a) z1 + z2 + z3

d) z2 /z3

b) z1 · z2 · z3

e) z1 /(z2 · z1 )

c) z1 (z2 + z3 )

f) z3 /(z1 /z2 )

La siguiente proposici´ on relaciona la divisi´ on de n´ umeros complejos con el inverso multiplicativo. Tambi´ en proporciona una manera equivalente de definir divisi´ on de n´ umeros complejos. Proposici´ on 1.2. Sean z, w ∈ C. Si w 6= 0, entonces z = z w−1 . w

5

´ MEROS COMPLEJOS CAP´ITULO 1. NU

z w y z w−1 son iguales. Tomemos z = x + i y, w = u + i v y asumamos que w = 6 0. Por definici´ on, se tiene que Demostraci´ on. Demostremos que las partes real e imaginaria de los n´ umeros complejos

z xu + yv yu − xv = 2 +i 2 . 2 w u +v u + v2

(1.1)

Ahora, veamos la forma que tiene w−1 . Asumamos que w−1 = a+i b. Por la Proposici´ on 1.1 se tiene que w w−1 = 1. Por tanto, podemos escribir: 1 = w w−1 = (u + i v)(a + i b) = (ua − vb) + i (ub + va), de donde se obtiene el siguiente sistema de ecuaciones lineales con inc´ ognitas a y b. 

ua − vb = 1 va + ub = 0

Resolviendo este u ´ ltimo sistema de ecuaciones lineales obtenemos: a=

u2

u + v2

y

b=

−v , + v2

u2

luego, w−1 =

u2

−v u +i 2 . 2 +v u + v2

Empleando es u ´ ltima ecuaci´ on se tiene que z w−1 =

xu + yv yu − xv +i 2 . 2 2 u +v u + v2

Por (1.1) y (1.2) podemos concluir que

(1.2)

z = z w−1 . w

1.3 Representaci´ on Geom´ etrica Un n´ umero complejo z = x + i y tambi´ en puede definirse como un u ´nico par ordenado (x, y), es decir, la parte real de z es la primera coordenada de (x, y) y la parte imaginaria es la segunda coordenada. As´ı, el n´ umero complejo z puede representarse geom´etricamente como un punto en el plano cartesiano xy, como ejemplo de ello observe la gr´afica que se muestra en la Figura 1.1. Cuando se utiliza el plano cartesiano para representar un n´ umero complejo, ´este se denomina plano complejo o plano z. Adem´ as, el eje x, o eje de las abscisas, se denomina eje real, mientras que el eje y, o eje de las ordenadas, se conoce como eje imaginario (ver Figura 1.1).

6

1.4. VALOR ABSOLUTO Y CONJUGADO

Eje imaginario

y

z = (x, y) b

x

Eje real

Figura 1.1. Representaci´ on de un n´ umero complejo como un par ordenado Otra representaci´ on posible de z = x + i y en el plano cartesiano es en forma de vector (ver Figura 1.2), es decir, z se puede representar como un vector orientado cuyo extremo inicial es el origen (0, 0) y su extremo final es el punto (x, y). De esta forma, a un n´ umero complejo lo podemos representar como un par ordenado o como un vector en el plano xy. ¿Por qu´e se toma a z como el vector orientado con origen (0, 0) y extremo final (x, y), y no otro vector?

Eje imaginario

y

z = x + iy

x

Eje real

Figura 1.2. Representaci´ on de un n´ umero complejo como un vector

1.4 Valor Absoluto y Conjugado En la secci´ on anterior notamos que un n´ umero complejo z se puede representar como un vector orientado. Este hecho permite que z herede algunas propiedades de los vectores, entre ellas la magnitud o valor absoluto. Definici´ on 1.3 (Valor absoluto). El valor absoluto del n´ umero complejo z = x + i y, denotado por |z|, se define como la longitud del vector z, la cual se calcula a trav´ es de la f´ ormula p |z| = x2 + y 2 .

Geom´etricamente, el valor absoluto |z| representa la distancia del origen (0, 0) al punto (x, y).

Definici´ on 1.4 (Conjugado). El conjugado del n´ umero complejo z = x + i y, denotado por z, se define como z = x + i (−y). Geom´etricamente, el conjugado z es la reflexi´ on del punto (x, y) con respecto al eje real. En la Figura 1.3 se muestra un ejemplo gr´afico del conjugado de un n´ umero complejo.

7

´ MEROS COMPLEJOS CAP´ITULO 1. NU

z = x + iy

y b

x −y

b

z = x − iy

Figura 1.3. Conjugado de un n´ umero complejo La Proposici´ on 1.3 presenta algunas propiedades del valor absoluto y el conjugado. Proposici´ on 1.3. Sean z1 y z2 , n´ umeros complejos. Las siguientes propiedades se cumplen: 1) z1 = z1 . 2) z1 ± z2 = z1 ± z2 . 3) z1 z2 = z1 z2 .   z1 z1 = , siempre y cuando z2 6= 0. 4) z2 z2 5) |z1 | = |z1 |. 6) z1 z1 = |z1 |2 . 7) z1−1 =

z1 , siempre y cuando z1 6= 0. |z1 |2

8) |z1 | ≥ |Re z1 | ≥ Re z1 . 9) |z1 | ≥ |Im z1 | ≥ Im z1 . 10) |z1 z2 | = |z1 | |z2 |. z1 |z1 | 11) = , siempre y cuando z2 6= 0. z2 |z2 |

Demostraci´ on. Se deja como ejercicio para el lector. La suma de los n´ umeros complejos z1 = x1 + i y1 y z2 = x2 + i y2 , tiene una interpretaci´ on simple en t´ erminos vectoriales. El vector que representa la suma de los n´ umeros z1 y z2 se obtiene sumando vectorialmente los vectores de z1 y z2 , es decir, empleando la regla del paralelogramo (ver Figura 1.4). Este esquema geom´etrico se puede utilizar para obtener la desigualdad triangular: la longitud de un lado cualquiera de un tri´ angulo es menor o igual que la suma de las longitudes de los otros lados. Esto es, la longitud correspondiente a z1 + z2 , |z1 + z2 |, es menor o igual que la suma de las longitudes, |z1 | y |z2 |. on matem´atica de la desigualdad triangular. En la Proposici´ on 1.4 damos la expresi´

8

1.4. VALOR ABSOLUTO Y CONJUGADO

z1

z1

+

z2 z2

Figura 1.4. Suma vectorial de n´ umeros complejos.

Proposici´ on 1.4 (Desigualdad triangular). Sean z1 y z2 n´ umeros complejos. Entonces, la siguiente desigualdad se cumple |z1 + z2 | ≤ |z1 | + |z2 |. Demostraci´ on. Usando Proposici´ on 1.3 6), podemos escribir: |z1 + z2 |2 = (z1 + z2 ) (z1 + z2 ) = (z1 + z2 ) (z1 + z2 ) = z1 z1 + z1 z2 + z2 z1 + z2 z2 = |z1 |2 + z1 z2 + z2 z1 + |z2 |2 = |z1 |2 + z1 z2 + z1 z2 + |z2 |2 ,

pero z1 z2 + z1 z2 = 2 Re (z1 z2 ) ≤ 2 |z1 z2 | = 2 |z1 | |z2 | = 2 |z1 | |z2 |, luego |z1 + z2 |2 ≤ |z1 |2 + 2 |z1 | |z2 | + |z2 |2 = (|z1 | + |z2 |)2 , de donde se deduce que |z1 + z2 | ≤ |z1 | + |z2 |, con lo cual queda establecida la desigualdad triangular. La desigualdad triangular se puede extender a m´ as de dos vectores, es decir, la longitud de la suma de un n´ umero finito de vectores es menor o igual que la suma de las longitudes de tales vectores. En la siguiente proposici´ on se muestra la expresi´ on matem´atica de este resultado, el cual se denomina desigualdad triangular generalizada. Proposici´ on 1.5 (Desigualdad triangular generalizada). Sean z1 , z2 , . . . , zn n´ umeros complejos, donde n es un entero mayor o igual que 2. Entonces, la siguiente desigualdad se cumple |z1 + z2 + · · · + zn | ≤ |z1 | + |z2 | + · · · + |zn |. Demostraci´ on. Realicemos la demostraci´ on por inducci´ on. Por la Proposici´ on 1.4, la desigualdad se cumple para n = 2. Supongamos, como hip´ otesis inductiva, que la desigualdad se cumple para n = h, es decir, se tiene que |z1 + z2 + · · · + zh | ≤ |z1 | + |z2 | + · · · + |zh |.

9

´ MEROS COMPLEJOS CAP´ITULO 1. NU

Ahora demostremos que esta desigualdad se cumple para n = h + 1. Por la hip´ otesis inductiva y la Proposici´ on 1.4, podemos escribir: |z1 + z2 + · · · + zh + zh+1 | ≤ |z1 + z2 + · · · + zh | + |zh+1 |

≤ |z1 | + |z2 | + · · · + |zh | + |zh+1 |,

para todo entero h ≥ 1. Con esto queda demostrada la desigualdad.

1.5 Coordenadas Polares Del hecho que un n´ umero complejo z = x+i y puede ser representado geom´etricamente como un vector, se puede derivar su representaci´ on en coordenadas polares (r, θ), donde r = |z| y θ es el argumento de z.

z = x + iy

r

=

|z |

Definici´ on 1.5 (Argumento de z). El argumento del n´ umero complejo z 6= 0, denotado por arg z, es cualquiera de los ´ angulos orientados comprendidos entre la parte positiva del eje real y el n´ umero complejo z.

θ

Figura 1.5. Coordenadas polares de z En la Figura 1.5 se muestra un ejemplo de la representaci´ on en coordenadas polares de un n´ umero complejo z = x + i y ubicado en el primer cuadrante. Se tiene que x = r cos θ e y = r sen θ. As´ı pues, la forma polar de z se define como z = r(cos θ + i sen θ). Los valores de r y θ = arg z definen de manera u ´ nica a z, es decir, para cada par (r, θ) existe un u ´ nico n´ umero complejo z que tiene como coordenadas polares a (r, θ). En cambio, el n´ umero complejo z caracteriza de manera u ´ nica a r, pero no a θ = arg z, esto es, dado un n´ umero complejo z, entonces existe un u ´ nico r > 0 tal que r = |z|, pero existen infinitos valores de θ = arg z. Observe el siguiente ejemplo. Ejemplo 1.1. Para el n´ umero complejo z = 1, se tiene que los valores de θ est´ an dados  por: θ = arg z ∈ {0, ±2π, ±4π, . . .}. La existencia de infinitos valores de θ es una consecuencia de la periodicidad de las funciones trigonom´ etricas. En efecto, si la forma polar de un n´ umero complejo z1 es z1 = r1 (cos θ1 + i sen θ1 ),

10

1.5. COORDENADAS POLARES

entonces se tiene que r1 (cos(θ1 + 2nπ) + i sen (θ1 + 2nπ)) = r1 (cos θ1 + i sen θ1 ) = z1 ,

para n ∈ Z,

en otras palabras, (r, θ1 + 2nπ) tambi´ en son coordenadas polares de z1 . Ahora bien, para obtener una u ´ nica representaci´ on en coordenadas polares de un n´ umero complejo z, el valor de θ se debe tomar en una determinaci´ on, es decir, se debe escoger un valor de θ en un intervalo de longitud 2π, por ejemplo, θ ∈ [0, 2π), o ´ θ ∈ (−π, π], etc. El valor de θ que generalmente se emplea es el valor del argumento de z que pertenece al intervalo (−π, π], el cual se denomina argumento principal. Definici´ on 1.6 (Argumento principal). El argumento principal de z 6= 0 o valor principal de arg z, denotado por Arg z, se define como el u ´ nico valor de arg z tal que −π < Arg z ≤ π.

Note que arctan(y/x) es un valor entre −π/2 y π/2.

Los valores de θ = arg z, y en especial el valor de Arg z, se pueden encontrar empleando la ecuaci´ on: y tan θ = . x En s´ı, para calcular el argumento principal de un n´ umero complejo z = x + i y se emplea la ecuaci´ on θ = arctan(y/x). La manera de emplear esta u ´ ltima ecuaci´ on en el c´ alculo del argumento principal de z 6= 0, se muestra en la siguiente expresi´ on:   0, x > 0, y = 0,      arctan(y/x), x > 0, y > 0,      π/2, x = 0, y > 0,    arctan(y/x) + π, x < 0, y > 0, Arg z = π, x < 0, y = 0,      arctan(y/x) − π, x < 0, y < 0,      −π/2, x = 0, y < 0,    arctan(y/x), x > 0, y < 0.

Note que la f´ ormula anterior considera la posici´ on z en el plano complejo para calcular el valor de Arg z, esto es, dependiendo del cuadrante donde se encuentre z, su argumento principal se calcula de una forma muy particular, por supuesto, usando para ello el valor de arctan(y/x). Observe el siguiente ejemplo. Ejemplo 1.2. Utilizando el argumento principal, represente en forma polar los siguientes n´ umeros complejos: z1 = 1 + i , z2 = −1 + i , z3 = −1 − i, y z4 = 1 − i. √ √ Soluci´ on. Se tiene que el valor absoluto de z1 es r = |z1 | = 12 + 12 = 2. Como z1 = 1+i est´ a en el primer cuadrante, su argumento principal es Arg z1 = arctan(1) =

π . 4

Por lo tanto, la f´ ormula polar de z1 = 1 + i empleando el argumento principal es: √ z1 = 2(cos(π/4) + i sen (π/4)).

11

´ MEROS COMPLEJOS CAP´ITULO 1. NU

Por un razonamiento similar, se puede verificar que la forma polar de los n´ umeros complejos z2 = −1 + i , z3 = −1 − i y z4 = 1 − i, es respectivamente: √ 2(cos(3π/4) + i sen (3π/4)), z2 = √ 2(cos(3π/4) − i sen (3π/4)), z3 = √ z4 = 2(cos(π/4) − i sen (π/4)). Se deja como ejercicio para el lector verificar cada una de estas formas polares.



En el ejemplo anterior calculamos el argumento principal de un n´ umero complejo. Es natural preguntarse: ¿a partir del valor del argumento principal se pueden obtener todos los valores de arg z? La respuesta a esta pregunta es afirmativa, m´ as a´ un, conocido un valor cualquiera del argumento (no necesariamente el principal) se pueden hallar todos los valores del argumento. Con la siguiente expresi´ on se encuentran todos los valores del argumento de un n´ umero complejo z, utilizando el valor de Arg z: arg z = Arg z + 2nπ,

para n ∈ Z.

Por otra parte, pasemos a describir algunas propiedades del argumento, las cuales se muestran en la siguiente proposici´ on. Proposici´ on 1.6. Si z1 y z2 son n´ umeros complejos distintos de cero, entonces las siguientes identidades se cumplen: 1. arg (z1 z2 ) = arg z1 + arg z2 . 2. arg (1/z2 ) = −arg (z2 ). 3. arg (z1 /z2 ) = arg z1 − arg z2 . Demostraci´ on. Sean z1 y z2 n´ umeros complejos cuya representaci´ on en forma polar es: z1 = r1 (cos θ1 + i sen θ1 ) y z2 = r2 (cos θ2 + i sen θ2 ). Utilizando identidades trigonom´etricas podemos escribir: z1 z2 = (r1 (cos θ1 + i sen θ1 )) (r2 (cos θ2 + i sen θ2 )) = (r1 r2 ) [(cos θ1 cos θ2 − sen θ1 sen θ2 ) + i (cos θ1 sen θ2 + sen θ1 cos θ2 )]

= (r1 r2 ) [cos(θ1 + θ2 ) + i sen (θ1 + θ2 )] ,

que es la forma polar de z1 z2 y θ1 + θ2 = arg z1 + arg z2 es un valor de arg (z1 z2 ). Por lo tanto, se cumple la identidad 1. Empleando la forma polar de z2 podemos escribir: 1 1 cos θ2 − i sen θ2 1 = = · = r2−1 (cos(−θ2 ) + i sen (−θ2 )), z2 r2 (cos θ2 + i sen θ2 ) r2 cos2 θ2 + sen 2 θ2 que es la forma polar de 1/z2 y arg (1/z2 ) = −θ2 ; luego, se cumple la identidad 2. Como 1 z1 = z1 · , z2 z2 entonces, por las identidades 1 y 2, se prueba que la identidad 3 se cumple.

12

1.5. COORDENADAS POLARES

Ejemplo 1.3. La Proposici´ on 1.6 se puede utilizar para calcular un valor del argumento de z1 z2 o z1 /z2 . Por ejemplo, tomemos z1 = i y z2 √ = −1 + i. La forma polar de estos n´ umeros complejos es z1 = cos(π/2) + i sen (π/2) y z2 = 2(cos(3π/4) + i sen (3π/4)), por supuesto, usando el argumento principal. As´ı, un valor del argumento de z1 z2 es 5π π 3π + = , 2 4 4 lo cual efectivamente es un valor del argumento de z1 z2 = −1 − i (verifique esta afirmaci´ on). Pero, este valor no es el argumento principal de z1 z2 = −1 − i, aunque π/2 y  3π/4 sean los argumentos principales de z1 y z2 , respectivamente. arg (z1 z1 ) =

Un ejercicio interesante es encontrar z1 y z2 tales que Arg (z1 z2 ) = Arg z1 + Arg z2

El Ejemplo 1.3 nos permite asegurar que, en general, Arg (z1 z2 ) 6= Arg z1 + Arg z2

y

Arg (z1 /z2 ) 6= Arg z1 − Arg z2 .

ormula de Euler 1.5.1 F´ La f´ ormula de Euler es una notaci´ on de la forma polar de un n´ umero complejo que permite ahorrar escritura e introduce la funci´ on exponencial compleja, la cual estudiaremos en los cap´ıtulos subsiguientes. El nombre de Euler es en honor a Leonhard Euler (17071783), matem´ atico y f´ısico suizo. Tal f´ ormula consiste en lo siguiente. Sea z = x + i y un n´ umero complejo con m´ odulo |z| = 1 y θ = arg z. Escribiendo eiθ = cos θ + i sen θ

se obtiene z = eiθ , que se conoce como f´ ormula de Euler. Ahora, para cualquier n´ umero complejo z = r(cos θ + i sen θ), con r > 0, se puede utilizar la f´ ormula de Euler para reescribir a z como: z = reiθ . Adem´ as, si z1 = r1 eiθ1 y z2 = r2 eiθ2 , entonces las siguientes identidades son ciertas: 1. z1 z2 = (r1 r2 )ei(θ1 +θ2 ) . 2. 1/z1 = (1/r1 ) ei(−θ1 ) =

1 −iθ e . r1

3. z1 /z2 = (r1 /r2 )ei(θ1 −θ2 ) . Considerando esta notaci´ on, la forma polar de z utilizando la f´ ormula de Euler es iθ z = re , donde r y θ son sus coordenadas polares (θ no necesariamente es el argumento principal de z). En el siguiente ejemplo se emplea la f´ ormula de Euler para expresar algunos n´ umeros complejos en su forma polar. Ejemplo 1.4. La forma polar, utilizando la f´ ormula de Euler, de los n´ umeros complejos z1 = 1 + i , z2 = −1 + i , z3 = −1 − i, y z4 = 1 − i, es √ √ √ √ z1 = 2 eiθπ/4 , z2 = 2 eiθ3π/4 , z3 = 2 e−iθ3π/4 , z4 = 2 e−iθπ/4 . Observe que en la forma polar de los n´ umeros dados se utiliz´ o el argumento principal, pero pudiera tambi´ en emplearse cualquier valor del argumento. 

13

´ MEROS COMPLEJOS CAP´ITULO 1. NU

1.6 Potencias y Ra´ıces Definici´ on 1.7 (Potencia n-´esima). Sean z = r eiθ y n un entero positivo. La potencia n-´esima de z, denotada por z n , se define como z n = r n einθ = r n (cos nθ + i sen nθ). Definici´ on 1.8 (Ra´ıces n-´esimas). Sean z = r eiθ y n un entero positivo. Las ra´ıces n-´esimas de z, denotadas por z 1/n , se definen como      √ √ θ + 2kπ θ + 2kπ 1/n i θ+2kπ n n z = r e n = r cos + i sen , n n √ para k = 0, 1, . . . , n − 1, donde n r denota la ra´ız n-´esima positiva del n´ umero real r. Observaci´ on 1.2. En las definiciones de potencia n-´esima y ra´ız n-´esima, se utiliza cualesquiera valor de θ, no es necesario que θ sea el argumento principal de z. Ejemplo 1.5. Se deja como ejercicio para el lector verificar que (1 + i)3 =

√ 3π 2 23 ei 4

y

(1 + i)1/5 =



10

2 ei

π +2kπ 4 5

, k = 0, 1, . . . , 4. 

1.7 Regiones en el Plano Complejo En esta secci´ on se describen conjuntos especiales de n´ umeros complejos. Comencemos con dos conjuntos que utilizaremos con mucha regularidad. Definici´ on 1.9. Los conjuntos de n´ umeros complejos {z ∈ C : |z| = 1}

y

{z ∈ C : |z| < 1}

se denominan circunferencia unitaria y disco unitario, respectivamente. on gr´afica de la circunferencia unitaria y el En la Figura 1.6 se muestra la representaci´ disco unitario. Note que la definici´ on dada de circunferencia unitaria es equivalente a la siguiente definici´ on: el conjunto de puntos (x, y) ∈ R2 tales que x2 + y 2 = 1, que es una circunferencia de centro 0 y radio 1.Ahora, una circunferencia de centro z0 ∈ C y radio r > 0, se define como el conjunto {z ∈ C : |z − z0 | = r} o, simplemente, |z − z0 | = r. Este u ´ ltimo conjunto nos permite introducir nuevos conjuntos en el plano complejo. Definici´ on 1.10 (Vecindad). Dados z0 ∈ C y ε > 0, el conjunto de puntos denotado por B(z0 , ε) = {z ∈ C : |z − z0 | < ε}, se denomina vecindad de z0 . En otras palabras, una vecindad es un disco de centro z0 y radio ε > 0 o, equivalentemente, el interior de la circunferencia de centro z0 y radio ε > 0. La representaci´ on geom´etrica de una vecindad B(z0 , ε) se aprecia en la Figura 1.7.

Si z = x + iy, entonces la ecuaci´ on |z| = 1 es equivalente a la ecuaci´ on x2 + y 2 = 1

14

1.7. REGIONES EN EL PLANO COMPLEJO

1

Circunferencia unitaria

b

0 Disco unitario

b

ε

Figura 1.6. Representaci´ on gr´afica de la circunferencia unitaria y el disco unitario

z

b

z0

Figura 1.7. Vecindad de un n´ umero complejo

Definici´ on 1.11 (Conjunto abierto). Se dice que un conjunto de n´ umeros complejos S es abierto, si para cada z ∈ S existe una vecindad de z, B(z, ε), tal que B(z, ε) ⊂ S. Ejemplo 1.6. Los conjuntos S1 , S2 y S3 que se muestran en la Figura 1.8 son abiertos. 1

S1

1

1

S2

S3

b

−1

1

1

−1

1

−1 a)

b)

c)

Figura 1.8. Ejemplos de conjuntos abiertos

¿Todo conjunto del plano que no contiene su borde es abierto?

La l´ınea de puntos indica que los puntos de la misma no pertenecen al conjunto. Por ello, ninguno de los conjuntos S1 , S2 y S3 contiene su borde.Los conjuntos S1 , S2 y S3 se definen de la siguiente manera. a) S1 = {z ∈ C : |z| < 1} b) S2 es la regi´ on del plano complejo formada por los puntos interiores del cuadrado cuyos v´ ertices son los puntos z1 = 0, z2 = 1, z3 = 1 + i, y z4 = i. Anal´ıticamente, S2

15

´ MEROS COMPLEJOS CAP´ITULO 1. NU

es el conjunto puntos z = x+i y que satisfacen el siguiente sistema de inecuaciones:  0≤x≤1 0≤y≤1 c) S3 es la regi´ on del plano complejo formada por los puntos z = x + i y que satisfacen el siguiente sistema de inecuaciones:   x − y > −1 x+y < 1  y > 0



Definici´ on 1.12 (Frontera de un conjunto). Sea S un conjunto de n´ umeros complejos. La frontera de S es el conjunto formado por los puntos z ∈ C tales que todas las vecindades de z contienen puntos de S y puntos que no est´ an en S. Ejemplo 1.7. En la Figura 1.9 observamos un conjunto S y tres puntos z1 , z2 y z3 . En este caso, z1 es un punto de la frontera de S, en cambio z2 y z3 no pertenecen a la frontera de S (se deja al lector justificar esta afirmaci´ on). z1

S

b

z2 b

z3 b

Figura 1.9. Frontera de un conjunto  Ejercicio 1.2. Determine la expresi´ on matem´atica de la frontera de los conjuntos S1 , S2 y S3 definidos en el Ejemplo 1.6. Definici´ on 1.13 (Punto de acumulaci´ on). Sea S un conjunto de n´ umeros complejos. Se dice que z0 es un punto de acumulaci´on si toda vecindad de z0 contiene por lo menos un punto de S diferente de z0 . Ejemplo 1.8. Considere el conjunto S de la Figura 1.9. Aqu´ı observamos que z1 y z2 son puntos de acumulaci´ on de S, pero z3 no es un punto de acumulaci´ on de S, ya que existe ε > 0 tal que B(z3 , ε) ∩ S = ∅.  Definici´ on 1.14 (Conjunto cerrado). Se dice que un conjunto de n´ umeros complejos S es cerrado, si todos sus puntos de acumulaci´ on pertenecen a ´el. De forma equivalente, S es cerrado si contiene a su frontera. ¿Todo conjunto del plano que contiene su borde es cerrado?

16

1.7. REGIONES EN EL PLANO COMPLEJO

Ejemplo 1.9. Considere todos los conjuntos mostrados en las Figuras 1.7, 1.8 y 1.9. Suponga que con estos conjuntos se forman nuevos conjuntos que contienen los puntos de la  frontera. Estos u ´ltimos conjuntos son todos cerrados. Definici´ on 1.15 (Conjunto conexo). Se dice que un conjunto de n´ umeros complejos S es conexo, si dados dos puntos cualesquiera de S, existe una trayectoria formada por segmentos de recta que los une y cuyos puntos pertenecen todos a S. Ejemplo 1.10. En la Figura 1.10 observamos dos conjuntos S1 y S2 . El conjunto S1 es conexo, en cambio S2 no lo es. (¿Por qu´e?) S1

S2

Figura 1.10. Conjuntos conexo y no conexo  Definici´ on 1.16 (Dominio). Se dice que un conjunto de n´ umeros complejos S es un dominio, si S es abierto y conexo. Ejemplo 1.11. Todos los conjuntos mostrados en las Figuras 1.7, 1.8 y 1.9 son dominios. en es un dominio. (¿Por qu´e?)  Asimismo, el conjunto S1 de la Figura 1.10 tambi´ Definici´ on 1.17 (Conjuntos acotado y no acotado). Se dice que un conjunto de n´ umeros complejos S es acotado, si existe un n´ umero real R > 0 tal que todo punto de S queda dentro de la circunferencia |z| = R. Por el contrario, si |z| > R para todo R > 0 y alg´ un z ∈ S, se dice que S es no acotado. ¿Todo conjunto cerrado es acotado?

Ejemplo 1.12. En la Figura 1.11 observamos dos conjuntos S1 y S2 . El conjunto S1 es acotado y S2 es no acotado.

S1

S2

Figura 1.11. Conjuntos acotado y no acotado 

´ MEROS COMPLEJOS CAP´ITULO 1. NU

17

1.8 Aritm´ etica Compleja con M ATLAB Las operaciones con n´ umeros complejos en M ATLAB son muy f´aciles de hacer, ya que on a este software trabaja con aritm´etica compleja. Ver Cap´ıtulo A para una introducci´ M ATLAB o el libro de Higham y Higham [4]. Un n´ umero complejo z = x + iy se representa en M ATLAB usando en esencia su misma estructura, por ejemplo, los n´ umeros complejos z1 = 1 + 3i y z2 = −1 − 2i se definen en M ATLAB como: ✄ >> z1 = 1+3i , z2 = −1−2i z1 = 1.0000 + 3.0000 i z2 = −1.0000 − 2.0000 i





Para M ATLAB , i o j representan la unidad imaginaria. Ahora, para las operaciones aritm´eticas de suma, resta, divisi´ on y multiplicaci´ on, se utilizan respectivamente los siguientes s´ımbolos: +, -, / y *. Por ejemplo, considerando los n´ umeros complejos z1 = 1+3i y z2 = −1 − 2i, las operaciones z1 + z2 , z1 − z2 , z1 z2 y z1 /z2 , se hacen en M ATLAB como: ✄ >> z1+z2 ans = 0 + 1.0000 i >> z1−z2 ans = 2.0000 + 5.0000 i >> z1 ∗ z2 ans = 5.0000 − 5.0000 i >> z1 / z2 ans = −1.4000 − 0.2000 i





Los comandos: real(z), imag(z), conj(z), abs(z) y angle(z), se utilizan para calcular la parte real, la parte imaginaria, el conjugado, el m´ odulo y el argumento de z, respectivamente. Observe el siguiente ejemplo. ✄ >> z = 1+i /2 z = 1.0000 + 0.5000 i >> r e a l ( z ) ans = 1 >> imag ( z ) ans = 0.5000 >> c o n j ( z ) ans = 1.0000 − 0.5000 i >> abs ( z ) ans = 1.1180 >> a n g l e ( z ) ans = 0.4636





¿Qu´e resultado arrojar´ıa el comando angle(i)?

18

´TICA COMPLEJA CON MATLAB 1.8. ARITME

Note que el valor angle(z) est´ a expresado en radianes, adem´as, es el argumento principal de z. Observe el siguiente ejemplo. ✄ >> z= 1+i ; a n g l e ( z ) ans = 0.7854 >> z= −1+i ; a n g l e ( z ) ans = 2.3562 >> z= −1−i ; a n g l e ( z ) ans = −2.3562 >> z= 1− i ; a n g l e ( z ) ans = −0.7854

✂ Para m´ as detalles de plot, teclee help plot en el escritorio de M ATLAB



El comando plotpuede usarse para ver la representaci´ on gr´afica de un n´ umero complejo como un par ordenado. Por ejemplo, con las instrucciones z = 1/3 + s q r t (2)∗ i ; p l o t ( z , ’ ob ’ , ’ MarkerEdgeColor ’ , ’ b ’ , ’ MarkerFaceColor ’ , ’ b ’ , ’ MarkerSize ’ , 5 ) t e x t ( 0 . 4 , 1 . 6 , ’ $z = \ f r a c {1}{3} + \ s q r t {2}\ , i $ ’ , ’ F o n t S i z e ’ , 1 4 , . . . ’ Backgrou ndColor ’ , ’w ’ , ’ I n t e r p r e t e r ’ , ’ l a t e x ’ ) xlabel ( ’ x ’ ) ylabel ( ’ y ’ ) g r i d on a x i s square

√ se genera la gr´afica de la Figura 1.12, donde se representa al n´ umero complejo z = 31 + 2i como un par ordenado. 2.5

2

z=

1 3

+



2i

y

1.5

1

0.5

0 −1

−0.5

0

0.5

1

1.5

x

Figura 1.12. Representaci´ on como un par ordenado del n´ umero z =

1 3

+



2i

El comando compass(z) dibuja a z = x + i y como un vector con extremo inicia en el origen y extremo final en el par ordenado (x, y). Las instrucciones zz = [1+i , −1−i , −(1/2)+ i , 1− i ] ; compass ( zz )

Para m´ as detalles de compass, teclee help compass en el escritorio de M A TLAB

se utilizaron para representar como vectores a los n´ umeros complejos z1 = 1 + i, z2 = 1 −1 − i, z3 = − 2 + i, z4 = 1 − i, cuya gr´afica se aprecia en la Figura 1.13.

19

´ MEROS COMPLEJOS CAP´ITULO 1. NU

90

1.5

120

60

1 30

150 0.5

180

0

210

330

240

300 270

Figura 1.13. Representaci´ on como vectores de los n´ umeros complejos z1 = 1 + i, z2 = −1 − i, z3 = − 12 + i, z4 = 1 − i Conocidas las coordenadas polares, (r, θ), de un n´ umero complejo z, se puede utilizar la funci´ on exp para hallar la forma cartesiana de z. Por ejemplo, la forma cartesiana, z = x + iy, del n´ umero complejo z = 2 eiπ/3 , que est´ a representado en forma polar usando la f´ ormula de Euler, se halla de la siguiente manera: ✄ >> z = 2∗ exp ( i ∗ p i /3) z = 1.0000 + 1.7321 i





El comando z^n permite calcular la potencia en´ esima del n´ umero complejo z. Observe el siguiente ejemplo donde se calcula (1 + i)5 . ✄ >> (1+ i )ˆ5 ans = −4.0000 − 4.0000 i





En cambio, con el comando z^(1/n) se puede calcular un valor de la ra´ız en´ esima de z. 1/5 Observe el siguiente ejemplo donde se calcula un valor de (1 + i) . ✄ >> (1+ i ) ˆ ( 1 / 5 ) ans = 1.0586 + 0.1677 i





Ahora, el Programa 1.1 muestra la funci´ on raizn que calcula las n ra´ıces en´ esimas de z. Programa 1.1. Funci´ on raizn.m f u n c t i o n szn = r a i z n ( z , n ) %r a i z n H a l l a l a s n r a´ı c e s de z . % szn e s un a r r e g o que c o n t i e n e l a s n r a´ı c e s de z t h e t a = angle ( z ) ; r = abs ( z ) ; k = 0 : 1 : n−1; szn = ( r ˆ(1/n ) ) ∗ exp ( ( t h e t a + 2∗k∗ p i )∗ i /n ) ; end

20

1.9. PROBLEMAS RESUELTOS

A continuaci´ on se utiliza la funci´ on raizn para calcular las ra´ıces quintas de z = 1 + i. ✄ >> r a i z n (1+ i , 5 ) ans = 1.0586 + 0.1677 i 0.4866 − 0.9550 i

0.1677 + 1.0586 i

−0.9550 + 0.4866 i

−0.7579 − 0.7579 i





Ejercicio 1.3. Emplee convenientemente las funciones √ compass y raizn para obtener la gr´afica de las ra´ıces s´ eptimas de la unidad, esto es, 7 1.

1.9 Problemas Resueltos Problema 1.1. Sean z1 y z2 n´ umeros complejos tales que z1 + z2 es un n´ umero real difeumero complejo puro. rente de cero y Re (z1 − z2 ) = 0. Pruebe que z12 − z22 es un n´ Soluci´ on. Sean z1 = x1 + iy1 y z2 = x2 + iy2 . Como z1 + z2 = (x1 + x2 ) + i(y1 + y2 ) es un n´ umero real diferente de cero y Re (z1 − z2 ) = x1 − x2 = 0, se tiene que y1 = −y2 y x1 = x2 . As´ı, z12 − z22 = (z1 + z2 )(z1 − z2 ) = (2x1 )(2y1 i) = 4x1 y1 i, umero complejo puro. en otras palabras, z12 − z22 es un n´



√ √ Problema 1.2. Demuestre que |(2 z + 5)( 2 − i)| = 3|2z + 5|. Soluci´ on. Utilizando la definici´ on y las propiedades del m´ odulo podemos escribir: √ √ √ √ |(2 z + 5)( 2 − i)| = |2 z + 5| | 2 − i| = 2z + 5 2 + 1 = 3|2z + 5|.



Problema 1.3. Sean z1 y z2 n´ umeros complejos. Si |z1 | = 6 |z2 |, entonces demuestre que |z1 + z2 | ≥ ||z1 | − |z2 ||. Soluci´ on. Por la Proposici´ on 1.3, podemos escribir: |z1 + z2 |2 = (z1 + z2 )(z1 + z2 ) = |z1 |2 + z1 z 2 + z2 z 1 + |z2 |2  = |z1 |2 + z1 z 2 + z1 z 2 + |z2 |2 = |z1 |2 + 2Re (z1 z 2 ) + |z2 |2 ≥ |z1 |2 − 2|z1 z 2 | + |z2 |2 = |z1 |2 − 2|z1 | |z2 | + |z2 |2 = (|z1 | − |z2 |)2 ,

de donde se deduce que |z1 + z2 | ≥ ||z1 | − |z2 ||.



Problema 1.4. Sean z1 , z2 y z3 n´ umeros complejos. Si |z2 | = 6 |z3 |, entonces demuestre que z1 |z1 | z2 + z3 ≤ ||z2 | − |z3 || .

21

´ MEROS COMPLEJOS CAP´ITULO 1. NU

Soluci´ on. Tomando z1 = z2 y z2 = z3 en el Problema 1.3, obtenemos: |z2 + z3 | ≥ ||z2 | − |z3 ||, de donde se deduce que

1 1 ≤ , |z2 + z3 | ||z2 | − |z3 ||

ahora, multiplicando por |z1 | en ambos lados de la desigualdad anterior se tiene |z1 | |z1 | ≤ , |z2 + z3 | ||z2 | − |z3 || lo que implica

z1 |z1 | z2 + z3 ≤ ||z2 | − |z3 || ,

que era lo que dese´ abamos demostrar.



Problema 1.5. Si z1 z2 6= 0, aplicar la forma polar para demostrar que Re (z1 z2 ) = |z1 | |z2 | si, y s´ olo si θ1 + θ2 = 2nπ, (n = 0, ±1, ±2, . . .), donde θ1 = arg z1 y θ2 = arg z2 . Soluci´ on. En la demostraci´ on utilizamos el s´ımbolo “⇔” que denota equivalencia. Sean z1 = r1 (cos θ1 + i sen θ1 ) y z2 = r2 (cos θ2 + i sen θ2 ). As´ı, tenemos que r1 = |z1 |, r2 = |z2 | y Re (z1 z2 ) = r1 r2 (cos θ1 cos θ2 − sen θ1 sen θ2 ) . Ahora, usando la ecuaci´ on anterior podemos escribir: Re (z1 z2 ) = |z1 | |z2 | ⇔ r1 r2 (cos θ1 cos θ2 − sen θ1 sen θ2 ) = r1 r2 ⇔ cos θ1 cos θ2 − sen θ1 sen θ2 = 1

⇔ cos(θ1 + θ2 ) = 1 ⇔ θ1 + θ2 = 2nπ,

para n = 0, ±1, ±2, . . . 

Problema 1.6. Demostrar que la identidad 1 + z + z2 + · · · + zn =

1 − z n+1 , 1−z

se satisface para todo entero n ≥ 1 y todo z 6= 1. Soluci´ on. Tenemos que (1 + z + z 2 + · · · + z n )(1 − z) = 1 − z n+1 ,

para todo z ∈ C.

Como z 6= 1, entonces dividiendo por (1 − z) en ambos lados de la ecuaci´ on anterior obtenemos: 1 − z n+1 1 + z + z2 + · · · + zn = , 1−z  para todo entero n ≥ 1 y todo z 6= 1, que era lo que dese´ abamos demostrar.

22

1.10. PROBLEMAS PROPUESTOS

1.10 Problemas Propuestos 1.1. Exprese los siguientes n´ umeros complejos en la forma cartesiana x + iy: e) (8 + 6i)3  2 3 f) 1 + 1+i 1 + 2i 2 − i + g) 3 − 4i 5i 4 h) (1 − i)

a) (2 + 3i) + (4 + i) 2 + 3i 4+i 1 3 c) + i 1+i d) (2 + 3i)(4 + i) b)

1.2. Encuentre las soluciones de las siguientes ecuaciones: a) z 2 = 3 − 4i

b) (z + 1)2 = 3 + 4i

1.3. Simplifique las siguientes expresiones: a) (−i)−1

d) 1 −

i)−1

b) (1 − 1+i c) 1−i

e)

1+



2i

1 √

−2i

1.4. Resuelva las siguientes ecuaciones: a) z 5 − 2 = 0

b) z 4 + i = 0

c) z 6 + 8 = 0 d) z 3 − 4 = 0

1.5. Determine el conjugado de los siguientes n´ umeros complejos: a)

(3 + 8i)4 (1 + i)10

c)

b)

(1 − 2i)10 (2 + 2i)5

d)

i(2 + 3i)(5 − 2i) (−2 − i)

(2 − 3i)2 (8 + 6i)2

1.6. Calcule el m´ odulo de los siguientes n´ umeros complejos: a)

(3 + 8i)4 (1 + i)10

c)

b)

(1 − 2i)10 (2 + 2i)5

d)

i(2 + 3i)(5 − 2i) (−2 − i)

(2 − 3i)2 (8 + 6i)2

1.7. Sea z ∈ C tal que |z| = 1. Entonces, calcular |1 + z|2 + |1 − z|2 .

23

´ MEROS COMPLEJOS CAP´ITULO 1. NU

1.8. Encontrar todos los valores del argumento de z cuando el n´ umero complejo z est´ a dado por: √ √ i 3 + (3 − 3)i 3 + a) z = c) z = i−1 3 + 3i √   √ (2 3 + 1) + ( 3 − 2) i (3 + i) b) z = 5 − 5i 1.9. Exprese las caracter´ısticas de los siguientes conjuntos de puntos, en cuanto a: cerrado o abierto, conexo o no, acotado o no. a) {z ∈ C : Re z = −2}

b) {z ∈ C : Re z ≤ Im z}

f) {z ∈ C : 0 < |z − 2 + 1| < 2}

g) {z ∈ C : −1 < Re z < 1}

c) {z ∈ C : zz > 2Re z}

h) {z ∈ C : |arg z| < π/2}

e) {z ∈ C : |z − 2 + 5i| ≤ 0}

j) {z ∈ C : |z + 2i| + |z − 2i| ≤ 10}

d) {z ∈ C : |z − 3 + 4i| ≤ 6}

i) {z ∈ C : Re (z − i) = 2}

2 Funciones de Variable Compleja Las funciones de variable compleja poseen muchas aplicaciones en distintas a´reas de la Ingenier´ıa, por ejemplo, en la teor´ıa de corrientes alternas, el movimiento de fluidos o el procesamiento de se˜ nales. En este cap´ıtulo se presentan los fundamentos matem´aticos de las funciones de variable compleja. Se presta mucha atenci´ on a la definici´ on y a las propiedades de continuidad y diferenciaci´ on de una funci´ on de variable compleja. Asimismo, se describe un procedimiento para transformar regiones del plano complejo a trav´ es de funciones lineales y el cociente de funciones lineales.

2.1 Definici´ on Una funci´ on f de variable compleja es una regla de asignaci´ on que le hace corresponder a un n´ umero complejo z = x + i y uno o varios n´ umeros complejos w = u + i v. El n´ umero w se llama valor o imagen de f en z y se designa por f (z), es decir, w = f (z) o, equivalentemente, u + i v = f (x + i y). Definici´ on 2.1 (Funciones monovaluadas y multivaluadas). Sea f (z) una funci´ on de variable compleja. Si a cada n´ umero complejo z la funci´ on f (z) le hace corresponder una y s´ olo una imagen w, se dice que f (z) es monovaluada. Ahora, si la funci´ on f le hace corresponder a z m´ as de una imagen, digamos w1 , w2 , . . ., se dice que f (z) es multivaluada.

¿Todas las funciones vistas en los cursos de C´ alculo son monovaluadas?

En la Figura 2.1 se muestra una representaci´ on gr´afica, a trav´ es de diagramas, de funciones monovaluadas y multivaluadas. w = f (z) b

z

w = f (z) b

b

w

z

b

w1 w2 .. . b

b

wn Funci´ on monovaluada

Funci´ on multivaluada

Figura 2.1. Funciones monovaluada y multivaluada

Definici´ on 2.2 (Dominio y rango). El conjunto de n´ umeros complejos z en donde la funci´ on f (z) est´ a bien definida (no hay divisi´ on por cero) se denomina dominio de f (z). El conjunto de n´ umeros complejos conformado con todas las im´ agenes w = f (z) es llamado rango de f (z). 24

25

CAP´ITULO 2. FUNCIONES DE VARIABLE COMPLEJA

Definici´ on 2.3 (Polinomio complejo). Sean n ≥ 0 un entero y a0 , a1 , . . . , an constantes complejas. La funci´ on p(z) = a0 + a1 z + a2 z 2 + · · · + an z n ,

(an 6= 0)

se denomina polinomio complejo o, simplemente, polinomio de grado n. Ejercicio 2.1. Demuestra que tanto el dominio como el rango de un polinomio de grado n, es todo el plano complejo. Definici´ on 2.4 (Funci´ on racional). Sean p(z) y q(z) polinomios. La funci´ on r(z) dada por p(z) r(z) = q(z) se denomina funci´on racional y est´ a definida en todo n´ umero complejo z, excepto donde q(z) = 0, esto es, el conjunto {z ∈ C : q(z) 6= 0} que no contiene ninguna de las ra´ıces del polinomio q(z). Ejemplo 2.1. Determinar el dominio y el rango de la funci´ on r(z) =

z+1 . z−i

Soluci´ on. Se tiene que el dominio de r(z) es el conjunto de n´ umero complejos z tales que r(z) est´ a bien definida, es decir, el conjunto de n´ umeros complejos que no produzcan una divisi´ on por 0, que en este caso es {z ∈ C : z 6= i}. Ahora, para determinar el rango de r(z) se utiliza el siguiente procedimiento. Se toma w = r(z) y luego se despeja a z en funci´ on de w. A la funci´ on obtenida (la inversa de r(z)), que depende de w, se le determina el dominio. Este u ´ ltimo conjunto de n´ umeros complejos constituye el rango de r(z). As´ı, se tiene z+1 , w= z−i despejando z en funci´ on de w, se obtiene z=

1 − iw , w−1

de lo cual se deduce que el rango de r(z) es {w ∈ C : w 6= 1}.



2.2 L´ımite y Continuidad Pasemos ahora a estudiar las propiedades de las funciones de variable compleja relacionadas con el an´alisis infinitesimal, esto es, l´ımite, continuidad y derivabilidad. El objetivo de esta parte es utilizar los resultados del an´alisis infinitesimal de variable real para desarrollar los conceptos de l´ımite, continuidad y diferenciaci´ on de funciones de variable compleja. Para ello, inicialmente se introduce el concepto de funciones componentes.

Recuerde el teorema fundamen´ tal del Algebra: todo polinomio de una variable no constante con coeficientes complejos tiene una ra´ız compleja

26

2.2. L´IMITE Y CONTINUIDAD

2.2.1 Funciones Componentes Toda funci´ on de variable compleja se puede expresar en t´erminos de una par de funciones de variable real monovaluadas. Esto es, si f (z) es una funci´ on de variable compleja y z = x + i y, entonces f (z) se puede expresar como f (z) = u(x, y) + i v(x, y), donde u : R2 → R y v : R2 → R, se denominan funciones componentes de f (z). Ejemplo 2.2. Las funciones componentes, u(x, y) y v(x, y), de f (z) = z 2 + a, donde a = a1 + i a2 , est´ an dadas por: u(x, y) = x2 − y 2 + a1 y v(x, y) = 2xy + a2 . Verifique esta  afirmaci´ on. 1 Ejemplo 2.3. Las funciones componentes, u(x, y) y v(x, y), de f (z) = z + est´ an dadas z por y(x2 + y 2 − 1) x(x2 + y 2 + 1) , v(x, y) = . u(x, y) = x2 + y 2 x2 + y 2 Verifique esta afirmaci´ on.



2.2.2 L´ımite Definici´ on 2.5 (L´ımite). Sea f (z) una funci´ on definida en todos los puntos de cierta vecindad de z0 excepto, posiblemente en el mismo z0 . Se dice que w0 es un l´ımite de w = f (z) cuando z tiende a z0 , si para cada n´ umero positivo ε, existe un n´ umero positivo δ tal que |f (z) − w0 | < ε,

siempre que

0 < |z − z0 | < δ.

Denotamos con l´ım f (z) = w0

z→z0

para indicar que w0 es un l´ımite de f (z), cuando z tiende a z0 . Note que la definici´ on de l´ımite de una funci´ on de variable compleja es una extensi´ on del l´ımite de una funci´ on de variable real.

Algunas de las f´ ormulas de l´ımites que se estudiaron en el c´ alculo elemental tienen sus hom´ ologas en el caso de funciones de variable compleja. Las f´ ormulas equivalentes, que se presentan aqu´ı, se demuestran usando la definici´ on de l´ımite. Teorema 2.1. Sean f (z) y g(z) funciones de variable complejas tales que l´ım f (z) = w0 z→z0

y l´ım g(z) = W0 . Entonces, z→z0

1. l´ım [f (z) + g(z)] = w0 + W0 ; z→z0

2. l´ım [f (z) · g(z)] = w0 · W0 ; z→z0

3. l´ım [f (z)/g(z)] = z→z0

w0 , W0 6= 0. W0

27

CAP´ITULO 2. FUNCIONES DE VARIABLE COMPLEJA

Demostraci´ on. Sea ε > 0. Como w0 y W0 son respectivamente los l´ımites de f (z) y g(z) cuando z tiende a z0 , podemos escribir: ε |f (z) − w0 | < , 2

siempre que 0 < |z − z0 | < δ1 ,

y

ε |g(z) − W0 | < , siempre que 0 < |z − z0 | < δ2 , 2 donde δ1 > 0 y δ2 > 0 dependen de ε. As´ı, por la desigualdad triangular tenemos que |[f (z) + g(z)] − [w0 + W0 ]| ≤ |f (z) − w0 | + |g(z) − W0 | < ε, siempre que 0 < |z − z0 | < δ, donde δ ≤ m´ın(δ1 , δ2 ). Por lo tanto, w0 + W0 es el l´ımite de f (z) + g(z) cuando z tiende a z0 . Supongamos que r ε |f (z) − w0 | < , siempre que 0 < |z − z0 | < δ3 , 2 y |g(z) − W0 | <

r

ε , 2

siempre que 0 < |z − z0 | < δ4 ,

donde δ3 > 0 y δ4 > 0 dependen de ε. Como W0 (f (z) − w0 ) = W0 f (z) − W0 w0 y w0 (g(z) − W0 ) = w0 g(z) − w0 W0 , entonces es claro que l´ım W0 f (z) = W0 w0 y l´ım w0 g(z) = w0 W0 ; z→z0

z→z0

luego, por 1., l´ım (W0 f (z) − w0 g(z)) = 0. Ahora, tenemos que z→z0

f (z)g(z) − w0 W0 = (f (z) − w0 )(g(z) − W0 ) − (W0 f (z) − w0 g(z)). As´ı, por todo lo anterior podemos escribir |f (z)g(z) − w0 W0 | = |(f (z) − w0 )(g(z) − W0 ) − (W0 f (z) − w0 g(z))|

≤ |(f (z) − w0 )| |(g(z) − W0 )| + |(W0 f (z) − w0 g(z))| r r ε ε ε < · + = ε, 2 2 2

siempre que 0 < |z − z0 | < δ, donde δ ≤ m´ın(δ3 , δ4 ). En otras palabras, w0 W0 es el l´ımite de f (z)g(z) cuando z tiende a z0 . La demostraci´ on de la identidad 3., se deja como ejercicio para el lector. El siguiente teorema muestra la relaci´ on entre el l´ımite de una funci´ on de variable compleja y el l´ımite de sus funciones componentes que, por supuesto, son funciones reales. Teorema 2.2. Sean f (z) = u(x, y) + i v(x, y) una funci´on de variable compleja y los n´ umeros complejos z0 = x0 + i y0 y w0 = u0 + i v0 . Entonces, l´ım f (z) = w0

z→z0

si, y s´olo si l´ım

(x,y)→(x0 ,y0 )

u(x, y) = u0

y

l´ım

(x,y)→(x0 ,y0 )

v(x, y) = v0 .

28

2.2. L´IMITE Y CONTINUIDAD

Demostraci´ on. Sean f1 (z) y f2 (z) funciones de variable compleja definidas respectivamente por f1 (z) = u(x, y) y f2 (z) = v(x, y). Es claro que f (z) = f1 (z) + i f2 (z), adem´as, si l´ım f (z) = w0 , entonces por el Teorema 2.1 z→z0 tenemos que l´ım f1 (z) = u0 =

u(x, y)

y

u(x, y) = l´ım f1 (z) = u0

y

z→z0

l´ım

(x,y)→(x0 ,y0 )

l´ım f2 (z) = v0 =

z→z0

l´ım

(x,y)→(x0 ,y0 )

v(x, y).

Ahora, si l´ım

(x,y)→(x0 ,y0 )

z→z0

l´ım

(x,y)→(x0 ,y0 )

v(x, y) = l´ım f2 (z) = v0 , z→z0

entonces, por el Teorema 2.1 obtenemos l´ım f (z) = w0 . z→z0

2.2.3 Continuidad

Observe que la definici´ on continuidad de una funci´ on compleja, es una extensi´ on de la definici´ on de continuidad de un funci´ on real.

La continuidad es uno de los conceptos m´ as importantes del an´alisis. A continuaci´ on veremos que la continuidad de una funci´ on de variable compleja se puede ver como una extensi´ on del concepto de continuidad de una funci´ on de variable real. Definici´ on 2.6. Se dice que una funci´ on f (z) es continua en z0 , si satisface las dos condiciones siguientes: (i) el valor f (z0 ) est´ a bien definido; (ii) l´ımz→z0 f (z) existe y l´ım f (z) = f (z0 ).

z→z0

Se dice que f (z) es continua en un dominio D, si es continua en todo z ∈ D. En el siguiente ejemplo se utiliza la definici´ on de continuidad combinada con los teoremas de l´ımite, para identificar el conjunto de n´ umeros complejos donde una funci´ on es continua. Ejemplo 2.4. Estudiemos la continuidad de la funci´ on  2  z − 1 , z 6= 1, f (z) = z − 1  1, z = 1.

Tomemos z = x + i y. Para z 6= 1, la funci´ on f (z) se puede expresar como f (z) =

(z − 1)(z + 1) = z + 1 = (x + 1) + i y, (z − 1)

luego, las funciones componentes de f (z) son u(x, y) = x + 1 y v(x, y) = y. Como el valor de f (z0 ) est´ a bien definido para todo z0 = x0 + i y0 6= 1, entonces por el Teorema 2.2 podemos escribir: l´ım f (z) =

z→z0

l´ım

(x,y)→(x0 ,y0 )

u(x, y) + i

l´ım

(x,y)→(x0 ,y0 )

v(x, y) = x0 + 1 + i y0 = z0 + 1 = f (z0 ),

29

CAP´ITULO 2. FUNCIONES DE VARIABLE COMPLEJA

en otras palabras, f (z) es continua en todo z0 6= 1. Para z = 1, se tiene que f (1) est´ a bien definido, pero l´ım f (z) = 2 6= f (1) = 1. Por lo z→1

tanto, f (z) no es continua en z = 1.



Pasemos ahora a ver algunos teoremas de continuidad. El siguiente teorema dice que las funciones definidas como la suma, multiplicaci´ on o divisi´ on de funciones continuas, son tambi´ en funciones continuas en su dominio de definici´ on. La demostraci´ on de este teorema se obtiene utilizando el Teorema 2.1. Teorema 2.3. Sean f (z) y g(z) dos funciones de variable compleja continuas en un dominio D. Entonces: 1. la funci´on f (z) + g(z) es continua en D; 2. la funci´on f (z)g(z) es continua en D; 3. la funci´on f (z)/g(z) es continua en D \ {z ∈ D : g(z) = 0}. on Observaci´ on 2.1. El Teorema 2.3 nos permite asegurar que un polinomio es una funci´ continua en todo el plano complejo; tambi´ en nos permite aseverar que una funci´ on racional es continua en todo el plano complejo excepto donde su denominador se anule. En el an´alisis real se tiene que la composici´ on de funciones de variable real continuas produce una nueva funci´ on continua. En el caso de las funciones de variable compleja tambi´ en se satisface este hecho, es decir, la composici´ on de funciones de variable compleja continuas es continua. Esta afirmaci´ on se plantea en el siguiente teorema, cuya demostraci´ on se obtiene directamente de la definici´ on de continuidad. Teorema 2.4. Sean f (z) y g(z) dos funciones de variable compleja definidas respectivamente en los dominios D y E, tales que f (D) ⊆ E. Si f es continua en D y g es continua en f (D), entonces la funci´ on h(z) = g(f (z)) es continua en D. Por otra parte, dado que el l´ımite de una funci´ on de variable compleja se puede calcular a trav´ es del l´ımite de sus funciones componentes (Teorema 2.2), es natural inferir que la continuidad de una funci´ on de variable compleja se corresponde con la continuidad de sus funciones componentes. Seguidamente se muestra un teorema que trata este aspecto, cuya demostraci´ on es inmediata utilizando la definici´ on de continuidad de funciones reales conjuntamente con el Teorema 2.2. Teorema 2.5. Sea f (z) = u(x, y) + i v(x, y) una funci´on de variable compleja. Entonces, f (z) es continua en un punto z0 = x0 + i y0 si, y s´olo si sus funciones componentes, u(x, y) y v(x, y), son continuas en el punto (x0 , y0 ). A continuaci´ on damos un ejemplo que muestra la aplicaci´ on de los Teoremas 2.3 y 2.5, en el estudio de la continuidad de una funci´ on de variable compleja. Ejemplo 2.5. Estudiemos la continuidad en z = 1 de la funci´ on f (z) = 2x + iy +

x − iy . x2 + y 2

Verifiquemos, a trav´ es de dos procedimientos, la continuidad de f (z). Primero, utilizando el Teorema 2.3; despu´es, empleando el Teorema 2.5. Escribamos a f (z) en funci´ on de z,

En general, los resultados de continuidad para funciones complejas son extensiones de los resultados de continuidad para funciones reales.

30

2.2. L´IMITE Y CONTINUIDAD

para ello consideremos las siguientes expresiones de x e y, x=

z+z , 2

y=

z−z . 2i

Sustituyendo convenientemente estas expresiones en la f´ ormula de definici´ on de f (z), obtenemos: 

   z+z z−z f (z) = 2 +i + 2 2i z z−z + = z+z+ 2 zz 1 1 3 z+ z+ . = 2 2 z

z+z 2

+ i z−z 2i zz

Definamos tres funciones f1 (z), f2 (z) y f3 (z) como f1 (z) =

3 z, 2

f2 (z) =

1 z, 2

f3 (z) =

1 . z

Las funciones f1 (z) y f3 (z) son, respectivamente, un polinomio de grado uno y una funci´ on racional. As´ı, f1 (z) es continua en todo el plano, particularmente en z = 1, y f3 (z) es continua en todo el plano excepto en z = 0, por lo tanto f3 (z) es continua en z = 1. En cuanto a la funci´ on f2 (z), se deja como ejercicio para el lector verificar que es continua en z = 1 (ayuda: utilice el Teorema 2.5). En consecuencia, como f1 (z), f2 (z) y f3 (z) son continuas en z = 1 y f (z) = f1 (z) + f2 (z) + f3 (z), entonces por el Teorema 2.3 se tiene que f (z) es continua en z = 1. Ahora empleemos el Teorema 2.5 para estudiar la continuidad de f (z) en z = 1. Para ello es necesario encontrar las funciones componentes, u(x, y) y v(x, y), de f (z). Operando obtenemos:     y x +i y− 2 , f (z) = 2x + 2 x + y2 x + y2 de donde se deduce que u(x, y) = 2x +

x2

x + y2

y

v(x, y) = y −

x2

y . + y2

Como u y v est´ an bien definidas en el punto (1, 0) y l´ım

(x,y)→(0,1)

¿Qu´e procedimiento le pareci´ o m´ as eficiente, usar el Teorema 2.3 o el Teorema 2.5? ¿Por qu´e?

u(x, y) = 3 = u(1, 0),

l´ım

(x,y)→(0,1)

v(x, y) = 0 = v(1, 0),

podemos concluir que u y v son continuas en (1, 0). Entonces, por el Teorema 2.5 la funci´ on  f (z) es continua en z = 1.

31

CAP´ITULO 2. FUNCIONES DE VARIABLE COMPLEJA

2.3 Diferenciaci´ on . Inmediatamente damos la definici´ on de derivada de una funci´ on de variable compleja. Definici´ on 2.7 (Derivada). Sea f (z) una funci´ on de variable compleja definida en un dominio D. Sea z0 un punto de acumulaci´ on de D. La derivada de f (z) en z0 , denotada d por f (z0 ) o f ′ (z0 ), es una funci´ on de variable compleja que se define mediante la dz ecuaci´ on d f (z0 + h) − f (z0 ) f (z0 ) = f ′ (z0 ) = l´ım , (2.1) h→0 dz h

¿Por qu´e en la definici´ on de derivada es necesario que z0 sea un punto de acumulaci´ on?

donde h es un n´ umero complejo. Se dice que f (z) es derivable en un dominio D ⊂ C, si f ′ (z) existe en cada punto z ∈ D. Teorema 2.6. Si la derivada de f (z) en z0 existe, entonces f (z) es continua en z0 . Demostraci´ on. Como la derivada de f (z) existe en z0 , entonces |f ′ (z0 )| < ∞ y f (z0 ) est´ a bien definido. Por tanto, l´ım |f (z0 + h) − f (z0 )| = l´ım |h| |f ′ (z0 )| = 0,

h→0

h→0

de donde se deduce que f (z) es continua en z0 . Observaci´ on 2.2. El Teorema 2.6 nos garantiza que toda funci´ on derivable es continua. Lo que no es cierto es que toda funci´ on continua es derivable; por ejemplo, f (z) = |z|2 es continua en todo el plano complejo, pero es derivable solamente en z = 0 (se deja al lector verificar este hecho).

2.3.1 F´ ormulas o Reglas de Diferenciaci´ on A continuaci´ on se describen las mismas reglas de diferenciaci´ on dadas en el C´ alculo Elemental, pero ahora se plantean para funciones de variable compleja. Supongamos que las funciones f (z), f1 (z), . . . , fn (z) son derivables en z ∈ C. Las siguientes propiedades son ciertas y su demostraci´ on se obtiene f´acilmente utilizando la definici´ on de derivada y los Teoremas 2.1 y 2.2. 1. Si f (z) = c, entonces f ′ (z) = 0, donde c ∈ C. 2. Si h(z) = c f (z), entonces h′ (z) = c f ′ (z), donde c ∈ C. 3. Si f (z) = z, entonces f ′ (z) = 1. 4. [f1 (z) + f2 (z) + · · · + fn (z)]′ = f1′ (z) + f2′ (z) + · · · + fn′ (z). 5. [f1 (z)f2 (z)]′ = f1′ (z)f2 (z) + f2′ (z)f1 (z). 6. [(f (z))m ]′ = m (f (z))m−1 f ′ (z), donde m es un entero. 7. Si f (z) = z m , entonces f ′ (z) = m z m−1 , donde m es un entero.

Tenga presente que para usar las reglas de derivaci´ on, las funciones involucradas deben ser derivables.

32

´ 2.3. DIFERENCIACION

8. Si f (z) = a0 + a1 z + a2 z 2 + · · · + am z m , entonces f ′ (z) = a1 + 2a2 z + 3a3 z 2 + · · · + m am z m−1 . 9.



f1 (z) f2 (z)

′

=

f1′ (z)f2 (z) − f2′ (z)f1 (z) , siempre y cuando f2 (z) 6= 0. (f2 (z))2

10. Regla de la Cadena. Sean f (z) derivable en z0 y g(w) derivable en f (z0 ). Entonces la funci´ on h(z) = g(f (z)) es derivable en z0 , y h′ (z0 ) = g′ (f (z0 )) f ′ (z0 ). Note que todas las reglas de diferenciaci´ on se cumplen, si las funciones involucradas son derivables. De esta forma, para aplicar alguna de estas reglas, es necesario primero verificar que las funciones consideradas sean derivables. En el C´ alculo Elemental, por lo general, era relativamente sencillo verificar si la derivada de una funci´ on exist´ıa o no. Por ejemplo, observando la gr´afica de una funci´ on f : R → R se pod´ıa identificar los puntos donde exist´ıa la derivada. Este procedimiento, que por dem´as es muy sencillo, no se puede usar para verificar que una funci´ on de variable compleja es derivable. En la siguiente secci´ on damos un procedimiento para estudiar la derivabilidad de las funciones de variable compleja.

2.3.2 Ecuaciones de Cauchy-Riemann

Las ecuaciones de CauchyRiemann tambi´ en se pueden expresar en la siguiente notaci´ on m´ as compacta: ux = vy , uy = −vx

Si una funci´ on de variable compleja es derivable en un punto, sus funciones componentes satisfacen las llamadas ecuaciones de Cauchy-Riemann en ese punto. A continuaci´ on damos una breve deducci´ on de estas ecuaciones, que son las condiciones necesarias para la existencia de la derivada de una funci´ on de variable compleja. Teorema 2.7. Sea f (z) = u(x, y) + i v(x, y) derivable en z0 = x0 + i y0 . Entonces, las funciones componentes, u(x, y) y v(x, y), satisfacen las ecuaciones de Cauchy-Riemann en el punto (x0 , y0 ), a saber:  ∂u ∂v  (x0 , y0 )   ∂x (x0 , y0 ) = ∂y    ∂u (x0 , y0 ) = − ∂v (x0 , y0 ) ∂y ∂x

(2.2)

Demostraci´ on. Como f (z) = u(x, y) + i v(x, y) es derivable en z0 = x0 + i y0 , se tiene f (z0 + h) − f (z0 ) . h→0 h

f ′ (z0 ) = l´ım

Ahora bien, utilizando las funciones componentes de f (z) y tomando h = h1 + i h2 , la ecuaci´ on anterior se expresa como: [u(x0 + h1 , y0 + h2 ) + i v(x0 + h1 , y0 + h2 )] − [u(x0 , y0 ) + i v(x0 , y0 )] h→0 h [u(x0 + h1 , y0 + h2 ) − u(x0 , y0 )] + i [v(x0 + h1 , y0 + h2 ) − v(x0 , y0 )] . = l´ım h→0 h

f ′ (z0 ) =

l´ım

33

CAP´ITULO 2. FUNCIONES DE VARIABLE COMPLEJA

Como la derivada de f (z) existe en z0 , podemos tender a 0 tomando h un real puro o un imaginario puro y el l´ımite, que define la derivada de f (z) en z0 , siempre es igual. Supongamos primero que h = h1 > 0, es decir, h2 = 0, entonces obtenemos: [u(x0 + h1 , y0 ) − u(x0 , y0 )] + i [v(x0 + h1 , y0 ) − v(x0 , y0 )] h1 u(x0 + h1 , y0 ) − u(x0 , y0 ) v(x0 + h1 , y0 ) − v(x0 , y0 ) = l´ım + i l´ım h1 →0 h1 →0 h1 h1 ∂v ∂u (x0 , y0 ) + i (x0 , y0 ). (2.3) = ∂x ∂x

f ′ (z0 ) =

l´ım

h1 →0

Ahora, si tomamos h = i h2 con h2 > 0, es decir, h1 = 0, entonces obtenemos: [u(x0 , y0 + h2 ) − u(x0 , y0 )] + i [v(x0 , y0 + h2 ) − v(x0 , y0 )] i h2 u(x0 , y0 + h2 ) − u(x0 , y0 ) v(x0 , y0 + h2 ) − v(x0 , y0 ) = l´ım + i l´ım h2 →0 h2 →0 i h2 i h2 ∂v ∂u (x0 , y0 ) + (x0 , y0 ). (2.4) = −i ∂y ∂y

f ′ (z0 ) =

l´ım

h2 →0

De las ecuaciones (2.3) y (2.4) se deduce que ∂u ∂v ∂v ∂u (x0 , y0 ) + i (x0 , y0 ) = (x0 , y0 ) − i (x0 , y0 ), ∂x ∂x ∂y ∂y es decir, f (z) satisface las ecuaciones de Cauchy-Riemann (2.2) en (x0 , y0 ). El hecho que una funci´ on satisfaga las ecuaciones de Cauchy-Riemann en un punto, no es suficiente para garantizar que f (z) sea derivable en tal punto. Por ejemplo, la funci´ on  2 z   , z 6= 0, f (z) = z   0, z = 0,

satisface las ecuaciones de Cauchy-Riemann en z = 0, pero no es derivable all´ı (se deja al lector verificar este hecho). As´ı pues, la validez de las ecuaciones de Cauchy-Riemman en un punto es una condici´ on necesaria para que exista la derivada en dicho punto. El solo hecho que las ecuaciones de Cauchy-Riemman sean v´ alidas para una funci´ on, no significa que todas las trayectorias por las que z0 + h se aproxime a z0 den lugar a un mismo valor l´ımite de la definici´ on de derivada. El siguiente teorema muestra las condiciones necesarias y suficientes para que una funci´ on sea derivable en un punto. Teorema 2.8. Sean f (z) = u(x, y) + i v(x, y) y z0 = x0 + i y0 . Si u(x, y) y v(x, y) tienen primeras derivadas parciales continuas, con respecto a x e y, en (x0 , y0 ), y satisfacen las ecuaciones de Cauchy-Riemann (2.2) en (x0 , y0 ), entonces f ′ (z0 ) existe y est´ a dada por: f ′ (z0 ) =

∂v ∂u (x0 , y0 ) + i (x0 , y0 ) o ∂x ∂x

f ′ (z0 ) =

∂v ∂u (x0 , y0 ) − i (x0 , y0 ). ∂y ∂y

34

2.4. FUNCIONES ANAL´ITICAS

Demostraci´ on. La demostraci´ on es inmediata utilizando el Teorema 2.7 y el hecho que u(x, y) y v(x, y) tienen primeras derivadas parciales continuas, con respecto a x e y, en (x0 , y0 ). Ejemplo 2.6. Comprobemos que la funci´ on f (z) = ex cos y + i ex sen y es derivable en todo punto z del plano complejo. Las funciones componentes de f (z) est´ an dadas por: u(x, y) = ex cos y,

v(x, y) = ex sen y.

As´ı, las derivadas parciales de u y v, con respecto a x e y, son: ∂u = ex cos y, ∂x

∂u = −ex sen y, ∂y

∂v = ex sen y, ∂x

La funci´ on f (z) del Ejemplo 2.6 satisface que f ′ (z) = f (z). ¿Ser´ a f (z) la funci´ on exponencial compleja?

∂v = ex cos y, ∂y

las cuales son continuas en todo R2 y, adem´as, satisfacen las ecuaciones de Cauchyon f (z) = ex cos y + Riemann en todo z = x + i y. Entonces, por el Teorema 2.8 la funci´ x i e sen y es derivable en todo el plano complejo. 

2.4 Funciones Anal´ıticas En esta secci´ on se introduce un conjunto de funciones de variable compleja denominadas anal´ıticas, que poseen una propiedad muy particular: si f (z) es anal´ıtica en un punto z0 , entonces es derivable en todo punto z muy cercano a z0 . Esta cualidad de las funciones anal´ıticas las hace muy importantes para el desarrollo te´ orico de aplicaciones en Ingenier´ıa. Definici´ on 2.8 (Funci´ on anal´ıtica). Se dice que una funci´ on f (z) es anal´ıtica en z0 ∈ C, si la derivada de f (z) existe en z0 y en todo punto z de alguna vecindad de z0 . Si f (z) es anal´ıtica en todos los puntos de un dominio D ⊂ C, se dice que f (z) es anal´ıtica en D. Ejemplo 2.7. Determinemos el conjunto de n´ umeros complejos z = x+i y donde la funci´ on 3 2 2 3 f (z) = (x − 3xy ) + i (3x y − y ) es anal´ıtica. Se tiene que las funciones componentes de f (z) est´ an dadas por: u(x, y) = x3 − 3xy 2

y

v(x, y) = 3x2 y − y 3 .

Ahora, las derivadas parciales de u y v, con respecto a x e y, son ∂u = 3x2 − 3y 2 , ∂x

∂u = −6xy, ∂y

∂v = 6xy, ∂x

∂v = 3x2 − 3y 2 . ∂y

Como las derivadas parciales de u y v son continuas y satisfacen las ecuaciones de CauchyRiemann en todos los puntos (x, y) del plano, entonces f (z) es derivable en todo el plano complejo. Por lo tanto, f (z) es anal´ıtica en todo el plano complejo. 

35

CAP´ITULO 2. FUNCIONES DE VARIABLE COMPLEJA

Ejemplo 2.8. Determinemos el conjunto de n´ umeros complejos z = x + i y donde la funci´ on f (z) = x2 + i y 2 es anal´ıtica. Las funciones componentes de f (z) son: u(x, y) = x2 y v(x, y) = y 2 . Al forzar que se satisfagan las ecuaciones de Cauchy-Riemann obtenemos: ∂v ∂u = 2x = 2y = ∂x ∂y

y

∂u ∂v =0=− , ∂y ∂x

de donde se deduce que f (z) es derivable u ´ nicamente en los puntos z = x + i y que pertenecen a la recta y = x. Si z0 es un punto de esta recta, toda vecindad de z0 contiene puntos en los que f ′ (z0 ) no existe. As´ı pues, f (z) no es anal´ıtica en ning´ un punto del plano  complejo. Los ejemplos anteriores muestran que la propiedad de analiticidad es muy fuerte, dado que al verificar que una funci´ on es anal´ıtica en un punto, se est´ a comprobando no solo que la funci´ on es derivable en dicho punto, sino que tambi´ en es derivable en todo punto de una vecindad del punto en cuesti´ on. Por ello, las funciones anal´ıticas, gracias a sus maravillosas propiedades, juegan un papel muy importante en la teor´ıa de las funciones de variable compleja. El siguiente teorema afirma que la suma, diferencia, multiplicaci´ on y divisi´ on de funciones anal´ıticas es otra funci´ on anal´ıtica. Tambi´ en establece que la composici´ on de funciones anal´ıticas es una funci´ on anal´ıtica. La demostraci´ on de este teorema se deja al lector. Teorema 2.9. Sean f (z) y g(z) dos funciones anal´ıticas en un dominio D ⊂ C. Entonces, f (z) + g(z), f (z) − g(z) y f (z)g(z), son funciones anal´ıticas en D. La funci´on f (z)/g(z) es anal´ıtica en el conjunto {z ∈ D : g(z) 6= 0}. Si la funci´on g(z) es anal´ıtica en el conjunto f (D), entonces la funci´ on h(z) = g(f (z)) es anal´ıtica en D.

Definici´ on 2.9 (Funci´ on entera). Se dice que una funci´ on f (z) es entera si es anal´ıtica en todo punto z del plano complejo.

Ejemplo 2.9. En el Ejemplo 2.6 vimos que la funci´ on f (z) = ex cos y+i ex sen y es derivable en todo punto z del plano complejo. Por lo tanto, f (z) es anal´ıtica en todo el plano, de lo cual se deduce que f (z) es entera.  Ahora bien, para ciertas funciones que son anal´ıticas en determinados conjuntos de n´ umeros complejos, existen puntos donde ellas no son anal´ıticas. Tales puntos se denominan puntos singulares y es de mucha importancia identificarlos. Seguidamente damos la definici´ on formal de punto singular. Definici´ on 2.10 (Punto singular). Sea f (z) una funci´ on de variable compleja. Se dice que un punto z0 ∈ C es un punto singular de f (z), si f (z) no es anal´ıtica en z0 y para toda vecindad de z0 f (z) es anal´ıtica en al menos un punto z diferente de z0 .

36

´ 2.5. FUNCIONES ARMONICAS

Otros puntos muy importantes que tambi´ en se deben identificar son los ceros de una funci´ on. Definici´ on 2.11 (Cero de una funci´ on). Sea f una funci´ on de variable compleja. Se dice z0 ∈ C es un cero de f (z) si l´ım f (z) = 0.

z→z0

Observaci´ on 2.3. La funci´ on racional r(z) =

a0 + a1 z + · · · + a n z n b0 + b1 z + · · · + bm z m

es anal´ıtica en todo el plano complejo, excepto en las ra´ıces del polinomio del denominador, en otras palabras, salvo en los puntos z tales que b0 + b1 z + · · · + bm z m = 0. Estas ra´ıces son los puntos singulares de r(z). Adem´ as, las ra´ıces del polinomio a0 + a1 z + · · · + an z n son los ceros de r(z). Ejemplo 2.10. Los puntos z0 = 0 y z1 = 1 son, respectivamente, el u ´ nico punto singular y  el u ´ nico cero de la funci´ on f (z) = (z − 1)/z.

onicas 2.5 Funciones Arm´ Pasemos ahora a estudiar la relaci´ on de las funciones anal´ıticas con una familia de funciones de variable real muy importante, denominadas arm´onicas. Definici´ on 2.12 (Funci´ on arm´ onica). Se dice que una funci´ on h : R2 → R es arm´onica 2 en un dominio D ⊂ R , si en todo punto (x, y) ∈ D tiene derivadas parciales, primera y segunda, continuas y satisface la ecuaci´ on en derivadas parciales ∇2 h(x, y) =

∂2h ∂2h (x, y) + 2 (x, y) = 0, 2 ∂x ∂y

conocida como ecuaci´ on de Laplace. Definici´ on 2.13 (Arm´ onica conjugada). Sean u : R2 → R y v : R2 → R dos funciones. Si u y v son arm´ onicas en un dominio D ⊂ R2 y sus primeras derivadas parciales satisfacen las ecuaciones de Cauchy-Riemann (2.2) para todo (x, y) ∈ D, se dice que v es arm´onica conjugada de u. El siguiente teorema establece una relaci´ on entre las funciones arm´ onicas y las funciones anal´ıticas. Teorema 2.10. Una funci´ on f (z) = u(x, y) + i v(x, y) es anal´ıtica en un dominio D ⊂ C si, y s´olo si v es arm´onica conjugada de u. Demostraci´ on. (⇒) Supongamos que f (z) = u(x, y) + i v(x, y) es anal´ıtica en un dominio D ⊂ C y demostremos que v es arm´ onica conjugada de u. Como f (z) es anal´ıtica, sus funciones componentes, u y v, satisfacen las ecuaciones de Cauchy-Riemann, adem´as, sus

37

CAP´ITULO 2. FUNCIONES DE VARIABLE COMPLEJA

primeras derivadas parciales son continuas en D. Por ello, basta demostrar que u y v son arm´ onicas para garantizar que v es arm´ onica conjugada de u. As´ı, derivando con respecto ∂v ∂u , obtenemos: a x la ecuaci´ on de Cauhy-Riemann ∂x = ∂y ∂2v ∂2u = . ∂x2 ∂x∂y Ahora, derivando con respecto a y la otra ecuaci´ on de Cauchy-Riemann nemos: ∂2u ∂2v = − . ∂y 2 ∂y∂x

(2.5) ∂u ∂y

∂v = − ∂x , obte-

(2.6)

Como las primeras derivadas parciales de u y v son continuas en D, se tiene que ∂2v ∂2v = . ∂x∂y ∂y∂x

(2.7)

Por (2.5), (2.6) y (2.7) podemos escribir: ∇2 u(x, y) =

∂2u ∂2u (x, y) + (x, y) = 0, ∂x2 ∂y 2

con lo cual se demuestra que u es arm´ onica en D. Utilizando un razonamiento similar se demuestra que v tambi´ en es arm´ onica. En consecuencia, u y v son funciones arm´ onicas que satisfacen las ecuaciones de Cauchy-Riemann en D, es decir, v es arm´ onica conjugada de u. (⇐) Supongamos que v es arm´ onica conjugada de u en D y demostremos que la funci´ on f (z) = u(x, y) + i v(x, y) es anal´ıtica en D. Como v es arm´ onica conjugada de u en D, entonces u y v son arm´ onicas en D y satisfacen las ecuaciones de Cauchy-Riemann; adem´as, sus primeras derivadas parciales son continuas en D. As´ı, por el Teorema 2.8, la funci´ on f (z) = u(x, y) + i v(x, y) es anal´ıtica en D. Observaci´ on 2.4. El Teorema 2.10 nos garantiza que una funci´ on f (z) = u(x, y) + i v(x, y) es anal´ıtica, si v es arm´ onica conjugada de u. Pero, no es cierto que si u y v son arm´ onicas, entonces f (z) = u(x, y) + i v(x, y) es anal´ıtica; por ejemplo, si tomamos u(x, y) = x + y y v(x, y) = x, observamos que u y v son arm´ onicas en todo el plano, pero f (z) = (x + y) + i x no es anal´ıtica en ning´ un punto del plano (se deja al lector demostrar esta afirmaci´ on). Es bueno aclarar que si v es arm´ onica conjugada de u en cierto dominio D, no es en general cierto que u sea arm´ onica conjugada de v en ese dominio. Por ejemplo, consideremos las funciones u(x, y) = x2 − y 2 y v(x, y) = 2xy. Vemos que u y v son las partes real e imaginaria, respectivamente, de f (z) = z 2 , que es una funci´ on entera. Por el Teorema 2.10, v es arm´ onica conjugada de u en todo el plano, pero u no puede ser una arm´ onica conjugada 2 2 de v, ya que la funci´ on g(z) = 2xy + i (x − y ) no es anal´ıtica en ning´ un punto del plano, porque sus funciones componentes no satisfacen las ecuaciones de Cauchy-Riemann. Por otra parte, es cierto que una arm´ onica conjugada, cuando existe, es u ´ nica, excepto por una constante aditiva. Expliquemos con un ejemplo un m´ etodo para obtener una arm´ onica conjugada de una funci´ on arm´ onica dada. Ejemplo 2.11. Determine v(x, y), la arm´ onica conjugada de u(x, y), si u(x, y) = x + y. Soluci´ on. Para que v sea arm´ onica conjugada de u, la funci´ on v debe ser arm´ onica y,

Note que g(z) = 2xy + i(x2 − y 2 ) solo es derivable en z = 0.

38

´ 2.5. FUNCIONES ARMONICAS

adem´as, se deben satisfacer las ecuaciones de Cauchy-Riemann (2.2). El m´ etodo para generar v, una arm´ onica conjugada de u, consiste en forzar que se cumplan las ecuaciones de Cauchy-Riemann y luego resolver ciertas ecuaciones diferenciales. Apliquemos tal metodolog´ıa al problema planteado. Como u(x, y) = x + y, se tiene que u es arm´ onica y ∂u = 1, ∂x

y

∂u = 1. ∂y

Ahora, utilizando una de las ecuaciones de Cauchy-Riemann, digamos ∂v ∂u = , ∂x ∂y podemos escribir: ∂v = 1. ∂y Integrando esta u ´ ltima ecuaci´ on con respecto a y, obtenemos: v(x, y) = y + φ(x),

(2.8)

donde φ : R → R. Ahora, utilizando la otra ecuaci´ on de Cauchy-Riemann, ∂u ∂v =− , ∂y ∂x podemos escribir: φ′ (x) =

∂u ∂v =− = −1, ∂x ∂y

es decir, φ′ (x) = −1. Integrando la ecuaci´ on anterior con respecto a x, obtenemos: φ(x) = −x + c, donde c es una constante real. Sustituyendo la expresi´ on de φ(x) en (2.8), la funci´ on v arm´ onica conjugada de u est´ a dada por v(x, y) = y − x + c, que es arm´ onica (se deja al lector verificar esta afirmaci´ on). Es decir, hemos construido una familia de funciones arm´ onicas conjugadas de u que se diferencian entre s´ı por una constante. Para obtener una de ellas es necesario un valor de v en alg´ un punto del plano. Por ejemplo, si v(0, 0) = 1, entonces la constante c correspondiente es c = 1 y la funci´ on v es v(x, y) = y − x + 1.  El procedimiento anterior para calcular funciones v arm´ onicas conjugadas de u, tambi´ en se puede utilizar, seg´ un el Teorema 2.10, para construir funciones anal´ıticas a partir de una de sus funciones componentes, como se muestra en el siguiente ejemplo.

39

CAP´ITULO 2. FUNCIONES DE VARIABLE COMPLEJA

Ejemplo 2.12. Sea v(x, y) = x. Determine una funci´ on anal´ıtica f (z) = u(x, y) + i v(x, y) tal que f (0) = −1. Soluci´ on. Como v(x, y) = x, se tiene que v es arm´ onica y ∂v =1 ∂x Es indiferente la ecuaci´ on de Cauchy-Riemann con que se comience el procedimiento. Un ejercicio interesante es determinar la funci´ on u(x, y), comenzando con la ecuaci´ on uy = −vx y luego utilizar la ecuaci´ on ux = vy .

y

∂v = 0. ∂y

Ahora, utilizando la ecuaci´ on de Cauchy-Riemann

∂u ∂x

=

∂v ∂y ,

podemos escribir:

∂u = 0. ∂x Integrando esta u ´ ltima ecuaci´ on con respecto a x, obtenemos: u(x, y) = φ(y), donde φ : R → R. Ahora, utilizando la otra ecuaci´ on de Cauchy-Riemann, podemos escribir: ∂v ∂u =− = −1, φ′ (y) = ∂y ∂x

∂u ∂y

∂v = − ∂x ,

es decir, φ′ (y) = −1. Integrando la ecuaci´ on anterior con respecto a y, obtenemos: φ(y) = −y + c, donde c es una constante real. Por lo tanto, considerando la expresi´ on de φ(y), la funci´ on u est´ a dada por u(x, y) = −y + c. Por construcci´ on, v es arm´ onica conjugada de u, luego, por el Teorema 2.10 las funciones f (z) est´ an dadas por f (z) = c − y + i x, las cuales constituyen una familia de funciones enteras que se diferencian entre s´ı por una constante real c. Ahora, la funci´ on deseada debe satisfacer f (0) = −1. Forzando esta u ´ ltima condici´ on y despejando c, se tiene que c = −1 y la funci´ on pedida es f (z) = −1 − y + i x = iz − 1. 

2.6 Funciones Elementales A continuaci´ on damos una breve descripci´ on de las funciones elementales de variable compleja.

40

2.6. FUNCIONES ELEMENTALES

2.6.1 Funci´ on Exponencial La funci´ on exponencial, denotada por ez , se define como ez = ex cos y + i ex sen y,

(2.9)

para todo n´ umero complejo z = x + i y. Las funciones componentes de ez son u(x, y) = ex cos y

y

v(x, y) = ex sen y,

las cuales satisfacen las ecuaciones de Cauchy-Riemann y sus derivadas parciales son continuas en todo el plano complejo. Luego, la funci´ on exponencial es anal´ıtica en todo el plano complejo (ver Ejemplo 2.6). Por lo tanto, ez es una funci´ on entera, cuya derivada est´ a dada por: ∂u ∂v d z e = +i = ex cos y + i ex sen y = ez , dz ∂x ∂x que coincide con la propiedad deseada de la funci´ on exponencial real. Otras propiedades de la funci´ on exponencial • Si z = a ∈ R, entonces ez = ea cos 0+i ea sen 0 = ea , es decir, ez se reduce a la funci´ on x e , si z toma valores reales. • El rango de la funci´ on exponencial es todo el plano complejo excepto z = 0. En z efecto, como |e | = ex > 0, para todo x ∈ R, entonces |ez | > 0 para todo z ∈ C. Por lo tanto, ez 6= 0 para todo z ∈ C. • La funci´ on exponencial es peri´ odica con un periodo imaginario puro de 2πi. En efecto, por la periodicidad de las funciones trigonom´etricas sen y cos, podemos escribir: e(z+2πi) = ex+i (y+2π) = ex (cos(y + 2π) + i sen (y + 2π)) = ex (cos y + i sen y) = ez , es decir, ez es peri´ odica con periodo 2πi. • Propiedades algebraicas. Sean z1 = x1 + i y1 y z2 = x2 + i y2 . Las siguientes propiedades son ciertas: i) ez1 ez2 = ez1 +z2 ; ez1 ii) z2 = ez1 −z2 ; e iii) e0 = 1; 1 iv) z1 = e−z1 ; e v) (ez1 )n = en z1 , (n = 0, ±1, ±2, . . .).

41

CAP´ITULO 2. FUNCIONES DE VARIABLE COMPLEJA

2.6.2 Funciones Trigonom´ etricas Se definen, para todo z ∈ C, la funci´ on seno complejo como sen z =

eiz − e−iz 2i

(2.10)

cos z =

eiz + e−iz . 2

(2.11)

y la funci´ on coseno complejo como

Las funciones sen z y cos z son combinaciones lineales de las funciones enteras eiz y e−iz , por lo tanto, sen z y cos z son funciones enteras; adem´as, sus derivadas son: i(eiz + e−iz ) d sen z = = cos z dz 2i y d i(eiz − e−iz ) cos z = = −sen z. dz 2i Otras propiedades de las funciones seno y coseno • Cuando z es un n´ umero real, sen z y cos z coinciden con las funciones reales sen x y cos x. En efecto, si z = x ∈ R, entonces por la definici´ on de sen z, cos z y ez , podemos escribir: sen z =

(cos x + i sen x) − (cos(−x) + i sen (−x)) 2i sen x eix − e−ix = = = sen x, 2i 2i 2i

cos z =

(cos x + i sen x) + (cos(−x) + i sen (−x)) 2 cos x eix + e−ix = = = cos x. 2 2 2i

• Las identidades trigonom´ etricas que satisfacen el seno y el coseno real tambi´ en son v´alidas en el caso complejo; por ejemplo: sen 2 z + cos2 z = 1, sen (z1 ± z2 ) = sen z1 cos z2 ± cos z1 sen z2 , cos(z1 ± z2 ) = cos z1 cos z2 ∓ sen z1 sen z2 ,

entre otras. (Se deja como ejercicio para el lector la comprobaci´ on de cada una de las identidades). • A partir de la definici´ on de sen z dada en (2.10) podemos escribir, para z = x + i y: ei(x+i y) − e−i(x+i y) 2i ey e−y (cos x + i sen x) − (cos x − i sen x) = 2i  2i    ey + e−y ey − e−y = sen x + i cos x . 2 2

sen z =

42

2.6. FUNCIONES ELEMENTALES

En otras palabras, la funci´ on sen z se puede expresar equivalentemente como: sen z = sen x cosh y + i cos x senh y.

(2.12)

De la misma manera, al utilizar la definici´ on de cos z dada en (2.11) obtenemos: cos z = cos x cosh y − i sen x senh y.

(2.13)

• Las funciones seno y coseno son peri´ odicas con periodo 2π. (Se deja como ejercicio para el lector la demostraci´ on). • Los ceros de sen z son todos los n´ umeros complejos z = nπ, para n = 0, ±1, ±2, . . .. En efecto, para que z ∈ C sea un cero de sen z, se debe cumplir que sen z = 0, luego, utilizando la ecuaci´ on (2.12) podemos escribir: sen x cosh y = 0,

cos x senh y = 0.

Ahora, utilizando la ecuaci´ on sen x cosh y = 0, se deduce que y = 0. Sustituyendo este valor de y en la ecuaci´ on sen x cosh y = 0, obtenemos sen x = 0, de lo cual se tiene que x = nπ, para n = 0, ±1, ±2, . . ... En consecuencia, los ceros de sen z son todos los n´ umeros complejos z = nπ, para n = 0, ±1, ±2, . . ..

• Los ceros de cos z son todos los n´ umeros complejos z = (n + 21 )π, para n = 0, ±1, . . .. (Se deja al lector verificar este hecho). Otras funciones trigonom´ etricas Las dem´as funciones trigonom´ etricas de argumento complejo se definen f´acilmente por analog´ıa con las funciones trigonom´etricas de argumento real, esto es, Tangente tan z =

sen z cos z

cot z =

cos z sen z

sec z =

1 cos z

Cotangente

Secante

Cosecante

1 sen z Las funciones tan z, cot z, sec z y csc z, son anal´ıticas en todos los n´ umeros complejos z donde no se anule el denominador de la expresi´ on que las define, adem´as, la derivada de cada una de ellas es, respectivamente, d tan z = sec2 z, dz d cot z = − csc2 z, dz d sec z = tan z sec z, dz d csc z = − cot z csc z. dz csc z =

43

CAP´ITULO 2. FUNCIONES DE VARIABLE COMPLEJA

2.6.3 Funciones Hiperb´ olicas Se definen, para todo z ∈ C, la funci´ on seno hiperb´olico complejo como senh z =

ez − e−z 2

(2.14)

y la funci´ on coseno hiperb´olico complejo como cosh z =

ez + e−z . 2

(2.15)

Las funciones senh z y cosh z son combinaciones lineales de las funciones enteras ez y e−z , por lo tanto, senh z y cosh z son funciones enteras; adem´as, sus derivadas son: d senh z = cosh z dz y d cosh z = senh z. dz Otras propiedades de las funciones seno y coseno hiperb´ olicos • Cuando z es un n´ umero real, senh z y cosh z coinciden con las funciones reales senh x y cosh x. • Las identidades hiperb´ olicas que satisfacen el seno y coseno hiperb´ olicos reales tambi´ en son v´ alidas en el caso complejo; por ejemplo: cosh2 z − senh2 z = 1,

senh(z1 + z2 ) = senh z1 cosh z2 + cosh z1 senh z2 , cosh(z1 + z2 ) = cosh z1 cosh z2 + senh z1 senh z2 ,

entre otras. (Se deja como ejercicio para el lector la comprobaci´ on de cada una de las identidades). • A partir de las definiciones de senh z y cosh z se deduce, para z = x + i y: senh z = senh x cos y + i cosh x sen y

(2.16)

cosh z = cosh x cos y + i senh x sen y.

(2.17)

y

• Las funciones seno y coseno hiperb´ olicos son peri´ odicas con periodo 2πi. • Los ceros de senh z son todos los n´ umeros complejos z = nπi, n = 0, ±1, ±2, . . . • Los ceros de cosh z son todos los n´ umeros complejos z = (n + 12 )πi, n = 0, ±1, ±2, . . .

44

2.6. FUNCIONES ELEMENTALES

Otras funciones hiperb´ olicas Las dem´as funciones hiperb´ olicas de argumento complejo se definen f´acilmente por analog´ıa con las funciones hiperb´ olicas de argumento real, esto es, Tangente hiperb´olica tanh z =

senh z cosh z

coth z =

cosh z senh z

sech z =

1 cosh z

csch z =

1 senh z

Cotangente hiperb´olica

Secante hiperb´olica

Cosecante hiperb´olica

Las funciones tanh z, coth z, sech z y csch z, son anal´ıticas en todos los n´ umeros complejos z donde no se anule el denominador de la expresi´ on que las define, adem´as, la derivada de cada una de ellas es, respectivamente, d tanh z dz d coth z dz d sech z dz d csch z dz

= sech 2 z, = −csch 2 z, = − tanh z sech z, = − coth z csch z.

2.6.4 Funci´ on Logaritmo Se define la funci´ on logaritmo, para todo z 6= 0, como log z = ln |z| + i arg z, En la ecuaci´ on (2.18) se puede utilizar un logaritmo en cualquier base, pero lo m´ as recomendable en la pr´ actica es utilizar la base e.

(2.18)

donde ln · denota el logaritmo natural. Veamos que log z es una funci´ on multivaluada. Se tiene que, para todo z 6= 0, el argumento de z se puede escribir como: arg z = Arg z + 2nπ

(n = 0, ±1, ±2, . . .).

De esta forma, la ecuaci´ on (2.18) se puede escribir equivalentemente como log z = ln |z| + i (Arg z + 2nπ)

(n = 0, ±1, ±2, . . .).

Observe que para cualquier z 6= 0, los valores de log z tienen la misma parte real y sus partes imaginarias difieren en m´ ultiplos enteros de 2π. Por lo tanto, log z es una funci´ on multivaluada. El siguiente ejemplo ilustra este hecho.

45

CAP´ITULO 2. FUNCIONES DE VARIABLE COMPLEJA

Ejemplo 2.13. Calculemos todos los valores de log(1 + i). Se tiene que Arg (1 + i) = √ |1 + i| = 2. As´ı,  π √ + 2nπ (n = 0, ±1, ±2, . . .). log(1 + i) = ln 2 + i 4

π 4

y



Definici´ on 2.14 (Valor principal del logaritmo). El valor principal de log z es el valor que se obtiene de la f´ ormula (2.18) cuando se utiliza el argumento principal de z. Ese valor se denota como Log z y se da por medio de la ecuaci´ on Log z = ln |z| + i Arg z,

para z 6= 0.

(2.19)

Algunas propiedades del valor principal del logaritmo

• La funci´ on w = Log z es monovaluada y su dominio de definici´ on es el conjunto de todos los n´ umeros complejos diferentes de cero; su rango es la franja −π < Im w ≤ π. • El valor principal del logaritmo se reduce al logaritmo natural si z es un n´ umero real positivo, en otras palabras, si z = r > 0, entonces Log z = ln r. • La funci´ on inversa de Log z es ez ; en otras palabras, si w es un n´ umero complejo tal que −π < Im w ≤ π, entonces w = Log z si, y s´ olo si z = ew . • La funci´ on Log z es continua en el dominio (|z| > 0, −π < arg z < π). (Se deja al lector la prueba). • La funci´ on Log z es anal´ıtica en el dominio (|z| > 0, −π < arg z < π), y su derivada est´ a dada por 1 d Log z = . dz z (Se deja al lector la verificaci´ on). Ramas del logaritmo Definici´ on 2.15 (Rama de una funci´ on multivaluada). Una rama de una funci´ on multivaluada f (z), es una funci´ on monovaluada F (z) que es anal´ıtica en cierto dominio D ⊂ C y que coincide con f (z) en D, es decir, F (z) = f (z) para todo z ∈ D. Definici´ on 2.16 (Corte y punto ramal). Un corte ramal es una l´ınea o curva de puntos singulares que se introducen al definir una rama de una funci´ on multivaluada. El punto com´ un a todos los cortes ramales de una funci´ on multivaluada se denomina punto ramal. Observe que Log z es una rama de la funci´ on multivaluada log z, adem´as, se tiene que Log z = log z, para todo z 6= 0 tal que |z| > 0 y −π < arg z < π. La funci´ on Log z se denomina rama principal del logaritmo. El corte ramal de Log z es el eje real negativo con el origen (ver Figura 2.2).

En general, se tiene que: Log (z a ) 6= aLog z, Log (a/b) 6= Log (a) − Log (b) ¿Por qu´e?

46

2.6. FUNCIONES ELEMENTALES

punto ramal

corte ramal b

Figura 2.2. Corte ramal de Log z

Otras ramas del logaritmo complejo se definen como log z = ln |z| + i arg z

(|z| > 0, α < arg z < α + 2π),

donde α es un n´ umero real fijo. El corte ramal de esta rama es el rayo arg z = α (ver Figura 2.3).

punto ramal

t cor

al am r e

b

α

Figura 2.3. Corte ramal de una rama de log z

Ejemplo 2.14. Determinemos el valor de log(1 + i), si log z est´ a definido por log z = ln |z| + i arg z (|z| > 0, π/2 < arg z < 5π/2). √ iπ/4 , pero π/4 6∈ (π/2, 5π/2). Luego, el argumento principal de Se tiene que 1 + i = 2e 1+i no es el valor indicado para calcular log(1+i). Para encontrar arg (1+i) ∈ (π/2, 5π/2), se utiliza el argumento principal de 1 + i. Se tiene que el valor del argumento de 1 + i buscado est´ a dado por: π 9π + 2π = ∈ (π/2, 5π/2); 4 4 por lo tanto, arg (1 + i) = 9π/4 es el valor indicado para calcular log(1 + i). As´ı, log(1 + i) = ln



2+i

9π . 4 

2.6.5 Funci´ on Exponente Complejo Sea c un n´ umero complejo. Se define la funci´ on exponente complejo como z c = ec log z ,

(2.20)

para todo z 6= 0. Esta funci´ on generalmente es multivaluada. Los siguientes ejemplos ilustran este hecho.

47

CAP´ITULO 2. FUNCIONES DE VARIABLE COMPLEJA

Ejemplo 2.15. Calculemos todos los valores de ii . Se tiene que 1

1

ii = ei log i = ei(ln 1+i(2n+ 2 )π) = e−(2n+ 2 )π , para n = 0, ±1, ±2, . . ..



Ejemplo 2.16. Calculemos el valor de i2 . Se tiene que 1

i2 = e2 log i = e2(ln 1+i(2n+ 2 )π) = e(4n+1)πi , para n = 0, ±1, ±2, . . .; pero, e(4n+1)πi = e4nπi eπi = −1, para todo n = 0, ±1, ±2, . . .; por lo tanto, i2 = −1.



El Ejemplo 2.15 muestra que la funci´ on exponente complejo es, generalmente, una funci´ on multivaluada. Por ello, se pueden definir ramas de esta funci´ on. Las ramas de la funci´ on exponente complejo se definen como z c = ec(ln |z|+i arg z) , donde |z| > 0 y α < arg z < α + 2π, con α un n´ umero real fijo. En otras palabras, las ramas de la funci´ on exponente complejo se definen seg´ un la rama del logaritmo que se est´ e utilizando. De esta forma, cuando α = −π, estaremos empleando la rama principal del logaritmo para definir la rama principal de la funci´ on exponente complejo. Para obtener la derivada de la funci´ on exponente complejo, se emplea la regla de la cadena. As´ı, la derivada de cada una de las ramas de la funci´ on exponente complejo est´ a dada por d c z = c z c−1 . dz Funci´ on exponencial de base c La funci´ on exponencial de base c, donde c es un n´ umero complejo distinto de cero, se define como cz = ez log c , (2.21) para todo z ∈ C. De igual manera que la funci´ on exponente complejo, la funci´ on exponencial de base c es generalmente multivaluada, cuyas ramas se definen seg´ un la rama del logaritmo que se est´ e empleando; por ejemplo, si utilizamos el valor principal de log c, entonces la rama que se obtiene es la rama principal de la funci´ on exponencial de base c. Para cada rama de la funci´ on exponencial de base c, su derivada est´ a dada por: d z c = cz log c. dz

48

2.6. FUNCIONES ELEMENTALES

2.6.6 Funciones Trigonom´ etricas Inversas Comencemos con la deducci´ on de una expresi´ on cerrada para la funci´ on inversa del seno. Un procedimiento similar se puede utilizar para la deducci´ on de las expresiones de las dem´as funciones trigonom´ etricas inversas. Para definir la funci´ on inversa de sen z, denotada por sen −1 z, se escribe w = sen −1 z, siempre que z = sen w. Ahora, utilizando la definici´ on de sen w podemos escribir: z=

eiw − e−iw 2i

o, equivalentemente, e2iw − 2iz eiw − 1 = 0.

Para expresar w en t´ erminos de z, primero se despeja eiw al resolver la ecuaci´ on anterior, iw que es una ecuaci´ on cuadr´ atica en e . Se tiene que eiw = iz + (1 − z 2 )1/2 ,

donde (1−z 2 )1/2 es una funci´ on bivaluada de z. Si tomamos logaritmo en ambos miembros y recordamos que w = sen −1 z, se llega a la siguiente f´ ormula cerrada de la funci´ on sen −1 z   (2.22) sen −1 z = −i log iz + (1 − z 2 )1/2 .

Las funciones inversa del coseno e inversa de la tangente se pueden obtener f´acilmente realizando un procedimiento similar al antes descrito. Las expresiones de cos−1 z y tan−1 z son, respectivamente:   (2.23) cos−1 z = −i log z + i(1 − z 2 )1/2 ,   i+z i . (2.24) tan−1 z = log 2 i−z

Busque en la red a trav´es de Google las expresiones de sec−1 z, csc−1 z y cot−1 z

Ejercicio 2.2. Deducir las expresiones cerradas de sec−1 z, csc−1 z y cot−1 z.

En general, las funciones trigonom´etricas inversas son multivaluadas. Por ejemplo, para la funci´ on sen −1 z podemos elegir dos valores igualmente v´alidos para la ra´ız cuadrada que aparece en su definici´ on. Una vez que hemos elegido una valor, existe un n´ umero infinito de valores posibles para el logaritmo de iz + (1 − z 2 )1/2 . En resumen, vemos que, debido a la ra´ız cuadrada y al logaritmo, hay dos conjuntos distintos de valores de sen −1 z, cada uno de los cuales posee un n´ umero infinito de elementos. Este hecho sucede de forma semejante para las restantes funciones trigonom´etricas inversas. Las derivadas de las funciones trigonom´etricas inversas se obtienen directamente de sus expresiones cerradas. Las derivadas de las funciones inversa del seno e inversa del coseno dependen de los valores escogidos de la ra´ıces cuadradas, a saber: d sen −1 z = dz d cos−1 z = dz

1 , (1 − z 2 )1/2 −1 . (1 − z 2 )1/2

En cambio, la derivada de la inversa de la tangente, 1 d tan−1 z = , dz 1 + z2 no depende de la manera en que la funci´ on se haga monovaluada.

49

CAP´ITULO 2. FUNCIONES DE VARIABLE COMPLEJA

Ejercicio 2.3. Determinar las expresiones cerradas de las derivadas de sec−1 z, csc−1 z y cot−1 z. El siguiente ejemplo ilustra la aplicaci´ on de las expresiones de las funciones inversas en la resoluci´ on de ecuaciones que involucran funciones trigonom´ etricas. Ejemplo 2.17. Resolver la ecuaci´ on sen 2 z + 2i sen z − 1 = 0. Soluci´ on. La ecuaci´ on dada se puede escribir equivalentemente como (sen z + i)2 = 0, la cual es cierta si, y s´ olo si la siguiente ecuaci´ on se cumple sen z = −i. Al tomar la inversa del seno en ambos miembros de la ecuaci´ on anterior, podemos escribir:   z = sen −1 (−i) = −i log i(−i) + (1 − (−i)2 )1/2    √  = −i log 1 + 21/2 = −i log 1 ± 2 .

Seg´ un el valor de la ra´ız cuadrada empleado, obtenemos dos conjuntos de valores de z que resuelven la ecuaci´ on dada. As´ı, uno de estos conjuntos de n´ umeros complejos est´ a dado por:   √  √  zk = −i log 1 + 2 = 2kπ − i ln 1 + 2 , para k = 0, ±1, ±2, . . .

El otro conjunto es:  √ √  zn = −i log 1 − 2 = (2n + 1)π − i ln 1 − 2 ,

para n = 0, ±1, ±2, . . . 

2.6.7 Funciones Hiperb´ olicas Inversas Las expresiones cerradas de las funciones hiperb´ olicas inversas se pueden obtener f´acilmente a trav´ es de un procedimiento similar al empleado en la deducci´ on de la funci´ on inversa del seno. Las expresiones cerradas de las funciones inversas del senh z, cosh z y tanh z, son, respectivamente:   senh−1 z = log z + (z 2 + 1)1/2 ,   cosh−1 z = log z + (z 2 − 1)1/2 ,   1 1+z −1 tanh z = log . 2 1−z Ejercicio 2.4. Deducir las expresiones cerradas de sech −1 z, csch −1 z y coth−1 z.

50

2.7. MAPEOS

Las derivadas de las funciones senh−1 z y cosh−1 z, dependen de los valores escogidos de las ra´ıces cuadradas, a saber: d senh−1 z = dz d cosh−1 z = dz

1 , (z 2 + 1)1/2 −1 . 2 (z − 1)1/2

La derivada de la funci´ on tanh−1 z, d 1 tanh−1 z = , dz 1 − z2 no depende de la manera en que la funci´ on se haga monovaluada.

2.7 Mapeos En esta secci´ on se estudia la transformaci´ on de regiones del plano complejo a trav´ es de funciones de variable compleja, tales como polinomios de grado 1 y funciones racionales obtenidas como el cociente de dos polinomios de grado 1. Recordemos que en el c´ alculo elemental se pod´ıa obtener la gr´afica de una funci´ on real y = f (x) y con ello estudiar su comportamiento; en cambio para una funci´ on w = f (z) de una variable compleja, no es tan f´acil elaborar su gr´afica. Para ello se requieren dos n´ umeros x e y para definir un valor z cualquiera y otros dos n´ umeros para los valores de u y v correspondientes. Por lo tanto, se requiere un espacio de cuatro dimensiones para representar w = f (z) en forma gr´afica. Evidentemente una gr´afica de cuatro dimensiones no es un medio conveniente para estudiar el efecto gr´afico de una funci´ on de variable compleja. Es preciso recurrir a otros medios para visualizar w = f (z). y

v S z

w = f (z)

f (S) w

Mapeo o Transformaci´ on b

b

x Plano z

u Plano w

Figura 2.4. Representaci´ on gr´afica de un mapeo Aqu´ı, visualizamos la relaci´ on w = f (z) como el efecto geom´etrico que tiene la funci´ on f (z) sobre un conjunto de puntos complejos. A este proceso lo denominamos mapear o transformar y a la funci´ on f (z) de variable compleja la denominamos transformaci´ on o mapeo (ver Figura 2.4). El objetivo principal de esta secci´ on es determinar anal´ıtica y gr´aficamente el efecto que tiene un mapeo sobre un conjunto de puntos del plano complejo; en particular, estudiamos el efecto de los mapeos lineales, inversi´ on y bilineales, sobre rectas y circunferencias. Para esta parte necesitamos la siguiente notaci´ on:

51

CAP´ITULO 2. FUNCIONES DE VARIABLE COMPLEJA

• Denotamos con w = f (z) la imagen de z bajo f , donde z = x + i y y w = u + i v. • Para todo conjunto S de n´ umeros complejos, denotamos con f (S) el transformado o la imagen de S bajo f ; • La imagen inversa de un punto w del rango de f es el conjunto de todos los puntos z, en el dominio de f , que tienen a w como su imagen. Definici´ on 2.17 (Mapeo inyectivo). Sean z1 , z2 ∈ C. Se dice que el mapeo w = f (z) es inyectivo, si z1 6= z2 implica que f (z1 ) 6= f (z2 ). Inicialmente describimos los mapeos lineales: w = z + c, w = bz y w = bz + c. Seguidamente mostramos el mapeo inversi´ on w = 1/z. Por u ´ ltimo, describimos el mapeo bilineal w = (az + b)/(cz + d).

2.7.1 Mapeo w = z + c El mapeo del plano z en el plano w definido por la ecuaci´ on w = z + c,

(2.25)

donde c es una constante compleja, es una traslaci´on en la direcci´ on del vector c. En la Figura 2.5 se aprecia el movimiento geom´etrico que tiene un punto z0 cuando se transforma con una traslaci´ on. y

z0

w =z+c

b

Traslaci´ on

v

z0 b

b

w0 = z0 + c

x b

u

c

Figura 2.5. Movimiento geom´etrico de la traslaci´ on

El mapeo (2.25) es inyectivo; por tanto, posee mapeo inverso definido como f −1 (w) = w − c.

(2.26)

A continuaci´ on demostramos que la traslaci´ on transforma rectas a rectas y circunferencias a circunferencias. Sean z = x + i y, w = u + i v y c = c1 + i c2 . Para obtener el transformado de una recta o una circunferencia bajo el mapeo w = z + c, utilizamos un procedimiento general que permite determinar, a trav´ es del mapeo inverso, el transformado de un conjunto S descrito por un conjunto de ecuaciones. El procedimiento es el siguiente: primero, se emplea la ecuaci´ on x + i y = f −1 (u + i v),

52

2.7. MAPEOS

para expresar a x e y en funci´ on de u y v; luego se sustituyen convenientemente estos valores en las ecuaciones que describen el conjunto S, con lo cual se obtiene el conjunto de ecuaciones (en funci´ on de u y v) que describen el transformado de S. En el caso de las rectas y circunferencias, se tiene que la ecuaci´ on general de una recta o una circunferencia est´ a dada por α(x2 + y 2 ) + βx + δy + γ = 0,

(2.27)

donde α, β, δ, γ ∈ R. Es decir, todo punto z = x + i y tal que x e y satisfacen la ecuaci´ on (2.27) pertenece a una recta o a una circunferencia. Ahora, utilizando el mapeo inverso (2.26) podemos escribir: x + i y = f −1 (u + i v) = u + i v − c1 + i c2 = (u − c1 ) + i (v − c2 ), de donde se deduce que x = u − c1

e

y = v − c2 .

Sustituyendo convenientemente estos u ´ ltimos valores en la ecuaci´ on (2.27), obtenemos la siguiente ecuaci´ on α((u − c1 )2 + (v − c2 )2 ) + β(u − c1 ) + δ(v − c2 ) + γ = 0, que describe anal´ıticamente el conjunto transformado de una recta o una circunferencia, bajo el mapeo w = z + c. Entonces, si α = 0, la ecuaci´ on (2.27) describe una recta en el plano z, y la ecuaci´ on del transformado de esta recta bajo el mapeo w = z + c est´ a dada por βu + δv + (δ − βc1 − δc2 ) = 0, que describe una recta en el plano w. En cambio, si α 6= 0, la ecuaci´ on (2.27) describe una circunferencia en el plano z, y la ecuaci´ on del transformado de esta circunferencia bajo el mapeo w = z + c est´ a dada por α(u2 + v 2 ) + (β − 2c1 α)u + (δ − 2c2 α)v + (αc21 + αc22 − βc1 − δc2 + γ) = 0, que tambi´ en describe una circunferencia en el plano w. A continuaci´ on damos un ejemplo donde se emplea la ecuaci´ on z = f −1 (w) para obtener el transformado, bajo el mapeo traslaci´ on, de un conjunto de n´ umeros complejos definido por inecuaciones. Ejemplo 2.18. Sea S el conjunto de n´ umeros complejos que se muestra en la Figura 2.6. Determinar el transformado de S bajo el mapeo w = f (z) = z − i. Soluci´ on. Se tiene que todo z = x + i y ∈ S satisface el siguiente conjunto de inecuaciones   2x − y ≥ 1 2x + y ≤ 7 S:  y ≥ 1

Adem´ as, la funci´ on inversa de f (z) = z −i es f −1 (w) = w+i. Ahora, utilizando la ecuaci´ on −1 x + i y = f (u + i v) obtenemos: x = u, e y = v + 1. Sustituyendo convenientemente estas u ´ ltimas expresiones en el conjunto de inecuaciones que describe a S, se tiene el siguiente sistema de inecuaciones   2u − v ≥ 2 f (S) : 2u + v ≤ 6  v ≥ 0

53

CAP´ITULO 2. FUNCIONES DE VARIABLE COMPLEJA

que describe los n´ umeros complejos w = u + i v del transformado de S bajo el mapeo  w = z − i, cuya gr´afica se aprecia en la Figura 2.5. y 3

S

2 1

1

2

3

x

Figura 2.6. Conjunto S del Ejemplo 2.18

v 3 2

f (S)

1

1

2

3

u

Figura 2.7. Conjunto f (S) del Ejemplo 2.18

2.7.2 Mapeo w = bz El mapeo del plano z en el plano w definido por la ecuaci´ on w = bz,

(2.28)

donde b es un n´ umero complejo distinto de cero, es una rotaci´ on en el a´ngulo Arg b y una expansi´on o ´ contracci´ on seg´ un sea el valor de |b|. Si |b| < 1, se dice que el mapeo (2.28) on es una rotaci´ on y contracci´ on; en cambio, si |b| ≥ 1, el mapeo (2.28) es una rotaci´ y expansi´ on. En la Figura 2.8 se aprecia el movimiento geom´etrico que tiene un punto z = reiθ cuando se transforma con el mapeo (2.28). Si tomamos b = leiβ y z = reiθ , entonces el transformado de z bajo el mapeo w = bz est´ a dado, en su forma polar, por on en el ´angulo β y una w = (lr)ei(β+θ) , de lo cual se infiere que (2.28) es una rotaci´ expansi´ on o contracci´ on, seg´ un sea el valor de l.

54

2.7. MAPEOS

y

v w = (lr)ei(θ+β) b

w = bz z = reiθ b

θ

b

β

b = leiβ

θ+β

x

u

Figura 2.8. Movimiento geom´etrico de la rotaci´ on y expansi´ on o contracci´ on

El mapeo (2.28) es inyectivo; por tanto, posee mapeo inverso definido como f −1 (w) =

w . b

(2.29)

Adem´ as, el mapeo (2.28) transforma rectas a rectas y circunferencias a circunferencias. Ejercicio 2.5. Demuestre que el mapeo lineal (2.28) transforma rectas a rectas y circunferencias a circunferencias. Ayuda: utilice la ecuaci´ on general de una recta o una circunferencia (2.27) conjuntamente con la expresi´ on del mapeo inverso (2.29). Ejemplo 2.19. Sea S el conjunto de n´ umeros complejos que se muestra en la Figura 2.9. Determinar el transformado de S bajo el mapeo w = f (z) = (1 − i)z. y 2 1

S b

1

2

x

Figura 2.9. Conjunto S del Ejemplo 2.19 Soluci´ on. Se tiene que todo z = x + i y ∈ S satisface que |z − (1 + i)| ≤ 1; adem´as, −1 f (w) = w/(1 − i). De esta forma, utilizando la ecuaci´ on z = f −1 (w) = w/(1 − i) convenientemente con |z − (1 + i)| ≤ 1, obtenemos: |w − 2| w − (1 + i) = ≤ 1, |z − (1 + i)| = 1−i |1 − i|

es decir, todo punto w que pertenece a f (S) satisface que |w − 2| ≤ se observa el transformado de S bajo el mapeo w = (1 − i)z.



2. En la Figura 2.8 

55

CAP´ITULO 2. FUNCIONES DE VARIABLE COMPLEJA

v 2

f (S) √

1

2

b

1

2

u

3

−1 Figura 2.10. Conjunto f (S) del Ejemplo 2.19

2.7.3 Mapeo w = bz + c El mapeo del plano z en el plano w definido por la ecuaci´ on w = bz + c,

(2.30)

donde b y c son n´ umeros complejos, es una rotaci´ on en el ´angulo Arg b y una expansi´on o ´ contracci´ on seg´ un sea el valor de |b|, seguida de una traslaci´on en la direcci´ on del vector c. En efecto, el mapeo (2.30) se puede expresar a trav´ es de los siguientes mapeos sucesivos: z

Rotaci´ on y Expansi´ on o ´ Contracci´ on

/ Z = bz

Traslaci´ on

/w =Z +c

El mapeo (2.30) es inyectivo ya que es la composici´ on de mapeos inyectivos; por tanto, posee mapeo inverso definido como f −1 (w) =

w−c , b

b 6= 0.

(2.31)

Adem´ as, como (2.30) es la composici´ on de una rotaci´ on y una traslaci´ on, entonces el mapeo w = bz + c transforma rectas a rectas y circunferencias a circunferencias. Ejemplo 2.20. Sea S el conjunto de n´ umeros complejos que se muestra en la Figura 2.11. Determinar el transformado de S bajo el mapeo w = f (z) = iz + 3 − i. y 3

S

2 1 b

1

1

2

3

x

Figura 2.11. Conjunto S del Ejemplo 2.20

56

2.7. MAPEOS

Soluci´ on. Se tiene que el mapeo inverso es f −1 (w) = −iw +1+3i, adem´as, el conjunto S se puede escribir como S = D1 ∪ D2 , donde D1 y D2 son los conjuntos de n´ umeros complejos definidos respectivamente por: D1 = {z = x + i y : 2x − y ≥ 1, 2x + y ≤ 7, y ≥ 1} y D2 = {z = x + i y : (x − 2)2 + (y − 1)2 ≤ 1, y ≤ 1}. De esta forma, como el mapeo es inyectivo, se tiene que el transformado de S est´ a dado por f (S) = f (D1 ) ∪ f (D2 ). Ahora, utilizando la ecuaci´ on x + i y = f −1 (u + i y), tenemos: x = v + 1, y = 3 − u. Sustituyendo los valores de x e y en las inecuaciones que describen a los conjuntos D1 y D2 , obtenemos que los conjuntos transformados de D1 y D2 , bajo el mapeo w = iz + 3 − i, est´ an dados por f (D1 ) = {w = u + i v : u + 2v ≥ 2, −u + 2v ≤ 2, u ≤ 2} y f (D2 ) = {w = u + i v : (u − 2)2 + (v − 1)2 ≤ 1, u ≥ 2}. As´ı, la uni´ on de f (D1 ) y f (D2 ) conforma el transformado de S bajo el mapeo w = iz +3−i, cuya representaci´ on gr´afica se aprecia en la Figura 2.12. v 3 f (S)

2 1 b

1

1

2

3

u

Figura 2.12. Conjunto f (S) del Ejemplo 2.20 

2.7.4 Mapeo Inversi´ on El mapeo del plano z en el plano w definido por la ecuaci´ on w=

1 , z

(2.32)

para todo z 6= 0, se denomina mapeo inversi´on, y establece una correspondencia uno a uno entre los puntos de los planos z y w distintos de cero. Por lo tanto, el mapeo inverso de f (z) = 1/z es f −1 (w) = 1/w, es decir, el mismo mapeo inversi´ on. en puede escribirse como Por otra parte, el mapeo (2.32) tambi´ w=

z , |z|2

para todo z 6= 0.

57

CAP´ITULO 2. FUNCIONES DE VARIABLE COMPLEJA

Por ello, el mapeo inversi´ on se puede expresar a trav´ es de los siguientes mapeos sucesivos: /Z= 1 z 2

z

/w=Z

|z|

El primero de estos mapeos se denomina inversi´ on con respecto a la circunferencia unitaria |z| = 1. Es decir, la imagen de un punto z 6= 0 es el punto Z = (1/|z|2 )z con las siguientes propiedades: 1 y arg Z = arg z. |Z| = |z|

En otras palabras, los puntos exteriores a la circunferencia |z| = 1 se transforman en los puntos diferentes de cero interiores a la misma y rec´ıprocamente. Cualquier punto sobre la circunferencia unitaria se transforma en s´ı mismo, bajo el mapeo Z = (1/|z|2 )z. El segundo mapeo w = Z es, simplemente, una reflexi´ on con respecto al eje real. Transformaci´ on de Rectas y Circunferencias Pasemos a ver el efecto que tiene el mapeo inversi´ on sobre las rectas y las circunferencias. Recordemos que la ecuaci´ on general de una recta o una circunferencia est´ a dada por α(x2 + y 2 ) + βx + δy + γ = 0, (2.33) donde α, β, δ, γ ∈ R. Tambi´ en recordemos que si α = 0, la ecuaci´ on anterior describe una recta en el plano z; en cambio, si α 6= 0, describe una circunferencia. Si z = x + i y es un punto que satisface la ecuaci´ on general de una recta o una circunferencia para ciertos valores de α, β, δ, γ, entonces su transformado w = u + i v = 1/z satisface la siguiente ecuaci´ on (se deja al lector verificar esta relaci´ on): γ(u2 + v 2 ) + βu − δv + α = 0.

(2.34)

A continuaci´ on consideramos ciertos valores de α, β, δ, γ, que describen algunas rectas y circunferencias muy particulares, que nos servir´ an como ejemplo para mostrar la utilidad de las ecuaciones (2.33) y (2.34), para transformar rectas y circunferencias bajo el mapeo inversi´ on. • Circunferencia que no pasa por el origen. La ecuaci´ on (2.33) con α 6= 0 y δ 6= 0, describe una circunferencia que no pasa por el origen en el plano z. Por la ecuaci´ on (2.34), esta circunferencia se transforma, bajo la inversi´ on, en una circunferencia que no pasa por el origen en el plano w. y

w=

x

1 z

v

u

Ejemplo 2.21. La circunferencia |z − (1 + i)| = 1 que no pasa por el origen en el plano z, se transforma, bajo la inversi´ on, en la circunferencia |w − (1 − i)| = 1 que no pasa por  el origen en el plano w.

58

2.7. MAPEOS

• Circunferencia que pasa por el origen. La ecuaci´ on (2.33) con α 6= 0 y δ = 0, describe una circunferencia que pasa por el origen en el plano z. Por la ecuaci´ on (2.34), esta circunferencia se transforma, bajo la inversi´ on, en una recta que no pasa por el origen en el plano w. y

w=

1 z

v

x

u

Ejemplo 2.22. La circunferencia (x − 1)2 + (y + 1)2 = 2 que pasa por el origen en el plano z, se transforma, bajo la inversi´ on, en la recta 2u + 2v = 1 que no pasa por el origen en el plano w.  • Recta que no pasa por el origen. La ecuaci´ on (2.33) con α = 0 y δ 6= 0, describe una recta que no pasa por el origen en el plano z. Por la ecuaci´ on (2.34), esta recta se transforma, bajo la inversi´ on, en una circunferencia que pasa por el origen en el plano w. y

w=

1 z

v

x

u

Ejemplo 2.23. La recta x−y = −1 que no pasa por el origen en el plano z, se transforma, bajo la inversi´ on, en la circunferencia (u + 1)2 + (v + 1)2 = 2 que pasa por el origen en el plano w.  • Recta que pasa por el origen. La ecuaci´ on (2.33) con α = 0 y δ = 0, describe una recta que pasa por el origen en el plano z. Por la ecuaci´ on (2.34), esta recta se transforma, bajo la inversi´ on, en una recta que pasa por el origen en el plano w. y

w=

x

1 z

v

u

Ejemplo 2.24. La recta x − y = 0 que pasa por el origen en el plano z, se transforma, bajo la inversi´ on, en la recta u + v = 0 que pasa por el origen en el plano w. 

59

CAP´ITULO 2. FUNCIONES DE VARIABLE COMPLEJA

Ejercicio 2.6. Compruebe cada uno de los conjuntos transformados de los Ejemplos 2.212.24. Ejemplo 2.25. Sea S el conjunto de n´ umeros complejos dado en el Ejemplo 2.18 y que se muestra en la Figura 2.6. Comprobemos que el transformado de S bajo el mapeo inversi´ on es el conjunto de n´ umeros complejos que se muestra en la siguiente figura. v 1 b

b

1 b

−1

2

u

f (S)

Figura 2.13. Conjunto f (S) del Ejemplo 2.25 Se tiene que S = {z = x + i y : 2x − y ≥ 1, 2x + y ≤ 7, y ≥ 1},

adem´as, la funci´ on inversa de f (z) = 1/z es f −1 (w) = 1/w. Ahora, utilizando la ecuaci´ on −1 x + i y = f (u + i v) obtenemos: x = u/(u2 + v 2 ), e y = −v/(u2 + v 2 ). Sustituyendo convenientemente estas u ´ ltimas expresiones en el conjunto de inecuaciones que describe a S, se tiene que los puntos w = u + i v del transformado de S bajo el mapeo inversi´ on satisfacen las siguientes inecuaciones:         1 2 1 2 1 1 2 5 1 2 5 2 2 u− , u + v+ ≤ , + v+ ≥ ≤ . (u − 1) + v − 2 4 7 14 196 2 4 Por lo tanto, el transformado de S bajo el mapeo inversi´ on es el conjunto de n´ umeros complejos que se muestra en la Figura 2.13. 

2.7.5 Mapeo Bilineal El mapeo del plano z en el plano w definido por la ecuaci´ on w=

az + b , cz + d

(2.35)

donde a, b, c y d son n´ umeros complejos, se denomina mapeo bilineal o transformaci´ on de M¨ obius. Este mapeo es inyectivo para todo z ∈ C tal que cz + d 6= 0; por tanto, en ese conjunto de puntos posee mapeo inverso definido como f −1 (w) =

−dw + b . cw − a

Cuando c = 0, el mapeo (2.35) adquiere la forma w=

a b z+ , d d

(2.36)

60

2.7. MAPEOS

lo cual indica que (2.35) es un mapeo lineal, estudiado en las secciones anteriores. Ahora, cuando c 6= 0, el mapeo (2.35) se puede escribir equivalentemente como   bc − ad 1 a , w= + c c cz + d donde el n´ umero ad − bc se denomina determinante del mapeo. Esto nos permite afirmar que el mapeo (2.35) se puede expresar a trav´ es de los siguientes mapeos sucesivos: z

Rotaci´ on con

/ Z = cz + d

Inversi´ on

/W = 1 Z

Rotaci´ on con

Expansi´ on o ´

Expansi´ on o ´

Contracci´ on, y

Contracci´ on, y

Traslaci´ on

Traslaci´ on

/w= a+ c



bc − ad c



W

on de mapeos lineales, por lo cual ´el En otras palabras, el mapeo (2.35) es la composici´ transforma circunferencias o rectas en el plano z a circunferencias o rectas en el plano w. Por otra parte, cuando el determinante del mapeo es cero, ad − bc = 0, el mapeo (2.35) adquiere la forma a w= , c es decir, el mapeo (2.35) transforma todo el plano complejo en el punto w = a/c. Ejemplo 2.26. Sea S el conjunto de n´ umeros complejos dado en el Ejemplo 2.18 y que se muestra en la Figura 2.6. Determinar el transformado de S bajo el mapeo bilineal w = f (z) =

iz + 1 + i . iz + i

Soluci´ on. Se tiene que S = {z = x + i y : 2x − y ≥ 1, 2x + y ≤ 7, y ≥ 1}. Adem´ as, la funci´ on inversa de f (z) = (iz + 1 + i)/(iz + i) es f −1 (w) =

i − 1. 1−w

Ahora, utilizando la ecuaci´ on x + i y = f −1 (u + i v) obtenemos: x=

−v − ((u − 1)2 + v 2 ) , (u − 1)2 + v 2

y=

1−u . (u − 1)2 + v 2

Sustituyendo convenientemente estas u ´ ltimas expresiones en el conjunto de inecuaciones que describe a S, se tiene que los puntos w = u + i v del transformado de S, bajo el mapeo bilineal dado, satisfacen las siguientes inecuaciones 

u−

7 6

2

  1 2 5 , + v+ ≤ 3 36



u−

17 18

2

  2 2 5 + v+ ≥ 2, 18 18

La gr´afica del transformado de S se aprecia en la Figura 2.14.



u−

1 2

2

1 + v2 ≤ . 4 

61

CAP´ITULO 2. FUNCIONES DE VARIABLE COMPLEJA

v 1 b b

1 −1

b

2

u

f (S)

Figura 2.14. Conjunto f (S) del Ejemplo 2.26

2.8 Funciones, L´ımite, Derivada y Mapeos con M ATLAB En esta secci´ on se utilizan las herramientas de matem´atica simb´ olica (Symbolic Math Toolbox, Matlab R2015a) de M ATLAB . Para usar esta herramienta es necesario definir las variables como objetos simb´ olicos. Por ejemplo, para declarar x e y como objetos simb´ olicos se usa el comando syms: ✄ >> syms x y





Los objetos simb´ olicos se pueden manipular siguiendo las reglas de las operaciones matem´ aticas, por ejemplo: ✄ >> x + y− 5∗x + yˆ2 ans = yˆ2 + y − 4∗x





Una funci´ on de variable compleja f (z) tambi´ en se puede crear como una funci´ on simb´ olica. Primero se crea el objeto simb´ olico z, luego se define la funci´ on simb´ olica f(z) seg´ un su forma algebraica. En el siguiente ejemplo se crea la funci´ on f (z) = z 2 + iz − 1. ✄

>> syms z >> f ( z ) = z ˆ2 + i ∗ z − 1 f (z) = z ˆ2 + z ∗ i − 1





Despu´ es de crear una funci´ on simb´ olica, se puede diferenciar, integrar o simplificar, evaluarla en valores y realizar otras operaciones matem´aticas. Considerando la funci´ on f (z) = z 2 + iz − 1 creada previamente, seguidamente se eval´ ua en z = i. ✄

>> f ( i ) ans = −3





Tambi´ en se pueden hacer operaciones entre funciones simb´ olicas: sumar, multiplicar, dividir funciones, entre otras. En el siguiente ejemplo se crean dos funciones, f1 (z) = z 2 −i y f2 (z) = (z − 1)(z − 2), con las que se crean otras tres funciones: g1 (z) = f1 (z) + f2 (z), g2 (z) = f1 (z)f2 (z) y g3 (z) = f1 (z)/f2 (z). ✄ >> syms z >> f 1 ( z ) = zˆ2−i , f 2 ( z ) = ( z −1)∗(z−2) f1 ( z ) = z ˆ2 − i f2 ( z ) = ( z − 1)∗( z − 2)

62

2.8. FUNCIONES, L´IMITE, DERIVADA Y MAPEOS CON MATLAB

>> g1 ( z)=f 1 ( z)+f 2 ( z ) , g2 ( z)=f 1 ( z )∗ f 2 ( z ) , g3 ( z)=f 1 ( z ) / f 2 ( z ) g1 ( z ) = ( z − 1)∗( z − 2) + z ˆ2 − i g2 ( z ) = ( z ˆ2 − i ) ∗ ( z − 1)∗( z − 2) g3 ( z ) = ( z ˆ2 − i ) / ( ( z − 1)∗( z − 2 ) )



Para m´ as detalles del comando limit, teclee help limit en el escritorio de M ATLAB



Una operaci´ on importante en matem´aticas es el c´ alculo de l´ımites, la cual tambi´ en se puede realizar con la herramienta simb´ olica de M ATLAB . En particular, el comando limit(f,z,a) calcula el l´ımite l´ımz→a f (z).Observe el siguiente ejemplo donde se calcula el l´ımite l´ım



z→0

z + z + 2 Re z (Im z + 1)i = 1 + i. z+z

>> syms z >> f ( z ) = ( z+c o n j ( z)+2∗ r e a l ( z ) ∗ ( imag ( z )+1)∗ i ) / ( z+c o n j ( z ) ) f (z) = ( z + c o n j ( z ) + r e a l ( z ) ∗ ( imag ( z ) + 1)∗2∗ i ) / ( z + c o n j ( z ) ) >> l i m i t ( f , z , 0 ) ans ( z ) = 1 + i





La derivada de una funci´ on f (z) se calcula usando el comando diff. Espec´ıficamente, diff(f,z,n) calcula la derivada en´ esima de f (z). Por ejemplo, la primera, segunda y tercera derivada de 2i − z 4 , f (z) = 3 z + iz 2 − z + 1 se calculan como sigue: ✄ >> syms z >> f ( z )=(2∗ i −z ˆ 4 ) / ( zˆ3+ i ∗ zˆ2−z+1) f (z) = −(z ˆ4 − 2 i ) / ( z ˆ3 + z ˆ2∗1 i − z + 1) >> d i f f ( f , z ) ans ( z ) = − (4∗ z ˆ 3 ) / ( z ˆ3 + z ˆ2∗1 i − z + 1) + ( ( z ˆ4 − 2 i )∗(3∗ z ˆ2 + z ∗2 i − 1 ) ) / ( z ˆ3 + z ˆ2∗1 i − z + 1)ˆ2 >> d i f f ( f , z , 2 ) ans ( z ) = − (12∗ z ˆ 2 ) / ( z ˆ3 + z ˆ2∗1 i − z + 1) − (2∗( z ˆ4 − 2 i )∗(3∗ z ˆ2 + z ∗2 i − 1)ˆ2)/ ( z ˆ3 + z ˆ2∗1 i − z + 1)ˆ3 + (8∗ z ˆ3∗(3∗ z ˆ2 + z ∗2 i − 1 ) ) / ( z ˆ3 + z ˆ2∗1 i − z + 1)ˆ2 + ((6∗ z + 2 i ) ∗ ( z ˆ4 − 2 i ) ) / ( z ˆ3 + z ˆ2∗1 i − z + 1)ˆ2 >> d i f f ( f , z , 3 ) ans ( z ) = (6∗( z ˆ4 − 2 i ) ) / ( z ˆ3 + z ˆ2∗1 i − z + 1)ˆ2 − (24∗ z ) / ( z ˆ3 + z ˆ2∗1 i − z + 1) + (6∗( z ˆ4 − 2 i )∗(3∗ z ˆ2 + z ∗2 i − 1 ) ˆ 3 ) / ( z ˆ3 + z ˆ2∗1 i − z + 1)ˆ4 + (12∗ z ˆ3∗(6∗ z + 2 i ) ) / ( z ˆ3 + z ˆ2∗1 i − z + 1)ˆ2 + (36∗ z ˆ2∗(3∗ z ˆ2 + z ∗2 i − 1 ) ) / ( z ˆ3 + z ˆ2∗1 i − z + 1)ˆ2 − (24∗ z ˆ3∗(3∗ z ˆ2 + z ∗2 i − 1)ˆ2)/ ( z ˆ3 + z ˆ2∗1 i − z + 1)ˆ3 − (6∗(6∗ z + 2 i ) ∗ ( z ˆ4 − 2 i )∗(3∗ z ˆ2 + z ∗2 i − 1 ) ) / ( z ˆ3 + z ˆ2∗1 i − z + 1)ˆ3





En este caso, las expresiones obtenidas de la segunda y tercera derivada de f (z) no est´ an simplificadas. Para buscar la forma m´ as simple de una expresi´ on simb´ olica se utiliza el comando simplify. Utilicemos tal comando para hallar las formas simplificadas de la primera, segunda y tercera derivada de f (z) en el ejemplo anterior.

63

CAP´ITULO 2. FUNCIONES DE VARIABLE COMPLEJA

✄ >> d2f=s i m p l i f y ( d i f f ( f , z , 2 ) ) ; d2f d2f ( z ) = (6∗( z ˆ5∗(1 + 1 i ) − z ˆ4∗(1 − 3 i ) − (8∗ z ˆ3)/3 − z ˆ2∗(2 + 4 i ) + z ∗(2 − 2 i ) + (2/3 + 2 i / 3 ) ) ) / ( z ˆ3 + z ˆ2∗1 i − z + 1)ˆ3 >> d3f=s i m p l i f y ( d i f f ( f , z , 3 ) ) ; d3f d3f ( z ) = −(( z ˆ7∗(2 − 2 i ) + z ˆ6∗(8 + 3 i ) + z ˆ5∗10 i − z ˆ4∗(19 − 10 i ) − z ˆ3∗(18 + 18 i ) + z ˆ2∗(10 − 13 i ) + z ∗(8 + 2 i ) + 2 i )∗12 i )/(− z ˆ3∗1 i + z ˆ2 + z ∗1 i − 1 i )ˆ4





M ATLAB est´ a provisto de un gran n´ umero de funciones elementales. Algunas de estas funciones se pueden apreciar en la Tabla ?? del Cap´ıtulo A. Los argumentos de las funciones elementales pueden ser (siempre que tenga sentido) reales o complejos y el resultado se devuelve en el mismo tipo del argumento. En los siguientes ejemplos mostramos como se utilizan algunas funciones elementales en M ATLAB . Observe el siguiente ejemplo: ✄ >> c o s (1− i ) ans = 0.8337 + 0.9889 i >> l o g (1+ i ) ans = 0.3466 + 0.7854 i >> exp ( p i ∗ i ) ans = −1.0000 + 0.0000 i





Con el comando cos(1-i) el valor de cos(1 − i), con log(1+i) se obtiene el √ se obtiene π valor de Log (1 + i) = ln 2 + i 4 , y con exp(pi*i) se obtiene eπi = −1. Con las funciones elementales se pueden crear cualquier tipo de funciones simb´ olicas. Observe el siguiente ejemplo donde se crea la funci´ on f (z) =

sen (1 − Log z) ez 2 +1 − 2

y se obtienen los valores |f (i)| y |f ′ (i)|. ✄

>> syms z >> f ( z)= s i n (1− l o g ( z ) ) / ( exp ( zˆ2+1)−2) f (z) = −s i n ( l o g ( z ) − 1 ) / ( exp ( z ˆ2 + 1) − 2) >> abs ( f ( i ) ) ans = ( cosh ( p i /2)ˆ2∗ s i n (1)ˆ2 + s i n h ( p i /2)ˆ2∗ c o s ( 1 ) ˆ 2 ) ˆ ( 1 / 2 ) >> d f ( z ) = d i f f ( f , z ) ; >> abs ( d f ( i ) ) ans = ( ( cosh ( p i /2)∗ c o s ( 1 ) + 2∗ cosh ( p i /2)∗ s i n ( 1 ) ) ˆ 2 + (2∗ s i n h ( p i /2)∗ c o s ( 1 ) − s i n h ( p i /2)∗ s i n ( 1 ) ) ˆ 2 ) ˆ ( 1 / 2 )





M ATLAB tambi´ en puede emplearse para hallar los conjuntos transformados de circunferencias y pol´ıgonos, bajo el mapeo lineal w = bz + c. El Programa 2.1 muestra la funci´ on MapeoPoligono que halla el transformado de un pol´ıgono con v´ertices z1 , z2 , . . . , zn ∈ C, on MapeoCircunferencia bajo el mapeo lineal w = bz+c. El Programa 2.2 muestra la funci´ que halla el transformado de una circunferencia de centro z0 ∈ C y radio r > 0, bajo el mapeo lineal w = bz + c.

64

2.8. FUNCIONES, L´IMITE, DERIVADA Y MAPEOS CON MATLAB

Programa 2.1. Funci´ on MapeoPoligono.m f u n c t i o n w = MapeoPoligono ( b , c , z ) %MapeoPoligono H a l l a h a l l a e l t r a n s f o r m a d o de un p o l´ıg o n o % cu yos v ´ e r t i c e s son l o s elementos d e l a r r e g l o z , % b a j o e l mapeo w = bz + c . % w e s un a r r e g o que c o n t i e n e l o s v ´e r t i c e s d e l % p o l´ıg o n o t r a n s f o r m a d o w = b∗ z + c ; subplot (1 ,2 ,1) p l o t ( [ z , z ( 1 ) ] , ’ r ’ , ’ LineWidth ’ , 2 ) xlabel ( ’ x ’ ) ylabel ( ’ y ’ ) g r i d on xmin = min ( [ min ( r e a l ( z ) ) min ( r e a l (w) ) ] ) − . 5 ; xmax = max( [ max( r e a l ( z ) ) max( r e a l (w) ) ] ) + . 5 ; ymin = min ( [ min ( imag ( z ) ) min ( imag (w) ) ] ) − . 5 ; ymax = max( [ max( imag ( z ) ) max( imag (w) ) ] ) + . 5 ; a x i s ( [ xmin xmax ymin ymax ] ) a x i s equal t i t l e ( ’ P o l´ıg o n o ’ ) % subplot (1 ,2 ,2) p l o t ( [w,w( 1 ) ] , ’ b ’ , ’ LineWidth ’ , 2 ) xlabel ( ’u ’ ) ylabel ( ’ v ’ ) g r i d on a x i s ( [ xmin xmax ymin ymax ] ) a x i s equal t e x t o = s p r i n t f ( ’w=(%s ) z+%s ’ , num2str ( b ) , num2str ( c ) ) ; t e x t o = [ ’ P o l´ıg o n o t r a n s f o r m a d o b a j o e l mapeo ’ , t e x t o ] ; t i t l e ( texto ) end

Programa 2.2. Funci´ on MapeoCircunferencia.m f u n c t i o n [w0, l ] = M a p e o C i r c u n f e r e n c i a ( b , c , z0 , r ) %M a p e o C i r c u n f e r e n c i a H a l l a e l t r a n s f o r m a d o de una c i r c u n f e − % r e n c i a con c e n t r o z0 y r a d i o r , b a j o e l % mapeo w = bz + c . % w0 e s e l c e n t r o y l e s e l r a d i o de l a c i r c u n f e r e n c i a % transformada w0 = b∗ z0 + c ; l = abs ( b )∗ r ; t = l i n s p a c e (0 ,2∗ p i ) ; subplot (1 ,2 ,1) x t = r ∗ c o s ( t )+ r e a l ( z0 ) ; y t = r ∗ s i n ( t )+imag ( z0 ) ; p l o t ( xt , yt , ’ r ’ , ’ LineWidth ’ , 2 ) xlabel ( ’ x ’ ) ylabel ( ’ y ’ ) g r i d on rmax = max( [ r l ] ) ; xmin = min ( [ r e a l ( z0 ) r e a l (w0)]) −rmax −.2; xmax = max( [ r e a l ( z0 ) r e a l (w0)])+rmax +.2; ymin = min ( [ imag ( z0 ) imag (w0)]) −rmax −.2; ymax = max( [ imag ( z0 ) imag (w0)])+rmax +.2;

65

CAP´ITULO 2. FUNCIONES DE VARIABLE COMPLEJA

a x i s ( [ xmin xmax ymin ymax ] ) a x i s equal t i t l e ( ’ Cricunferencia ’ ) % subplot (1 ,2 ,2) u t = l ∗ c o s ( t )+ r e a l (w0) ; v t = l ∗ s i n ( t )+imag (w0) ; p l o t ( ut , vt , ’ b ’ , ’ LineWidth ’ , 2 ) xlabel ( ’u ’ ) ylabel ( ’ v ’ ) g r i d on a x i s ( [ xmin xmax ymin ymax ] ) a x i s equal t e x t o = s p r i n t f ( ’w=(%s ) z+%s ’ , num2str ( b ) , num2str ( c ) ) ; t e x t o = [ ’ C i r c u n f e r e n c i a t r a n s f o r m a d a b a j o e l mapeo ’ , t e x t o ] ; t i t l e ( texto ) end

A continuaci´ on usamos la funci´ on MapeoPoligono para hallar el transformado del pol´ıgono con v´ ertices 1 + 2i, 2i, 2, 2 − 2i, 0 y −2 − i, bajo el mapeo w = (−1 + i)z + i. ✄

>> z=[−1+2 i 2 i 2 2−2 i 0 −2−i ] ; b=−1+i ; c=i ; >> w = MapeoPoligono ( b , c , z ) w= Columns 1 throu gh 4 −1.0000 − 2.0000 i −2.0000 − 1.0000 i −2.0000 + 3.0000 i Columns 5 throu gh 6 0 + 1.0000 i 3.0000

0 + 5.0000 i



✁ Polígono

Polígono transformado bajo el mapeo w=(−1+1i)z+0+1i

4

4

3

3

2

2 v

5

y

5

1

1

0

0

−1

−1

−2

−2 −2

0 x

2

−2

−1

0

1

2

3

u

Seguidamente usamos la funci´ √on MapeoCircunferencia para hallar el transformado de la circunferencia |z − 1 + i| = 2, bajo el mapeo w = (−2 − i)z + 4. ✄

>> z0=1−i ; r=s q r t ( 2 ) ; b=−2−i ; c =4; >> [w0, l ] = M a p e o C i r c u n f e r e n c i a ( b , c , z0 , r ) w0 = 1.0000 + 1.0000 i l = 3.1623





66

2.9. PROBLEMAS RESUELTOS

Circunferencia transformada bajo el mapeo w=(−2−1i)z+4

4

4

3

3

2

2

1

1

0

0

v

y

Cricunferencia

−1

−1

−2

−2

−3

−3

−4

−4 −2

0

2

4

−2

x

0

2

4

u

2.9 Problemas Resueltos Problema 2.1. Pruebe que los siguientes l´ımites no existen en punto alguno z0 ∈ C. z − z0 . a) l´ım z→z0 z − z0 b) l´ım

z→z0

Re z − Re z0 . z − z0

Soluci´ on. Demostremos que los l´ımites dados alcanzan valores distintos cuando z tiende a z0 por los siguientes caminos: i) una recta horizontal que pasa por z0 , esto es, tomando z = x + iy0 ; y ii) una recta vertical que pasa por z0 , esto es, tomando z = x0 + iy. 0 a dado por: a) El l´ımite de z−z z−z0 cuando z tiende a z0 por el camino i) est´ l´ım

z→z0

Ahora, el l´ımite de l´ım

z→z0

x − x0 z − z0 x + iy0 − x0 + iy0 = l´ım = l´ım = 1. z→z z→z z − z0 0 x + iy0 − x0 − iy0 0 x − x0

z−z0 z−z0

cuando z tiende a z0 por el camino ii) est´ a dado por:

i(−y + y0 ) z − z0 x0 + iy − x0 + iy0 = −1. = l´ım = l´ım z − z0 z→z0 x0 + iy − x0 − iy0 z→z0 i(y − y0 ) z−z0 no existe. z→z0 z−z0 Re z−Re z0 cuando z tiende z−z0

Por lo tanto, el l´ımite l´ım b) El l´ımite de l´ım

z→z0

El l´ımite de l´ım

z→z0

a z0 por el camino i) est´ a dado por:

Re z − Re z0 Re (x + iy0 ) − Re (x0 + iy0 ) x − x0 = l´ım = l´ım = 1. z→z0 z→z0 x − x0 z − z0 x + iy0 − x0 − iy0

Re z−Re z0 z−z0

cuando z tiende a z0 por el camino ii) est´ a dado por:

Re z − Re z0 Re (x0 + iy) − Re (x0 + iy0 ) x0 − x0 = l´ım = l´ım = 0. z→z0 z→z0 i(y − y0 ) z − z0 x0 + iy − x0 − iy0

Por lo tanto, el l´ımite l´ım

z→z0

Re z−Re z0 z−z0

no existe.



67

CAP´ITULO 2. FUNCIONES DE VARIABLE COMPLEJA

Problema 2.2. Sea f (z) una funci´ on de variable compleja definida como  2  z , f (z) = 2(Re z)2 i,   (Im z)z,

Im z < Re z, Im z = Re z, Im z > Re z.

Determine el conjunto de todos los n´ umeros complejos z donde la funci´ on f (z) es continua. Soluci´ on. Es claro que f (z) est´ a bien definida en todo z ∈ C. Como f (z) = z 2 para Im z < Re z y z 2 es un polinomio, entonces f (z) es continua en z ∈ C tal que Im z < Re z. Ahora, para z = x + iy tal que Im z > Re z, se tiene que f (z) = (Im z)z = yx + iy 2 ; de lo cual se deduce que las funciones componentes de f (z) son u(x, y) = yx y v(x, y) = y 2 , para y > x, que son continuas en todo (x, y) ∈ R2 , en particular, en todo (x, y) tal que y > x. De esta forma, f (z) es continua en todo z tal que Im z > Re z o Im z < Re z. Veamos ahora que f (z) no es continua en z tal que Im z = Re z 6= 0. Sea z0 tal que Im z0 = Re z0 6= 0. Calculemos el l´ımite l´ım f (z) en la direcci´ on de la recta Im z = Re z0 − Re z + Im y0 , con z→z0

Im z > Re z. As´ı,

l´ım f (z) = l´ım (Im z)z = (Im z0 )z0 = (Re z0 )2 + i(Re z0 )2 6= 2(Re z0 )2 i = f (z0 ),

z→z0

z→z0

Por lo cual f (z) no es continua en z tal que Im z = Re z 6= 0. Es claro que f (z) es continua z0 = 0. De todo lo anterior, se tiene que el conjunto de todos los n´ umeros complejos z donde f (z) es continua est´ a dado por: C − {z ∈ C : Im z = Re z 6= 0} 

Problema 2.3. Si u y v se expresan en t´erminos de las coordenadas polares (r, θ), muestre que las ecuaciones de Cauchy-Riemann pueden escribirse de la forma, para r > 0,    

∂u ∂r    1 ∂u r ∂θ

1 ∂v r ∂θ ∂v = − ∂r

=

denominada forma polar de las ecuaciones de Cauchy-Riemann. Soluci´ on. Asumamos que u(x, y) y v(x, y) satisfacen las ecuaciones de Cauchy-Riemann en su forma cartesiana, esto es  ∂u ∂v    ∂x = ∂y  ∂u ∂v   = − ∂y ∂x

68

2.9. PROBLEMAS RESUELTOS

Ahora, haciendo el cambio de variable x = r cos θ e y = r sen θ y aplicando la regla de la cadena, obtenemos: ∂u ∂r

=

∂u ∂x ∂u ∂y ∂u ∂u + = cos θ + sen θ, ∂x ∂r ∂y ∂r ∂x ∂y

∂u ∂θ

=

∂u ∂x ∂u ∂y ∂u ∂u + = −r sen θ + r cos θ, ∂x ∂θ ∂y ∂θ ∂x ∂y

∂v ∂r

=

∂v ∂x ∂v ∂y ∂v ∂v + = cos θ + sen θ, ∂x ∂r ∂y ∂r ∂x ∂y

∂v ∂θ

=

∂v ∂x ∂v ∂y ∂v ∂v + = −r sen θ + r cos θ. ∂x ∂θ ∂y ∂θ ∂x ∂y

De esta forma, usando las ecuaciones anteriores conjuntamente con las ecuaciones de Cauchy-Riemann en su forma cartesiana, podemos escribir:   ∂u ∂u 1 ∂v ∂v ∂u 1 ∂v − = cos θ + sen θ − sen θ + r cos θ −r ∂r r ∂θ ∂x ∂y r ∂x ∂y     ∂u ∂v 1 ∂v ∂u ∂v ∂u = − + = , cos θ + sen θ = 0 ⇒ ∂x ∂y ∂y ∂x ∂r r ∂θ 1 ∂u ∂v + r ∂θ ∂r

  1 ∂u ∂v ∂v ∂u = sen θ + r cos θ + cos θ + sen θ −r r ∂x ∂y ∂x ∂y     ∂u ∂v ∂u ∂v = − + + cos θ + sen θ = 0 ⇒ ∂x ∂y ∂y ∂x

∂v 1 ∂u =− . r ∂θ ∂r 

Problema 2.4. Sea f (z) una funci´ on de variable compleja definida como   1 f (z) = log , z−1 donde log z = ln |z| + iarg z, |z| > 0 y −π/4 < arg z < 7π/4. Determine el conjunto de todos los n´ umeros complejos z donde f (z) es anal´ıtica. Soluci´ on. Por las condiciones del enunciado del problema, se tiene que la funci´ on log z es anal´ıtica en todo el plano complejo excepto en el conjunto {z ∈ C : Re z ≥ 0, Im z = −Re z}. Ahora, como     1 1 Re z − 1 −Im z Re e Im , = = 2 2 z−1 (Re z − 1) + (Im z) z−1 (Re z − 1)2 + (Im z)2   1 es anal´ıtica en todo el plano complejo excepto en el entonces la funci´ on f (z) = log z−1 conjunto   Re z − 1 −Im z Re z − 1 z∈C: ≥ 0, =− (Re z − 1)2 + (Im z)2 (Re z − 1)2 + (Im z)2 (Re z − 1)2 + (Im z)2

69

CAP´ITULO 2. FUNCIONES DE VARIABLE COMPLEJA

o, equivalentemente, {z ∈ C : Re z ≥ 1, Im z = Re z − 1}. Por lo tanto, el conjunto de todos los n´ umeros complejos z donde f (z) es anal´ıtica est´ a dado por: C − {z ∈ C : Re z ≥ 1, Im z = Re z − 1}  Problema 2.5. Sea u(x, y) una funci´ on definida de R2 en R dada por ( Re (z 2 + 1), si x ≤ 0, u(x, y) = x−3 y 3 , si x > 0. Determine el conjunto de todos los pares ordenados (x, y) donde u(x, y) es arm´ onica, adem´as, construya una funci´ on f (z) = u(x, y) + iv(x, y) y el mayor dominio D ⊂ C tales que f (z) es anal´ıtica en D y f (−1 − i) = 1 − i. Soluci´ on. Se tiene que la funci´ on z 2 + 1 es entera; por tanto, sus funciones componentes 2 2 Re (z + 1) e Im (z + 1) son funciones arm´ onicas en todo R2 . Como u(x, y) = Re (z 2 + 1) = x2 − y 2 + 1, para cada z = x + iy tal que x < 0, entonces u(x, y) es arm´ onica en el conjunto {(x, y) ∈ R2 : x ≤ 0}. Ahora, para (x, y) ∈ R2 tal que x > 0, se tiene que u(x, y) = x−3 y 3 ; as´ı ∂2u ∂2u (x, y) + (x, y) ∂x2 ∂y 2 = 12x−5 y 3 + 6x−3 y  = 6x−3 y x−2 y 2 + 1 = 0

∇2 u(x, y) =



y = 0,

es decir, u(x, y) tambi´ en es arm´ onica en el conjunto {(x, y) ∈ R2 : x > 0, y = 0}, que no es un dominio. De esta forma, u(x, y) es arm´ onica en el conjunto {(x, y) ∈ R2 : x ≤ 0} ∪ {(x, y) ∈ R2 : x > 0, y = 0}. En consecuencia, el dominio D ⊂ C donde la funci´ on f (z) = u(x, y) + iv(x, y) es anal´ıtica es D = {z = x + iy : x < 0}.

Como u(x, y) = Re (z 2 + 1) = x2 − y 2 + 1 para todo z ∈ D, entonces es claro que la funci´ on v(x, y) = Im (z 2 + 1) + c = 2xy + c es arm´ onica conjugada de u(x, y), donde c es una constante real. Por lo tanto, forzando f (−1 − i) = 1 − i, se obtiene 1 − i = f (−1 − i) = u(−1, −1) + iv(−1, −1) = 1 + (2 + c)i



c = −3.

Es decir, la funci´ on f (z) = (x2 − y 2 + 1) + i(2xy − 3) = z 2 + 1 − 3i es anal´ıtica en D y f (−1 − i) = 1 − i.



Problema 2.6. Sean α un n´ umero real y z0 un n´ umero complejo tal que Im z0 > 0. Demostrar que el mapeo bilineal   iα z − z0 , w=e z − z0 transforma al semiplano superior Im z ≥ 0 sobre en el disco unitario |w| ≤ 1.

70

2.9. PROBLEMAS RESUELTOS

Soluci´ on. Sean z = x + iy y z0 = x0 + iy0 . Se tiene que   2 2 2 iα z − z0 2 iα 2 z − z0 2 2 = |z − z0 | = (x − x0 ) + (y − y0 ) . = e |w| = e z − z0 z − z0 |z − z 0 |2 (x − x0 )2 + (y + y0 )2

Como Im z0 = y0 > 0 e Im z = y ≥ 0, entonces de la ecuaci´ on anterior se deduce que |w|2 ≤ 1,

la cual implica que |w| ≤ 1.



Problema 2.7. Sea A ⊂ C el conjunto sombreado que se muestra en la siguiente figura. y 1

A 1 2

− 21

-1

1 2

1

x

Determine la expresi´ on anal´ıtica y luego grafique el transformado del conjunto A bajo el mapeo z−1 . w= i−z Soluci´ on. El conjunto A se puede expresar inecuaciones:   |z| A: |z|  y

anal´ıtica a trav´ es del siguiente sistema de ≤ 1, ≥ 21 , ≥ 0.

(2.37)

Ahora, despejemos z = x + iy en funci´ on w = u + iv, a partir de la ecuaci´ on w = Se tiene que iw − wz = z − 1



z(w + 1) = iw + 1



z=

iw + 1 , w+1

z−1 . i−z (2.38)

de donde se deduce x + iv =

i(u + iv) + 1 u + iv + 1



x + iv =

[(1 − v) + iu] [(u + 1) − iv] , (u + 1)2 + v 2

lo que implica que x=

u−v+1 , (u + 1)2 + v 2

y=

u2 + v 2 + u − v . (u + 1)2 + v 2

(2.39)

Sustituyendo convenientemente en (2.37) las expresiones de x, y y z dadas en (2.38) y (2.39), y luego operando, se obtienen las ecuaciones que describen el transformado de A,  iw + 1 2      w + 1 ≤ 1, |iw + 1|2 ≤ |w + 1|2 ,           iw + 1 2 1 ⇔ ⇔ f (A) : |iw + 1|2 ≥ 14 |w + 1|2 , , ≥ w+1 4          2   u + v 2 + u − v ≥ 0,  u2 + v 2 + u − v   ≥ 0, (u + 1)2 + v 2

71

CAP´ITULO 2. FUNCIONES DE VARIABLE COMPLEJA

          

     

     

|(1 − v) + iu|2 ≤ |(u + 1) + iv|2 , |(1 − v) + iu|2 ≥

1 4

|(u + 1) + iv|2 , ⇔

(1 − v)2 + u2 ≤ (u + 1)2 + v 2 , 

≥ (u + 1)2 + v 2 ,      (u + 1 )2 + (v − 1 )2 ≥ 1 , 2 2 2   v ≥ −u,     ⇔ u2 + v 2 − 32 u − 83 v + 1 ≥ 0,      (u + 21 )2 + (v − 12 )2 ≥ 12 .

u2 + v 2 + u − v ≥ 0, v 2 − 2v + 1 + u2 ≤ u2 + 2u + 1 + v 2 ,

4v 2 − 8v + 4 + 4u2 ≥ u2 + 2u + 1 + v 2 ,      (u + 1 )2 + (v − 1 )2 ≥ 1 . 2 2 2

4 (1 − v)2 + u2

De esta forma, el transformado de A est´ a dado por:   v ≥ −u,     (u − 31 )2 + (v − 43 )2 ≥ 89 , f (A) :      (u + 1 )2 + (v − 1 )2 ≥ 1 . 2 2 2

cuya representaci´ on gr´afica se muestra en la siguiente figura. v

.. .

4

f (A)

..

3

.

2 1 -4

-3

-2

-1

1

3

4

u

..

-1

2

. 

2.10 Problemas Propuestos 2.1. Encuentre el dominio de definici´ on de cada una de las siguientes funciones. 1 1 − |z|2 1 b) f (z) = 2 z +1 z c) g(z) = z+z 1−z d) g(z) = 3 z + iz 2 + 3z − i a) f (z) =

|z|2 − 1 |z + i| 1 f) h(z) = 2 − (z − i)2

e) g(z) =

g) h(z) = Arg (1/z) h) h(z) =

z 2 − z + (1 − z)i z3 + z

72

2.10. PROBLEMAS PROPUESTOS

2.2. Encuentre las funciones componentes, u(x, y) y v(x, y), de las siguientes funciones f (z), donde z = x + iy. 1 . z2 z+1 b) f (z) = . 2z − 5

c) f (z) = z 2 − 3z + 2.

a) f (z) =

d) f (z) =

2.3. Dada la funci´ on f (z) = z ∈ C tales que:

z+2 . 2z − 1 + i

z+2 , establecer en cada caso el conjunto de puntos 2z − 1

a) f (z) = i

b) f (z) = z

c) f (z) = 2

2.4. Calcule los siguientes l´ımites: 1−z z→2i 1 + z b) l´ım (2z 3 − 3z 2 + z + i) a) l´ım

f) l´ım

z→i

g)

z→2i

c)

l´ım z→e

πi 4

z2 z4 + z + 1

h)

z 2 − 2i z d) l´ım z→2+i z 2 + 4 z 2 − 2i z e) l´ım z→2i z 2 + 4

z2 + 1 z6 + 1

l´ım

z z+z

l´ım

|z|2 − 1 |z + i|

z→(3−4i)

z→(1−i)

i) l´ım Arg (1/z) z→i

j) l´ım (ln |z| + i Arg z) z→−i

2.5. Dada la funci´ on f : C → C definida como f (z) =

(z + z) + 2(Re z)(Im z + 1)i , z+z

calcule l´ımite de f (z) cuando z tiende a 0. 2.6. Determine el conjunto de n´ umeros complejos donde las siguientes funciones son continuas:  2 3z − 5iz + 2   , z 6= 2i; z 2 + iz + 6 a) f (z) =  7 z = 2i. 5,  2 z +1   , z 6= −i; z+i b) g(z) =   −2i, z = −i.  Re z  , Re z 6= Im z, Im z 6= 0;    Im z   c) h(z) = 1, Re z = Im z, Im z 6= 0;       Re z, Im z = 0.

73

CAP´ITULO 2. FUNCIONES DE VARIABLE COMPLEJA

2.7. Sea f (z) una funci´ on definida por  (z + i)2 , si y 6= x, f (z) = (−2x − 1) + i 2(x2 + x), si y = x,

donde z = x + iy. Determine el conjunto de todos los n´ umeros complejos z donde f (z) es continua.

2.8. Pruebe que cada una de las siguientes funciones satisfacen las ecuaciones de Cauchy-Riemann: a) f (z) = ex (−sen y + i cos y). b) f (z) = cos x cosh y − i sen x senh y.

c) f (z) = sen x cosh y + i cos x senh y.

d) f (z) = ex

2 −y 2

(cos(2xy) + i sen (2xy)).

2.9. Determine en qu´ e conjunto D de n´ umeros complejos z = x + i y, las siguientes funciones son derivables y calcule su derivada. a) f (z) = x2 + i y 2 b) f (z) = x2 + 2x − i y

c) f (z) = 2xy + i (x + 23 y 3 )

2.10. Utilizando las condiciones necesarias y suficientes para la existencia de derivada, demostrar que f ′ (z) no existe en punto alguno del plano complejo si f (z) est´ a dada por: a) f (z) = z.

c) f (z) = 2x + ixy 2 .

b) f (z) = z − z.

d) f (z) = ex e−iy .

2.11. Determine el conjunto D de todos los n´ umeros complejos donde cada una de las siguientes funciones f (z) es anal´ıtica y calcule su derivada: a) f (z) = (z + 1)2 1 b) f (z) = z + z 3z − 1 c) f (z) = 3−z 2z + 1 d) f (z) = z(z 2 + 1)

z4 1 + z4 Log (z + 4) f) f (z) = z2 + i

e) f (z) =

g) f (z) = ee

z

h) f (z) = sen (ez )

2.12. Comprobar que cada una de las siguientes funciones son enteras: a) f (z) = 3x + y + i(3y − x).

b) f (z) = e−y eix .

c) f (z) = (z 2 − 2)e−z .

d) f (z) = sen (x2 − y 2 ) cosh(2xy) + i cos(x2 − y 2 ) senh(2xy).

74

2.10. PROBLEMAS PROPUESTOS

2.13. Determine el conjunto de todos los n´ umeros complejos z = x+iy donde la funci´ on y 2 − x2 + i|1 − xy| es anal´ıtica. f (z) = 2 2.14. Determine en qu´ e conjunto D ⊂ R2 cada una de las siguientes funciones son arm´ onicas, y encuentre de ser posible su arm´ onica conjugada. a) u(x, y) = Im (z 2 + 3z + 1) x−1 b) u(x, y) = 2 x + y 2 − 2x + 1

c) v(x, y) = Im (z + 1/z) −y − 2 +3 d) v(x, y) = (x + 1)2 + (y + 2)2

2.15. Demostrar las siguientes identidades. a) e(2±3πi) = −e2

b) ez+πi = −ez

c) exp(z) = exp(iz), para z ∈ C

d) senh(z + πi) = − senh z, para z ∈ C e) cosh(z + πi) = − cosh z, para z ∈ C

f) senh 2z = 2 senh z cosh z, para z ∈ C

g) Log (ei) = 1 + (π/2)i

h) Log (1 + i) = 21 (ln 2 + (π/2)i) i) log 1 = 2nπi, para n = 0, ±1, ±2, . . .

j) log(−1) = (2n + 1)πi, para n = 0, ±1, . . .  k) log i1/2 = (n + 1/4)πi, para n = 0, ±1, . . .

l) (1 + i)i = exp(−π/4 + 2nπ) exp((i/2) ln 2), para n = 0, ±1, ±2, . . .

m) (−1)1/π = exp((2n + 1)i), para n = 0, ±1, ±2, . . . 2.16. Demostrar que:

a) si log z = ln r + iθ, para todo z tal que r = |z| > 0 y π/4 < θ = arg z < 9π/4, entonces log(i2 ) = 2 log i. b) si log z = ln r + iθ, para todo z tal que r = |z| > 0 y 3π/4 < θ = arg z < 11π/4, entonces log(i2 ) 6= 2 log i. 2.17. Encontrar todas las ra´ıces de cada una de las siguientes ecuaciones: c) tanh z = −2 d) log z = (π/2)i

a) cosh z = 1/2 b) senh z = i

e) ez = −3

2.18. Encontrar el valor principal de: a) ii

b) (1 − i)4i 

c) (−i)3πi

 1 2.19. Sea f (z) = Log . Determine el conjunto D de todos los n´ umeros complez+i jos z donde f (z) es anal´ıtica.

75

CAP´ITULO 2. FUNCIONES DE VARIABLE COMPLEJA

2.20. Sea f (z) = Log (z 2 + 1). Determine el conjunto D de todos los n´ umeros complejos z donde f (z) es anal´ıtica. 2.21. Determine la rama principal y su derivada, para cada una de las siguientes funciones multivaluadas: a) f (z) =



2

d) f (z) = z log z ≡ e(log z)

ez + 1

e) f (z) = icos z ≡ ecos z log i

b) f (z) = cos(log z) c) f (z) = log(ez + 1)

f) f (z) = z sen z ≡ esen z log z

2.22. Sea f (z) una funci´ on de variable compleja definida como z

f (z) = ie ≡ ee

z

log i

,

donde se utiliza un rama tal que se satisface f ′ (iπ) = 3π/2. Determine f ′ (z) y f ′ (πi/2). 2.23. Demostrar que la transformaci´ on w = iz + i mapea el semiplano x > 0 sobre el semiplano v > 1. 2.24. Demostrar que si c1 < 0, la imagen del semiplano x < c1 bajo la transformaci´ on w = 1/z es el interior de un c´ırculo. ¿Cu´al es la imagen cuando c1 = 0? 2.25. Demostrar que la imagen del semiplano y > c2 bajo la transformaci´ on w = 1/z es el interior de un c´ırculo, siempre y cuando c2 > 0. Encontrar la imagen cuando c2 < 0; tambi´ en cuando c2 = 0. 2.26. Sean α un n´ umero real y z0 un n´ umero complejo tal que |z0 | ≤ 1. Demostrar que la transformaci´ on bilineal   z − z0 iα , w=e zz 0 − 1 mapea el disco |z| ≤ 1 sobre disco |w| ≤ 1. 2.27. Considere el conjunto D de n´ umeros complejos que se muestra en la siguiente figura. y 1

-1

1 -1

x

D

Determine el conjunto transformado de D bajo el mapeo w =

z+1+i . z+i

76

2.10. PROBLEMAS PROPUESTOS

2.28. Sea S el conjunto de n´ umeros complejos que se muestra en la siguiente figura. y 1

−1

S

1

x

Determine el transformado de S bajo cada uno de los siguientes mapeos: 1 . z 1−z b) w = . i+z a) w =

i−z . i+z z+2 . d) w = z − 2i c) w =

3 Series de Potencias y Singularidades Aisladas En este cap´ıtulo se estudian los conceptos b´asicos de las series de n´ umeros complejos, particularmente, la representaci´ on en serie de potencias de las funciones de variable compleja, hecho ´este de gran importancia para la resoluci´ on de problemas en Ingenier´ıa, por ejemplo, ecuaciones diferenciales. Inicialmente se dan los conceptos de sucesi´ on y serie de n´ umeros complejos; despu´es se presenta un resultado fundamental de la convergencia de las series de potencias y se describen los desarrollos de Taylor y de Laurent. Finalmente, se caracterizan los puntos singulares aislados de una funci´ on de variable compleja.

3.1 Serie de N´ umeros Complejos Definici´ on 3.1 (Sucesi´ on de n´ umeros complejos). El conjunto de n´ umeros complejos {z0 , z1 , z3 , . . .}, se denomina sucesi´ on de n´ umeros complejos y se denota por {zn }∞ n=0 . Definici´ on 3.2 (Sucesi´ on convergente). La sucesi´ on de n´ umeros complejos {zn }∞ n=0 tiene l´ımite o converge a un n´ umero complejo z, si para todo ε > 0 existe un n´ umero entero N > 0 tal que |zn − z| < ε siempre que n ≥ N . Cuando la sucesi´ on {zn }∞ n=0 converge a z, se escribe l´ım zn = z.

n→∞

Observaci´ on 3.1. Para cada sucesi´ on de n´ umeros complejos {zn }∞ n=0 , existen sucesiones de ∞ ∞ n´ umeros reales {xn }n=0 y {yn }n=0 tales que zn = xn + i yn ,

para n = 0, 1, 2, . . .

on de n´ umeros complejos definida por Ejemplo 3.1. Sea {zn }∞ n=0 la sucesi´ zn =

1 . (1 + i)n

∞ Entonces, las sucesiones de n´ umeros reales {xn }∞ n=0 y {yn }n=0 tales que zn = xn + i yn est´ an definidas como

xn = 2−n/2 cos(nπ/4)

y

yn = −2−n/2 sen (nπ/4). 

77

78

´ MEROS COMPLEJOS 3.1. SERIE DE NU

El siguiente teorema permite estudiar la convergencia de las sucesiones de n´ umeros complejos a trav´ es de la convergencia de sucesiones de n´ umeros reales. La demostraci´ on de este teorema se establece usando la definici´ on de convergencia de sucesiones. Teorema 3.1. Sean zn = xn + i yn (n = 0, 1, 2, . . .) con xn , yn ∈ R, y z = x + i y con x, y ∈ R. Entonces, l´ım zn = z n→∞

si, y s´olo si l´ım xn = x

n→∞

y

l´ım yn = y.

n→∞

on de En el siguiente ejemplo se emplea el Teorema 3.1 para calcular el l´ımite de sucesi´ n´ umeros complejos. Ejemplo 3.2. Determinar si la sucesi´ on   (−1)n 1 zn = + i 1 + n n

(n = 1, 2, . . .)

converge y halle el l´ımite si es el caso. Soluci´ on. Se tiene que zn = xn + i yn , donde xn =

1 n

y

yn = 1 +

(−1)n . n

on {zn }∞ Como l´ım xn = 0, y l´ım yn = 1, entonces por el Teorema 3.1 la sucesi´ n=0 con→∞

n→∞

nverge y su l´ımite es

  1 (−1)n l´ım zn = l´ım xn + i l´ım yn = l´ım + i l´ım 1 + = i. n→∞ n→∞ n→∞ n→∞ n n→∞ n  Definici´ on 3.3 (Serie de N´ umeros Complejos). La suma de los t´erminos de una sucesi´ on ∞ de n´ umeros complejos {zn }n=0 , se denomina serie de n´ umeros complejos, esto es ∞ X

n=0 Un ejercicio interesantes es definir las sumas parciales de la serie onica P∞ arm´ n n ρ , con 0 < ρ < 1, y demostrar que esta ser´ıa converge a 1/(1 − ρ)

zn = z0 + z1 + z2 + · · ·

Definici´ on 3.4 (Serie Convergente). Se dice que una serie n´ umero complejo S, si la sucesi´ on de sumas parciales, SN =

N X

zn ,

n=0

converge a S. En este caso se escribe ∞ X

n=0

zn = l´ım SN = S. N →∞

P∞

n=0 zn

converge a un

79

CAP´ITULO 3. SERIES DE POTENCIAS Y SINGULARIDADES AISLADAS

Un hecho muy importante que relaciona las series de n´ umeros complejos P con las series de n´ umeros reales es el siguiente: toda serie de n´ umeros complejos ∞ n=0 zn se puede escribir como: ∞ ∞ ∞ X X X yn , xn + i zn = n=0

n=0

n=0

P∞

P∞

umeros reales, adem´as, estas series se denominan donde n=0 xn y n=0 yn son series P de n´ parte real y parte imaginaria de ∞ z , n=0 n respectivamente. El hecho descrito arriba permite estudiar la convergencia de las series de n´ umeros complejos a trav´ es de la convergencia de series de n´ umeros reales. En el siguiente teorema se establece que una serie de n´ umeros complejos converge si, y s´ olo si las series de sus partes real e imaginaria convergen. Teorema 3.2. Sean zn = xn + i yn (n = 0, 1, 2, . . .) con xn , yn ∈ R, y S = X + i Y con X, Y ∈ R. Entonces, ∞ X zn = S n=0

si, y s´olo si

∞ X

xn = X

y

∞ X

yn = Y.

n=0

n=0

∞ Demostraci´ on. Sean {XN }∞ n=0 y {XN }n=0 las sucesiones de sumas parciales definidas respectivamente por N N X X yn . xn y Y N = XN = n=0

n=0

As´ı,

SN =

N X

n=0

zn =

N X

xn + i

N X

yn = XN + i YN .

n=0

n=0

Como S = X + i Y = l´ımN →∞ SN = l´ımN →∞ XN + i l´ımN →∞ YN si, y s´ olo si l´ım XN = X

N →∞

y

l´ım YN = Y,

N →∞

P entonces, concluir que la serie ∞ olo si las n=0 zn converge a S = X + i Y si, y s´ P∞podemos P∞ series n=0 xn y n=0 yn convergen respectivamente a X e Y .

3.1.1 Serie de Potencias

Definici´ on 3.5 (Serie de potencias). Dada una sucesi´ on {an }∞ umeros complejos n=0 de n´ y z0 ∈ C, a la serie ∞ X an (z − z0 )n (3.1) n=0

se le llama serie de potencias, donde z ∈ C. Los n´ umeros an se denominan coeficientes de la serie y z0 se denomina centro de la serie.

El siguiente teorema da una idea completa del campo de convergencia de las series de potencias. La demostraci´ on de este teorema se puede apreciar en [8].

80

´ MEROS COMPLEJOS 3.1. SERIE DE NU

Teorema 3.3 (Teorema de Cauchy-Hadamard). Consideremos la serie de potencias (3.1). p n ´nico punto Sea α = l´ımn→∞ |an |. Entonces: si α = ∞, la serie es convergente en el u z = z0 ; si 0 < α < ∞, la serie es absolutamente convergente en el c´ırculo |z − z0 | < 1/α y es divergente en el exterior de este c´ırculo; y si α = 0, la serie es absolutamente convergente en todo el plano complejo. El Teorema 3.3 es conocido como criterio de la ra´ız; cuando α se define por α = |an+1 | l´ım , n→∞ |an | este teorema se conoce como criterio del cociente

De este modo, cuando 0 < α < ∞, existe un c´ırculo con centro en el punto z = z0 , en el interior del cual la serie (3.1) es absolutamente convergente y en el exterior del cual ´ ste se llama c´ırculo de convergencia de la serie de potencias y su la serie es divergente. E radio R = 1/α, radio de convergencia de la misma. Los casos α = ∞ y α = 0 se pueden considerar como casos l´ımites. En el primero de ellos, el c´ırculo de convergencia se reduce a un punto z0 y su radio R es igual a cero. En el segundo, el c´ırculo de convergencia se extiende a todo el plano, de modo que se puede considerar que su radio es igual a ∞. Llamando en los tres casos al n´ umero R radio de convergencia de la serie de potencias, el contenido de la f´ ormula de Cauchy-Hadamard puede expresarse por la f´ ormula R=

1 . α

Esta u ´ ltima se llama f´ ormula de Cauchy-Hadamard. Para las aplicaciones de la f´ ormula de Cauchy-Hadamard, en muchos casos suele ser u ´ til la relaci´ on siguiente: r 1 n n! (3.2) = . l´ım n n→∞ n e La demostraci´ on de esta relaci´ on se deja como ejercicio para el lector. Ejemplo 3.3. Hallar el c´ırculo y el radio de convergencia de la serie ∞ X nn

n=1

n!

zn.

Soluci´ on. Se tiene que

nn y z0 = 0. n! Ahora, utilizando la ecuaci´ on (3.2) obtenemos r nn α = l´ım n = e. n→∞ n! an =

Por lo tanto, el radio de convergencia de la serie es R=

1 , e

y el c´ırculo de convergencia de la misma es 1 |z| < . e 

CAP´ITULO 3. SERIES DE POTENCIAS Y SINGULARIDADES AISLADAS

81

En muchos casos resulta conveniente determinar el radio de convergencia de una serie de potencias mediante el criterio del cociente, es decir, tomando an+1 . α = l´ım n→∞ an

As´ı, para la serie del Ejemplo 3.3 se tiene (n+1)n+1   an+1 (n+1)! 1 n = l´ım α = l´ım = l´ ım = e, 1 + nn n→∞ n→∞ an n→∞ n n! por consiguiente, el radio de convergencia de la serie es R = 1/e.

Ejemplo 3.4. Determine el c´ırculo de convergencia de la serie de potencias ∞ X

zn.

n=0

Tambi´ en determine a qu´ e converge esta serie. Soluci´ on. Como an = 1, para n ≥ 0, y z0 = 0, entonces el radio de convergencia de la serie es R = 1 y su c´ırculo de convergencia es |z| < 1. Ahora, para determinar a qu´e converge la serie dada, consideremos la sucesi´ on de sumas parciales SN (z) =

N X

zn

n=0

= 1 + z + z2 + · · · + zN =

1 − z N +1 , 1−z

z 6= 1.

Por lo tanto, utilizando la ecuaci´ on anterior podemos escribir ∞ X

n=0

z n = l´ım SN (z) N →∞

1 − z N +1 N →∞ 1−z 1 = , |z| < 1. 1−z

= l´ım



3.1.2 Serie de Taylor En esta parte se estudia una propiedad muy importante de las funciones anal´ıticas, a saber: toda funci´ on anal´ıtica se puede expresar como una serie de potencias que converge a dicha funci´ on en alg´ un dominio. En el siguiente teorema se establece que si una funci´ on f (z) es anal´ıtica en un c´ırculo centrado en un punto z0 ∈ C, entonces f (z) se puede representar como una serie de potencias en dicho c´ırculo. Para la descripci´ on formal del siguiente teorema, es necesario suponer que f (z) es infinitamente diferenciable, lo cual, como veremos m´ as adelante en el on anal´ıtica. Cap´ıtulo 4, es una propiedad que posee toda funci´

82

´ MEROS COMPLEJOS 3.1. SERIE DE NU

Teorema 3.4 (Teorema de Taylor). Si f (z) es una funci´on anal´ıtica en todo punto del disco |z − z0 | < r0 , entonces f (z) se expresa como f (z) =

∞ X f (n) (z0 )

n=0

n!

(z − z0 )n ,

(3.3)

para |z − z0 | < r0 . Observaci´ on 3.2. P f (n) (z0 ) (z − z0 )n converge a f (z), • El Teorema de Taylor garantiza que la serie ∞ n=0 n! para todo z tal que |z − z0 | < r0 . El nombre de Taylor es en honor al matem´atico brit´anico Brook Taylor (1685-1731). • La serie de potencias (3.3) se denomina desarrollo en serie de Taylor o, simplemente, desarrollo de Taylor de f (z) centrado en el punto z0 . • Si z0 = 0, entonces el desarrollo de Taylor adquiere la forma f (z) =

∞ X f (n) (0)

n=0

n!

zn

y se denomina desarrollo de MacLaurin de f (z). El nombre de MacLaurin es en honor al matem´ atico escoc´ es Colin MacLaurin (1698-1746). El siguiente teorema nos garantiza que el desarrollo de Taylor de una funci´ on f (z) alrededor de z0 , es la u ´ nica serie de potencias que converge a f (z) en un disco centrado en z0 . Teorema 3.5. El desarrollo en serie de Taylor de una funci´on f (z) alrededor de z0 es la u ´nica serie en potencias de (z − z0 ) que converge a f (z) en todo punto de un disco centrado en z0 . Demostraci´ on. Por el absurdo. Supongamos que f (z) posee dos desarrollos de Taylor distintos alrededor de z0 v´ alidos en un mismo dominio D ⊂ C, a saber: f (z) =

∞ X

n=0

an (z − z0 )n

y

f (z) =

∞ X

n=0

bn (z − z0 )n

Como estos desarrollos son distintos, entonces se debe satisfacer que an 6= bn , para alg´ un n ≥ 0. Ahora, tambi´ en tenemos que, para todo z 6= z0 , 0 = f (z) − f (z) =

∞ X

(an − bn )(z − z0 )n

n=0

⇐⇒

a n = bn ,

para todo n ≥ 0.

Esto es una contradicci´ on. Observaci´ on 3.3. Sean f (z) y g(z) funciones tales que su desarrollo de Taylor centrado en z0 est´ a dado respectivamente por f (z) =

∞ X

n=0

an (z − z0 )n ,

para z tal que |z − z0 | < r1 ,

CAP´ITULO 3. SERIES DE POTENCIAS Y SINGULARIDADES AISLADAS

y g(z) =

∞ X

n=0

bn (z − z0 )n ,

83

para z tal que |z − z0 | < r2 .

Entonces del desarrollo de Taylor de f (z) + g(z) centrado en z0 es f (z) + g(z) =

∞ X

(an + bn ) (z − z0 )n ,

n=0

para z tal que |z − z0 | < m´ın(r1 , r2 ).

El siguiente teorema nos permite calcular el radio de convergencia del mayor c´ırculo para el cual existe la serie de Taylor de una funci´ on f (z). Teorema 3.6. Consideremos el desarrollo en serie de Taylor (3.3) de una funci´on f (z) alrededor de z0 . Entonces, |z − z0 | < r es el mayor c´ırculo dentro del cual la serie converge a f (z) para cada z en ´este c´ırculo, donde r es la distancia entre z0 y el punto singular de f (z) m´ as cercano. Demostraci´ on. Como f (z) es anal´ıtica en z0 , entonces existe r0 > 0 tal que para todo z en el disco |z − z0 | < r0 se tiene que f (z) es derivable. Afirmamos que r0 = r, donde r es la distancia entre z0 y el punto singular de f (z) m´ as cercano, ya que de lo contrario, si r0 > r, esto ser´ıa una contradicci´ on al Teorema de Taylor. Obs´ ervese que este teorema no afirma que la serie de Taylor no converja fuera de |z − z0 | = r. S´ olo asevera que ´este es el mayor c´ırculo en todo punto del cual la serie converge a f (z). El c´ırculo en todo punto del cual la serie de Taylor (3.3) converge a f (z) y el c´ırculo en todo punto del cual la serie converge no son necesariamente iguales. El segundo de ellos puede tener un radio mayor. No obstante, se puede mostrar que cuando el punto singular m´ as cercano a z0 es tal que |f (z)| se hace infinito, los dos c´ırculos coinciden. ´ ste es el caso en la mayor parte de las funciones que consideraremos. E Ejemplo 3.5. Comprobar cada uno de los siguientes desarrollos de Taylor. a) ez =

∞ X 1 n z , n! n=0

b) sen z =

|z| < ∞.

∞ X (−1)n 2n+1 z , (2n + 1)!

n=0

c) cos z =

∞ X (−1)n

n=0

(2n)!

z 2n ,

|z| < ∞.

∞ X

d) senh z =

1 z 2n+1 , (2n + 1)! n=0

e) cosh z =

∞ X

n=0 ∞

1 z 2n , (2n)!

X 1 f) zn, = 1−z n=0

|z| < ∞.

|z| < ∞.

|z| < ∞.

|z| < 1.

84

´ MEROS COMPLEJOS 3.1. SERIE DE NU



X 1 (−1)n z n , = g) 1+z n=0

h)

i)



1 X (−1)n (z − 1)n , = z n=0

|z| < 1. |z − 1| < 1.



i X1h 1 (−1)n + 2−(n+1) (z − 1)n , = z(3 − z) 3 n=0

|z − 1| < 1.

Soluci´ on. a) Se tiene que f (z) = ez es una funci´ on entera y f (n) (z) = ez . As´ı, por los Teoremas 3.4 y 3.6, el desarrollo de MacLaurin de ez es ∞ X 1 n e = z , n! z

n=0

|z| < ∞.

b) Se tiene que f (z) = sen z es una funci´ on entera, adem´as, sen z = forma, utilizando el desarrollo de Macalurin de ez podemos escribir:

sen z =

eiz − e−iz = 2i

∞ ∞ X X 1 1 n (iz) − (−iz)n n! n! n=0 n=0

2i

=

eiz − e−iz . De esta 2i

∞ 1 X 1 n (i − (−i)n )z n . 2i n! n=0

Ahora, utilizando la expresi´ on anterior y el hecho que ( 0, si n es par, (in − (−i)n ) = m 2i(−1) , si n = 2m + 1 es impar, obtenemos que el desarrollo de MacLaurin de sen z es ∞ X (−1)n 2n+1 z , sen z = (2n + 1)! n=0

|z| < ∞.

La verificaci´ on de c), d) y e), se deje como ejercicio pare el lector. 1 f) Se tiene que f (z) = 1−z es anal´ıtica en todo z ∈ C tal que z 6= 1, entonces por el 1 Teorema 3.6 el desarrollo de MacLaurin de f (z) = 1−z es v´alido para todo |z| < 1. Ahora, 1 por el Ejemplo 3.4, se tiene que el desarrollo de MacLaurin de f (z) = 1−z es ∞

X 1 zn , = 1−z n=0

1 1+z

|z| < 1.

g) Como f (z) = es anal´ıtica en todo z ∈ C tal que z 6= 1, entonces por el Teorema 3.6 1 el desarrollo de Maclaurin de f (z) = 1+z es v´alido para todo |z| < 1. A continuaci´ on 1 hallamos el desarrollo de MacLaurin de f (z) = 1+z empleando el desarrollo de Taylor de 1 1−z dado en f). Se tiene que ∞ ∞ X X 1 1 (−z)n = (−1)n z n , = = 1+z 1 − (−z) n=0 | {z } n=0 |−z|<1

|z| < 1.

85

CAP´ITULO 3. SERIES DE POTENCIAS Y SINGULARIDADES AISLADAS

h) Aqu´ı utilizamos el desarrollo de Taylor dado en g) para hallar el desarrollo de Taylor de f (z) = 1/z centrado en z0 = 1. Se tiene que ∞

X 1 1 (−1)n (z − 1)n , = = z 1 + (z − 1)

|z − 1| < 1.

n=0

1 i) Para hallar el desarrollo de Taylor de f (z) = z(3−z) centrado en z0 = 1, utilizamos fracciones simples, desarrollos de Taylor conocidos y operaciones algebraicas simples. Se tiene que

1 z(3 − z)

= =

1/3 −1/3 1/3 −1/3 + = + z z−3 1 + (z − 1) (z − 1) − 2 1 1 1 1 1 · + · · 3 1 + (z − 1) 3 2 1 − (z−1) 2

=

∞ 1X

3

(−1)n (z − 1)n +

n=0 ∞ X

∞ 1 X (z − 1)n

1 · 3 2

n=0

2n

∞ X 1 1 −(n+1) n n = (−1) (z − 1) + 2 (z − 1)n 3 3 n=0 {z } |n=0 {z } |

=

|z−1|<1

∞ X

1h

3 n=0

|z−1|<2

i

(−1)n + 2−(n+1) (z − 1)n ,

|z − 1| < 1. 

3.1.3 Serie de Laurent El desarrollo en serie de Laurent o, simplemente, desarrollo de Laurent de una funci´ on f (z) centrado en el punto z0 es de la forma f (z) =

∞ X

n=−∞

cn (z − z0 )n ,

donde la serie converge a f (z) en cierto dominio o regi´ on y cn ∈ C. El nombre de Laurent es en honor al matem´ atico y oficial militar franc´es Pierre Alphonse Laurent (1813-1854). As´ı pues, un desarrollo de Laurent, a diferencia del desarrollo de Taylor, puede contener uno o m´ as t´ erminos con (z − z0 ) elevado a una potencia negativa. Tambi´ en puede contener potencias positivas de (z − z0 ). Normalmente los desarrollos de Laurent se obtienen a partir de los desarrollos de Taylor. Por ejemplo, para encontrar el desarrollo de Laurent de f (z) = e1/z centrado en z0 = 0, se considera el desarrollo de MacLaurin de es , ∞ X 1 n s , e = n! s

n=0

|s| < ∞

despu´es, se hace s = 1/z en la ecuaci´ on anterior para obtener 1/z

e

∞ X 1 −n z , = n! n=0

|z| > 0,

86

´ MEROS COMPLEJOS 3.1. SERIE DE NU

que es, efectivamente, el desarrollo de Laurent de e1/z centrado en z0 = 0, v´alido en todo el plano complejo excepto en z = 0. ¿Qu´ e clase de funciones pueden representarse por medio de series de Laurent y en qu´ e regi´ on del plano complejo ser´ a v´ alida dicha representaci´ on? La respuesta se encuentra en el siguiente teorema. Antes de dar el teorema, definiremos anillo o dominio anular. Definici´ on 3.6 (Anillo o dominio anular). Sean r1 > 0 y r2 > 0 tales que r1 < r2 . Se denomina anillo o dominio anular centrado en z0 ∈ C, al conjunto de n´ umeros complejos z tales que r1 < |z − z0 | < r2 . Teorema 3.7 (Teorema de Laurent). Si f (z) es una funci´on anal´ıtica en el anillo r1 < |z − z0 | < r2 , centrado en z0 , entonces f (z) se puede expresar como f (z) =

∞ X

n=0

an (z − z0 )n +

∞ X

n=1

bn (z − z0 )−n ,

para todo z tal que r1 < |z − z0 | < r2 . Observaci´ on 3.4. P P∞ n −n • El Teorema de Laurent garantiza que la serie ∞ n=0 an (z − z0 ) + n=1 bn (z − z0 ) converge a f (z) para todo z tal que r1 < |z − z0 | < r2 . • Si f (z) es anal´ıtica en todos los puntos de la regi´ on |z − z0 | < r2 , entonces el desarrollo de Laurent de f (z) centrado en z0 , se convierte en el desarrollo de Taylor de f (z) centrado en z0 . • Si f (z) es anal´ıtica en todos los puntos de la regi´ on |z − z0 | < r2 excepto en en el punto z0 , entonces el desarrollo de Laurent es v´alido en toda la regi´ on 0 < |z − z0 | < r2 . • Los coeficientes de la serie de Laurent se pueden definir como una integral que involucra a la funci´ on f (z). Este u ´ltimo hecho permite calcular integrales utilizando los resultados de series de potencias, lo cual se estudiar´a en el Cap´ıtulo 4. Ejemplo 3.6. Determine el desarrollo de Laurent en potencias de z de la funci´ on f (z) =

1 . 1+z

Soluci´ on. Observamos que el u ´ nico punto singular de f (z) es z = −1, entonces existen dos regiones donde f (z) posee desarrollos de Laurent centrados en z0 = 0, a saber: a) |z| < 1, y b) |z| > 1. a) Como f (z) es anal´ıtica en todo punto del disco |z| < 1, el desarrollo de Laurent de f (z) centrado en z0 = 0 v´ alido en el dominio |z| < 1, coincide con su desarrollo de Maclaurin. En otras palabras, el desarrollo de Laurent de f (z) centrado en z0 = 0 v´alido en el dominio |z| < 1, es: ∞ X (−1)n z n , |z| < 1. f (z) = n=0

87

CAP´ITULO 3. SERIES DE POTENCIAS Y SINGULARIDADES AISLADAS

b) Ahora bien, el desarrollo de Laurent de f (z) centrado en z0 = 0 v´alido en el dominio |z| > 1, est´ a dado por: ∞ ∞ X X 1 1 1 (−1)n z −(n+1) , (−1)n z −n = = · −1 = z −1 1+z z z +1 } n=0 | n=0 {z

|z| > 1.

|z −1 |<1



Ejercicio 3.1. Compruebe que todos los desarrollos de Laurent centrados en z0 = i de la funci´ on 1 f (z) = (z − 1)(z − 2) est´ an dados por: • f (z) =

∞ h X

n=0

• f (z) = − • f (z) =

i √ (1 − i)−(n+1) − (2 − i)−(n+1) (z − i)n , |z − i| < 2,

∞ X

n

−(n+1)

(1 − i) (z − i)

n=0

∞ X

n=0



∞ X

(2 − i)−(n+1) (z − i)n ,



n=0

[(2 − i)n − (1 − i)n ] (z − i)−(n+1) , |z − i| >



2 < |z − i| <



5,

5.

3.1.4 Propiedades Adicionales de las Series Diferenciaci´ on t´ ermino a t´ ermino Si una funci´ on f (z) est´ a representada con una serie de potencias en una regi´ on anular R, la serie que se obtiene por diferenciaci´ on t´ermino a t´ermino converge a f ′ (z) dentro de R. Este procedimiento puede repetirse un n´ umero indefinido de veces. Los siguientes ejemplos utilizan la propiedad de diferenciaci´ on t´ermino a t´ermino para encontrar desarrollos de Taylor. Ejemplo 3.7. Usando el desarrollo de Taylor de 1/z centrado en z0 = 1, obtenga el desarrollo de 1/z 2 centrado en el mismo punto. Soluci´ on. Primero determinemos el desarrollo de Taylor de 1/z centrado en z0 = 1, luego diferenciando t´ ermino a t´ ermino ´este desarrollo obtendremos el desarrollo de 1/z 2 centrado en z0 = 1. Se tiene que ∞

X 1 1 (−1)n (z − 1)n , = = z 1 + (z − 1) n=0 ahora, como d 1 1 = − 2, dz z z

|z| < 1,

88

3.2. SINGULARIDADES AISLADAS

entonces utilizando el desarrollo de Taylor de 1/z centrado en z0 = 1, podemos escribir: ∞ ∞ X X d d 1 1 n n (−1)n+1 n (z − 1)n−1 , = = − [(−1) (z − 1) ] = − z2 dz z dz {z } n=0 | n=0

|z| < 1.

|z|<1



Ejemplo 3.8. Calcular el desarrollo de MacLaurin de f (z) =

z . (1 − z)2

Soluci´ on. Se tiene que el desarrollo de MacLaurin de 1/(1 − z) est´ a dado por: ∞

X 1 zn, = (1 − z) n=0

Como

|z| < 1.

d 1 1 = , dz (1 − z) (1 − z)2

entonces utilizando el desarrollo de MacLaurin de 1/(1 − z), podemos escribir: ∞ ∞ X z 1 d n X n d =z =z z = nz , (1 − z)2 dz (1 − z) dz n=0 n=0 | {z }

|z| < 1.

|z|<1



3.2 Singularidades Aisladas Definici´ on 3.7 (Punto singular aislado). Se dice que z0 ∈ C es un punto singular aislado de una funci´ on f (z), si z0 es un punto singular de f (z) y existe una vecindad de z0 en todo punto de la cual f (z) es anal´ıtica excepto en z0 . Ejemplo 3.9. Cada punto z0 indicado es un punto singular aislado de la funci´ on dada. • f (z) =

1 , z

• f (z) =

z+1 , (z − 1)(z − i)

• f (z) = cot z,

z0 = 0 z0 = 1, z1 = i

zn = nπ, n = 0, ±1, ±2, . . . 

89

CAP´ITULO 3. SERIES DE POTENCIAS Y SINGULARIDADES AISLADAS

Ejemplo 3.10. El punto z0 = 0 no es un punto singular aislado de la funci´ on f (z) =

1 . sen (1/z) 

¿z0 = 0 es un punto de acumulaci´ on de los puntos singulares de f (z) = 1/sen (1/z)?

Ejercicio 3.2. Verificar lo afirmado en los Ejemplos 3.9 y 3.10. Definici´ on 3.8 (Parte principal). Sea z0 un punto singular aislado de f (z). Sea f (z) =

∞ X

n=0

an (z − z0 )n +

∞ X

n=1

bn (z − z0 )−n ,

0 < |z − z0 | < r0 ,

(3.4)

el desarrollo de Laurent de f (z) centrado en z0 v´alido en el anillo 0 < |z − z0 | < r0 . Se denomina parte principal de f (z) en z0 , a la parte de Laurent (3.4) que P∞ del desarrollo −n posee potencias negativas de (z − z0 ), esto es, n=1 bn (z − z0 ) .

Ejemplo 3.11. Determine la parte principal de la funci´ on f (z) =

1 z(z − 1)

en cada uno de sus puntos singulares. Soluci´ on. Los puntos singulares aislados de f (z) son z1 = 0 y z2 = 1. Como 1 −1 1 = + , z(z − 1) z (z − 1) el desarrollo de Laurent de f (z) centrado en z1 = 0 v´alido en 0 < |z| < 1, est´ a dado por: ∞

1 1 X 1 (−1)z n + (−1)z −1 , =− − = z(z − 1) (1 − z) z n=0

0 < |z| < 1,

por lo tanto, la parte principal de f (z) en z1 = 0 es: (−1)z −1 . Ahora, se tiene que 1 z(z − 1)

1 1 1 = − + =− + (z − 1)−1 z (z − 1) 1 + (z − 1) ∞ X (−1)n+1 (z − 1)n + (z − 1)−1 , = | {z } |z−1|>0 |n=0 {z } |z−1|<1

as´ı, el desarrollo de Laurent de f (z) centrado en z2 = 1 v´alido en 0 < |z − 1| < 1, est´ a dado por: ∞

X 1 = (−1)n+1 (z − 1)n + (z − 1)−1 , z(z − 1) n=0

0 < |z − 1| < 1;

por lo tanto, la parte principal de f (z) en z2 = 1 es (z − 1)−1 .



90

3.2. SINGULARIDADES AISLADAS

Los puntos singulares aislados se pueden caracterizar seg´ un la forma que adquiere la parte principal de la funci´ on en tales puntos. Se distinguen tres tipos de singularidades aisladas: polo de orden m, punto singular esencial y punto singular removible. A continuaci´ on se describen cada una de ellas.

3.2.1 Polo de Orden m Definici´ on 3.9. Sea z0 un punto singular aislado de f (z). Se dice que z0 es un polo de orden m de f (z), si la parte principal de f (z) en z0 tiene un n´ umero finito de t´erminos, esto es, el desarrollo de Laurent de f (z) centrado en z0 , v´alido en el anillo 0 < |z − z0 | < r0 , tiene la forma f (z) =

∞ X

n=0

an (z − z0 )n +

b1 b2 bm + + ··· + , (z − z0 ) (z − z0 )2 (z − z0 )m

donde bm 6= 0 y bm+1 = bm+2 = · · · = 0. Los polos de orden m = 1 se llaman polos simples. La definici´ on anterior nos indica que para determinar si z0 es un polo de f (z), se debe observar el desarrollo de Laurent de f (z) centrado en z0 ; pero, en general, este procedimiento no es pr´ actico. Existen otros procedimientos m´ as adecuados para verificar si un punto es o no un polo. El siguiente teorema nos da un procedimiento para determinar si un punto es o no un polo de una funci´ on, sin construir su serie de Laurent. Teorema 3.8. Si f (z) tiene un polo en z0 , entonces l´ımz→z0 |f (z)| = ∞. Demostraci´ on. Como z0 es un polo de orden m de f (z), el desarrollo de Laurent de f (z) centrado en z0 , v´ alido en el anillo 0 < |z − z0 | < r0 , tiene la forma f (z) =

∞ X

n=0

an (z − z0 )n +

b1 b2 bm + + ··· + , 2 (z − z0 ) (z − z0 ) (z − z0 )m

donde bm 6= 0. Multiplicando en ambos lados de la ecuaci´ on anterior por (z−z0 )m tenemos (z − z0 )m f (z) =

∞ X

n=0

an (z − z0 )n+m + b1 (z − z0 )m−1 + b2 (z − z0 )m−2 + · · · + bm ,

de lo cual se deduce que l´ım (z − z0 )m f (z) = bm 6= 0, ∞.

z→z0

(3.5)

Como l´ımz→z0 |(z − z0 )m | = 0, entonces por (3.5) se tiene que l´ımz→z0 |f (z)| = ∞. El teorema anterior no solo nos permite identificar si un punto es un polo, sino tambi´ en el orden del mismo. Bas´ andose en este teorema y, particularmente, en la ecuaci´ on (3.5), las siguientes reglas nos permiten identificar si un punto singular aislado z0 es un polo y, adem´as, calcular el orden del mismo. Regla I Si existe el l´ımz→z0 (z − z0 )m f (z) y si dicho l´ımite no es cero ni infinito, entonces z0 es un polo de orden m de f (z).

91

CAP´ITULO 3. SERIES DE POTENCIAS Y SINGULARIDADES AISLADAS

Regla II Si z0 es un polo es un polo de orden m de f (z), entonces ( 0, si n > m, n l´ım (z − z0 ) f (z) = z→z0 ∞, si n < m. 1 . Probar que 0 y 1 son polos simples de f (z). z(z − 1)

Ejemplo 3.12. Sea f (z) =

Soluci´ on. Utilicemos el Teorema 3.8 y las Reglas I y II, para determinar que los puntos 0 y 1 son polos simples de f (z). Se tiene que l´ım |f (z)| = ∞

z→0

y

l´ım |f (z)| = ∞,

z→1

luego, los puntos 0 y 1 son polos de f (z). Adem´ as,   si n > 1, 0, z n−1 n l´ım z f (z) = l´ım = −1, si n = 1, z→0 z→0 (z − 1)   ∞, si n < 1,

y

(z − 1)n−1 z→1 z

l´ım (z − 1)n f (z) = l´ım

z→1

  si n > 1, 0, = 1, si n = 1,   ∞, si n < 1.

Por lo tanto, los puntos 0 y 1 son polos simples de f (z).



Es com´ un encontrarse con problemas en los que se quiere determinar el orden de los polos de una funci´ on de la forma f (z) = p(z)/q(z), por ello les dedicaremos una atenci´ on especial. El siguiente teorema nos permite identificar si un punto es un polo, y adem´as su orden, para una funci´ on f (z) = p(z)/q(z). Teorema 3.9. Sea z0 ∈ C. Sea f (z) una funci´on tal que se puede escribir como f (z) =

p(z) , q(z)

donde p(z) y q(z) son anal´ıticas en z0 y p(z0 ) 6= 0. Entonces, z0 es un polo de orden m de f (z) si, y s´olo si q(z0 ) = q ′ (z0 ) = · · · = q (m−1) (z0 ) = 0 y q (m) (z0 ) 6= 0. Demostraci´ on. (⇒) Supongamos que z0 es un polo de orden m de f (z) y demostremos que q(z0 ) = q ′ (z0 ) = · · · = q (m−1) (z0 ) = 0 y q (m) (z0 ) 6= 0. Como z0 es un polo de orden m de f (z), el desarrollo de Laurent de f (z) centrado en z0 , v´alido en el anillo 0 < |z − z0 | < r0 , tiene la forma ∞

b1 b2 bm p(z) X an (z − z0 )n + = + + ··· + , f (z) = 2 q(z) (z − z0 ) (z − z0 ) (z − z0 )m n=0

92

3.2. SINGULARIDADES AISLADAS

donde bm 6= 0. Como q(z) es anal´ıtica en z0 , entonces posee desarrollo de Taylor centrado en z0 de la forma ∞ X q n (z0 ) (z − z0 )n . q(z) = n! n=0

As´ı, obtenemos que p(z) = =

!

! b b 1 m an (z − z0 )n + (z − z0 )n + ··· + n! (z − z0 ) (z − z0 )m n=0 n=0 ! ! ∞ m ∞ ∞ X X X X q n (z0 ) q n (z0 ) n n + an (z − z0 ) bk (z − z0 ) (z − z0 )n−k . n! n! n=0 n=0 n=0 ∞ X q n (z0 )

∞ X

k=1

Como p(z) es anal´ıtica en z0 , entonces la expresi´ on anterior es el desarrollo de Taylor de p(z) centrado z0 , por lo que no puede tener potencias negativas de (z − z0 ). Para que esto sea cierto, se debe cumplir que q(z0 ) = q ′ (z0 ) = · · · = q (m−1) (z0 ) = 0. Adem´ as, haciendo z = z0 en el desarrollo de Taylor de p(z), podemos escribir: 0 6= p(z0 ) = bm

q (m) (z0 ) , m!

de donde se deduce que q (m) (z0 ) 6= 0. (⇐) Supongamos que q(z0 ) = q ′ (z0 ) = · · · = q (m−1) (z0 ) = 0 y q (m) (z0 ) 6= 0, y demostremos que z0 es un polo de orden m de f (z). Como p(z) y q(z) son anal´ıticas en z0 con p(z0 ) 6= 0 y q(z0 ) = 0, entonces z0 es un punto singular aislado de f (z). Ahora utilizando el desarrollo de Taylor de q(z) centrado en z0 se tiene que ∞ X q n (z0 ) q (m) (z0 ) q(z) + (z − z0 )n−m , = (z − z0 )m m! n! n=m+1

por tanto, l´ım

z→z0

lo cual implica que m

q(z) q (m) (z0 ) 6= 0, ∞, = m (z − z0 ) m!

l´ım (z − z0 ) f (z) =

z→z0



  (z − z0 )m l´ım p(z) l´ım 6= 0, ∞. z→z0 z→z0 q(z)

Seg´ un la Regla I, lo anterior indica que z0 es un polo de orden m de f (z). Ejemplo 3.13. Verificar que todos los puntos singulares aislados de la funci´ on f (z) =

ez sen z

son polos simples. Soluci´ on. Se tiene que f (z) = p(z)/q(z), donde p(z) = ez y q(z) = sen z. Ahora, los puntos singulares aislados de f (z) son: zn = nπ, para n = 0, ±1, ±2, . . .. Como p(zn ) 6= 0, q(zn ) = 0 y q ′ (zn ) = cos(nπ) 6= 0, para todo n, entonces por el Teorema 3.9 todos los  puntos singulares de f (z) son polos simples.

93

CAP´ITULO 3. SERIES DE POTENCIAS Y SINGULARIDADES AISLADAS

Ejercicio 3.3. Verifique la existencia de los polos y el orden de los mismos para cada una de las funciones dadas. a) El punto z0 = 0 es un polo de orden m = 2 de la funci´ on f (z) =

1 . − 1)

z(ez

b) Los puntos z0 = 3i y z1 = −3i son polos simples de la funci´ on f (z) = c) El punto z0 = 2 es un polo simple de la funci´ on f (z) =

(z + 1) . (z 2 + 9)

z 2 − 2z + 3 . (z − 2)

d) El punto z0 = 0 es un polo de orden m = 3 de la funci´ on f (z) =

senh z . z4

3.2.2 Punto Singular Esencial Definici´ on 3.10. Sea z0 un punto singular aislado de f (z). Se dice que z0 es un punto singular esencial de f (z), si la parte principal de f (z) en z0 tiene un n´ umero infinito de t´erminos diferentes de cero. De la definici´ on de punto singular esencial se deduce que z0 es un punto singular esencial de f (z) si, y s´ olo si l´ımz→z0 f (z) no existe (ni finito ni infinito). Observe el siguiente ejemplo. Ejemplo 3.14. Verificar que z0 = 0 es un punto singular esencial de f (z) = e1/z . Soluci´ on. El desarrollo de Laurent de e1/z centrado en z0 = 0 es e1/z = 1 +

∞ X 1 −n z , n! n=1

|z| > 0.

Se observa que la parte principal de e1/z en z0 = 0 tiene un n´ umero infinito de t´erminos diferentes de cero; por lo tanto, z0 = 0 es un punto singular esencial de e1/z . Otra manera de ver que z0 = 0 es un punto singular esencial de e1/z , es probar que el l´ımite, l´ımz→0 e1/z , no existe. Si nos acercamos al origen por la recta y = 0, x > 0, obtenemos que l´ım e1/z = l´ım e1/x = ∞. z→0

x→0

Ahora, si nos acercamos al origen por la recta x = 0, y > 0, tenemos que e1/z = cos(1/y) − i sen (1/y) = e−i/y , es un n´ umero complejo de m´ odulo 1 para todo valor de y. Luego, l´ım e1/z = l´ım e−i/y 6= ∞.

z→0

y→0

Por lo tanto, el l´ımite l´ımz→0 e1/z no existe. En consecuencia, z0 = 0 es un punto singular  esencial de e1/z .

94

3.3. SERIES DE POTENCIAS Y SINGULARIDADES CON MATLAB

3.2.3 Punto Singular Removible Definici´ on 3.11. Sea z0 un punto singular aislado de f (z). Se dice que z0 es un punto singular removible de f (z), si todos los coeficientes de la parte principal de f (z) en z0 son cero. Si z0 es un punto singular removible de f (z), el desarrollo de Laurent de f (z) centrado en z0 , v´ alido en el anillo 0 < |z − z0 | < r0 , tiene la forma f (z) =

∞ X

n=0

an (z − z0 )n ,

de donde se deduce que l´ım f (z) = a0 .

z→z0

Por lo tanto, para determinar si un punto singular aislado z0 es un punto singular removible de f (z), basta con verificar que el l´ımz→z0 f (z) existe y es finito. Ejemplo 3.15. Verificar que z0 = 0 es un punto singular removible de la funci´ on f (z) = Soluci´ on. Se tiene que

sen z . z

sen z L′ H = l´ım cos z = 1, z→0 z→0 z l´ım

por tanto, z0 = 0 es un punto singular removible de f (z) =

sen z . z



3.3 Series de Potencias y Singularidades con M ATLAB En esta secci´ on tambi´ en usaremos la caja de herramientas de matem´atica simb´ olica (Symbolic Math Toolbox) de M ATLAB . En particular el comando symsum(expr,n,a,b) que permite evaluar la suma una la serie, donde expr define los t´erminos de una serie con respecto de z y n, y n var´ıa de a hasta b. Observe el siguiente ejemplo donde se calcula: ∞ X

zn =

n=0

1 , 1−z

|z| < 1.

✄ >> syms z n >> symsum( z ˆn , n , 0 , I n f ) ans = p i e c e w i s e ( [ 1 <= z , I n f ] , [ abs ( z ) < 1 , −1/(z − 1 ) ] )





En esta caso, la respuesta que da M ATLAB , piecewise([1 <= z, Inf], [abs(z) < 1, -1/(z - 1)]), indica que la serie no converge para |z| ≥ 1, pero si converge a 1/(1 − z) cuando |z| < 1. Veamos otros ejemplos. Seguidamente se calcula: •

∞ X zn

n=1

n

= −Log (1 − z),

|z| < 1

CAP´ITULO 3. SERIES DE POTENCIAS Y SINGULARIDADES AISLADAS

∞ X n n z = • 2n 2 n=1

• ✄

∞ X n(z + 2)n

n=0

2n

z z 2

=

2 , −1

2z + 4 , z2

95

|z| < 2 |z + 2| < 2

>> syms z n >> symsum( z ˆn/n , n , 1 , I n f ) ans = p i e c e w i s e ( [ 1 <= z , I n f ] , [ abs ( z ) <= 1 & z ˜= 1 , −l o g (1 − z ) ] ) >> symsum( ( n∗ z ˆn )/2ˆn , n , 1 , I n f ) ans = p i e c e w i s e ( [ abs ( z ) < 2 , z / ( 2 ∗ ( z /2 − 1 ) ˆ 2 ) ] ) >> symsum( ( n ∗( z+2)ˆn )/2ˆn , n , 1 , I n f ) ans = p i e c e w i s e ( [ abs ( z /2 + 1) < 1 , (2∗( z + 2 ) ) / z ˆ 2 ] )





Hemos visto que el comando symsum permite calcular a qu´e y en d´ onde converge una serie de potencias. Ahora utilicemos el comando taylor para determinar el desarrollo de Taylor de una funci´ on f (z). Con taylor(f,z,a) se calculan, por defecto, los cinco primeros t´ erminos (f (a) + f ′ (a)(z − a) + f ′′ (a)(z − a)2 + f ′′′ (a)(z − a)3 + f (iv) (a)(z − a)4 ) del desarrollo de Taylor de f (z) al rededor de z0 = a. El desarrollo de Maclaurin de f (z) se calcula usando taylor(f). Observe el siguiente ejemplo donde se calculan los cinco primeros t´ erminos de los desarrollos de Maclaurin de ez , cos z y 1/(1 − z). ✄

>> syms z >> t a y l o r ( exp ( z ) ) ans = z ˆ5/120 + z ˆ4/24 + z ˆ3/6 + z ˆ2/2 + z + 1 >> t a y l o r ( c o s ( z ) ) ans = z ˆ4/24 − z ˆ2/2 + 1 >> t a y l o r (1/(1− z ) ) ans = z ˆ5 + z ˆ4 + z ˆ3 + z ˆ2 + z + 1





La opci´ on taylor(f,z,a,’Order’,m) permite calcular los m primeros t´erminos del desarrollo de Taylor de f (z) al rededor de z0 = a. Observe el siguiente ejemplo donde se calculan los 8 primeros t´ erminos del desarrollo de Taylor de f (z) = (z + 1)/(z − 5)6 al rededor de z0 = −1. ✄

>> syms z >> t a y l o r ( ( z +1)/(z −5)ˆ6 ,z , −1 , ’ Order ’ , 8 ) ans = z /46656 + ( z + 1)ˆ2/46656 + (7∗( z + 1)ˆ3)/559872 + (7∗( z + 1)ˆ4)/1259712 + (7∗( z + 1)ˆ5)/3359232 + (7∗( z + 1)ˆ6)/10077696 + (77∗( z + 1)ˆ7)/362797056 + 1/46656





M ATLAB no tiene comandos que permiten hallar directamente el desarrollo de Laurent. Para ello se deben utilizar desarrollos de Taylor conocidos. Para funciones particulares como las funciones racionales propias, f (z) =

b1 z m + b2 z m−1 + b3 z m−2 + · · · + bm , a1 z n + a2 z n−1 + a3 z n−2 + · · · + an

con n > m, hemos dise˜ nado la funci´ on LaurentRacional (ver Programa 3.1), que permite hallar el desarrollo de Laurent de f (z) alrededor de uno de sus puntos singulares z0 ∈ C.

96

3.3. SERIES DE POTENCIAS Y SINGULARIDADES CON MATLAB

Programa 3.1. Funci´ on LaurentRacional.m f u n c t i o n [ d s l ]= L a u r e n t R a c i o n a l ( b , a , z0 ,m) %L a u r e n t R a c i o n a l H a l l a e l d e s a r r o l l o de L a u r e n t de l a f u n c i o ´n % r a c i o n a l propia % b ( 1 ) z ˆm + b ( 2 ) z ˆ(m−1) +...+ b (m) % f ( z ) = −−−−−−−−−−−−−−−−−−−−−−−−−−−−−−−−−−−− % a ( 1 ) z ˆn + a ( 2 ) z ˆ( n−1) +...+ a ( n ) % a l rededor de uno de s u s pu ntos s i n g u l a r e s z0 syms z ; d s l = sym ( 0 ) ; d s l 1 = sym ( 0 ) ; [ r , p , k] = r e sidu e (b , a ) ; np = l e n g t h ( r ) ; f i = sym ( z e r o s ( s i z e ( r ) ) ) ; f i ( 1 ) = sym ( r ( 1 ) ) / ( z−sym ( p ( 1 ) ) ) ; mr = 1 ; i f np>1 f o r t =2:np i f p ( t ) == p ( t −1) f i ( t ) = sym ( r ( t ) ) / ( z−sym ( p ( t ) ) ) ˆ ( sym (mr+1)); mr = mr+1; else f i ( t ) = sym ( r ( t ) ) / ( z−sym ( p ( t ) ) ) ; mr = 1 ; end end end f o r t =1:np i f sym ( p ( t ) ) == z0 dsl = dsl + f i ( t ); else dsl1 = dsl1 + f i ( t ) ; end end d s l = d s l + t a y l o r ( dsl1 , z , z0 , ’ Order ’ ,m) ; i f ˜ isempty (k ) d s l = d s l + sym ( k ) ; end end

Observe el siguiente ejemplo donde se calcula el desarrollo de Laurent de f (z) =

z+1 z+1 = 3 (z − 1)2 (z + 3) z + z 2 − 5z + 3

alrededor de z0 = −3, usando la funci´ on LaurentRacional. ✄

>> b = [1 1 ] ; a = [1 1 −5 3 ] ; >> d s l = L a u r e n t R a c i o n a l ( b , a , −3 ,5) dsl = z /128 − 1/(8∗( z + 3 ) ) + ( z + 3)ˆ2/256 + (3∗( z + 3)ˆ3)/2048 + ( z + 3)ˆ4/2048 + 3/128





De esta forma, el desarrollo de Laurent de f (z) alrededor de z0 = −3 obtenido previamente, se expresa como: f (z) = −

1 (z + 3) (z + 3)2 3 (z + 3)3 (z + 3)4 + + + + + ··· 8 (z + 3) 128 256 2048 2048

CAP´ITULO 3. SERIES DE POTENCIAS Y SINGULARIDADES AISLADAS

97

Por otra parte, el comando limit puede usarse para determinar la naturaleza de una singularidad aislada, es decir, podemos identificar si es un polo, un punto singular removible o un punto singular esencial. A continuaci´ on verificamos cada una de las siguientes afirmaciones: a) El punto z0 = 0 es un polo de orden m = 2 de la funci´ on f (z) = 1/(z(ez − 1)). b) El punto z0 = i es un punto singular esencial de f (z) = e1/(z−i) . c) El punto z0 = π/2 es un punto singular removible de f (z) = cos(z)/(2z − π). a) Se tiene ✄ >> syms z >> f ( z ) = 1/( z ∗( exp ( z ) −1)) f (z) = 1/( z ∗( exp ( z ) − 1 ) ) >> l i m i t ( abs ( f ) , z , 0 ) ans ( z ) = Inf





Esto nos indica que z0 = 0 es un polo de f (z) = 1/(z(ez − 1)). Ahora, ✄ >> l i m i t ( z ˆ2∗ f , z , 0 ) ans ( z ) = 1





indica que z0 = 0 es un polo de orden 2 de f (z) = 1/(z(ez − 1)). b) Se tiene ✄ >> syms z >> f ( z ) = exp ( 1 / ( z−i ) ) f (z) = exp ( 1 / ( z − i ) ) >> l i m i t ( f , z , i ) ans ( z ) = NaN





En este caso, el s´ımbolo NaN indica que el l´ımite no existe. Por lo tanto, z0 = i es un punto singular esencial de f (z) = e1/(z−i) . c) Se tiene ✄ >> syms z >> f ( z ) = c o s ( z ) / ( 2 ∗ z−p i ) f (z) = −c o s ( z ) / ( p i − 2∗ z ) >> l i m i t ( f , z , p i /2) ans ( z ) = −1/2



Por lo tanto, z0 = π/2 es un punto singular removible de f (z) = cos(z)/(2z − π).



98

3.4. PROBLEMAS RESUELTOS

3.4 Problemas Resueltos Problema 3.1. Determine la regi´ on de convergencia de la serie de potencias ∞ h X

n=0

i (−1)n − 2−(n+1) (z − 1)n .

Soluci´ on. Es claro que la serie de potencias dada se puede escribir como la resta de dos series, ∞ h X n=0

n

−(n+1)

(−1) − 2

i

n

(z − 1) =

∞ X

n

n

(−1) (z − 1) −

n=0

∞ X

n=0

2−(n+1) (z − 1)n .

De esta forma, la regi´ on de convergencia de la serie de potencias P inicial es la intersecn n ci´ onPde las regiones de convergencia de las series de potencias: a) ∞ n=0 (−1) (z − 1) y ∞ −(n+1) (z − 1)n . b) n=0 2 P n n a) Los coeficientes y el centro de la serie ∞ n=0 (−1) (z − 1) son: an = (−1)n

y

z0 = 1.

As´ı, aplicando el criterio de la ra´ız obtenemos: α = l´ım

n→∞

p n

|an | = l´ım

n→∞

p n

|(−1)n | = 1.

P n n Luego, |z − 1| < 1 es el disco de convergencia de la serie ∞ n=0 (−1) (z − 1) . P −(n+1) (z − 1)n son: b = 2−(n+1) y z = 1. b) Los coeficientes y el centro de la serie ∞ n 0 n=0 2 As´ı, aplicando el criterio del cociente obtenemos: 1 2−(n+2) |bn+1 | = l´ım −(n+1) = . n→∞ 2 n→∞ |bn | 2

α = l´ım

P −(n+1) (z − 1)n . Luego, |z − 1| < 2 es el disco de convergencia de la serie ∞ n=0 2   P n −(n+1) (z − 1)n es el Por lo tanto, la regi´ on de convergencia de la serie ∞ n=0 (−1) − 2 disco |z − 1| < 1.  Problema 3.2. Determine el desarrollo de Taylor de f (z) =

2z + 1 , z(2z − 1)

centrado en z0 = − 12 . Soluci´ on. Expresemos a f (z) en fracciones simples: f (z) =

2 z−

1 2

1 − . z

CAP´ITULO 3. SERIES DE POTENCIAS Y SINGULARIDADES AISLADAS

99

Hallemos el desarrollo de Taylor de cada uno de los tiene: ! 2 −1 =2 z − 21 1 − (z + 12 )  ∞  X 1 n = (−2) z+ , 2

sumandos, centrado en z0 = − 12 . Se

n=0

z +

! −1 1 − 2(z + 12 )   ∞ X 1 n n 2 z+ , = (−2) 2

1 =2 z

n=0

1 < 1, 2

z +

1 1 < . 2 2

De esta forma, considerando los desarrollos de Taylor hallados previamente, obtenemos a dado por: que el desarrollo de Taylor de f (z) centrado en z0 = − 12 , est´ ∞  X

   ∞ X 1 n 1 n n f (z) = (−2) z+ 2 z+ − (−2) 2 2 n=0 n=0 {z } | {z } | |z+ 21 |<1 |z+ 21 |< 21   ∞ X 1 n 1 1 n+1 , z + < . (2 − 2) z + = 2 2 2 n=0



Problema 3.3. Determine el desarrollo de Maclaurin de f (z) = Soluci´ on. escribir:

ez . 1−z

Considerando el desarrollo de Macalurin de ez y el de 1/(1 − z), podemos

 1 f (z) = e 1−z ! ! ∞ ∞ X 1 X n n = z z n! n=0 n=0 | {z } | {z } z



|z|<∞

|z|<1

1 = 1+ 1+ 1!



1 1 z+ 1+ + 1! 2!

n ∞ X X 1 = k!

!

zn,



n=0

k=0







1 1 1 z + 1+ + + 1! 2! 3! 2



z3 + · · ·

|z| < 1. 

100

3.4. PROBLEMAS RESUELTOS

Problema 3.4. Determine el desarrollo de Taylor de f (z) =

i , z2

centrado en z0 = −i. Soluci´ on. Primero hallemos el desarrollo de Taylor de g(z) = i/z, centrado en z0 = −i. Se tiene que   −1 i =i g(z) = z i − (z + i) ∞ X −1 i−n (z + i)n = =− 1 − i−1 (z + i) | n=0 {z } = −

∞ X

|i−1 (z+i)|<1

i−n (z + i)n ,

n=0

|z + i| < 1.

Como f (z) = −g′ (z), entonces el desarrollo de Taylor de f (z) centrado en z0 = −i, est´ a dado por: # " ∞ ∞ X X d −n n i−n n (z + i)n−1 , |z + i| < 1. f (z) = − i (z + i) = − dz n=0

n=0



Problema 3.5. Determine todos los posibles desarrollos de Laurent de f (z) =

1 , z(z + i)

centrados en z0 = 1. Soluci´ on. Primero expresemos a f (z) en fracciones simples: f (z) =

i i − . z+i z

La funci´ on f (z) dada tiene tres desarrollos de Laurent centrados en z√ 0 = 1, cuyos dominios √ de validez son los siguientes anillos: a) |z − 1| < 1, b) 1 < |z − 1| < 2, y c) |z − 1| > 2. a) El desarrollo de Laurent de f (z) centrado en z0 = 1, v´alido en |z − 1| < 1, est´ a dado por: f (z) = = =

=

i i − z+i z    1 1 i −i 1 + i 1 + (1 + i)−1 (z − 1) 1 + (z − 1) ∞ ∞ X i X n −n n (−1) (1 + i) (z − 1) − i (−1)n (z − 1)n 1+i |n=0 {z } {z } |n=0

∞ X

n=0

√ |z−1|< 2

h i i(−1)n (1 + i)−(n+1) − 1 (z − 1)n ,

|z−1|<1

|z − 1| < 1.

101

CAP´ITULO 3. SERIES DE POTENCIAS Y SINGULARIDADES AISLADAS

b) El desarrollo de Laurent de f (z) centrado en z0 = 1, v´alido en 1 < |z − 1| < dado por:



2, est´ a

i i − z+i z    1 1 i i = − 1 + i 1 + (1 + i)−1 (z − 1) z − 1 1 + (z − 1)−1 ∞ ∞ X X n −(n+1) n i(−1) (1 + i) (z − 1) − = i(−1)n (z − 1)−(n+1)

f (z) =

=

|n=0 ∞ X

{z

}

√ |z−1|< 2

n

−(n+1)

i(−1) (1 + i)

n=0

n

(z − 1) +

|n=0

∞ X

n=0

{z

|z−1|>1

}

i(−1)n+1 (z − 1)−(n+1) , 1 < |z − 1| <

c) El desarrollo de Laurent de f (z) centrado en z0 = 1, v´alido en |z − 1| > por:



2.

√ 2, est´ a dado

i i − z+i z    1 1 i i − = z − 1 1 + (1 + i)(z − 1)−1 z − 1 1 + (z − 1)−1 ∞ ∞ X X n n −(n+1) i(−1)n (z − 1)−(n+1) − i(−1) (1 + i) (z − 1) =

f (z) =

n=0

=

|

∞ X

n=0

{z

√ |z−1|> 2

}

|n=0

i(−1)n [(1 + i)n − 1] (z − 1)−(n+1) ,

{z

|z−1|>1

|z − 1| >



2.

}



Problema 3.6. Determine todos los posibles desarrollos de Laurent, centrados en z0 = 0, de la funci´ on 1 f (z) = , z (z − 1) (z − 2) Soluci´ on. Primero expresemos a f (z) en fracciones simples: f (z) =

1/2 1 1/2 − + . z−2 z−1 z

La funci´ on f (z) tiene tres desarrollos de Laurent centrados en z0 = 0, cuyos dominios de validez son los siguientes anillos: a) 0 < |z| < 1, b) 1 < |z| < 2, y c) |z| > 2.

102

3.4. PROBLEMAS RESUELTOS

a) El desarrollo de Laurent de f (z) centrado en z0 = 0, v´alido en 0 < |z| < 1, est´ a dado por: 1 1/2 1/2 − + z − 2 z−1  z 1 1 1 1 + z −1 + = − 4 1 − 2−1 z 1−z 2 ∞ ∞ 1 X −n n X n 1 −1 = − 2 z + z + z 4 |2 {z } n=0 n=0 | {z } | {z } |z|>0

f (z) =

=

∞ h X

n=0

|z|<2

|z|<1

i

1 − 2−(n+2) z n +

1 −1 z , 2

0 < |z| < 1.

b) El desarrollo de Laurent de f (z) centrado en z0 = 0, v´alido en |z| > 2, est´ a dado por: 1 1/2 1/2 − + z − 2 z−1  z   1 1 1 1 −1 + z −1 = − −z −1 −1 4 1−2 z 1−z 2 ∞ ∞ 1 X −n n X −(n+1) 1 −1 = − 2 z − z + z 4 |2 {z } n=0 n=0 | {z } | {z } |z|>0

f (z) =

=

∞ X

|z|<2

|z|>1 ∞ X

(−1)2−(n+2) z n +

(−1)z −(n+1) +

n=0

n=0

1 −1 z , 2

1 < |z| < 2.

c) El desarrollo de Laurent de f (z) centrado en z0 = 0, v´alido en 1 < |z| < 2, est´ a dado por: 1/2 1 1/2 − + z−2 z−1 z     1 1 1 z −1 −1 −z + z −1 = −1 −1 2 1 − 2z 1−z 2 ∞ ∞ X X 1 2n−1 z −(n+1) − = z −(n+1) + z −1 |2 {z } n=0 | {z } |n=0 {z } |z|>0

f (z) =

=

∞ X 

n=0

|z|>2

|z|>1

 1 2n−1 − 1 z −(n+1) + z −1 , 2

|z| > 2.



103

CAP´ITULO 3. SERIES DE POTENCIAS Y SINGULARIDADES AISLADAS

Problema 3.7. Determine y clasifique las singularidades aisladas de la funci´ on f (z) =

1 (z +

2)3 e1/(z+2)3

.

Soluci´ on. El u ´ nico punto singular de f (z) es z0 = −2, el cual es un punto singular esencial. En efecto, se tiene que el desarrollo de Laurent de f (z) centrado en z0 = −2, est´ a dado por: 3

f (z) = (z + 2)−3 e−1/(z+2) ∞ X (−1)n (z + 2)−3n = (z + 2)−3 n! n=0

=

∞ X (−1)n

n!

n=0

(z + 2)−3(n+1) ,

|z + 2| > 0.

Por lo que la parte principal de f (z) en z0 = −2 posee infinitos t´erminos distintos de cero;  por lo tanto, z0 = −2 es un punto singular esencial de f (z). Problema 3.8. Determine y clasifique las singularidades aisladas de la funci´ on f (z) =

sen z . z5

Soluci´ on. El u ´ nico punto singular de f (z) es z0 = 0, el cual es un polo de orden 4. En efecto, f (z) se puede escribir como p(z) , q(z)

f (z) = donde p(z) =

( sen z z

1,

, z 6= 0,

y

z = 0,

q(z) = z 4 .

Adem´ as, se tiene que: p(z) y q(z) son anal´ıticas en z0 = 0, p(0) = 1 6= 0, q(0) = q ′ (0) = q ′′ (0) = q ′′′ (0) = 0 y q (iv) (0) = 24 6= 0. Entonces, z0 = 0 es un polo de orden 4 de f (z).  Problema 3.9. Determine y clasifique las singularidades aisladas de la funci´ on f (z) =

1 z 2 ez 2

.

Soluci´ on. El u ´ nico punto singular de f (z) es z0 = 0, el cual es un polo de orden 2. En efecto, f (z) se puede escribir como f (z) = 2

p(z) , q(z)

as, se tiene que: p(z) y q(z) son anal´ıticas en z0 = 0, donde p(z) = e−z y q(z) = z 2 . Adem´ ′ ′′ p(0) = 1 6= 0, q(0) = q (0) = 0 y q (0) = 2 6= 0. Por lo tanto, z0 = 0 es un polo de orden 2  de f (z).

104

3.5. PROBLEMAS PROPUESTOS

3.5 Problemas Propuestos 3.1. Determine el dominio de convergencia de cada una de las siguientes series: a) b) c) d) e) f)

∞ X zn

n=1 ∞ X

n=0 ∞ X

n=1 ∞ X

n=1 ∞ X

n=0 ∞ X

n

zn n!

nn z n n n z 2n  −n  2 + (−i)n (z + i)n n

z2

n=0

∞ X n! n g) z nn

h) i) j) k) l)

n=1 ∞ X

n=0 ∞ X

n=0 ∞ X

n=1 ∞ X

n=0 ∞ X

cos(in)z n an z n , a ∈ R (z − 1)−n 2n n3 n(z + 2)n 2n

(−1)n−1 n z n n=1

3.2. Encuentre todas las representaciones en serie de potencias de (z − z0 ) para cada una de las siguientes funciones, seg´ un el centro z0 dado. a) f (z) = 1/z, z0 = 2i. 1 , z0 = 2. z−3 1 c) f (z) = 2 , z0 = 0. z +1 1 , z0 = 0. d) f (z) = 2 (z + 1)2 z e) f (z) = , z0 = −1. 3z − z 2 − 2 b) f (z) =

z+1 , z0 = 1. (z − 1)2 (z + 3) cos z g) f (z) = 2 , z0 = 0. z   1 3 h) f (z) = z cos , z0 = 2. z−2 sen (2z) , z0 = −1. i) f (z) = (z + 1)3 f) f (z) =

j) f (z) = (1 + z + z 2 )e1/(z−1) , z0 = 1.

3.3. Determine y clasifique todas las singularidades aisladas de las siguientes funciones complejas. 1 . z 2 (z 2 − 4z + 5) z+i b) f (z) = 3 . z + z 2 − 8z − 12 z−1 c) f (z) = 4 . z − z 2 (1 + i) + i cos(z − 1) d) f (z) = 4 . z − z 2 (1 + i) + i a) f (z) =

e) f (z) =

cos z . z2

f) f (z) = z 3 e1/z . g) f (z) =

z . sen z

h) f (z) =

ez z(1 − e−z )

4 Integraci´ on Compleja Es de notar que en los cursos de c´ alculo elemental, particularmente, en el tema de integraci´ on de funciones definidas de R en R, el concepto de integral tiene la siguiente inRb terpretaci´ on geom´etrica: el valor de la integral a h(x) dx corresponde al a ´rea comprendida entre el eje real y la gr´ afica de la funci´on h(x) en el intervalo [a, b]. Desafortunadamente, esta interpretaci´ on geom´etrica a trav´ es de a´reas no se aplica para funciones de variable compleja. Por ello se hace necesario interpretar de forma diferente la integral de una funci´ on de variable compleja. En s´ı, la integraci´ on de funciones de variable compleja es una herramienta esencial para el desarrollo te´ orico de las ideas del c´ alculo operacional, en particular, el estudio de las transformadas de Laplace, Fourier y z, que son temas indispensables para la compresi´ on de ciertos conceptos estudiados en Ingenier´ıa El´ectrica. En este cap´ıtulo se describen los conceptos b´asicos de la integraci´ on compleja, comenzando con la integral definida, pasando luego por la integral de l´ınea y la primitiva de una funci´ on, para finalizar con la teor´ıa de residuos.

4.1 Integral Definida Definici´ on 4.1. Sea F (t) una funci´ on de variable real con valores complejos definida como F (t) = U (t) + i V (t), donde U : [a, b] → R y V : [a, b] → R son funciones continuas a trozos definidas en el intervalo acotado y cerrado a ≤ t ≤ b. Bajo estas condiciones, la funci´ on F es continua a trozos y la integral definida de F (t) en el intervalo a ≤ t ≤ b se define como: Z

b

F (t) dt =

a

Z

b

U (t) dt + i a

Z

b

V (t) dt a

y se dice que F (t) es integrable en [a, b].

Ejemplo 4.1. Calcular la integral Z

π/4

eit dt.

0

Soluci´ on. Como eit = cos t + i sen t, se tiene que Z

0

π/4

it

e dt =

Z

π/4

cos t dt + i 0

Z

π/4 0

105

√ 2− 2 2 +i . sen t dt = 2 2 √



106

´ DE L´INEA 4.2. INTEGRACION

Propiedades de la integral definida Sean F (t) = U (t) + i V (t), F1 (t) = U1 (t) + i V1 (t) y F2 (t) = U2 (t) + i V2 (t), integrables en [a, b]. A partir de la definici´ on de integral definida se deducen f´acilmente las siguientes propiedades:  Z b Z b Re [F (t)] dt. F (t) dt = i) Re a

a

ii) Im

Z

a

iii)

Z

b

Z

b

b



F (t) dt =

c F (t) dt = c

a

a

iv)

Z

Z

b

Im [F (t)] dt. a

b

F (t) dt, para todo c ∈ C.

[F1 (t) + F2 (t)] dt =

a

Z

b

F1 (t) dt + a

Z

b

F2 (t) dt. a

Z b Z b v) F (t) dt ≤ |F (t)| dt. a

a

Ejercicio 4.1. Demostrar las propiedades de la integral definida.

4.2 Integraci´ on de L´ınea 4.2.1 Contornos A continuaci´ on presentamos conjuntos de puntos muy particulares denominados contornos que nos permitir´ an estudiar la integral de una funci´ on de variable compleja. Definici´ on 4.2 (Curva). Una curva C es un conjunto de puntos z = x + i y en el plano complejo tales que x = x(t), y = y(t) (a ≤ t ≤ b), donde x(t) y y(t) son funciones continuas en el intervalo [a, b]. Los puntos de C se pueden describir mediante la funci´ on continua z(t) = x(t) + i y(t), a ≤ t ≤ b denominada parametrizaci´on de C (ver Figura 4.1). y

C α = z(a)

β = z(b)

b

b

b

z(t)

b

b

a

t

b

b x

Figura 4.1. Representaci´ on gr´afica de una curva C

107

´ COMPLEJA CAP´ITULO 4. INTEGRACION

En la Figura 4.1 se aprecia la representaci´ on gr´afica de una curva. El valor α = z(a) se denomina extremo inicial de C y β = z(b) extremo final. Ahora, si x(t) y y(t) son diferenciables, entonces la parametrizaci´ on z(t) = x(t) + i y(t) tambi´ en es diferenciable, cuya derivada est´ a dada por z ′ (t) = x′ (t) + i y ′ (t),

a ≤ t ≤ b.

Definici´ on 4.3 (Curva suave). Una curva C se llama curva suave, si la derivada de la parametrizaci´ on z ′ (t) existe, es continua y nunca se hace cero en el intervalo a ≤ t ≤ b. Ejemplo 4.2. A continuaci´ on se muestran las gr´aficas de las curvas C1 y C2 , y sus respectivas parametrizaciones z1 (t) y z2 (t). La curva C1 es suave, en cambio la curva C2 no lo es (se deja al lector verificar esta aseveraci´ on). y 1

−1

y 1

C1

1

x

−1

C2

1

x

−1 z1 (t) = eit ,

( −t + i(1 + t), z2 (t) = −t + i(1 − t),

0 ≤ t ≤ 2π

−1 ≤ t ≤ 0, 0 < t ≤ 1. 

Definici´ on 4.4 (Contorno). Un contorno o curva suave a tramos, es una curva que consta de un n´ umero finito de curvas suaves unidas por sus extremos. Ejemplo 4.3. Seguidamente se aprecia una representaci´ on gr´afica de un contorno conformado por seis curvas suaves C1 , C2 , . . . , C6 . C2 C3

C1

C4 C6

C5 

Definici´ on 4.5 (Contorno cerrado simple). Se dice que C es un contorno cerrado simple, si C es un contorno y z(a) = z(b) y z(t1 ) 6= z(t2 ), para todo t1 6= t2 ∈ (a, b).

108

´ DE L´INEA 4.2. INTEGRACION

Ejemplo 4.4. En la siguiente gr´afica se muestran dos curvas C1 y C2 . La curva C1 es un contorno cerrado simple, en cambio C2 no lo es. C1

C2

z2 (a) = z2 (b)

z2 (t1 ) = z2 (t2 ) z1 (a) = z1 (b) 

Observaci´ on 4.1. A todo contorno C representado por la ecuaci´ on z(t) = x(t) + i y(t),

a≤t≤b

se le asocia el contorno −C, cuya parametrizaci´ on est´ a definida por la ecuaci´ on z = z(−t), donde −b ≤ t ≤ −a. Gr´aficamente, el contorno −C es el mismo contorno C pero recorrido en sentido contrario.

4.2.2 Integral de L´ınea Definici´ on 4.6. Sean f (z) una funci´ on y C un contorno representado por la ecuaci´ on z(t) = x(t) + i y(t),

a ≤ t ≤ b,

con extremo inicial α = z(a) y extremo final β = z(b). Supongamos que f (z) = u(x, y)+ i v(x, y) es continua a trozos en C, es decir, las partes real e imaginaria, u(x(t), y(t)) y v(x(t), y(t)), de f (z(t)) son funciones de t continuas a trozos. Bajo estas condiciones, se define la integral de l´ınea de f (z) a lo largo de C como: Z

f (z) dz =

Z

b

f (z(t)) z ′ (t) dt,

a

C

donde z ′ (t) = x′ (t) + i y ′ (t). Propiedades de la Integral de L´ınea Sean f (z) y g(z) funciones de variable compleja continuas a trozos sobre un contorno C con parametrizaci´ on z(t) = x(t) + i y(t), a ≤ t ≤ b. A partir de la definici´ on de integral de l´ınea se deducen f´acilmente las siguientes propiedades. Z Z f (z) dz, para k ∈ C. k f (z) dz = k i) C

ii)

Z

C

C

[f (z) + g(z)] dz =

Z

C

f (z) dz +

Z

C

g(z) dz.

109

´ COMPLEJA CAP´ITULO 4. INTEGRACION

iii)

Z

f (z) dz =

Z

−a

−b

−C

f (z(−t)) z ′ (−t) dt = −

Z

f (z) dz. C

iv) Si C consta de una curva C1 desde α1 hasta β1 y de la curva C2 desde α2 hasta β2 , donde α2 = β1 , se cumple: Z Z Z f (z) dz. f (z) dz + f (z) dz = C2

C1

C

Z Z b v) f (z) dz ≤ |f (z(t)) z ′ (t)| dt. C

a

Ejercicio 4.2. Demostrar las propiedades de la integral de l´ınea. Ejemplo 4.5. Calcular la integral

Z

|z|=1

1 dz, z

donde |z| = 1 es la circunferencia de centro 0 y radio 1, recorrida en sentido positivo. Soluci´ on. Una parametrizaci´ on de la circunferencia |z| = 1 es: z(t) = eit = cos t + i sen t, As´ı,

Z

1 dz = z

|z|=1

Z

2π 0

0 ≤ t ≤ 2π.

1 it ie dt = i eit

Z



dt = 2πi. 0

 Ejemplo 4.6. Calcular la integral

Z

C

(z − i) dz,

donde C es el contorno descrito por la ecuaci´ on ( −t + i(1 + t), −1 ≤ t ≤ 0, z(t) = −t + i(1 − t), 0 < t ≤ 1. Soluci´ on. Utilizando la propiedad iv) de la integral de l´ınea, podemos escribir: Z

C

(z − i) dz =

Z

0

−1

(−t + i(1 + t) − i) (−1 + i) dt +

= (−1 + i)

Z

2

= (−1 + i)

0

t(−1 + i) dt + (−1 − i)

−1 Z 0

−1

2

t dt + (−1 − i)

Z

0

1

Z

1

(−t + i(1 − t) − i) (−1 − i) dt

0

Z

1 0

t(−1 − i) dt

t dt = −2i ·

−1 1 + 2i · = 2i. 2 2 

110

4.3. TEOREMA DE CAUCHY-GOURSAT

4.3 Teorema de Cauchy-Goursat El siguiente resultado se conoce como Teorema de Cauchy-Goursat. El nombre de Cauchy es en honor al matem´ atico franc´es Augustin Louis Cauchy Mel (1789-1857), quien descrubri´ o el teorema en 1825, y el nombre Goursat es en honor al matem´atico franc´es Edouard Goursat (1858-1936), quien dio una demostraci´ on que no requer´ıa de la hip´ otesis de que la derivada de la funci´ on fuera continua. La demostraci´ on del Teorema de Cauchyon dada por Goursat se puede ver en [8] y es esencialmente la misma que la demostraci´ Goursat. Teorema 4.1 (Teorema de Cauchy-Goursat). Sea C un contorno cerrado simple. Sea f (z) una funci´on anal´ıtica sobre y en el interior de C. Entonces, Z f (z) dz = 0. C

El Teorema de Cauchy-Goursat es uno de los resultados m´ as importantes del an´alisis complejo. Desde el punto de vista pr´actico, este teorema puede ahorrar una gran cantidad de trabajo al realizar cierto tipo de integraciones. Por ejemplo, integrales como Z

sen z dz, C

Z

cosh z dz C

y

Z

ez dz C

deben anularse para cualquier contorno cerrado simple C. En todos estos casos, el integrando es una funci´ on entera. Ahora, desde el punto de vista te´ orico, el Teorema de Cauchy-Goursat permite introducir el concepto de primitiva de una funci´ on (lo cual se estudiar´ a m´ as adelante), entre otros conceptos. R Note que la direcci´ on de integraci´ on en la integral C f (z) dz = 0 no afecta el resultado, pues Z Z f (z) dz. f (z) dz = − C

−C

En el siguiente ejemplo se verifica la validez del Teorema de Cauchy-Goursat. Ejemplo 4.7. Verifique que Z

z n dz = 0, C

donde n es un entero positivo y C es la circunferencia |z| = r, con r > 0.

Soluci´ on. Como n > 0, la funci´ on f (z) = z n es entera, luego por el Teorema de CauchyGoursat Z z n dz = 0. C

Veamos que esto es efectivamente cierto. Una parametrizaci´ on de |z| = r es: z(t) = r eit ,

0 ≤ t ≤ 2π.

111

´ COMPLEJA CAP´ITULO 4. INTEGRACION

Luego, Z

n



it n

it

n+1

Z



(r e ) (ir e ) dt = ir e(n+1)it dt 0 0 Z 2π = ir n+1 [cos((n + 1)t) + i sen ((n + 1)t)] dt 0   sen ((n + 1)t) − i cos((n + 1)t) 2π = ir n+1 = 0. (n + 1) 0

z dz =

C

Z

As´ı, hemos verificado el Teorema de Cauchy-Goursat para un caso particular.



4.3.1 Extensi´ on del Teorema de Cauchy-Goursat Pasemos ahora a describir la extensi´ on del Teorema de Cauchy-Goursat. Para ello necesitamos definir algunos dominios muy particulares. Definici´ on 4.7 (Dominio simplemente conexo). Un dominio D se dice simplemente conexo si todo contorno cerrado simple dentro del mismo encierra s´ olo puntos de D. Definici´ on 4.8 (Dominio multiplemente conexo). Un dominio D se dice multiplemente conexo si no es simplemente conexo. Ejemplo 4.8. En la siguiente gr´afica se muestran dos dominios D1 y D2 , y tres contornos cerrados simples C, C1 y C2 . El Dominio D1 es simplemente conexo, en cambio D2 es multiplemente conexo.

D1

D2 C1

C

C2

 El siguiente teorema es la extensi´ on del Teorema de Cauchy-Goursat para dominios simplemente conexos. La demostraci´ on de esta teorema se obtiene de forma inmediata al emplear el Teorema de Cauchy-Goursat. Teorema 4.2. Si f (z) es anal´ıtica en un dominio simplemente conexo D, entonces para todo contorno cerrado simple C, dentro de D, se cumple Z f (z) dz = 0. C

112

4.3. TEOREMA DE CAUCHY-GOURSAT

Veamos ahora un ejemplo de aplicaci´ on del Teorema de Cauchy-Goursat el cual nos permitir´ a extenderlo para dominios multiplemente conexos. Ejemplo 4.9. Pruebe que

Z

1 B

z 2 (z 2

− 1)

dz = 0,

donde B consta de la circunferencia |z| = 2 descrita en la direcci´ on positiva, y de las circunferencias |z + 1| = 1/2, |z| = 1/2 y |z − 1| = 1/2, descritas en la direcci´ on negativa. Soluci´ on. Sea f (z) la funci´ on definida por f (z) =

1 z 2 (z 2

− 1)

.

Sean los contornos cerrados simples C1 y C2 indicados respectivamente con los colores azul y rojo, que se muestran en las siguientes figuras. y

y

2

2

C1

1

−2

1

−1 −1

1

2

x

−2

−2 Es claro que

−1 −1 −2

Z

f (z) dz =

Z

f (z) dz +

2

x

C2

f (z) dz.

C2

C1

B

Z

1

Como f (z) es una funci´ on anal´ıtica sobre y en el interior de los contornos cerrados simples C1 y C2 , entonces por el Teorema de Cauchy-Goursat obtenemos: Z Z f (z) dz = 0. f (z) dz = 0 y C2

C1

Por lo tanto, podemos concluir que Z

1 B

z 2 (z 2

− 1)

dz = 0. 

El procedimiento realizado en el Ejemplo 4.9 permite definir la extensi´ on del Teorema de Cauchy-Goursat para dominios multiplemente conexos, lo cual se expresa en el Teoon de este teorema se deja como ejercicio para el lector. (Ayuda: rema 4.3. La demostraci´ utilice un razonamiento similar al empleado en el Ejemplo 4.9.)

113

´ COMPLEJA CAP´ITULO 4. INTEGRACION

Teorema 4.3. Se denota a C como un contorno cerrado simple y a Cj (j = 1, 2, . . . , n) como un n´ umero finito de contornos cerrados simples interiores a C tales que los conjuntos interiores a cada Cj no tienen puntos en com´ un. R es la regi´on cerrada que consta de todos los puntos dentro y sobre C excepto los puntos interiores a cada Cj (R es un dominio multiplemente conexo). Se denota por B la frontera completa orientada de R que consta de C y todos los Cj , descrita en una direcci´ on tal que los puntos de R se encuentran a la izquierda de B. En este caso, si una funci´on f (z) es anal´ıtica en R, entonces Z f (z) dz = 0. B

4.4 Integral Indefinida El Teorema de Cauchy-Goursat es una herramienta valiosa cuando se trata de integrar una funci´ on anal´ıtica alrededor de un contorno cerrado. En caso de que el contorno no sea cerrado, existen m´ etodos que se pueden deducir a partir de dicho teorema y que facilitan el c´ alculo de la integral considerada. El siguiente teorema se conoce como principio de independencia de la trayectoria, cuya demostraci´ on se fundamenta en el Teorema de Cauchy-Goursat. Teorema 4.4 (Principio de independencia de la trayectoria). Sea f (z) anal´ıtica en un dominio simplemente conexo D y sean z1 , z2 ∈ D. Entonces, el valor de la integral Z z2 f (z) dz z1

no depende del contorno, dentro de D, utilizado para ir de z1 a z2 . Demostraci´ on. Podemos suponer, sin p´erdida de generalidad, que los contornos C1 y C2 van de z1 a z2 sin intersecci´ on en puntos intermedios. Se tiene que los contornos C1 y −C2 forman un contorno cerrado simple, que denominamos C. Luego, por el Teorema de Cauchy-Goursat Z f (z) dz = 0, C

pero Z

C

f (z) dz =

Z

f (z) dz +

Z

f (z) dz =

−C2

C1

Z

C1

f (z) dz −

Z

f (z) dz, C2

por lo tanto, Z

f (z) dz = C1

Z

f (z) dz,

C2

lo cual indica que la integral desde z1 hasta z2 es as´ı independiente del contorno seguido, en tanto ese contorno se encuentre dentro de D. Del principio de la independencia de la trayectoria se define la primitiva de una funci´ on de variable compleja.

114

4.4. INTEGRAL INDEFINIDA

Definici´ on 4.9 (Integral indefinida o primitiva). Sea f (z) una funci´ on anal´ıtica en un dominio simplemente conexo D ⊂ C. Sea z0 ∈ D. La funci´ on F (z) definida en D por Z z F (z) = f (s) ds + c, (4.1) z0

se denomina integral indefinida o primitiva de f (z), donde c es un constante compleja. En realidad, f (z) posee un n´ umero infinito de primitivas. Dichas primitivas difieren en valores constantes y son anal´ıticas en D, y satisfacen F ′ (z) = f (z), Usamos la notaci´ on de integral indefinida Z

para todo z ∈ D.

f (z) dz

para indicar todas las posibles primitivas de f (z). Note que la primitiva de cada una de las funciones elementales complejas, son las mismas primitivas de las funciones elementales de variable real; por ejemplo, la primitiva de ez es, por supuesto, ella misma, y la primitiva de sen z es − cos z, etc. Por otra parte, el valor de la constante c correspondiente a una primitiva espec´ıfica Z z f (s) ds z0

queda determinado por el l´ımite inferior de integraci´ on, como se muestra en el siguiente ejemplo. Ejemplo 4.10. a) Encuentre las primitivas de f (z) = z sen z. R π/2 b) Encuentre la primitiva necesaria para calcular la integral π z sen z dz y halle el valor de la integral. Soluci´ on. a) Usando integraci´ on por partes obtenemos Z Z z sen z dz = −z cos z + cos z dz = −z cos z + sen z + c = F (z). b) Usando el resultado de a) tenemos Z z s sen s ds = −z cos z + sen z + c. π

Para determinar el valor de c observemos que el lado izquierdo de esta ecuaci´ on es cero cuando z = π. Por lo tanto, 0 = −π cos π + sen π + c, de donde se deduce que c = −π. De esta forma, Z z s sen s ds = −z cos z + sen z − π. π

115

´ COMPLEJA CAP´ITULO 4. INTEGRACION

Utilizando esta u ´ ltima ecuaci´ on tenemos que Z π/2 z sen z dz = −(π/2) cos(π/2) + sen (π/2) − π = 1 − π. π

 De la ecuaci´ on (4.1) se infiere que una integral definida se puede evaluar de igual forma que en el c´ alculo elemental: Z

β α

β f (z) dz = F (β) − F (α) = F (z) , α

es decir, usando la Regla de Barrow, que es una aplicaci´ on del Teorema Fundamental del C´ alculo.

4.5 F´ ormula Integral de Cauchy En esta secci´ on veremos que si una funci´ on es anal´ıtica en un punto, sus derivadas de todos los o ´rdenes existen en ese punto y son tambi´ en anal´ıticas ah´ı. Previo a este resultado veremos un resultado curioso que se obtiene a trav´ es del Teorema de Cauchy-Goursat, a saber: si consideramos una funci´ on anal´ıtica sobre y en el interior de un contorno cerrado simple, basta con conocer los valores que ella toma sobre ese contorno, para determinar los valores que toma en el interior del mismo. Este resultado se conoce como f´ ormula integral de Cauchy. La demostraci´ on de este teorema se puede ver en [8]. Teorema 4.5 (F´ ormula Integral de Cauchy). Sea f (z) una funci´on anal´ıtica en un dominio simplemente conexo D ⊂ C. Sea C un contorno cerrado simple C dentro de D. Sea z0 ∈ D un punto interior a C. Entonces, Z 1 f (z) f (z0 ) = dz. (4.2) 2πi C (z − z0 ) ormula integral de Cauchy. El siguiente ejemplo aclara La f´ ormula (4.2) se denomina f´ el uso de esta f´ ormula en la evaluaci´ on de integrales. Ejemplo 4.11. Hallar el valor de la integral Z |z−i|=2

1 dz. z2 + 4

Soluci´ on. Factorizando el integrando tenemos z2

1 1 = . +4 (z − 2i)(z + 2i)

Observamos que el factor (z −2i) se anula dentro del contorno de integraci´ on y que (z +2i) no se anula ni sobre el contorno ni en su interior. Escribiendo la integral considerada en la forma 1 Z (z + 2i) dz |z−i|=2 (z − 2i)

116

´ 4.5. FORMULA INTEGRAL DE CAUCHY

notamos que la funci´ on f (z) = 1/(z + 2i) es anal´ıtica tanto en la circunferencia |z − i| = 2 como en su interior. Por tanto, podemos usar la f´ ormula integral de Cauchy dada en el Teorema 4.5, podemos escribir: Z Z f (z) 1 1 1 dz = dz. f (2i) = 2 2πi |z−i|=2 (z − 2i) 2πi |z−i|=2 z + 4 Como f (2i) =

1 , entonces el valor de la integral considerada es 4i Z 1 π dz = 2πif (2i) = . 2+4 z 2 |z−i|=2 

Pasemos ahora a ver que si una funci´ on es anal´ıtica en un punto, entonces sus derivadas de todos los o ´rdenes existen en ese punto y son tambi´ en anal´ıticas. Tal resultado se conoce como extensi´ on de la f´ ormula integral de Cauchy. Teorema 4.6 (Extensi´ on de la F´ ormula Integral de Cauchy). Sea f (z) una funci´on anal´ıtica en un dominio simplemente conexo D ⊂ C. Sea C un contorno cerrado simple C dentro de D. Sea z0 ∈ D un punto interior a C. Entonces, f (z) es infinitamente diferenciable en D y Z n! f (z) (n) f (z0 ) = dz. 2πi C (z − z0 )n+1 Adem´ as, f (n) (z) es anal´ıtica en D para cada n.

El siguiente ejemplo aclara el uso de la extensi´ on de la f´ ormula integral de Cauchy en la evaluaci´ on de integrales. Ejemplo 4.12. Hallar el valor de la integral Z |z−i|=2

1 dz. (z 2 + 4)2

Soluci´ on. Factorizando el integrando tenemos (z 2

1 1 = . 2 2 + 4) (z − 2i) (z + 2i)2

Tomando f (z) = 1/(z + 2i)2 y z0 = 2i, por el Teorema 4.6 podemos escribir: Z Z f (z) 1 1 1 ′ dz = dz. f (2i) = 2 2 2πi |z−i|=2 (z − 2i) 2πi |z−i|=2 (z + 4)2 Como f ′ (z) =

−2 , entonces el valor de la integral considerada es (z + 2i)3 Z π 1 dz = 2πif ′ (2i) = . 2 2 16 |z−i|=2 (z + 4) 

117

´ COMPLEJA CAP´ITULO 4. INTEGRACION

4.6 Residuo La teor´ıa de residuos proporciona una t´ecnica de evaluaci´ on (r´ apida y) efectiva de integrales de la forma Z f (z) dz, C

donde C es un contorno cerrado simple. El punto esencial de la teor´ıa es el concepto de residuo y su relaci´ on con la serie de Laurent. Definici´ on 4.10. Sean C un contorno cerrado simple y z0 ∈ C un punto interior a C. Sea f (z) una funci´ on anal´ıtica sobre C y en todo punto de su interior, salvo en z0 . El residuo de f (z) en z0 , denotado por Res [f (z)], se define como z=z0

1 Res [f (z)] = z=z0 2πi

Z

f (z) dz.

C

El siguiente teorema describe la relaci´ on que existe entre el residuo Res [f (z)] y una y z=z0

s´ olo una serie de Laurent de f (z) centrada en z0 ; adem´as, dice que Res [f (z)] es igual al z=z0

coeficiente b1 de esa serie de Laurent. Teorema 4.7. Sea z0 un punto singular aislado de una funci´on f (z). Entonces, el residuo de f (z) en z0 es igual al coeficiente de (z − z0 )−1 en la serie de Laurent que representa a f (z) en el anillo 0 < |z − z0 | < r, para cierto n´ umero real r > 0. Demostraci´ on. Como z0 es un punto singular aislado de f (z), entonces existe r > 0 tal que el desarrollo de Laurent de f (z) centrado en z0 , v´alido en el anillo 0 < |z − z0 | < r, est´ a dado por: ∞ ∞ X X n bn (z − z0 )−n . an (z − z0 ) + f (z) = n=1

n=0

Sea C un contorno cerrado simple contenido en el anillo 0 < |z − z0 | < r tal que z0 es un punto interior de C. As´ı, integrando en ambas partes de la ecuaci´ on anterior obtenemos: # Z "X Z ∞ ∞ X bn (z − z0 )−n dz an (z − z0 )n + f (z) dz = C

C

=

∞ X

n=0

n=0

an

Z

n

C

n=1 ∞ X

(z − z0 ) dz +

n=1

bn

Z

C

(z − z0 )−n dz,

adem´as, aplicando convenientemente el Teorema de Cauchy-Goursat, la f´ ormula integral de Cauchy o la extensi´ on de la f´ ormula integral de Cauchy, tenemos: Z (z − z0 )n dz = 0, para n ≥ 0, Z C (z − z0 )−1 dz = 2πi, ZC (z − z0 )−n dz = 0, para n ≥ 2. C

118

4.6. RESIDUO

Por lo tanto,

Z

f (z) dz = b1 2πi,

C

de donde se deduce que

Res [f (z)] = b1 ,

z=z0

que era lo que dese´ abamos demostrar. Ejercicio 4.3. Utilice el Teorema 4.7 para calcular los siguientes residuos:   a) Res e1/z z=0

b) Res z=i



1 (z − i)2



  c) Res z 4 sen (1/z) z=0

alculo del Residuo 4.6.1 C´ Como observamos en el Ejercicio 4.3, el Teorema 4.7 se puede aplicar para calcular el residuo de una funci´ on f (z) en un punto singular aislado z0 . De esta forma, por el Teorema 4.7 se tiene: • si z0 es un punto singular removible de la funci´ on f (z), entonces su residuo en ese punto es cero; • si z0 es un punto singular esencial de f (z), la u ´nica manera para determinar el residuo en dicho punto, consiste en obtener el desarrollo de Laurent centrado z0 , v´alido en el anillo dado en el Teorema 4.7, y tomar el residuo igual al coeficiente b1 . Pero, si la funci´ on tiene un polo en z0 , no es necesario obtener todo el desarrollo de Laurent centrado en z0 para encontrar el coeficiente que buscamos. Existen diversos m´ etodos que podemos usar siempre y cuando sepamos que la singularidad es un polo, los cuales estudiaremos a continuaci´ on. C´ alculo del Residuo en un Polo El siguiente teorema nos permite identificar si un punto z0 es un polo de f (z) y, adem´as, nos dice como calcular el residuo Res [f (z)]. z=z0

Teorema 4.8. Sea z0 un punto singular aislado de una funci´on f (z). Si para cierto entero positivo m, la funci´on φ(z) = (z − z0 )m f (z) se puede definir en z0 de modo que sea anal´ıtica ah´ı y φ(z0 ) 6= 0, entonces f (z) tiene un polo de orden m en z0 y, adem´ as,   φ(z0 ) = l´ım (z − z0 )m f (z), m = 1;   z→z0  (4.3) Res [f (z)] = z=z0  φ(m−1) (z0 )    , m > 1. (m − 1)!

119

´ COMPLEJA CAP´ITULO 4. INTEGRACION

Demostraci´ on. Como φ(z) = (z − z0 )m f (z) es anal´ıtica en z0 y φ(z0 ) 6= 0, entonces la funci´ on f (z) se puede escribir como f (z) =

p(z) , q(z)

donde p(z) = φ(z) y q(z) = (z − z0 )m , adem´as, q(z0 ) = q ′ (z0 ) = · · · = q (m−1) (z0 ) = 0,

y

q (m) (z0 ) = m! 6= 0.

Luego, por el Teorema 3.9 z0 es un polo de orden m de f (z). Pasemos ahora a demostrar la f´ ormula (4.3). Como φ(z) es anal´ıtica en z0 ella posee desarrollo de Taylor centrado en z0 , v´alido en cierto disco |z − z0 | < r, dado por φ(z) = (z − z0 )m f (z) = φ(z0 ) + φ′ (z0 )(z − z0 ) + · · · +

φ(m) (z0 ) (z − z0 )m + · · · . m!

As´ı, en cada punto z del disco |z − z0 | < r excepto en z0 , se tiene que f (z) =

φ(m) (z0 ) φ′ (z0 ) φ(m−1) (z0 ) φ(z0 ) + + ··· , + + · · · + (z − z0 )m (z − z0 )m−1 (m − 1)!(z − z0 ) m!

que es el desarrollo de Laurent de f (z) centrado en z0 v´alido en el anillo 0 < |z − z0 | < r, con φ(m−1) (z0 ) . b1 = (m − 1)! Entonces, por el Teorema 4.7 la ecuaci´ on (4.3) es cierta.

En el siguiente ejemplo se muestra la utilidad del teorema anterior en el c´ alculo del residuo en un polo. ez en cada uno de sus puntos singulares. sen z ez Soluci´ on. Los puntos singulares aislados de f (z) = son: sen z Ejemplo 4.13. Hallar el residuo de f (z) =

zn = nπ,

n = 0, ±1, ±2, . . . ,

adem´as, cada zn es un polo simple de f (z). Definamos la funci´ on φn (z) como   l´ım (z − zn )f (z) = (−1)n enπ , si z = zn ,  z→zn φn (z) = (z − zn )f (z) = z    (z − zn )e , si z 6= zn . sen z

Es claro que φn (z) es anal´ıtica en zn y φn (zn ) 6= 0. As´ı. utilizando la ecuaci´ on (4.3) se tiene ez que el residuo de f (z) = en cada uno de sus puntos singulares zn es sen z  z  e = (−1)n enπ , n = 0, ±1, ±2, . . . Res z=zn sen z 

120

4.6. RESIDUO

Ejercicio 4.4. Calcule los siguientes residuos:   ez a) Res 2 2 . z=0 z (z + 1) # " z 1/2 . (Utilizando la rama principal de z 1/2 ) b) Res z=2 z(z − 2)2 

 Log z c) Res 4 . z=1 z (z − 1)2   sen z − z d) Res , para n = 0, ±1, ±2, . . . z=nπi z senh z

4.6.2 Teorema de los Residuos El siguiente teorema nos permite calcular la integral de una funci´ on f (z) a lo largo de un contorno cerrado simple C, tal que f (z) es anal´ıtica en C y en su interior, salvo en un n´ umero finito de puntos singulares interiores a C. Teorema 4.9 (Teorema de los Residuos). Sea C un contorno cerrado simple orientado positivamente, dentro y sobre el cual una funci´on f (z) es anal´ıtica excepto en un n´ umero finito de puntos z1 , z2 , . . . , zn interiores a C. Entonces, Z

f (z) dz = 2πi

C

n X k=1

Res [f (z)] .

z=zk

Demostraci´ on. Se deja como ejercicio para el lector. Ayuda: n R defina convenientemente Pn R f (z)dz, contornos cerrados simples, C1 , . . . , Cn , de manera que C f (z)dz = k=1 Ck luego aplique la f´ o rmula integral de Cauchy para calcular cada una de las integrales R f (z)dz. Ck En el siguiente ejemplo utilizamos el Teorema de los Residuos para calcular la integral de una funci´ on f (z), a lo largo de un contorno cerrado simple que posee en su interior un n´ umero finito de puntos singulares de f (z).

Ejemplo 4.14. Calcular la integral Z

C

z−2 dz, (z − 1)z

donde C es la circunferencia |z| = 3 orientada positivamente.

z−2 . Los puntos singulares de f (z) son z0 = 0 y z1 = 1, que son (z − 1)z puntos interiores a C, adem´as, z0 y z1 son polos simples de f (z). Luego, Soluci´ on. Sea f (z) =

Res [f (z)] = 2 y

z=z0

Res [f (z)] = −1.

z=z1

121

´ COMPLEJA CAP´ITULO 4. INTEGRACION

Como f (z) es anal´ıtica dentro y sobre C excepto en z0 y z1 , entonces por el Teorema de los Residuos obtenemos   Z z−2 dz = 2πi Res [f (z)] + Res [f (z)] = 2πi. z=z0 z=z1 C (z − 1)z 

4.6.3 Expansi´ on en Fracciones Parciales Una aplicaci´ on de gran importancia del c´ alculo de residuos, es la expansi´ on en fracciones parciales de algunas funciones racionales particulares. La expansi´ on en fracciones parciales se aplica a funciones racionales propias, esto es, a funciones del tipo f (z) =

b0 + b1 z + · · · + bM z M , a0 + a1 z + · · · + z N

donde M < N . La expansi´ on en fracciones parciales consiste en expresar la funci´ on racional propia f (z) como una suma de fracciones simples. El siguiente teorema nos muestra expl´ıcitamente la forma de la expansi´ on en fracciones parciales, dependiendo de la multiplicidad de los polos de f (z). Teorema 4.10. Sea f (z) una funci´on racional propia dada por f (z) =

c0 + c1 z + · · · + cM z M , d0 + d1 z + · · · + dN −1 z N −1 + z N

donde M < N . Sean pk los polos de f (z) y rk sus respectivas multiplicidades, para k = P 1, 2, . . . , T , donde T es un entero positivo tal que N = Tk=1 rk . Entonces:

(i) Si todos los polos de f (z) son simples, la expansi´on en fracciones parciales de f (z) es: f (z) =

A2 AN A1 + + ··· + , (z − p1 ) (z − p2 ) (z − pN )

donde los n´ umeros complejos Ak , denominados coeficientes, se calculan como Ak = Res [f (z)] , z=pk

para k = 1, 2, . . . , N .

(ii) Si todos los polos de f (z) son simples, excepto el polo pℓ que es de orden rℓ , la expansi´ on en fracciones parciales de f (z) es: f (z) =

donde

A2 Aℓ−1 A1 + + ··· + (z − p1 ) (z − p2 ) (z − pℓ−1 ) Aℓ,1 Aℓ,2 Aℓ,rℓ + + + ··· + (z − pℓ ) (z − pℓ )2 (z − pℓ )rℓ Aℓ+1 Aℓ+2 AT + + + ··· + , (z − pℓ+1 ) (z − pℓ+2 ) (z − pT )

Ak = Res [f (z)] , para k = 1, 2, . . . , T , y k = 6 ℓ; z=pk # " d(rℓ −) 1 rℓ f (z)] [(z − p ) , para  = 1, 2, . . . , rℓ . · Aℓ, = ℓ (r −) (rℓ − )! dz ℓ z=pℓ

122

4.6. RESIDUO

Demostraci´ on. S´ olo demostraremos la parte (i), la parte (ii) se deja como ejercicio para el lector. Realicemos la demostraci´ on por inducci´ on en el n´ umero de polos simples de f (z). Supongamos que f (z) posee un s´ olo polo simple p1 y demostremos que f (z) =

A1 (z − p1 )

(4.4)

con A1 = Res [f (z)] . z=p1

Como f (z) es una funci´ on racional propia de la forma f (z) =

c0 + c1 z + · · · + cM z M , d0 + d1 z + · · · + dN −1 z N −1 + z N

entonces todos los polos de f (z) son las ra´ıces del polinomio d0 +d1 z+· · ·+dN −1 z N −1 +z N ; pero f (z) posee un s´ olo polo simple p1 , luego el polinomio a0 + a1 z + · · · + z N adquiere la forma (z − p1 ) y el polinomio c0 + c1 z + · · · + cM z M debe ser de grado 0, es decir, c0 6= 0 y c1 = c2 = · · · = cM = 0. Por lo tanto, f (z) tiene la forma f (z) =

c0 , z − p1

adem´as, utilizando el desarrollo de Laurent de f (z) centrado en p1 obtenemos: b1 c0 = f (z) = a0 + = a0 + z − p1 z − p1

Res [f (z)]

z=p1

z − p1

,



a0 = 0 y c0 = Res [f (z)] , z=p1

en otras palabras, (4.4) es cierta. Como hip´ otesis inductiva supongamos que una funci´ on h(z) posee N = L polos simples p1 , . . . , pL y que adem´as se cumple h(z) =

A1 A2 AL + + ··· + , (z − p1 ) (z − p2 ) (z − pL )

(4.5)

donde Ak = Res [h(z)] , z=pk

para k = 1, 2, . . . , L.

Demostremos que si f (z) posee N = L + 1 polos simples p1 , . . . , pL , pL+1 , entonces se cumple A2 AL AL+1 A1 + + ··· + + , (4.6) f (z) = (z − p1 ) (z − p2 ) (z − pL ) (z − pL+1 ) donde AL+1 = Res [f (z)] . z=pL+1

Como f (z) es una funci´ on racional propia y p1 , . . . , pL , pL+1 son polos simples, entonces f (z) tiene la forma f (z) =

p(z) c + , (z − p1 )(z − p1 ) · · · (z − pL ) (z − pL+1 )

123

´ COMPLEJA CAP´ITULO 4. INTEGRACION

donde p(z) es un polinomio de grado < L y c ∈ C. Tomando h1 (z) y h2 (z) definidas respectivamente como: h1 (z) =

p(z) (z − p1 )(z − p1 ) · · · (z − pL )

h2 (z) =

y

c , (z − pL+1 )

se tiene f (z) = h1 (z) + h2 (z), adem´as, por (4.5) podemos escribir: h1 (z) =

A1 A2 AL + + ··· + (z − p1 ) (z − p2 ) (z − pL )

y

h2 (z) =

AL+1 , (z − pL+1 )

donde Ak = Res [h(z)] , z=pk

para k = 1, 2, . . . , L + 1.

Por lo tanto, de todo lo anterior obtenemos: f (z) =

A2 AL AL+1 A1 + + ··· + + , (z − p1 ) (z − p2 ) (z − pL ) (z − pL+1 )

es decir, (4.6) es cierta. El teorema anterior se puede extender a funciones racionales propias que tienen dos o m´ as polos cuyo orden es mayor que 1. En el siguiente ejemplo se muestra tal extensi´ on. Ejemplo 4.15. Halle la expansi´ on en fracciones parciales de f (z) =

144 z 2 + 144 z + 144 . (z − 3)2 (z − 2)2 (z + 1)

Soluci´ on. Los puntos p1 = −1, p2 = 2 y p3 = 3 son los polos de f (z). Se observa que p1 es de orden 1 y p2 y p3 son de orden 2. De esta forma, la expansi´ on en fracciones parciales de f (z) es de la forma f (z) =

A2,1 A2,2 A3,2 A3,1 A1 + + + + . 2 (z + 1) (z − 2) (z − 2) (z − 3) (z − 3)2

Se tiene que A1 =

Res [f (z)] = 1, z=−1   A2,2 = (z − 2)2 f (z) z=2 = 336,   d 2 [(z − 2) f (z)] = 800, A2,1 = dz z=2   A3,2 = (z − 3)2 f (z) z=3 = 468,   d A3,1 = [(z − 3)2 f (z)] = −801, dz z=3 as´ı, la expansi´ on en fracciones parciales de la funci´ on dada es f (z) =

800 336 801 468 1 + + − + . 2 (z + 1) (z − 2) (z − 2) (z − 3) (z − 3)2 

124

´ Y RESIDUOS CON MATLAB 4.7. INTEGRACION

4.7 Integraci´ on y Residuos con M ATLAB La herramienta de matem´ atica simb´ olica de M ATLAB cuenta con el comando int que permite calcular integrales de funciones complejas. Con int(f,z) se calcula la primitiva de la funci´ on f (z). En el siguiente ejemplo se calculan las primitivas de las siguientes funciones: z5 z+1 , h(z) = . f (z) = sen (z + 1), g(z) = (z − 2)2 ez−1 ✄

>> syms z >> f ( z ) = s i n ( z +1); i n t ( f , z ) ans ( z ) = −c o s ( z + 1) >> g ( z ) = ( z +1)/(z −2)ˆ2; i n t ( g , z ) ans ( z ) = l o g ( z − 2) − 3/( z − 2) >> h ( z ) = z ˆ5/ exp ( z −1); i n t ( h , z ) ans ( z ) = −exp (1 − z ) ∗ ( z ˆ5 + 5∗ z ˆ4 + 20∗ z ˆ3 + 60∗ z ˆ2 + 120∗ z + 120)





As´ı, las primitivas de las funciones dadas son: F (z) = − cos(z + 1) + c, H(z) = −

G(z) = Log (z − 2) −

3 + c, z−2

z 5 + 5 z 4 + 20 z 3 + 60 z 2 + 120 z + 120 + c. ez−1

Con el comando int(f,z,a,b) se calcula la integral definida Z

b

f (z) dz. a

donde a, b ∈ C. Observe el siguiente ejemplo donde se calcula la integral Z

−1 −1+i

ez + z 3 − i dz = i e + e2 (−2 + i) + cos(1) e2 (2 − 3i) + sen (1)e2 (3 + 2i). ez−1

✄ >> syms z >> f ( z ) = ( exp ( z)+zˆ3− i ) / exp ( z −1); i n t ( f , z,−1−i , −1) ans ( z ) = exp (1)∗ i + exp(2)∗( − 2 + i ) + c o s (1)∗ exp ( 2 ) ∗ ( 2 − 3∗ i ) + exp (2)∗ s i n ( 1 ) ∗ ( 3 + 2∗ i )





En M ATLAB no existe una comando que permita calcular, en forma directa, una integral de l´ınea. Nosotros hemos dise˜ nado la funci´ on IntegralLinea (ver Programa 4.1), que permite calcular la integral de l´ınea Z

C

f (z) dz =

Z

b

f (z(t)) z ′ (t) dt,

a

donde C es una curva con parametrizaci´ on z(t) = x(t) + iy(t), a ≤ t ≤ b y f (z) es una funci´ on de variable compleja.

125

´ COMPLEJA CAP´ITULO 4. INTEGRACION

Programa 4.1. Funci´ on IntegralLinea.m f u n c t i o n [ v i ]= I n t e g r a l L i n e a ( f , zt , a , b ) %I n t e g r a l L i n e a C a l c u l a e l v a l o r de l a i n t e g r a l de f ( z ) a l o % l a r g o de l a cu rva C con p a r a m e t r i z a c i o ´n % z ( t ) , a <= t <= b % v i : v a l o r de l a i n t e g r a l de l´ı n e a % syms z t ; gt = f ( zt ) ; d z t = d i f f ( zt , t ) ; v i = i n t ( g t ∗ dzt , t , a , b ) ; end

En el siguiente ejemplo, se utiliza la funci´ on IntegralLinea para calcular las integrales: Z

C1

Z

z3 dz = 0, z2

C2

Log z dz = π



1 i − 4 4



+ ln(2)



i 1 − 2 2



− 1 + 2 i,

donde C1 es el arco de la circunferencia de centro 0 y radio 2, desde z = 2i a z = −2, y C2 es el segmento de recta que va de z = i a z = 1 − i. ✄ >> syms z t >> f ( z)= c o n j ( z )ˆ3/ z ˆ 2 ; z t =2∗( c o s ( t )+ i ∗ s i n ( t ) ) ; >> v i= I n t e g r a l L i n e a ( f , zt , p i /2 , p i ) vi = 0 >> f ( z)= l o g ( z ) ; z t=t +(1−2∗ t )∗ i ; >> v i= I n t e g r a l L i n e a ( f , zt , 0 , 1 ) vi = p i ∗(1/4 − i /4) + l o g (2)∗(1/2 − i /2) − 1 + 2∗ i





Por otra parte, M ATLAB no cuenta con un comando que calcule directamente el residuo de una funci´ on f (z) en z0 . Hemos dise˜ nado la funci´ on ResiduoPolo (ver Programa 4.2), que calcula el residuo Res [f (z)], cuando z0 es un polo de orden m. z=z0

Programa 4.2. Funci´ on ResiduoPolo.m f u n c t i o n [ r e s]= Residu oPolo ( f , z0 ,m) %Residu oPolo C a l c u l a e l r e s i u d o de f ( z ) en e l polo % z0 de orden m % res : valor del residuo % syms z ; p h i = ( z−z0 )ˆm∗ f ; i f m==1 r e s = l i m i t ( phi , z , z0 ) ; else dphi = d i f f ( phi , z ,m−1); r e s = l i m i t ( dphi , z , z0 ) / f a c t o r i a l (m−1); end end

126



´ Y RESIDUOS CON MATLAB 4.7. INTEGRACION

En el siguiente ejemplo, se utiliza la funci´ on ResiduoPolo para calcular los residuos:   ez = 1. a) Res 2 2 z=0 z (z + 1)   Log z = 0. b) Res 4 z=1 z (z − 1)2   sen z − z c) Res . = π−senh(π) π z=πi z senh z >> syms z >> f ( z ) = exp ( z ) / ( z ˆ2∗( z ˆ2+1)); z0 = 0 ; m = 2 ; >> r e s = Residu oPolo ( f , z0 ,m) res (z) = 1 >> f ( z ) = l o g ( z ) / ( z ˆ4∗( z −1)); z0 = 1 ; m = 2 ; >> r e s = Residu oPolo ( f , z0 ,m) res (z) = 0 >> f ( z ) = ( s i n ( z)−z ) / ( z ∗ s i n h ( z ) ) ; z0 = p i ∗ i ; m = 1 ; >> r e s = Residu oPolo ( f , z0 ,m) res (z) = ( pi − sinh ( pi ))/ pi





Ahora bien, para calcular el residuo de una funci´ on f (z) en un punto singular esencial z0 , es necesario hallar el desarrollo de Laurent para obtener tal residuo. Para ello podemos usar convenientemente el comando taylor para determinar solo aquellos coeficientes del desarrollo de Laurent necesarios para hallar Res [f (z)]. Tenga presente que el procediz=z0

miento que explicaremos a continuaci´ on, se aplica a funciones f (z) que pueden expresarse como f (z) = f1 (z)f2 (z) (o un producto finito de funciones), donde z0 es un punto singular esencial de f2 (z) y f1 (z) es anal´ıtica en z0 . Suponga que se desea hallar el residuo de f (z) = (1 + z 2 )e1/(z−2i) en z0 = 2i, que, por supuesto, es un punto singular esencial de f (z). Es claro que f (z) = f1 (z)f2 (z), donde f1 (z) = 1 + z 2

y

f2 (z) = e1/(z−2i) .

Primero, hallamos el desarrollo de Taylor de f1 (z) centrado en z0 = 2i (en este caso se hallan todos los t´ erminos, que son tres), ✄ >> syms z >> f 1 ( z ) = 1+z ˆ 2 ; d t f 1 ( z ) = t a y l o r ( f1 , z , 2 i , ’ Order ’ , 3 ) dtf1 ( z) = z ∗4∗ i + ( z − 2∗ i )ˆ2 + 5





o, equivalentemente, f1 (z) = −3 + 4i(z − 2i) + (z − 2i)2 .

(4.7)

Luego, hallamos los primeros 5 t´ erminos del desarrollo de Maclaurin de h(w) = ew , ✄ >> syms w >> h (w) = exp (w) ; dth (w)= t a y l o r ( h , w, 0 , ’ Order ’ , 5 ) dth (w) = wˆ4/24 + wˆ3/6 + wˆ2/2 + w + 1





127

´ COMPLEJA CAP´ITULO 4. INTEGRACION

Seguidamente realizamos la operaci´ on dth(1/(z-2i)), ✄ >> dth ( 1 / ( z−2i ) ) ans = 1/( z − 2∗ i ) + 1/(2∗( z − 2∗ i ) ˆ 2 ) + 1/(6∗( z − 2∗ i ) ˆ 3 ) + 1/(24∗( z − 2∗ i ) ˆ 4 ) + 1





que corresponden a los primeros 5 t´erminos del desarrollo de Laurent de f2 (z) centrado en z0 = 2i, 1 1 1 1 + + +1 (4.8) + 2 3 z − 2 i 2 (z − 2 i) 6 (z − 2 i) 24 (z − 2 i)4 Multiplicando las ecuaciones (4.7) y (4.8), obtenemos que el coeficiente b1 del desarrollo de Laurent de f (z) centrado en z0 = 2i, est´ a dado por: b1 = −3 +

17 4i 1 + = − + 2i, 2 6 6

en otras palabras, el residuo de f (z) en z0 = 2i es Res [f (z)] = −

z=2i

17 + 2i. 6

Para finalizar esta secci´ on, mostramos la funci´ on ExpFracParc (ver Programa 4.3) que halla la expansi´ on en fracciones parciales de una funci´ on racional propia del tipo f (z) =

c0 + c1 z + · · · + cM z M , d0 + d1 z + · · · + dN −1 z N −1 + z N

donde M < N . En el siguiente ejemplo, se utiliza la funci´ on ExpFracParc para hallar la expansi´ on en fracciones parciales de z+1 . f (z) = (z + 3)(z + 2)3 (z − 1)2 ✄ >> syms z >> f ( z)=( z +1)/(( z +3)∗( z +2)ˆ3∗(z −1)ˆ2); p=[−3 −2 1 ] ; v=[1 3 2 ] ; >> e f p = E x p F r a c P a rc ( f , p , v ) efp = 1/(54∗(z − 1)ˆ2) − 1/(72∗( z − 1 ) ) − 1/(9∗( z + 2 ) ) + 4/(27∗( z + 2)ˆ2) + 1/(8∗( z + 3 ) ) − 1/(9∗( z + 2)ˆ3)





o, equivalentemente, f (z) = −

1/9 1/9 1/8 4/27 1/72 1/54 − + + − + 2 3 (z + 2) (z + 2) (z + 3) (z − 1) (z − 1)2 (z + 2)

128

4.8. PROBLEMAS RESUELTOS

Programa 4.3. Funci´ on ExpFracParc.m f u n c t i o n [ e f p]= E x p F r a c P a r c ( f , p , v ) %E x p F r a c P a rc H a l l a l a expansi o ´ n en f r a c c i o n e s p a r c i a l e s % de l a f u n c i o ´n r a c i o n a l propia f ( z ) % p : v e c t o r de p o l o s % v : v e c t o r de m u t i p l i c i d a d e s de l o s p o l o s % e f p : expansi o ´ n en f r a c c i o n e s p a r c i a l e s syms z ; np = l e n g t h ( p ) ; efp = 0; f o r t =1:np i f v ( t ) == 1 dphi ( z ) = ( z−p ( t ) ) ∗ f ( z ) ; r = dphi ( p ( t ) ) ; e f p = e f p + sym ( r / ( z−p ( t ) ) ) ; else f o r s =1:v ( t ) dphi ( z ) = d i f f ( ( z−p ( t ) ) ˆ ( v ( t ) ) ∗ f ( z ) , z , v ( t )−s ) ; r = dphi ( p ( t ) ) / f a c t o r i a l ( v ( t )− s ) ; e f p = e f p + sym ( r / ( z−p ( t ) ) ˆ s ) ; end end end end

4.8 Problemas Resueltos Problema 4.1. Determine el valor de la integral Z Re (z(z − 1)) dz, C

donde C es el arco desde z = −1 + i hasta z = 1 + i a lo largo de la curva y = |x|. Soluci´ on. La parametrizaci´ on de C es ( t − it, −1 ≤ t ≤ 0, z(t) = t + it, 0 < t ≤ 1. As´ı, usando la definici´ on y las propiedades de la integral de l´ınea podemos escribir: Z

C

Re (z(z − 1)) dz =

Z

1 −1

Re (z(t)(z(t) − 1))z ′ (t) dt

= (1 − i)

Z

0 −1

2

(2t − t) dt + (1 + i)

7 1 8 = (1 − i) + (1 + i) = − i. 6 6 6

Z

1 0

(2t2 − t) dt



129

´ COMPLEJA CAP´ITULO 4. INTEGRACION

Problema 4.2. Sea f (z) una funci´ on de variable compleja definida por ( 1, si Re z < 0; f (z) = 4 Re z, si Re z ≥ 0. Determine el valor de la integral

Z

f (z) dz,

C

donde C es el arco desde z = −1 − i hasta z = 1 + i, a lo largo de la curva y = x. Soluci´ on. La parametrizaci´ on de C es z(t) = t + it, −1 ≤ t ≤ 1. As´ı, considerando la definici´ on de f (z) y las propiedades de la integral de l´ınea, podemos escribir: Z 1 Z f (z(t)) z ′ (t) dt f (z) dz = −1 C  Z 0 Z 1 4t dt = 3 + 3i. dt + = (1 + i) −1

0

 Problema 4.3. Sea f (z) = Log (z + 1). Determine la expresi´ on anal´ıtica de la primitiva Rz F (z) = f (s) ds + c, en el dominio simplemente conexo i−1

D = {z ∈ C : |z + 1| > 0, −π < arg (z + 1) < π} .

Soluci´ on. Las primitivas de f (z) = Log (z + 1), en el dominio D, tienen la forma F (z) = (Log (z + 1) − 1)(z + 1) + c La primitiva pedida debe satisfacer F (i − 1) = 0, en otras palabras, 0 = F (i − 1) = (Log (i) − 1)i + c = −

π −i+c 2



c=

π + i. 2

Por lo tanto, la expresi´ on anal´ıtica de la primitiva pedida es F (z) = (Log (z + 1) − 1)(z + 1) +

π + i. 2 

Problema 4.4. Sea f (z) una funci´ on de variable compleja definida como f (z) = z i ≡ ei log z , donde log z = ln |z| + i arg z, con |z| > 0 y −π/2 < arg z < 3π/2. Determine el valor de la integral Z f (z) dz, C

donde C es el arco de la circunferencia |z| = 1 desde z = −1 hasta z = 1 situado en el semiplano superior.

130

4.8. PROBLEMAS RESUELTOS

Soluci´ on. La funci´ on f (z) = ei log z es anal´ıtica en el dominio simplemente conexo D = {z ∈ C : |z| > 0, −π/2 < arg z < 3π/2}. Por lo tanto, f (z) tiene primitiva en D dada por: F (z) =

e(i+1) log z z i+1 ≡ + c, i+1 i+1

adem´as, la curva C est´ a dentro de D. Luego, podemos escribir: # 1 "  0  Z 1 Z e − e−π eπi 1 + e−π e(i+1) log z i +c = = (1 − i). z dz = f (z) dz = i+1 i+1 2 −1 C −1



Problema 4.5. Determine el valor de la integral Z sen (z + 1) dz. 34 |z+1|=1 (z + 1) Soluci´ on. Sea f (z) = sen (z+1). Es claro que f (z) es anal´ıtica en la circunferencia |z+1| = 1 y en el interior de la misma; adem´as, z0 = −1 es un punto interior de esta circunferencia. As´ı, usando la extensi´ on de la f´ ormula integral de Cauchy, podemos escribir: Z sen (z + 1) 2πi f (33) (−1) . dz = 34 33! |z+1|=1 (z + 1) Hallemos f (33) (−1). Para ello utilizaremos el desarrollo de Taylor de f (z) centrado en z0 = −1. Tal desarrollo tiene la forma f (z) =

∞ X

n=0

donde an =

f (n) (−1) . n!

an (z + 1)n ,

|z + 1| < ∞,

As´ı, f (33) (−1) = 33! a33 . Como

f (z) = sen (z + 1) =

∞ X (−1)n (z + 1)2n+1 , (2n + 1)! n=0

|z + 1| < ∞,

entonces f (33) (−1) = 1. Por lo tanto, el valor de la integral dada est´ a dado por: Z 2πi sen (z + 1) . dz = 34 (z + 1) 33! |z+1|=1  Problema 4.6. Eval´ ue la integral Z

|z|=2

(z + 1) cos(1/(z − 1)) dz. (1 − z)2

131

´ COMPLEJA CAP´ITULO 4. INTEGRACION

cos(1/(z−1)) . Es claro que f (z) es anal´ıtica en la circunferencia Soluci´ on. Sea f (z) = (z+1) (1−z) 2 |z| = 2 y en el interior de la misma, salvo en el punto z0 = 1. As´ı, aplicando el teorema de los residuos tenemos: Z (z + 1) cos(1/(z − 1)) dz = 2πi Res [f (z)] . z=1 (1 − z)2 |z|=2

Como z0 = 1 es un punto singular esencial de f (z), entonces debemos hallar el desarrollo de Laurent para obtener el residuo en tal punto. Se tiene que el desarrollo de Laurent de f (z) centrado en z0 = 1, v´ alido en el anillo |z − 1| > 0, est´ a dado por:   1 1 f (z) = (z + 1) cos (1 − z)2 1−z ∞ X 1 (−1)n = (2 + (z − 1)) (z − 1)−2n 2 (1 − z) (2n)! n=0

=

∞ X 2(−1)n

n=0

(2n)!

(z − 1)−2(n+1) +

∞ X (−1)n

n=0

(2n)!

(z − 1)−2n−1 ,

|z − 1| > 0,

en consecuencia, el residuo de f (z) en z0 = 1 es: Res [f (z)] = 1. Por lo tanto, el valor de z=1

la integral dada es

Z

|z|=2

(z + 1) cos(1/(z − 1)) dz = 2πi. (1 − z)2 

Problema 4.7. Eval´ ue la integral Z

z e1/(z+i) +

|z|=3/2

z3 dz. (z + 2)2 (z − 1)

Soluci´ on. Por las propiedades de la integral tenemos: Z

|z|=3/2

z e1/(z+i) +

z3 dz = (z + 2)2 (z − 1)

Z

z e1/(z+i) dz +

|z|=3/2

Z

|z|=3/2

z3 dz. (z + 2)2 (z − 1)

Sean f1 (z) y f2 (z) funciones definidas como: f1 (z) = z e1/(z+i) ,

f2 (z) =

z3 . (z + 2)2 (z − 1)

Es claro que f1 (z) y f2 (z) son funciones anal´ıticas en la circunferencia |z| = 3/2 y en el interior de la misma, salvo en los puntos z0 = −i y z1 = 1 y z2 = −2, respectivamente. Observamos que z2 = −2 no es punto interior de la circunferencia |z| = 3/2. As´ı, aplicando el teorema de los residuos se tiene Z Z z3 1/(z+i) ze dz = 2πi Res [f1 (z)] , dz = 2πi Res [f2 (z)] . z=z0 z=z1 (z + 2)2 (z − 1) |z|=3/2

|z|=3/2

132

4.8. PROBLEMAS RESUELTOS

Ahora, como z0 = −i es un punto singular esencial de f1 (z), entonces debemos hallar el desarrollo de Laurent para calcular el residuo en tal punto. Se tiene que el desarrollo de Laurent de f1 (z) centrado en z0 = −i, v´alido en el anillo |z + i| > 0, est´ a dado por: 1/(z+i)

f1 (z) = z e =

∞ X −i

n=0

n!

∞ X 1 (z + i)−n = (−i + (z + i)) n! n=0 −n

(z + i)

∞ X 1 + (z + i)−n+1 , n! n=0

luego, Res [f1 (z)] =

z=z0

|z + i| > 0,

1 − i. 2

Por otra parte, como z1 = 1 es un polo simple de f2 (z), se tiene que   1 z3 = . Res [f2 (z)] = z=z1 (z + 2)2 z=1 9

En consecuencia, el valor de la integral dada es     Z 1 1 π z3 1/(z+i) dz = 2πi − i + 2πi = (18 + 11 i) . ze + 2 (z + 2) (z − 1) 2 9 9 |z|=3/2

 Problema 4.8. Calcular el valor de la integral Z (1 + z + z 2 )(e1/z + e1/(z−1) + e1/(z−2) ) dz, C

donde C es un contorno cerrado simple que contiene en su interior a los puntos 0, 1 y 2. Soluci´ on. Sean f1 (z), f2 (z) y f3 (z) funciones definidas como: f1 (z) = (1 + z + z 2 ) e1/z ,

f2 (z) = (1 + z + z 2 ) e1/(z−1) ,

f3 (z) = (1 + z + z 2 ) e1/(z−2) .

As´ı, f (z) = f1 (z) + f2 (z) + f3 (z), luego, Z Z Z Z 2 1/z 1/(z−1) 1/(z−2) f3 (z)dz. f2 (z)dz + f1 (z)dz + (1 + z + z )(e + e +e ) dz = C

C

C

C

Ahora, los u ´ nicos puntos singulares de f1 (z), f2 (z) y f3 (z) son, respectivamente, 0, 1 y 2, adem´as, cada uno de ellos son puntos singulares esenciales, lo cual nos obliga a determinar el desarrollo de Laurent para calcular el residuo en tales puntos. Se tiene que el desarrollo de Laurent de f1 (z) centrado 0, v´ alido en el anillo |z| > 0, est´ a dado por ∞ ∞ ∞ ∞ X 1 −n X 1 −n X 1 1−n X 1 2−n f1 (z) = (1 + z + z ) z = z + z + z , n! n! n! n! 2

n=0

n=0

n=0

n=0

133

´ COMPLEJA CAP´ITULO 4. INTEGRACION

de donde se deduce que

10 1 1 + = . 2 6 6 Asimismo, el desarrollo de Laurent de f2 (z) centrado 1, v´alido en el anillo |z − 1| > 0, est´ a dado por Res [f1 (z)] = 1 + z=0

∞ X 1 (z − 1)−n f2 (z) = (3 + 3(z − 1) + (z − 1) ) n! 2

n=0

∞ ∞ ∞ X X X 3 1 3 −n 1−n (z − 1) + (z − 1) + (z − 1)2−n , = n! n! n! n=0 n=0 n=0

que implica

3 1 28 + = . z=1 2 6 6 Ahora, el desarrollo de Laurent de f3 (z) centrado 2, v´alido en el anillo |z − 2| > 0, est´ a dado por Res [f2 (z)] = 3 +

f3 (z) = (7 + 5(z − 2) + (z − 2)2 ) =

∞ X 1 (z − 2)−n n! n=0

∞ ∞ ∞ X X X 5 1 7 (z − 2)−n + (z − 2)1−n + (z − 2)2−n , n! n! n!

n=0

n=0

n=0

luego

58 5 1 + = . z=1 2 6 6 De esta forma, utilizando el Teorema de los Residuos obtenemos Z 10 f1 (z)dz = πi, 3 ZC 28 f2 (z)dz = πi, 3 C Z 58 πi, f3 (z)dz = 3 C Res [f2 (z)] = 7 +

en consecuencia, el valor de la integral dada es Z 10 28 58 (1 + z + z 2 )(e1/z + e1/(z−1) + e1/(z−2) ) dz = πi + πi + πi = 32πi. 3 3 3 C  Problema 4.9. Sea f (z) =

(z + i)2 + z cos (z − i − 1)(ez−1 − 1)



 z . z+1

a) Halle el residuo de f (z) en cada uno de sus puntos singulares aislados. Z f (z) dz , donde C es el contorno que se muestra b) Determine el valor de la integral C

en la siguiente figura. El punto inicial y el fine de C coinciden en α = 2.

134

4.8. PROBLEMAS RESUELTOS

y C

2 1

−2

−1 −1

1

2

x

Soluci´ on. a) Los puntos singulares de f (z) son: z0 = 1, z1 = 1 + i, z2 = −1. Como z0 y z1 son polos simples de la funci´ on h(z) =

(z + i)2 (z − i − 1)(ez−1 − 1)

y la funci´ on g(z) = z cos



z z+1



es anal´ıtica en z0 y z1 , entonces el residuo de f (z) en los puntos z0 = 0 y z1 = 1 es:   (z + i)2 (z − 1)(z + i)2 Res [f (z)] = Res = −2 = l´ ım z=z0 z=z0 (z − i − 1)(ez−1 − 1) z→1 (z − i − 1)(ez−1 − 1)   (z + i)2 −3 + 4i (z + i)2 Res [f (z)] = Res = i = l´ ım z−1 z=z1 z=z1 (z − i − 1)(ez−1 − 1) z→1+i (e − 1) e −1 El residuo de f (z) en z2 = −1, est´ a dado por:    z . Res [f (z)] = Res z cos z=z2 z=z2 z+1   z centrado en z2 = −1 es, para Ahora, el desarrollo de Laurent de g(z) = z cos z+1 |z + 1| > 0:   (z + 1) − 1 g(z) = ((z + 1) − 1) · cos z+1      1 1 = ((z + 1) − 1) cos(1) cos + sen (1)sen z+1 z+1 ∞ ∞ n X X (−1) (−1)n = cos(1) (z + 1)−2n+1 + sen (1) (z + 1)−2n (2n)! (2n + 1)! n=0 n=0 + cos(1)

∞ X (−1)n+1

n=0

(2n)!

(z + 1)−2n + sen (1)

∞ X (−1)n+1 (z + 1)−(2n+1) . (2n + 1)!

n=0

Por tanto, z2 es un punto singular esencial de g(z) y 1 Res [g(z)] = − cos(1) − sen (1) z=z2 2



1 Res [f (z)] = − cos(1) − sen (1) z=z2 2

135

´ COMPLEJA CAP´ITULO 4. INTEGRACION

b) Es claro que C no es un contorno cerrado simple, pero ´este se puede escribir como la uni´ on de dos contornos cerrados simples: C1 (trazado de color azul recorrido en sentido negativo) y C2 (trazado de color rojo recorrido en sentido positivo), que se muestran en la figura adjunta. Luego, Z Z Z f (z) dz. f (z) dz + f (z) dz = C

y

C1

C2

C1

C2

2

−2

1

−1 −1

1

2

x

Los puntos singulares de f (z), z0 = 1 y z1 = 1 + i, son puntos interiores del contorno C2 . Entonces, por el Teorema de los Residuos se tiene     Z −3 + 4i . f (z) dz = 2πi Res [f (z)] + Res [f (z)] = 2πi −2 + i z=z0 z=z1 e −1 C2 Ahora, el punto singular de f (z), z2 = −1, es un punto interior del contorno C1 . De esta forma, por el Teorema de los Residuos se tiene   Z 1 f (z) dz = −2πi Res [f (z)] = 2πi cos(1) + sen (1) . z=z2 2 C1 De todo lo anterior se obtiene que el valor de la integral es:   Z −3 + 4i 1 f (z) dz = 2πi −2 + cos(1) + sen (1) + i . 2 e −1 C 

4.9 Problemas Propuestos 4.1. Calcule la integral

Z

1/z dz si la curva C es:

C

a) el segmento de recta que va de z = i a z = 1; b) la semicircunferencia |z| = 1, −π/2 ≤ arg z ≤ π/2 (el camino se inicia en el punto z = −i). 4.2. Eval´ ue la integral Z

C

z3 dz, z2

donde C es la circunferencia |z| = 1 tomada en sentido positivo. 4.3. Eval´ ue la integral

Z

Log z dz,

C

donde C es la curva z(t) = eit , 0 ≤ t ≤ π.

136

4.9. PROBLEMAS PROPUESTOS

4.4. Eval´ ue la integral

Z

f (z) dz, |z|=2

donde f (z) es una funci´ on de variable compleja definida por   0,    z 2 , f (z) =  z,    0,

√ si Re z < − 2; √ si − 2 ≤ Re z ≤ 0; √ si 0 < Re z ≤ 2; √ si Re z > 2.

4.5. Para cada una de las siguientes integrales, diga las caracter´ısticas del contorno cerrado simple C para que el valor de la integral sea cero, seg´ un el Teorema de Cauchy-Goursat. (Justifique su respuesta.) a)

Z

b)

Z

C

cos z dz z+2

c)

Z

1 dz 1 + ez

d)

Z

1 dz 1 − ez

C

Log z dz C

C

4.6. Para la funci´ on de f (z) =

1 determine la primitiva de f (z) tal que: (z − i)2

a) est´ e definida en el dominio Re z > 0. b) sea igual a cero cuando z = i + 1. 4.7. Eval´ ue las siguientes integrales. Use la f´ ormula integral de Cauchy (o su extensi´ on) o bien el Teorema de Cauchy-Goursat donde sea necesario. En cada una de las integrales, C es un contorno cerrado simple. Z z dz, a > 1. a) 4 |z−a|=a z − 1 b)

Z

c)

Z

d)

Z

e)

Z

f)

Z

C

C

1 x2 y2 dz, donde C es el contorno + = 1. ez (z − 2) 9 16 x2 sen (ez + cos z) dz, donde C es el contorno + y 2 = 1. (z − 1)2 (z + 3) 2

|z|=1

C

C

cos(2z) dz. z 21

ez dz, donde C es un contorno cerrado simple que contiene a |z| ≤ a. z 2 + a2 z ez dz, donde a es un punto interior del contorno cerrado simple C. (z − a)3

137

´ COMPLEJA CAP´ITULO 4. INTEGRACION

4.8. En cada una de las siguientes integrales determine el m´ınimo valor de la constante positiva M que satisface Z f (z) dz ≤ M, C

para cada funci´ on f (z) considerada en cada caso. Z 1 dz a) z |z|=1

Z

b)

z2

1 dz +1

|z−i|=1/2

c)

Z

ez

|z|=2

d)

1 dz −1

Z

z dz, donde C est´ a conformado con los puntos del cuadrado con v´ertices C z−i en los puntos −1 + 2i, 1 + 2i, 1 y −1.

4.9. Determine los residuos de las siguientes funciones complejas en cada polo o en el polo indicado: a) f (z) =

2z + 1 . −z−2

z2

1 . b) f (z) = 2 z (1 − z)

e)

z+1 (z − 1)2 (z + 3)

f)

cos z , z0 = 0 z

g) c) f (z) =

d)

3z 2 + 2 . (z − 1)(z 2 + 9)

h)

z3 − z2 + z + 1 z 2 + 4z

i)

π z4 − 1 , z0 = e 4 i 4 z +1

z4

π sen z , z0 = e 3 i 2 +z +1

z , z0 = π sen z

4.10. Use el Teorema de los Residuos para evaluar las siguientes integrales: Z z dz, para a) z2 + 1 C ( i) |z| = 1/2 C= ii) |z| = 2 b)

Z

z 2 + 3i − 2 dz, para z 3 + 9z

C

C=

(

i) |z| = 1 ii) |z| = 4

138

4.9. PROBLEMAS PROPUESTOS

c)

Z

(z 2 + 2)(z 2 + 4) dz (z 2 + 1)(z 2 + 6)

para

C

C=

d)

Z

      

1 dz, z 2 (1 + z 2 )2

i) |z| = 2 ii) |z − i| = 1 iii) |z| = 4

para

C

C=

e)

Z

C

3z 2 + 2 dz, (z 2 + 4)(z − 1)

f)

C

z 2 − 2z dz, (z 2 + 4)(z − 1)2

Z

C

(z +

1)3 (z

ii) |z| = 2

(

i) |z − 2| = 2 ii) |z| = 4

para

C=

g)

i) |z| = 1/2

para

C=

Z

(

(

i) |z| = 3 ii) |z + i| = 2

1 dz, para − 1)(z − 2)

C=

(

i) |z + 1| = 1 ii) es el rect´ angulo de v´ertices en ± i, 3 ± i

Parte II

C´ alculo Operacional

139

5 Funciones de Dominios Continuo y Discreto Los conceptos de de funciones de dominio continuo (se˜ nales de tiempo continuo) o funciones de dominio discreto (se˜ nales de tiempo discreto) surgen en casi cualquier ´area de las ciencias e ingenier´ıa, en los circuitos el´ ectricos, dispositivos para comunicaciones, rob´ otica y automatizaci´ on, dispositivos biom´edicos, procesos qu´ımicos, entre otros. En este cap´ıtulo se da una introducci´ on al c´ alculo operacional, comenzando con la caracterizaci´ on de las funciones de dominio continuo, pasando luego por la convoluci´ on en el dominio continuo. En esta parte se estudia con inter´ es la funci´ on impulso unitario. Finalmente, se describen las funciones de dominio discreto y se define la convoluci´ on de funciones de dominio discreto.

5.1 Funciones de Dominio Continuo Definici´ on 5.1. Una funci´ on f que depende de una variable t se dice de dominio continuo, si t toma valores en el conjunto de los n´ umeros reales; en otras palabras, toda funci´ on definida en el conjunto de n´ umeros reales es una funci´ on de dominio continuo. Ejemplo 5.1. Las siguientes funciones son de dominio continuo: • f (t) = et , para todo t ∈ R. • f (t) = sen (t), para todo t ∈ R. • f (t) = cos(t), para todo t ∈ R. ( 0, t < 0; • f (t) = 1, t > 0.  Note que normalmente en los libros textos de se˜ nales y sistemas, a una funci´ on de dominio continuo se le denomina se˜ nal continua o se˜ nal en tiempo continuo. Tal denominaci´ on corresponde a que la representaci´ on matem´atica de una se˜ nal continua es una funci´ on definida en el conjunto de n´ umeros reales. Recuerde que una se˜ nal es cualquier fen´ omeno o magnitud f´ısica que puede ser representado de manera cuantitativa. En este material no se usa el concepto de se˜ nal, pero el lector debe tener presente la relaci´ on que existe entre se˜ nal en tiempo continuo y funciones de dominio continuo. A continuaci´ on se describen las funciones de dominio continuo de uso m´ as frecuente. 140

141

CAP´ITULO 5. FUNCIONES DE DOMINIOS CONTINUO Y DISCRETO

5.1.1 Impulso Unitario El impulso unitario o delta de Dirac no es una funci´ on ordinaria. El nombre de Dirac es en honor a Paul Adrien Maurice Dirac (1902-1984), ingeniero el´ ectrico, matem´atico y f´ısico te´ orico brit´anico. Las funciones ordinarias se especifican definiendo la relaci´ on para obtener el valor de la funci´ on (un n´ umero) para cada valor de su argumento, en alg´ un conjunto espec´ıfico. Pero, el impulso unitario se define por el efecto que produce en la interacci´ on con una funci´ on ordinaria de dominio continuo. Hay muchos fen´ omenos f´ısicos que se pueden modelar como funciones delta, por ejemplo, fuentes puntuales, cargas puntuales, cargas concentradas en estructuras, fuentes puntuales de tensi´ on y de corriente que act´ uan durante intervalos de tiempo extremadamente cortos. Matem´ aticamente la funci´ on impulso se define de la siguiente manera.

Definici´ on 5.2. La funci´ on impulso unitario es una funci´on generalizada o distribuci´on, denotada con δ(t), que satisface: Z ∞ ϕ(t)δ(t) dt = ϕ(0), −∞

para toda funci´ on ϕ(t) continua en R.

En la Figura 5.1, se aprecia la representaci´ on gr´afica de δ(t). Por supuesto, el impulso unitario no es la “flecha vertical con altura 1”, sino que esto es simplemente la representaci´ on usual de δ(t), que no puede ser justificada matem´aticamente. Esto se usa como una ayuda nemot´ecnica. δ(t) 1 t Figura 5.1. Representaci´ on gr´afica de δ(t)

La representaci´ on gr´afica de la funci´ on f (t) = A δ(t − t0 ),

con A ∈ R y t0 ∈ R,

se aprecia en la Figura 5.2. Aqu´ı, A δ(t − t0 ) se representa como una flecha vertical con altura A, cuyo extremo inicial est´ a localizado en t0 .

142

5.1. FUNCIONES DE DOMINIO CONTINUO

Aδ(t − t0 )

Aδ(t − t0 ) A

t0 t

t0

A a) A < 0

t

b) A > 0

Figura 5.2. Representaci´ on gr´afica de A δ(t − t0 ). a) A < 0; b) A > 0.

Propiedades del impulso unitario Seguidamente describimos algunas propiedades de la funci´ on δ(t). i) l´ım δ(t) = ∞. t→0

ii) δ(t) = 0, para todo t 6= 0. Z ∞ δ(t) dt = 1. iii) −∞

iv) δ(t) = δ(−t), en otras palabras, δ(−t) se comporta como δ(t) (si δ(t) se toma como una funci´ on ordinaria, esta propiedad indicar´ıa que el impulso unitario es una funci´ on par). v) Desplazamiento. Si ϕ(t) es continua en t0 ∈ R, entonces Z ∞ ϕ(t)δ(t − t0 ) dt = ϕ(t0 ). −∞

Esta propiedad tambi´ en se puede aplicar en un intervalo [t1 , t2 ], ( Z t2 ϕ(t0 ), si t0 ∈ [t1 , t2 ], ϕ(t)δ(t − t0 ) dt = 0, si t0 6∈ [t1 , t2 ]. t1 vi) Muestreo. Si ϕ(t) es continua en t0 ∈ R, entonces ϕ(t) δ(t − t0 ) = ϕ(t0 ) δ(t − t0 ). vii) Escalado. Para a ∈ R se cumple δ(at) =

1 δ(t). |a|

Particularmente, para a, t0 ∈ R se cumple   t0 1 δ t− δ(at − t0 ) = . |a| a

CAP´ITULO 5. FUNCIONES DE DOMINIOS CONTINUO Y DISCRETO

143

viii) Derivada en´esima. Si ϕ(t) es n veces continuamente diferenciable en t0 ∈ R, entonces Z ∞ ϕ(t)δ(n) (t − t0 ) dt = (−1)n ϕ(n) (t0 ). −∞

Esta propiedad tambi´ en se puede aplicar en un intervalo [t1 , t2 ], ( Z t2 (−1)n ϕ(n) (t0 ), si t0 ∈ [t1 , t2 ], ϕ(t)δ(n) (t − t0 ) dt = 0, si t0 6∈ [t1 , t2 ]. t1

Ejercicio 5.1. Demostrar cada una de las propiedades del impulso unitario. Ejemplo 5.2. Eval´ ue las siguientes expresiones: a) 3t4 δ(t − 1) b) tδ(3 − 2t) Z ∞ sen (t2 ) δ′ (t − π 1/2 ) dt c) −∞

Soluci´ on. a) Aplicando la propiedad de muestreo se tiene: 3t4 δ(t − 1) = [3t4 ] t=1 δ(t − 1) = 3δ(t − 1).

b) Se tiene que

 3 . tδ(3 − 2t) = t δ −2 t − 2 Ahora, aplicando la propiedad de escalado seguido de la propiedad de muestreo, se obtiene:         1 t 3 3 3 3 tδ(3 − 2t) = t δ t− δ t− = = δ t− . | − 2| 2 2 t=3/2 2 4 2 



c) Aplicando la propiedad de la derivada n-´esima se tiene:   Z ∞   √ d 2 2 ′ 1/2 = (−1) 2t cos(t2 ) t=π1/2 = 2 π. sen (t ) sen (t )δ (t − π ) dt = (−1) dt −∞ t=π 1/2



5.1.2 Escal´ on Unitario La funci´ on escal´ on unitario o funci´on escal´ on de Heaviside es una funci´ on cuyo valor es 0 para cualquier argumento negativo y es 1 si el argumento es positivo. El nombre Heaviside es en honor a Oliver Heaviside (1850-1925), f´ısico, ingeniero el´ ectrico, radiotelegrafista y matem´atico ingl´ es. Matem´ aticamente el escal´ on unitario se define de la siguiente forma. Definici´ on 5.3. El escal´ on unitario, denotado por u(t), se define como ( 0, t < 0, u(t) = 1, t > 0, on u(t − t0 ) La representaci´ on gr´afica de u(t) se aprecia en la Figura 5.3. La funci´ representa un desplazamiento en tiempo de la funci´ on escal´ on unitario, para t0 ∈ R, lo cual se muestra con un ejemplo gr´afico en la Figura 5.4.

144

5.1. FUNCIONES DE DOMINIO CONTINUO

u(t) 1 t Figura 5.3. Representaci´ on gr´afica de u(t) u(t − t0 ) 1 t

t0

Figura 5.4. Representaci´ on gr´afica de u(t − t0 )

5.1.3 Pulso Rectangular A partir de la definici´ on del escal´ on unitario se pueden definir otras funciones b´asicas de gran utilidad en an´alisis de sistemas, como lo es el pulso rectangular. Un pulso rectangular de duraci´ on 2T con T > 0, se puede concebir como una funci´ on que escoge dos valores espec´ıficos: inicialmente el pulso rectangular es igual a cero hasta cierto argumento −T , luego cambia abruptamente a 1 manteni´ endose en este valor hasta un argumento T , para luego cambiar su valor a 0. Matem´ aticamente, la definci´ on del pulso rectangular es la siguiente. Definici´ on 5.4. El pulso rectangular, denotado por pT (t), se define para T > 0 como ( 1, |t| < T, pT (t) = 0, |t| > T. La representaci´ on gr´afica de pT (t) se muestra en la Figura 5.5. pT (t) 1

−T

T

t

Figura 5.5. Representaci´ on gr´afica de pT (t)

El pulso rectangular se puede escribir como una suma de dos funciones escalones: pT (t) = u(t + T ) − u(t − T ).

145

CAP´ITULO 5. FUNCIONES DE DOMINIOS CONTINUO Y DISCRETO

En efecto, las funciones u(t + T ) y u(t − T ) est´ an definidas respectivamente por: ( ( 0, t < T, 0, t < −T, y u(t − T ) = u(t + T ) = 1, t > T, 1, t > −T, luego, se sigue que u(t + T ) − u(t − T ) = pT (t).

5.1.4 Pulso Triangular El pulso triangular tiene un comportamiento similar al pulso rectangular pT (t). El pulso triangular inicialmente es cero hasta llegar a −T , luego toma el valor de una recta con pendiente T −1 hasta llegar a 0, luego toma el valor de una recta con pendiente −T −1 hasta llegar a T , para luego cambiar su valor a 0. Matem´ aticamente, el pulso triangular se define de la siguiente forma. Definici´ on 5.5. El pulso triangular, denotado por qT (t), se define para T > 0 como  |t|  1 − , |t| ≤ T, qT (t) = T 0, |t| > T, La representaci´ on gr´afica de qT (t) se muestra en la Figura 5.6. qT (t) 1

−T

t

T

Figura 5.6. Representaci´ on gr´afica de qT (t) El pulso triangular se puede expresar a trav´ es de funciones escalones o pulsos rectangulares, multiplicadas por una recta. He aqu´ı algunas representaciones u ´ tiles.     t t pT (t) = 1 + (u(t + T ) − u(t)) + 1 − (u(t) − u(t − T )) T T

o

pT (t) =



t 1+ T



pT /2



T t+ 2





t + 1− T



pT /2



T t− 2



Se deja como ejercicio para el lector demostrar que las representaciones anteriores son ciertas.

5.1.5 Funci´ on Signo La funci´ on signo adquiere el signo del argumento, es decir, ella adquiere el valor −1 cuando el argumento es negativo y es igual a 1 si el argumento es positivo; en cero la funci´ on signo no est´ a definida, por supuesto, ya que el cero no tiene signo. La definici´ on matem´atica de la funci´ on signo es la siguiente.

146

5.1. FUNCIONES DE DOMINIO CONTINUO

Definici´ on 5.6. La funci´ on signo, denotada por sgn(t), se define como ( −1, t < 0, sgn(t) = 1, t > 0. on de sgn(t) La representaci´ on gr´afica de sgn(t) se aprecia en la Figura 5.7. La definici´ permite inferir que ella se puede expresar como la suma de dos funciones escal´ on, a saber: sgn(t) = u(t) − u(−t)

o, equivalentemente,

sgn(t) = −1 + 2 u(t).

sgn(t) 1 t −1

Figura 5.7. Representaci´ on gr´afica de sgn(t) Tambi´ en el escal´ on unitario se puede definir utilizando la funci´ on signo: u(t) =

1 1 + sgn(t). 2 2

5.1.6 Pulso Exponencial El pulso exponencial se obtiene como la multiplicaci´ on de una funci´ on exponencial por el escal´ o unitario. Matem´ aticamente, el pulso exponencial se define de la siguiente forma. Definici´ on 5.7. El pulso exponencial, denotado por Exp (t), se define, para A > 0 y α > 0, como ( 0, t<0 Exp (t) = −αt A e , t ≥ 0, La representaci´ on gr´afica de Exp (t) se aprecia en la Figura 5.8. Exp (t) A

t Figura 5.8. Representaci´ on gr´afica de Exp (t)

147

CAP´ITULO 5. FUNCIONES DE DOMINIOS CONTINUO Y DISCRETO

5.1.7 Funci´ on Rampa La funci´ on rampa tiene un comportamiento similar al escal´ on unitario. La funci´ on rampa inicialmente es cero hasta llegar a 0, luego toma el valor de la recta t. La funci´ on rampa se puede ver como la multiplicaci´ on de la funci´ on t por el escal´ o unitario. Definici´ on 5.8. La funci´ on rampa, denotada por r(t), se define como ( 0, t < 0, r(t) = t, t ≥ 0, La representaci´ on gr´afica de r(t) se aprecia en la Figura 5.9. r(t) 2 1 1

2

t

Figura 5.9. Representaci´ on gr´afica de r(t)

Es claro que la funci´ on rampa se puede expresar como: r(t) = tu(t). Adem´ as, de las definiciones de r(t) y u(t) se obtienen las siguientes relaciones:

r(t) =

Z

t

−∞

u(τ ) dτ

y

u(t) =

d r(t). dt

5.1.8 Funciones Sa y sinc La funci´ on de muestreo es de mucha utilidad en an´alisis de sistemas y, particularmente, en el muestreo de se˜ nales. La definici´ on matem´atica de la funci´ on de muestreo es la siguiente. Definici´ on 5.9. La funci´ on de muestreo, denotada por Sa (t), se define como  1, t = 0, Sa (t) = sen t  , t 6= 0, t La representaci´ on gr´afica de Sa (t) se aprecia en la siguiente figura.

148

5.1. FUNCIONES DE DOMINIO CONTINUO

Sa (t) 1

t −6π

−5π

−4π

−3π

−2π

−π

π











Figura 5.10. Representaci´ on gr´afica de Sa (t) La funci´ on seno cardinal o simplemente funci´ on sinc est´ a estrechamente relacionada con la funci´ on de muestreo. La definici´ on matem´atica de la funci´ on seno cardinal es la siguiente. Definici´ on 5.10. La funci´ on sinc (t) est´ a definida como  t = 0, 1, sinc (t) = sen (πt)  , t 6= 0. πt

Es claro que sinc (t) = Sa (πt). La representaci´ on gr´afica de sinc (t) se muestra en la siguiente figura. sinc (t) 1

t −6

−5

−4

−3

−2

−1

1

2

3

4

5

6

Figura 5.11. Representaci´ on gr´afica de sinc (t)

5.1.9 Relaci´ on entre u(t) y δ(t) Las funciones escal´ on unitario e impulso unitario est´ an relacionadas de la siguiente manera: Z t d δ(τ ) dτ y δ(t) = u(t). u(t) = dt −∞ Demostremos estas relaciones. Primero verifiquemos que la integral h(t) =

Z

t

δ(τ ) dτ −∞

se comporta como el escal´ on unitario, en otras palabras, verificaremos que h(t) = 0, para t < 0 y h(t) = 1, para t > 0. Como δ(t) = 0 para todo t 6= 0, entonces es claro que h(t) = 0,

149

CAP´ITULO 5. FUNCIONES DE DOMINIOS CONTINUO Y DISCRETO

para t < 0. Ahora, para t > 0 podemos escribir: h(t) =

Z

t

δ(τ ) dτ = −∞

Z

0−

δ(τ ) dτ +

−∞

pero δ(t) = 0 para todo t 6= 0 y

Z

luego

Z

0+

δ(τ ) dτ + 0−

Z

t

δ(τ ) dτ,

0+

0+

δ(τ ) dτ = 1, 0−

h(t) = 1,

para t > 0.

Por lo tanto, todo lo anterior nos indica que h(t) se comporta como el escal´ on unitario, es decir, Z t

δ(τ ) dτ.

u(t) =

−∞

Ahora, derivando en ambos lados de la ecuaci´ on anterior se tiene que δ(t) =

d u(t). dt

5.1.10 Derivada Generalizada La expresi´ on de la derivada generalizada de una funci´ on se deduce del hecho de que toda funci´ on diferenciable a trozos o, en general, toda funci´ on continua a trozos, puede expresarse como una suma de funciones que involucran escalones unitarios. Definici´ on 5.11. Sea f (t) una funci´ on diferenciable a trozos en R, esto es, f (t) es derivable en todo R excepto en t1 , t2 , . . . , tn ∈ R, adem´as, en cada uno de estos puntos f (t) tiene saltos h1 , h2 , . . . , hn ∈ R, respectivamente. La derivada generalizada de f (t) se define como n X d ′ hk δ(t − tk ). (5.1) + f (t) = f (t) dt t6=tk k=1

Observe el siguiente ejemplo.

Ejemplo 5.3. Sea f (t) la funci´ on dada por   0,    −t   e ; f (t) = t,    3,     1,

t ≤ −1, −1 < t ≤ 0, 0 < t ≤ 2, 2 < t ≤ 3, t > 3.

cuya gr´afica se muestra en la Figura 5.12. Note que f (t) no es diferenciable en los puntos t1 = −1, t2 = 0, t3 = 2 y t4 = 3, con saltos h1 = e, h2 = −1, h3 = 1 y h4 = −2, respectivamente. As´ı, por (5.1) la derivada generalizada de f (t) es f ′ (t) = g(t) + e δ(t + 1) − δ(t) + δ(t − 2) − 2δ(t − 3),

(5.2)

150

5.1. FUNCIONES DE DOMINIO CONTINUO

f (t) 3 2 1

-1

1

2

3

4

t

Figura 5.12. Representaci´ on gr´afica de f (t)

donde  −t  −e , −1 < t < 0, g(t) = 1, 0 < t < 2,   0, en otros casos. Determinemos la derivada generalizada de f (t) por otro procedimiento, usando el hecho que f (t) puede expresarse equivalentemente como: f (t) = e−t (u(t + 1) − u(t)) + t(u(t) − u(t − 2)) + 3(u(t − 2) − u(t − 3)) + u(t − 3). Recuerde la regla de derivaci´ on del producto

Ahora, derivando la expresi´ on anterior se tiene:

f ′ (t) =

 d d d  −t e (u(t + 1) − u(t)) + [t(u(t) − u(t − 2))] + [3(u(t − 2) − u(t − 3))] dt dt dt d + u(t − 3) dt

= − e−t (u(t + 1) − u(t)) + e−t (δ(t + 1) − δ(t)) + (u(t) − u(t − 2)) + t(δ(t) − δ(t − 2)) + 3(δ(t − 2) − δ(t − 3)) + δ(t − 3)

= − e−t (u(t + 1) − u(t)) + e δ(t + 1) − δ(t) + (u(t) − u(t − 2)) − 2 δ(t − 2) + 3 δ(t − 2) − 3 δ(t − 3) + δ(t − 3)

= − e−t (u(t + 1) − u(t)) + (u(t) − u(t − 2)) + e δ(t + 1) − δ(t) + δ(t − 2) − 2δ(t − 3), que es equivalente a la expresi´ on dada en (5.2).



151

CAP´ITULO 5. FUNCIONES DE DOMINIOS CONTINUO Y DISCRETO

5.1.11 Convoluci´ on en el Dominio Continuo La convoluci´ on es una operaci´ on entre funciones de dominio continuo de mucha utilidad en aplicaciones en ingenier´ıa y, especialmente, en an´alisis de sistemas. La definici´ on matem´atica de la convoluci´ on de funciones de dominio continuo es la siguiente. Definici´ on 5.12. Sean f y g dos funciones de dominio continuo. La convoluci´ on entre f y g es una funci´ on de dominio continuo, denotada por f ∗ g, la cual est´ a definida como: Z ∞ f (τ ) g(t − τ ) dτ, para t ∈ R. f ∗ g (t) = −∞

En la siguiente proposici´ on se establecen algunas propiedades de la convoluci´ on. Proposici´ on 5.1. Sean f , g y h funciones de dominio continuo. Entonces, las siguientes propiedades se cumplen. i) f ∗ g = g ∗ f .

(Conmutatividad)

ii) f ∗ (g + h) = f ∗ g + f ∗ h.

(Distributiva con respecto a la suma)

iii) Si h(t) = f ∗ g(t) es la convoluci´ on de las funciones de dominio continuo f (t) y g(t), entonces f (t − t0 ) ∗ g(t) = h(t − t0 ). Demostraci´ on. i) Se tiene que f ∗ g(t) =

Z



−∞

f (τ ) g(t − τ ) dτ ;

ahora, haciendo el cambio de variable λ = t − τ se obtiene: Z ∞ Z −∞ f (t − λ) g(λ) dλ = g ∗ f (t). f (t − λ) g(λ) dλ = f ∗ g(t) = − −∞



Demostraci´ on de ii). Se tiene que Z ∞ f (τ ) [g(t − τ ) + h(t − τ )] dτ f ∗ (g + h)(t) = −∞ Z ∞ Z ∞ f (τ ) h(t − τ ) dτ f (τ ) g(t − τ ) dτ + = −∞

= f ∗ g(t) + f ∗ h(t).

−∞

Demostraci´ on de iii). Sean h(t) = f ∗ g(t) y t0 . Por definici´ on, la convoluci´ on f (t − t0 ) ∗ g(t) est´ a dada por: Z f (t − t0 ) ∗ g(t) =



−∞

f (τ − t0 ) g(t − τ ) dτ ;

ahora, haciendo el cambio de variable λ = τ − t0 se obtiene: Z ∞ f (λ) g((t − t0 ) − λ) dλ = h(t − t0 ). f (t − t0 ) ∗ g(t) = −∞

Esto completa la demostraci´ on.

152

5.1. FUNCIONES DE DOMINIO CONTINUO

En los siguientes ejemplos se calcula la convoluci´ on entre dos funciones de dominio continuo. Ejemplo 5.4. Sean f (t) = p1 (t) y g(t) = u(t). Determinar la convoluci´ on f ∗ g. Soluci´ on. Tomemos h(t) = f ∗ g (t). Considerando las expresiones de f (t) y g(t), se puede escribir: Z ∞ p1 (τ ) u(t − τ ) dτ h(t) = f ∗ g (t) = Z

=

−∞ 1−

−1+

u(t − τ ) dτ,

para cada t ∈ R.

Determinemos la expresi´ on de h(t) para: t ≤ −1, −1 < t < 1 y t ≥ 1. • Para t ≤ −1, la gr´afica de la funci´ on u(t − τ ) es u(t − τ ) 1

-2

t -1

1

2

τ

2

τ

Entonces, por (5.3), h(t) = 0, para t ≤ −1. • Para −1 < t < 1, la gr´afica de u(t − τ ) es u(t − τ ) 1

-2

Entonces, por (5.3), h(t) =

Z

-1

t

1

t −1+

dτ = t + 1, para −1 < t < 1.

• Para t ≥ 1, la gr´afica de u(t − τ ) es 1

-2

Entonces, por (5.3), h(t) =

Z

-1

u(t − τ )

1

t 2

1− −1+

dτ = 2, para t ≥ 1.

τ

(5.3)

153

CAP´ITULO 5. FUNCIONES DE DOMINIOS CONTINUO Y DISCRETO

De esta forma, la expresi´ on anal´ıtica de la convoluci´ on entre f y g est´ a dada por   t ≤ −1, 0, h(t) = t + 1, −1 < t < 1,   2, t ≥ 1, cuya representaci´ on gr´afica es

h(t) 2 1

-2

-1

1

t

2

 Ejemplo 5.5. Sean f y g las funciones de dominio continuo cuyas gr´aficas se muestran en la siguiente figura. f (t)

g(t)

1

1

-1

1

t

-1

t

1

Determinar la convoluci´ on f ∗ g. Soluci´ on. Tomemos h(t) = f ∗ g (t). Se tiene que f y g se pueden expresar como f (t) = (1 − t)(u(t) − u(t − 1))

g(t) = u(t) − u(t − 1).

y

Luego, considerando la expresi´ on de f (t), se puede escribir, para cada t ∈ R: Z 1− Z ∞ f (τ ) g(t − τ ) dτ = (1 − τ )g(t − τ ) dτ. h(t) = f ∗ g (t) = 0+

−∞

Adem´ as, para cada t ∈ R, la funci´ on g(t − τ ) se puede escribir como g(t − τ ) = u(t − τ ) − u((t − 1) − τ ),

para τ ∈ R.

Determinemos la expresi´ on de h(t) para: t ≤ 0, 0 < t < 1, 1 ≤ t < 2 y t ≥ 2. • Para t ≤ 0, la gr´afica de g(t − τ ) es 1

t−1

t

g(t − τ )

1

2

τ

(5.4)

154

5.1. FUNCIONES DE DOMINIO CONTINUO

Entonces, por (5.4), h(t) = 0, para t ≤ 0. • Para 0 < t < 1, la gr´afica de g(t − τ ) es 1

g(t − τ )

t−1

t

1

2

τ

Entonces, por (5.4)  t Z t  (τ − 1)2 1 (1 − τ ) dτ = − h(t) = = 1 − (t − 1)2 , 2 2 0+ 0

para 0 < t < 1.

• Para 1 ≤ t < 2, la gr´afica de g(t − τ ) es 1

g(t − τ )

t−1

1 t

2

τ

Entonces, por (5.4) Z

1−



(τ − 1)2 (1 − τ ) dτ = − h(t) = 2 t−1

1

t−1

=

(t − 2)2 , 2

para 1 ≤ t < 2.

• Para t ≥ 2, la gr´afica de g(t − τ ) es 1

g(t − τ )

1t−12 t

τ

Entonces, por (5.4), h(t) = 0, para t ≥ 2. De esta forma, la expresi´ on anal´ıtica de la convoluci´ on h(t) = f ∗ g (t) est´ a dada por:   0, t ≤ 0,        1 1 − (t − 1)2 , 0 < t < 1, 2 h(t) = 1  2  1 ≤ t < 2,  2 (t − 2) ,     0, t ≥ 2, cuya representaci´ on gr´afica se aprecia en la siguiente figura.

155

CAP´ITULO 5. FUNCIONES DE DOMINIOS CONTINUO Y DISCRETO

h(t) 1 1/2 -3

-2

-1

1

2

3

t 

5.2 Funciones de Dominio Discreto Una funci´ on f que depende de una variable n se dice de dominio discreto, si n toma valores en el conjunto de los n´ umeros enteros; en otras palabras, toda funci´ on definida en el conjunto de n´ umeros enteros es una funci´ on de dominio discreto. Una de las formas m´ as comunes en las que surgen funciones de dominio discreto es discretizando funciones de dominio continuo. Ejemplo 5.6. Seguidamente mostramos funciones de dominio discreto que se obtienen respectivamente como la discretizaci´ on de las funciones de dominio continuo: f (t) = et , f (t) = sen (t), f (t) = ln(t), f (t) = t2 y u(t). • f (n) = en , para todo n ∈ Z. • f (n) = sen (n), para todo n ∈ Z. • f (n) = ln(n), para todo n ≥ 0. • f (n) = n2 , para todo n ∈ Z. ( 0, n < 0, • f (n) = 1, n ≥ 0. 

5.2.1 Impulso Unitario Discreto El impulso unitario discreto es la contraparte discreta del impulso unitario continuo, pero el impulso unitario discreto no puede verse como la discretizaci´ on de δ(t), ya que δ(0) no est´ a definido. Definici´ on 5.13. El impulso unitario discreto, denotado por δ(n), se define como ( 1, n = 0, δ(n) = 0, n 6= 0, La gr´afica de δ(n) se aprecia en la Figura 5.13.

Sean t0 , t1 , . . . . La funci´ on g(n) = f (tn ) se denomina discretizaci´ on de la funci´ on f (t)

156

5.2. FUNCIONES DE DOMINIO DISCRETO

δ(n) 1

b

b

b

b

b

b

b

b

b

-4

-3

-2

-1

1

2

3

4

n

Figura 5.13. Gr´afica de δ(n)

5.2.2 Escal´ on Unitario Discreto El escal´ on unitario discreto puede obtenerse discretizando el impulso unitario, a pesar de que u(0) no est´ a definido. En este caso, el escal´ on unitario discreto alcanza el valor 1 en 0, es decir, toma el valor del l´ımite por la derecha de u(t) cuando t tiende a 0. Definici´ on 5.14. El escal´ on unitario discreto, denotado por u(n), se define como ( 0, n < 0, u(n) = 1, n ≥ 0, La gr´afica de u(n) se muestra en la Figura 5.14. u(n) 1 b

b

b

b

-4

-3

-2

-1

b

b

b

b

b

··· 1

2

3

4

n

Figura 5.14. Gr´afica de u(n)

5.2.3 Funci´ on Rampa Discreta La funci´ on rampa discreta se obtiene discretizando la funci´ on rampa continua. Definici´ on 5.15. La funci´ on rampa discreta, denotada por r(n), se define como ( 0, n < 0, r(n) = n, n ≥ 0, La gr´afica de r(n) se aprecia en la Figura 5.15. Las funciones escal´ on unitario discreto y la rampa discreta est´ an relacionadas de la siguiente manera: r(n) =

n X

k=−∞

u(k)

y

u(n) = r(n) − r(n − 1).

Se deja como ejercicio para el lector demostrar estas relaciones.

157

CAP´ITULO 5. FUNCIONES DE DOMINIOS CONTINUO Y DISCRETO

r(n) b

3 b

2

b

b

b

b

-4

-3

-2

-1

··· b

1 b

1

2

3

n

Figura 5.15. Gr´afica de r(n)

5.2.4 Funci´ on Signo Discreto La funci´ on signo discreto puede obtenerse discretizando de la funci´ on signo de dominio continuo, a pesar de que sgn(0) no est´ a definido. En este caso la funci´ on signo discreto discreto alcanza el valor 0 en 0. Definici´ on 5.16. La funci´ on signo discreto, denotado por sgn(n), se define como   −1, n < 0, sgn(n) = 0, n = 0,   1, n > 0, La gr´afica sgn(n) se aprecia en la siguiente figura. sgn(n) b

1 −4

···

−3

−2

−1

b

b

b

b

b

··· b

1 b

b

2

3

4

n

-1

Figura 5.16. Gr´afica de sgn(n)

Igual que en el dominio continuo, las funciones u(n) y sgn(n) tambi´ en est´ an estrechamente relacionadas. La definici´ on de sgn(n) permite inferir que ella se puede expresar como la suma de dos funciones escal´ on, a saber:

sgn(n) = u(n) − u(−n). Se deja como ejercicio para el lector demostrar esta relaci´ on.

158

5.2. FUNCIONES DE DOMINIO DISCRETO

5.2.5 Relaci´ on entre u(n) y δ(n) Las funciones escal´ on unitario discreto e impulso unitario discreto est´ an relacionadas de la siguiente manera:

u(n) =

n X

δ(k)

y

k=−∞

δ(n) = u(n) − u(n − 1).

Demostremos estas relaciones. Primero veamos que la suma

f (n) =

n X

δ(k)

k=−∞

se comporta como el escal´ on unitario discreto. Como δ(k) = 0 para todo k 6= 0, entonces es claro que f (n) = 0 para n < 0. Para n = 0, se tiene que

f (0) =

−1 X

δ(k) + δ(0) = 1.

k=−∞

Ahora, para n > 0 se obtiene

f (n) =

−1 X

δ(k) + δ(0) +

n X

δ(k) = 1.

k=1

k=−∞

Por lo tanto, todo lo anterior nos dice que f (n) se comporta como el escal´ on unitario discreto, es decir, u(n) =

n X

δ(k).

k=−∞

Veamos ahora que la funci´ on g(n) = u(n) − u(n − 1) se comporta como el impulso unitario discreto. Como u(n) = 0 para todo n < 0, entonces es claro que g(n) = 0 para n < 0. Para n = 0, se tiene que g(0) = u(0) − u(−1) = 1. Ahora, se tiene que u(n) = 1 y u(n − 1) = 1 para n ≥ 1; luego, g(n) = 0 para n ≥ 1. Por lo tanto, todo lo anterior nos indica que g(n) se comporta como el impulso unitario discreto, es decir, δ(n) = u(n) − u(n − 1).

159

CAP´ITULO 5. FUNCIONES DE DOMINIOS CONTINUO Y DISCRETO

5.2.6 Convoluci´ on en el Dominio Discreto La convoluci´ on en el dominio discreto es un concepto de mucha utilidad en an´alisis de sistemas, especialmente, en el procesamiento digital de se˜ nales. La definici´ on matem´atica de la convoluci´ on de funciones de dominio discreto es la siguiente. Definici´ on 5.17. Sean f y g dos funciones de dominio discreto. La convoluci´ on entre f y g, denotada por f ∗ g, es una funci´ on de dominio discreto definida como f ∗ g (n) =

∞ X

k=−∞

f (k) g(n − k),

para n ∈ Z.

En la siguiente proposici´ on se establecen algunas propiedades de la convoluci´ on de funciones de dominio discreto. Proposici´ on 5.2. Sean f , g y h funciones de dominio discreto. Entonces, las siguientes propiedades se cumplen. i) f ∗ g = g ∗ f .

(Conmutatividad)

ii) f ∗ (g + h) = f ∗ g + f ∗ h.

(Distributiva con respecto a la suma)

iii) Si h(n) = f ∗ g(n) es la convoluci´ on de las funciones de dominio discreto f (n) y g(n), entonces f (n − n0 ) ∗ g(n) = h(n − n0 ). Demostraci´ on. i) Se tiene que f ∗ g(n) =

∞ X

k=−∞

f (k) g(n − k);

ahora, haciendo el cambio m = n − k en la sumatoria, se obtiene: f ∗ g(n) =

−∞ X

m=∞

f (n − m) g(m) =

∞ X

m=−∞

g(m) f (n − m) = g ∗ f (n).

Demostraci´ on de ii). Se tiene que f ∗ (g + h)(n) = =

∞ X

k=−∞ ∞ X

k=−∞

f (k) [g(n − k) + h(n − k)] f (k) g(n − k) +

= f ∗ g(n) + f ∗ h(n).

∞ X

k=−∞

f (k) h(n − k)

Demostraci´ on de iii). Sean h(n) = f ∗ g(n) y n0 . Por definici´ on, la convoluci´ on f (n − n0 ) ∗ g(n) est´ a dada por: f (n − n0 ) ∗ g(n) =

∞ X

k=−∞

f (k − n0 ) g(n − k);

160

5.2. FUNCIONES DE DOMINIO DISCRETO

ahora, haciendo el cambio m = k − n0 en la sumatoria, se obtiene: f (n − n0 ) ∗ g(n) =

∞ X

k=−∞

f (m) g((n − n0 ) − m) = h(n − n0 ).

Esto completa la demostraci´ on. En los siguientes ejemplos se calcula la convoluci´ on entre dos funciones de dominio discreto. Ejemplo 5.7. Determinar la convoluci´ on f ∗ g, donde f (n) = u(n − 2) − u(n − 4) y

g(n) = u(n).

Soluci´ on. Considerando las expresiones de f (n) y g(n), se tiene que la expresi´ on anal´ıtica de la convoluci´ on h(n) = f ∗ g (n), para cada n ∈ Z, es: h(n) = f ∗ g (n) = =

∞ X

k=−∞ 3 X k=2

f (k)g(n − k)

g(n − k)

= g(n − 2) + g(n − 3)

= u(n − 2) + u(n − 3), cuya representaci´ on gr´afica se muestra en la siguiente figura. h(n) b

2

b

b

b

-3

-2

-1

b

b

b

1

···

b

b

··· b

1

2

3

4

5

6

n 

Ejemplo 5.8. Si f (n) = (1/3)n u(n) y g(n) = 2n u(−n), entonces determine la convoluci´ on f ∗ g. Soluci´ on. Tomemos h(n) = f ∗ g (n). Considerando las expresiones de f (n) y g(n), se tiene que h(n) = f ∗ g (n) =

∞ X

f (k)g(n − k) =

k=−∞ ∞ X −k n

6

= 2

k=0

∞ X

u(k − n),

Determinemos la expresi´ on de h(n) para: n ≤ 0 y n > 0.

k=0

3−k 2n−k u(k − n)

para todo n ∈ Z.

(5.5)

161

CAP´ITULO 5. FUNCIONES DE DOMINIOS CONTINUO Y DISCRETO

• Para n ≤ 0, la gr´afica de u(n − k) es

b

b

b

u(k − n)

1 b

b

b

b

··· b

n

n−1

··· −1

1

!

= (6/5)2n ,

2

k

Entonces, por (5.5) ∞ X (1/6)k = 2n h(n) = 2

1 1−

n

k=0

1 6

para n ≤ 0.

• Para n > 0, la gr´afica de u(n − k) es u(k − n) b

1 b

b

b

b

··· b

b

··· b

n

n−1

1

b

n+1 n+2

k

Entonces, por (5.5) h(n) = 2n = 2n n

= 2

∞ X

(1/6)k

k=n "∞ X

"

k=0

(1/6)k −

1 1−

1 6

n−1 X

(1/6)k

k=0

1 − (1/6)n − 1 − 16

#

#

= (6/5)2n (1/6)n = (6/5)3−n ,

para n > 0.

De esta forma, la expresi´ on anal´ıtica de la convoluci´ on h(n) = f ∗ g (n) est´ a dada por h(n) = (6/5)2n u(−1 − n) + (6/5)3−n u(n), cuya representaci´ on gr´afica se aprecia en la siguiente figura. h(n) b

1 b

b

b

-5

-4

b

-3

b b

-2

b

-1

1

2

b b

b

3

4

5

n 

162

´ AL CALCULO ´ 5.3. INTRODUCCION OPERACIONAL CON MATLAB

5.3 Introducci´ on al C´ alculo Operacional con M ATLAB Comencemos utilizando la herramienta de matem´atica simb´ olica de M ATLAB para trabajar con funciones de dominio continuo y calcular la convoluci´ on en el dominio continuo. Una funci´ on de dominio continuo f (t) se puede crear como una funci´ on simb´ olica. Primero se crea el objeto simb´ olico t, luego se define la funci´ on simb´ olica f(t) seg´ un su forma algebraica. Observe el siguiente ejemplo, donde creamos la funci´ on f (t) = t2 (sen (t) + e−t ). ✄ >> syms t >> f ( t ) = t ˆ2∗( s i n ( t )+exp(− t ) ) f(t) = t ˆ2∗( exp(− t ) + s i n ( t ) )





El comando ezplot(f,[tmin,tmax]) dibuja la funci´ on f (t) en el intervalo [tmin, tmax]. 2 Por ejemplo, la gr´afica de la funci´ on f (t) = t (sen (t) + e−t ), creada previamente, en el intervalo [−5, 5] se obtiene de la siguiente forma: ✄ >> e z p l o t ( f , [ − 5 , 5 ] ) , t i t l e ( ’ t ˆ2( s i n ( t )+exp(− t ) ) ’ )





t2(sin(t)+exp(−t))

1000

800

600

400

200

0

−5

−4

−3

−2

−1

0 t

1

2

3

4

5

Las funciones simb´ olicas creadas en M ATLAB se pueden operar entre s´ı, diferenciar, integrar, etc. Ahora, las funciones elementales ya est´ an predefinidas y las funciones como el impulso unitario y el escal´ on unitario, tambi´ en est´ an predefinidas. El impulso unitario, δ(t), es dirac(t), y el escal´ on unitario, u(t), es heaviside(t). La gr´afica de u(t) en el intervalo [−10, 10], se obtiene de la siguiente forma: ✄ >> syms t >> e z p l o t ( h e a v i s i d e ( t ) ,[ −10 ,10])





163

CAP´ITULO 5. FUNCIONES DE DOMINIOS CONTINUO Y DISCRETO

heaviside(t)

1

0.8

0.6

0.4

0.2

0

−10

−8

−6

−4

−2

0 t

2

4

6

8

10

Las otras funciones de dominio continuo de uso com´ un, por ejemplo, la funci´ on signo sgn(t), la rampa r(t), el pulso rectangular pT (t) o el pulso triangular qT (t), se pueden definir a trav´ es de su relaci´ on con el escal´ on unitario. Observe el siguiente ejemplo donde se definen las funciones sgn(t), r(t), p1 (t) y q1 (t). ✄ >> syms t >> u ( t ) = h e a v i s i d e ( t ) ; >> sgn ( t ) = u ( t )−u(− t ) sgn ( t ) = h e a v i s i d e ( t ) − h e a v i s i d e (− t ) >> r ( t ) = t ∗u ( t ) r(t) = t ∗heaviside ( t ) >> p1 ( t ) = u ( t +1)−u ( t −1) p1 ( t ) = h e a v i s i d e ( t + 1) − h e a v i s i d e ( t − 1) >> q1 ( t ) = (1+ t ) ∗ ( u ( t +1)−u ( t ))+(1− t ) ∗ ( u ( t )−u ( t −1)) q1 ( t ) = ( t − 1)∗( h e a v i s i d e ( t − 1) − h e a v i s i d e ( t ) ) + ( t + 1)∗( h e a v i s i d e ( t + 1) − h e a v i s i d e ( t ) )





Tambi´ en se puede definir la composici´ on de funciones de dominio continuo. Observe el siguiente ejemplo donde se define la funci´ on f (t) = u(cos(t)) y se obtiene su gr´afica en el intervalo [−6π, 6π]. ✄ >> syms t >> f ( t ) = h e a v i s i d e ( c o s ( t ) ) ; >> e z p l o t ( f ,[ −6∗ pi , 6 ∗ p i ] )





Utilice el comando ezplot para obtener las gr´aficas de las funciones sgn(t), r(t), p1 (t) y q1 (t), en el intervalo [−10, 10]

164

´ AL CALCULO ´ 5.3. INTRODUCCION OPERACIONAL CON MATLAB

heaviside(cos(t))

1

0.8

0.6

0.4

0.2

0

−15

−10

−5

0 t

5

10

15

Con el comando int se pueden calcular integrales que involucren a la funci´ on impulso unitario. Seguidamente se calculan los valores de las integrales Z

3 0

1 e(t−2) δ(2t − 4) dt = , 2

Z

3 0

e(t−2) δ′′ (2t − 4) dt =

1 . 2

✄ >> syms t >> f 1 ( t ) = exp ( t −2)∗ d i r a c (2∗ t −4); >> i n t ( f1 , t , 0 , 3 ) ans ( t ) = 1/2 >> f 2 ( t ) = exp ( t −2)∗ d i f f ( d i r a c (2∗ t −4) , t , 2 ) ; >> i n t ( f2 , t , 0 , 3 ) ans ( t ) = 1/2





M ATLAB no tiene un comando que permita calcular en forma expl´ıcita la convoluci´ on de dos funciones de demonio continuo. En el Programa 5.1 se muestra la funci´ on ConvCont, que permite calcular el valor de la convoluci´ on h(t) = f ∗ g(t) en t = t0 , en otras palabras, halla el valor h(t0 ). Veamos un ejemplo. Consideremos las funciones f (t) = p1 (t) y g(t) = on u(t) del Ejemplo 5.4, donde hallamos la convoluci´ h(t) = f ∗ g(t) = (t + 1)(u(t + 1) − u(t − 1)) + 2u(t − 1). Programa 5.1. Funci´ on ConvCont.m f u n c t i o n [ h0]= ConvCont ( f , g , t 0 ) %ConvCont H a l l a e l v a l o r de l a c o n v o l u c i o ´ n h ( t ) = f ∗g ( t ) % en t 0 % h0 = h ( t 0 ) syms t tau ; h0 = i n t ( f ( tau )∗ g ( t0−tau ) , tau ,− I n f , I n f ) ; end

Seguidamente se utiliza la funci´ on ConvCont para hallar el valor h(1/2) = 3/2. ✄ >> syms t >> f ( t ) = h e a v i s i d e ( t +1)−h e a v i s i d e ( t −1);

165

CAP´ITULO 5. FUNCIONES DE DOMINIOS CONTINUO Y DISCRETO

>> g ( t ) = h e a v i s i d e ( t ) ; >> h0 = ConvCont ( f , g , 1 / 2 ) h0 = 3/2





La funci´ on ConvCont no s´ olo permite hallar el valor de f ∗ g(t) en t = t0 , sino que tambi´ en podemos usarla para obtener una gr´afica de f ∗ g(t). A continuaci´ on, con las funciones f (t) y g(t) creadas previamente, se construye la gr´afica de h(t) = f ∗ g(t) en el intervalo [−4, 4]. ✄ >> >> >> >> >>

t 0 = −4:.1:4; h0 = ConvCont ( f , g , t 0 ) ; p l o t ( t0 , h0 , ’ LineWidth ’ , 2 ) a x i s ([ −4 ,4 , −1 ,3]); g r i d on t i t l e ( ’ h ( t ) = f ∗g ( t ) ’ )





h(t) = f*g(t) 3

2.5

2

1.5

1

0.5

0

−0.5

−1 −4

−3

−2

−1

0

1

2

3

4

En este caso, t0 es un vector, por ello ConvCont arroja como salida un vector h0, cuyas componentes son los valores que alcanza h(t) en cada componente del vector t0. En cuanto a las funciones de dominio discreto, con M ATLAB tambi´ en se pueden manejar, pero en forma limitada. Una funci´ on f (n) de dominio discreto es en s´ı una sucesi´ on de n´ umeros reales. Este hecho nos permite definir funciones de dominio discreto como un arreglo de n´ umeros reales. Por ejemplo, la funci´ on sen (πn/2) puede definirse en M ATLAB como una discretizaci´ on de la funci´ on sen (πt), primero creando n como un arreglo de n´ umeros enteros y despu´es evaluando la funci´ on sen (πt/2) en ese arreglo. Esto puede observarlo a continuaci´ on. ✄ >> n=−10:10; >> f = s i n ( p i ∗n / 2 ) ;





Con n=-10:10 se crea el arreglo [-10,-9,-8,...,8,9,10]. As´ı, el resultado de las operaciones anteriores es un arreglo f cuyas componentes son sen (πn/2), para n = −10, . . . , 10; en otras palabras, se ha definido la funci´ on f (n) = sen (πn/2), para n = −10, −9, . . . , 10. La gr´afica de esta funci´ on puede generarse con el comando stem(n,f,’fill’,’--’). ✄ >> stem ( n , f , ’ f i l l ’ , ’−− ’ ) >> t i t l e ( ’ sen ( n\ p i /2) ’ )





166

´ AL CALCULO ´ 5.3. INTRODUCCION OPERACIONAL CON MATLAB

sen(nπ/2) 1

0.8

0.6

0.4

0.2

0

−0.2

−0.4

−0.6

−0.8

−1 −10

−8

−6

−4

−2

0

2

4

6

8

10

El proceso anterior se puede aplicar para generar todas las funciones elementales de dominio discreto (cos(n), en , tan(n), etc.). En cuanto a las funciones impulso unitario discreto δ(n) y escal´ on unitario discreto u(n), hemos definido las funciones impulso (ver Programa 5.2) y escalon (ver Programa 5.3), que hallan los valores que alcanzan δ(n) y u(n) en las componentes del arreglo n, respectivamente. A continuaci´ on usamos las funciones impulso y escalon, para generar la gr´afica de δ(n) y la de u(n) en el intervalo [−10, 10]. ✄ >> >> >> >> >>

n=−10:10; s u b p l o t ( 2 , 1 , 1 ) , stem (n , impu lso ( n ) , ’ f i l l ’ , ’−− ’ ) t i t l e ( ’ \ de lta (n) ’ ) s u b p l o t ( 2 , 1 , 2 ) , stem (n , e s c a l o n ( n ) , ’ f i l l ’ , ’−− ’ ) t i t l e ( ’ u(n) ’ )





Programa 5.2. Funci´ on impulso.m f u n c t i o n [ d ] = impu lso ( n ) %impu lso H a l l a l o s v a l o r e s de d e l t a ( n ) % d : a r r e g l o de v a l o r e s de d e l t a ( n ) d = zeros ( s i z e (n ) ) ; d ( n==0) = 1 ; end

Programa 5.3. Funci´ on escalon.m f u nct i on [u] = escalon (n) %e s c a l o n H a l l a l o s v a l o r e s de u ( n ) % u : a r r e g l o de v a l o r e s de u (n ) u = zeros ( s i z e (n ) ) ; u (n>=0) = 1 ; end

167

CAP´ITULO 5. FUNCIONES DE DOMINIOS CONTINUO Y DISCRETO

δ(n) 1 0.8 0.6 0.4 0.2 0 −10

−8

−6

−4

−2

0

2

4

6

8

10

2

4

6

8

10

u(n) 1 0.8 0.6 0.4 0.2 0 −10

−8

−6

−4

−2

0

Los Programas 5.4, 5.5 y 5.6, muestran las funciones signo, rampa y pulsorectangular, que definen respectivamente las funciones signo discreto,

rampa discreta,

  −1, n < 0, sgn(n) = 0, n = 0,   1, n > 0, r(n) = n u(n),

y pulso rectangular discreto   0, pn1 ,n2 (n) = 1,   0,

n < n1 , n1 ≤ n ≤ n2 , n > n2 ,

donde n1 , n2 ∈ Z tales que n1 < n2 .

Programa 5.4. Funci´ on signo.m f u nct i on [ s ] = signo (n) %s i g n o H a l l a l o s v a l o r e s de sgn ( n ) % s : a r r e g l o de v a l o r e s de sgn (n ) s = zeros ( s i z e (n ) ) ; s (n<0) = −1; s (n>0) = 1 ; end

Utilice el comando stem para obtener las gr´aficas de las funciones sgn(n), r(n) y p−7,−1 (n), en el intervalo [−10, 10]

168

´ AL CALCULO ´ 5.3. INTRODUCCION OPERACIONAL CON MATLAB

Programa 5.5. Funci´ on rampa.m f u n c t i o n [ r ] = rampa( n ) %rampa H a l l a l o s v a l o r e s de r ( n ) % r : a r r e g l o de v a l o r e s de r ( n ) r = zeros ( s i z e (n ) ) ; r (n>=0) = n ( n>=0); end

Programa 5.6. Funci´ on pulsorectangular.m f u n c t i o n [ p ] = p u l s o r e c t a n g u l a r (n , n1 , n2 ) %p u l s o r e c t a n g u l a r H a l l a l o s v a l o r e s de un p u l s o % r e c t a n g u l a r e n t r e n1 y n2 % p : a r r e g l o de v a l o r e s d e l p u l s o r e c t a n g u l a r p = ones ( s i z e ( n ) ) ; p (nn2 ) = 0 ; end

Con las funciones impulso, escalon, signo, rampa y pulsorectangular, se pueden definir nuevas funciones, por ejemplo, las funciones f1 (n) = u(4 − n), f3 (n) = u(−n) − δ(n − 1) − δ(n − 2), se definen, en el intervalo [−10, 10], como:

f2 (n) = n (u(n + 2) − u(n − 3)),   −1, −5 ≤ n ≤ −1, f4 (n) = 1, 0 ≤ n ≤ 4,   0, en otros casos.

✄ >> >> >> >> >>

n = −10:10; f 1 = e s c a l o n ( n−4); f 2 = n . ∗ ( e s c a l o n ( n+2)−e s c a l o n ( n −3)); f 3 = e s c a l o n (−n)−impu lso ( n−1)−impu lso ( n−2); f 4 = p u l s o r e c t a n g u l a r ( n ,0 ,4) − p u l s o r e c t a n g u l a r ( n, −5 , −1);





cuyas gr´aficas se obtienen de la siguiente forma:

✄ >> >> >> >>



subplot (2 ,2 ,1) , subplot (2 ,2 ,2) , subplot (2 ,2 ,3) , subplot (2 ,2 ,4) ,

stem (n , f1 stem (n , f2 stem (n , f3 stem (n , f4

, , , ,

’ ’ ’ ’

fill fill fill fill

’ ’ ’ ’

, , , ,

’−− ’ ) ; ’−− ’ ) ; ’−− ’ ) ; ’−− ’ ) ;

title title title title

( ( ( (

’f ’f ’f ’f

1 (n) 2 (n) 3 (n) 4 (n)

’) ’) ’) ’)



169

CAP´ITULO 5. FUNCIONES DE DOMINIOS CONTINUO Y DISCRETO

f (n)

f (n)

1

2

1

2

0.8

1

0.6 0 0.4 −1

0.2 0 −10

−5

0

5

−2 −10

10

−5

f (n)

0.5

0.5

0

0

−0.5

−0.5

0

10

5

10

4

1

−5

5

f (n)

3

1

−1 −10

0

5

10

−1 −10

−5

0

M ATLAB no tiene un comando que permita calcular en forma expl´ıcita la convoluci´ on de dos funciones cualesquiera de demonio discreto. Pero, M ATLAB si cuenta con el comando conv(f,g) que calcula la convoluci´ on de funciones f (n) y g(n) definidas en intervalos finitos, es decir, f (n) debe estar definida en un intervalo [n1 , n2 ] y g(n) debe estar definida en un intervalo [n3 , n4 ], donde n1 , n2 , n3 , n4 ∈ Z tales que n1 < n2 y n3 < n4 . Cuando se aplica el comando h = conv(f,g), se obtiene la convoluci´ on h(n) = f ∗ g(n) definida en el intervalo n ∈ [n1 + n3 , n2 + n4 ]. Veamos un ejemplo. Consideremos las funciones f (n) = u(n − 2) − u(n − 4),

para n ∈ [0, 5]

y g(n) = u(n + 2) − u(n − 5),

para n ∈ [−2, 8]

En esta caso, la convoluci´ on h(n) = f ∗ g(n) est´ a dada por: h(n) = u(n − 1) + u(n − 2) − u(n − 7) − u(n − 8). Utilicemos el comando h = conv(f,g) para hallar los valores de h(n), por supuesto, en el intervalo [−2, 13]. ✄ >> >> >> >>

n1=0; n2=5; n3=−2; n4=8; n=n1 : n2 ; f=e s c a l o n ( n−2)−e s c a l o n ( n−4); n=n3 : n4 ; g=e s c a l o n ( n+1)−e s c a l o n ( n−5); h = conv ( f , g ) ;





Las gr´aficas de f (n), g(n) y h(n) se obtienen de la siguiente forma: ✄ >> s u b p l o t ( 2 , 2 , 1 ) , stem ( n1 : n2 , f , ’ f i l l ’ , ’−− ’ ) ; t i t l e ( ’ f ( n ) ’ ) >> s u b p l o t ( 2 , 2 , 2 ) , stem ( n3 : n4 , g , ’ f i l l ’ , ’−− ’ ) ; t i t l e ( ’ g ( n ) ’ ) >> s u b p l o t ( 2 , 2 , 3 : 4 ) , stem ( n1+n3 : n2+n4 , h , ’ f i l l ’ , ’−− ’ ) ; t i t l e ( ’ h ( n ) ’ )





170

5.4. PROBLEMAS RESUELTOS

f(n)

g(n)

1

1

0.8

0.8

0.6

0.6

0.4

0.4

0.2

0.2

0

0

1

2

3

4

0 −2

5

0

2

4

6

8

h(n) 2

1.5

1

0.5

0 −2

0

2

4

6

8

10

12

14

5.4 Problemas Resueltos Problema 5.1. Sean f (t) y g(t) funciones de dominio continuo definidas respectivamente por: f (t) = pa (t − a) y g(t) = u(t). Determinar la convoluci´ on h(t) = f ∗ g(t). Soluci´ on. Tenemos que la convoluci´ on h(t) = f ∗ g(t) se define como: Z ∞ f (τ )g(t − τ ) dτ, para t ∈ R. h(t) = f ∗ g(t) = −∞

Ahora bien, considerando las expresiones de f (t) y g(t) dadas en el enunciado del problema, obtenemos: h(t) =

Z

∞ −∞

pa (τ − a)u(t − τ ) dτ =

Z

2a− 0+

u(t − τ ) dτ,

para t ∈ R.

(5.6)

Pasemos a determinar la expresi´ on matem´atica de h(t) considerando la ecuaci´ on (5.6). Debemos tener presente que esta ecuaci´ on es v´alida para todo t ∈ R. El procedimiento que seguiremos para hallar la expresi´ on de h(t) es el siguiente: fijar valores de t, luego construir la gr´afica de la funci´ on u(t − τ ) (en funci´ on de τ ) y, por u ´ ltimo, determinar el valor de la integral de (5.6), para cada valor de t. • Para t ≤ 0, la gr´afica de u(t − τ ) es: u(t − τ ) 1

t

2a

τ

171

CAP´ITULO 5. FUNCIONES DE DOMINIOS CONTINUO Y DISCRETO

Lo que implica, seg´ un (5.6), que h(t) = 0, para t ≤ 0. • Para 0 < t < 2a, la gr´afica de u(t − τ ) es: 1

u(t − τ )

t

τ

2a

As´ı, usando (5.6) obtenemos: h(t) =

Z

t

dτ = t,

0+

para t ∈ (0, 2a).

• Para t ≥ 2a, la gr´afica de u(t − τ ) es: 1

u(t − τ )

2a

τ

t

As´ı, usando (5.6) obtenemos: h(t) =

Z

2a+

dτ = 2a,

0+

para t ≥ 2a.

De todo lo anterior se tiene que la expresi´ on matem´atica de la convoluci´ on h(t) = f ∗g(t) est´ a dada por: h(t)   0, h(t) = t,   2a,

t ≤ 0, 0 < t < 2a, t ≥ 2a.

2a

t

2a 

Problema 5.2. Sean f (t) y g(t) funciones de dominio continuo definidas respectivamente por: f (t) = r(t) y g(t) = sgn(t) + u(−t − 1). Determinar la convoluci´ on h(t) = f ∗ g(t).

172

5.4. PROBLEMAS RESUELTOS

Soluci´ on. Usando la distributividad de la convoluci´ on podemos escribir, para t ∈ R, h(t) = f ∗ g(t) = r(t) ∗ sgn(t) + r(t) ∗ u(−t − 1). Calculemos cada una de las convoluciones r(t) ∗ sgn(t) y r(t) ∗ u(−t − 1). Usando la definici´ on de convoluci´ on obtenemos: Z ∞ r(τ ) sgn(t − τ ) dτ, para t ∈ R. r(t) ∗ sgn(t) =

(5.7)

−∞

• Para t ≤ 0, la gr´afica de la funci´ on r(τ ) sgn(t − τ ) es: r(τ ) sgn(t − τ )

τ −τ As´ı, por (5.7) la convoluci´ on r(t) ∗ sgn(t) no existe, para t ≤ 0. • Para t > 0, la gr´afica de la funci´ on r(τ ) sgn(t − τ ) es: r(τ ) sgn(t − τ ) t t −t

τ −τ

As´ı, usando (5.7) obtenemos: r(t) ∗ sgn(t) = =

Z

t

Z



(−τ )dτ τ dτ + t 0+  2 s τ t2 = −∞, − l´ım s→∞ 2 2 t

para t > 0;

en otras palabras, la convoluci´ on r(t) ∗ sgn(t) no existe, para t ≥ 0. De esta forma, la convoluci´ on r(t) ∗ sgn(t) no existe para todo t ∈ R. En consecuencia, podemos concluir que la convoluci´ on f ∗ g(t) no existe para todo t ∈ R, sin considerar el resultado de la convoluci´ on r(t) ∗ u(−t − 1). Se deja como ejercicio para el lector, verificar  que la convoluci´ on r(t) ∗ u(−t − 1) no existe para todo t ∈ R.

173

CAP´ITULO 5. FUNCIONES DE DOMINIOS CONTINUO Y DISCRETO

Problema 5.3. Sean f (t) y g(t) funciones de dominio continuo definidas respectivamente por: f (t) = e−t u(t) y g(t) = e−2t u(t). Determinar la convoluci´ on h(t) = f ∗ g(t). Soluci´ on. Considerando la definici´ on de la convoluci´ on h(t) = f ∗ g(t) y las expresiones de f (t) y g(t) dadas en el enunciado del problema, obtenemos: Z ∞ Z ∞ −τ −2(t−τ ) −2t e e u(t − τ ) dτ = e h(t) = eτ u(t − τ ) dτ, para t ∈ R. (5.8) 0+

−∞

• Para t ≤ 0, la gr´afica de eτ u(t − τ ) es: eτ u(t − τ ) et τ

t

As´ı, por (5.8), h(t) = 0, para t ≤ 0. • Para t > 0, la gr´afica de eτ u(t − τ ) es: eτ u(t − τ ) et

t

τ

As´ı, por (5.8) obtenemos: h(t) = e−2t

Z

t

0+

 eτ dτ = e−2t et − 1 ,

para t > 0.

De todo lo anterior se tiene que la expresi´ on matem´atica de la convoluci´ on h(t) = f ∗ g(t) est´ a dada por: h(t)  h(t) = e−2t et − 1 u(t) t 

174

5.4. PROBLEMAS RESUELTOS

Problema 5.4. Determinar la convoluci´ on h(t) = f ∗ g(t) de las funciones de dominio continuo f (t) y g(t) que se muestran en la siguiente figura. Tome en cuenta que solo se muestra la gr´afica de las funciones donde ellas son distintas de cero. g(t)

f (t) 1 −1 −1

1

1 t

t −1

Soluci´ on. Considerando la definici´ on de la convoluci´ on h(t) = f ∗ g(t) y la gr´afica de f (t) dada en el enunciado del problema, obtenemos: Z 1 Z 0 g(t − τ ) dτ, para t ∈ R. (5.9) (τ + 1)g(t − τ ) dτ + h(t) = 0

−1

Determinemos los valores de las integrales de (5.9) para cada t ∈ R. • Para t ≤ −2, las gr´aficas de g(t − τ ) y (τ + 1)g(t − τ ) son: g(t − τ )

g(t − τ )(τ + 1)

(τ + 1)(τ − t)

(τ + 1)(t − τ ) t−1

t+1

t

−1

τ

t−1

t+1

t

−1

−1

τ

As´ı, por (5.9), h(t) = 0, para t ≤ −2. • Para −2 < t ≤ −1, las gr´aficas de g(t − τ ) y (τ + 1)g(t − τ ) son: g(t − τ )

g(t − τ )(τ + 1) (τ + 1)(τ − t)

t−1 −2

t

t+1

τ

−1 −1

t+1 t t − 1 −2 −1 (τ + 1)(t − τ )

As´ı, por (5.9) obtenemos: Z t+1 (t − 1) (t + 2)2 , (τ + 1)(t − τ ) dτ = h(t) = 6 −1

τ

para − 2 < t ≤ −1.

175

CAP´ITULO 5. FUNCIONES DE DOMINIOS CONTINUO Y DISCRETO

• Para −1 < t ≤ 0, las gr´aficas de g(t − τ ) y (τ + 1)g(t − τ ) son: g(t − τ )

g(t − τ )(τ + 1) (τ + 1)(τ − t)

t−1

t+1

t

τ

1

−1

t+1

t t−1

(τ + 1)(t − τ )

−1

−1

As´ı, por (5.9) obtenemos: Z Z 0 Z t (τ + 1)(t − τ ) dτ + (τ + 1)(τ − t) dτ + h(t) = =

t

−1 2 t

2



(t + 1)3 t2 (t + 3) 1 − − , 6 6 2

τ

1

t+1

(t − τ ) dτ

0

para − 1 < t ≤ 0.

• Para 0 < t ≤ 1, las gr´aficas de g(t − τ ) y (τ + 1)g(t − τ ) son: g(t − τ )

t−1

g(t − τ )(τ + 1)

t+1

t 1

t−1

t+1

t

τ

2

1

τ

2

(τ + 1)(τ − t) −1 (τ + 1)(t − τ )

As´ı, por (5.9) obtenemos: Z 1 Z t Z 0 (t − τ ) dτ (τ − t) dτ + (τ + 1)(τ − t) dτ + h(t) = t 0 t−1  (t − 1) t2 + 4 t + 1 (t − 1)2 t2 = − − , para 0 < t ≤ 1. 6 2 2 • Para 1 < t ≤ 2, las gr´aficas de g(t − τ ) y (τ + 1)g(t − τ ) son: g(t − τ ) t−1

g(t − τ )(τ + 1) t+1

t 1

2

t−1 3

τ

t+1

t 1

2

−1 (τ + 1)(τ − t)

(τ + 1)(t − τ )

3

τ

176

5.4. PROBLEMAS RESUELTOS

As´ı, por (5.9) obtenemos: Z h(t) =

1

t−1

(τ − t) dτ =

t (t − 2) , 2

para 1 < t ≤ 2.

• Para t > 2, las gr´aficas de g(t − τ ) y (τ + 1)g(t − τ ) son: g(t − τ )

g(t − τ )(τ + 1)

t−1 1

t+1

t 2

t−1 4 τ

3

1

t+1

t 2

3

4

τ

−1

(τ + 1)(τ − t)

(τ + 1)(t − τ )

As´ı, por (5.9), h(t) = 0, para t > 2. De todo lo anterior se tiene que la convoluci´ on h(t) = f  (t−1) (t+2)2  ,  6    (t+1)3 t2 (t+3) t2 1    2 − 6 − 6 − 2, 2 2 h(t) = (t−1) (t +4 t+1) − (t−1) − t2 ,  6 2 2   t (t−2)  ,  2    0,

∗ g(t) est´ a dada por: −2 < t ≤ −1, −1 < t ≤ 0,

0 < t ≤ 1, 1 < t ≤ 2, en otros casos.

h(t)

−2

2

t −2/3

 Problema 5.5. Sean f (n) y g(n) funciones de dominio discreto definidas respectivamente por: f (n) = u(n + 1) − u(n − 2) y g(n) = u(n + 1) − u(n − 2). Determinar la convoluci´ on h(n) = f ∗ g(n).

177

CAP´ITULO 5. FUNCIONES DE DOMINIOS CONTINUO Y DISCRETO

Soluci´ on. Considerando la expresi´ on de f (n) dada en el enunciado del problema y la definici´ on de la convoluci´ on de funciones de dominio discreto, podemos escribir: h(n) = f ∗ g(n) =

∞ X

k=−∞

f (k)g(n − k) =

1 X

k=−1

g(n − k),

para n ∈ Z.

(5.10)

Calculemos los valores que toma h(n) para cada n ∈ Z. Seguidamente construiremos gr´aficas de g(n − k) para cada n. Aqu´ı solo se mostrar´a el gr´afico de g(n − k) = u((n + 1) − k) − u((n − 2) − k) donde ella es distinta de cero. • Para n ≤ −3, la gr´afica de g(n − k) es: g(n − k) b

b

b

n−1

n

n+1

1

···

−1

1

k

As´ı, por (5.10), h(n) = 0, para n ≤ −3. • Para n = −2, la gr´afica de g(n − k) es: g(n − k)

-5

-4

b

b

b

-3

-2

-1

1

1

2

3

4

5

k

2

3

4

5

k

2

3

4

5

k

As´ı, por (5.10), h(−2) = 1. • Para n = −1, la gr´afica de g(n − k) es:

-5

-4

-3

b

b

-2

-1

b

1

g(n − k)

1

As´ı, por (5.10), h(−1) = 2. • Para n = 0, la gr´afica de g(n − k) es: b

-5

-4

-3

-2

-1

1

b

g(n − k) b

1

178

5.4. PROBLEMAS RESUELTOS

As´ı, por (5.10), h(0) = 3. • Para n = 1, la gr´afica de g(n − k) es:

1

b

-5

-4

-3

-2

g(n − k)

-1

b

b

1

2

3

b

b

b

1

2

3

4

5

k

4

5

k

As´ı, por (5.10), h(1) = 2. • Para n = 2, la gr´afica de g(n − k) es: g(n − k) 1

-5

-4

-3

-2

-1

As´ı, por (5.10), h(2) = 1. • Para n ≥ 3, la gr´afica de g(n − k) es: g(n − k) b

b

b

n−1

n

n+1

1

−1

···

1

k

As´ı, por (5.10), h(n) = 0, para n ≥ 3. De todo lo anterior se tiene que la convoluci´ on h(n) = f ∗ g(n) est´ a dada por: h(n) = (3 − |n|)(u(n + 2) − u(n − 3)) h(n) 3 b

b

b

b

b

-5

-4

-3

-2

2

b

b

b

1

-1

1

2

b

b

b

3

4

5

n 

179

CAP´ITULO 5. FUNCIONES DE DOMINIOS CONTINUO Y DISCRETO

Problema 5.6. Sean f (n) y g(n) funciones de dominio discreto definidas respectivamente por: f (n) = u(n + 2) − u(n − 4) y g(n) = nu(n). Determinar la convoluci´ on h(n) = f ∗ g(n). Soluci´ on. Considerando las expresiones de f (n) y g(n) dadas en el enunciado del problema y la definici´ on de la convoluci´ on de funciones de dominio discreto, podemos escribir para n ∈ Z: h(n) = f ∗ g(n) =

∞ X

k=−∞

f (k)g(n − k) =

3 X

(n − k) u(n − k).

k=−2

Haciendo el cambio de ´ındices m = n−k y luego sustituyendo m por k, la ecuaci´ on anterior adquiere la forma: n+2 X h(n) = k u(k), para n ∈ Z. (5.11) k=n−3

Calculemos los valores que toma h(n) para cada n ∈ Z. Para ello construiremos la gr´afica de ku(k) y usaremos la ecuaci´ on (5.11) variando los valores de n. La gr´afica de k u(k) es: ku(k) b

6 5 b

b

4 b

3 b

2 b

1 b

b

b

b

b

-5

-4

-3

-2

-1

b

1

2

3

• Para n ≤ −2, por (5.11), h(n) = 0. • Para n = −1, por (5.11), h(−1) = 1. • Para n = 0, por (5.11), h(0) = 1 + 2 = 3. • Para n = 1, por (5.11), h(1) = 1 + 2 + 3 = 6. • Para n = 2, por (5.11), h(2) = 1 + 2 + 3 + 4 = 9. • Para n = 3, por (5.11), h(3) = 1 + 2 + 3 + 4 + 5 = 14. • Para n = 4, por (5.11), h(4) = 1 + 2 + 3 + 4 + 5 + 6 = 20.

4

5

6

k

180

5.4. PROBLEMAS RESUELTOS

• Para n = 5, por (5.11), h(5) = 2 + 3 + 4 + 5 + 6 + 7 = 27. • Para n = 6, por (5.11), h(6) = 3 + 4 + 5 + 6 + 7 + 8 = 33. .. . De todo lo anterior se tiene que la convoluci´ on h(n) = f ∗ g(n) est´ a dada por:

h(n) =

  0,   

n ≤ −2,

(n+2)(n+3)

2     (n+2)(n+3)

−1 ≤ n ≤ 4,

, −

2

(n−4)(n−3) , 2

n ≥ 5.

h(n) b

b

b

··· b

b b b b

b

b

b

-5

-4

-3

-2

b

-1

1

2

3

4

5

6

n 

4 X (−1)m δ(n − m) y g(n) m

Problema 5.7. Determinar la convoluci´ on f ∗ g, donde f (n) =

m=1

es una funci´ on cuya gr´afica se muestra en la siguiente figura. g(n) b

b

· · · −4

b

1

b

−3

b

−2

b

b

3 b

4 b

··· n

Se tiene que la funci´ on g(n) se puede expresar como g(n) = g1 (n) − g2 (n), g1 (n) = u(−1 − n) − u(−3 − n)

Verifique que g(n) = g1 (n) − g2 (n), para las funciones g1 (n) y g2 (n) dadas

2

−1 -1

Soluci´ on. donde

1

y

g2 (n) = u(n − 1) − u(n − 3).

181

CAP´ITULO 5. FUNCIONES DE DOMINIOS CONTINUO Y DISCRETO

As´ı, considerando las expresiones de f (n) y g(n), y aplicando la propiedad distributiva de la convoluci´ on, se puede escribir, para todo n ∈ Z: h(n) = f ∗ g (n)

= f ∗ g1 (n) − f ∗ g2 (n) 4 4 X X (−1)m (−1)m δ(n − m) ∗ g1 (n) − δ(n − m) ∗ g2 (n). = m m m=1 m=1

(5.12)

Calculemos las convoluciones δ(n − m) ∗ g1 (n) y δ(n − m) ∗ g2 (n), para m = 1, 2, 3, 4. Se tiene que, para todo n ∈ Z: δ(n − m) ∗ g1 (n) = δ(n − m) ∗ u(−1 − n) − δ(n − m) ∗ u(−3 − n) ∞ ∞ X X = δ(k − m)u(−1 − n + k) − δ(k − m)u(−3 − n + k) k=−∞

k=−∞

= u((m − 1) − n) − u((m − 3) − n),

para m = 1, 2, 3, 4,

y δ(n − m) ∗ g2 (n) = δ(n − m) ∗ u(n − 1) − δ(n − m) ∗ u(n − 3) ∞ ∞ X X δ(k − m)u(n − k − 3) δ(k − m)u(n − k − 1) − = k=−∞

k=−∞

= u(n − (m + 1)) − u(n − (m + 3)),

para m = 1, 2, 3, 4.

Ahora, usando convenientemente estas u ´ ltimas expresiones en (5.12), se obtiene que la expresi´ on anal´ıtica de la convoluci´ on h(n) = f ∗ g (n) est´ a dada por: 4 X (−1)m h(n) = [u((m − 1) − n) − u((m − 3) − n) − u(n − (m + 1)) + u(n − (m + 3))] m m=1

2 1 1 1 1 =u(−2 − n) − u(−1 − n) − u(−n) + u(1 − n) − u(2 − n) + u(3 − n) 2 3 4 3 4 1 2 1 1 1 + u(n − 2) − u(n − 3) − u(n − 4) + u(n − 5) − u(n − 6) + u(n − 7), 2 3 4 3 4

cuya representaci´ on gr´afica se aprecia en la siguiente figura. h(n) 1

· · · −3 b

−2 b

−1

b

b

4 b

b

1

b b

2

3

b

6 5

b

b

7

b

8···

n

-1



182

5.5. PROBLEMAS PROPUESTOS

5.5 Problemas Propuestos 5.1. Dibujar las siguientes funciones de dominio continuo: a) f1 (t) = u(t) + 3u(t − 1) − 2u(t − 2)

b) f2 (t) = r(t) − r(t − 1) − u(t − 2) c) f3 (t) = u(t)u(t − a),

a>0

d) f4 (t) = u(t)u(a − t),

a>0

e) f5 (t) = u(cos t)

f) f6 (t) = p1/2 (t2 + 2t + 1)   1 g) f7 (t) = f1 (t)f2 t + 2   t 1 h) f8 (t) = f1 − + + f2 (t) 3 2

5.2. Calcular las siguientes integrales:

a) b)

Z Z

1 −2 4 −2

2

(t + t )δ(t − 3) dt

c)

Z

3

Z

4

0

2

(t + t )δ(t − 3) dt

d)

−4

e(t−2) δ(2t − 4) dt 2 ′

(t − 2) δ



1 1 − t+ 3 2



dt

5.3. Determinar la convoluci´ on h(t) = f ∗ g (t) de la parejas de funciones f (t) y g(t) de dominio continuo que se enumeran a continuaci´ on: a) f (t) = pa (t − a), g(t) = δ(t − b), b > a

e) f (t) = e−t u(t), g(t) = e−2t u(t) + u(−t)

b) f (t) = u(t), g(t) = sgn(t)

f) f (t) = t e−t u(t), g(t) = u(t)

c) f (t) = t[u(t) − u(t − 1)], g(t) = u(t)

g) f (t) = u(t), g(t) = e−2t u(t) + δ(t)

d) f (t) = [u(t + 1) − u(t − 1)] sgn(t), g(t) = u(t)

h) f (t) = δ(t − 1) + e−t u(t), g(t) = e−2t u(t)

5.4. Determinar la convoluci´ on h(t) = f ∗ g (t) de la pareja de funciones f (t) y g(t), cuyas gr´aficas se muestran en siguiente figura. f (t)

g(t)

1

1 −1

a) −1

1

t

1 −1

t

183

CAP´ITULO 5. FUNCIONES DE DOMINIOS CONTINUO Y DISCRETO

f (t)

g(t)

1

1

b)

1 −1

t

1

t

−1 −1

f (t)

g(t)

2

c)

1

1

−1

t

2

−1

f (t)

−1

t

g(t)

1

d)

1

1

t

1

−1

1

t

5.5. Sea f (n) la funci´ on de dominio discreto cuya gr´afica se muestra en la siguiente figura. f (n) b

2 b

1

−2

1 −1

n

2 b

-1 b

3

-2

b

Dibujar las siguientes funciones: a) g(n) = f (n − 2) b) g(n) = f (3n − 4)   n+8 c) g(n) = f − 4

d) g(n) = f (n)u(n) e) g(n) = u(f (n)) f) g(n) = δ(f (n))

184

5.5. PROBLEMAS PROPUESTOS

5.6. Determine la convoluci´ on h(n) = f ∗ g (n) de la pareja de funciones f (n) y g(n) de dominio discreto que se enumeran a continuaci´ on: a) f (n) = u(n − 1), e) f (n) = u(n + 2) − u(n − 4),  g(n) = nu(n)  −1, −5 ≤ n ≤ −1, g(n) = 1, 0 ≤ n ≤ 4, f) f (n) = n (u(n + 2) − u(n − 3)),   0, en otros casos. g(n) = u(−n)−(δ(n − 1) + δ(n − 2))  n 2 1 X (−1)k u(n), b) f (n) = δ(n − k) g) f (n) = 2 2k+2  n k=−2 1 g(n) = δ(n) + δ(n − 1) + u(n) 3 X 3 g(n) = (−1)k δ(n − k) k=0 c) f (n) = u(n + 1) − u(n − 2), g(n) = u(n + 1) − u(n − 2)

d) f (n) = −n(1/4)n u(−n − 1), g(n) = u(n)

h) f (n) =

5 X (−1)k+1 k=0

k+1

δ(n − k)

g(n) = u(n) − u(n − 5)

6 Transformada de Fourier Este cap´ıtulo presenta la Transformada de Fourier, tambi´ en conocida como la Integral de Fourier. El nombre de Fourier es en honor al f´ısico franc´es Jean Baptiste Fourier (17681830). En Ingenier´ıa, la transformada de Fourier es una herramienta sumamente u ´ til. Generalmente se emplea para pasar una se˜ nal al dominio de frecuencia para obtener informaci´ on que no es evidente en el dominio del tiempo. Por ejemplo, se puede usar en el ´ambito de procesamiento de se˜ nales, para determinar en qu´e ancho de banda se concentra la energ´ıa de una se˜ nal o tambi´ en en el procesamiento digital de im´ agenes, para mejorar ciertas zonas de una imagen fotogr´ afica. El cap´ıtulo comienza con la definici´ on matem´atica de la Transforma de Fourier, luego se prueban todos los teoremas y propiedades. Seguidamente se derivan las transformadas de Fourier de funciones comunes. Finalmente, se calculan la magnitud y la fase de una funci´ on de dominio continuo.

6.1 Definici´ on Pasemos definir una de las herramientas matem´aticas m´ as importantes para la ingenier´ıa como lo es la Transformada de Fourier. Definici´ on 6.1. Sea f (t) una funci´ on de dominio continuo definida de R en C. La transformada de Fourier de f (t) es una transformada integral lineal definida por: Z ∞ f (t) e−jωt dt, F (ω) = F{f (t)} = −∞

para ω ∈ R, donde j denota la unidad imaginaria. A la variable ω, generalmente, se le denomina frecuencia. La notaci´ on de la definici´ on anterior la usaremos en este y en los subsiguientes cap´ıtulos. La transformada de Fourier, F (ω), es una funci´ on definida de R en C, es decir, es una funci´ on que toma valores complejos. Por ello, se puede expresar como la suma de su parte real e imaginaria, o en su forma polar, esto es, F (ω) = Re {F (ω)} + j Im {F (ω)} = |F (ω)| ejφ(ω) , donde |F (ω)| se denomina magnitud y φ(ω) se denomina fase. Si f (t) es una funci´ on generalizada, entonces F (ω) se denomina transformada de Fourier generalizada. Para que la transformada de Fourier de una funci´ on f (t) exista en forma ordinaria (no generalizada), f (t) debe satisfacer las siguientes propiedades denominadas condiciones de Dirichlet: • f (t) es absolutamente integrable, esto es, Z ∞ |f (t)| dt < ∞; −∞

185

186

´ 6.1. DEFINICION

• f (t) posee un n´ umero finito de discontinuidades en cualquier intervalo de longitud finita. En el siguiente ejemplo mostramos una funci´ on que no posee transformada de Fourier ordinaria, pero si generalizada. Ejemplo 6.1. Sea f (t) dada por ( 1, |t| ≤ 1, = 2u(−t − 1) + u(t + 1) + u(t − 1) f (t) = 2, |t| > 1.

Intente hallar la transformada de Fourier de f (t) usando la definici´ on

La transformada de Fourier ordinaria de f (t) no existe, ya que f (t) no es absolutamente integrable. En cambio, f (t) si tiene transformada de Fourier generalizada, la cual est´ a dada por  eω j j e−2 ω j − j F (ω) = 4 π δ(ω) − . ω  En el resto del cap´ıtulo, llamaremos transformada de Fourier de una funci´ on a su transformada de Fourier generalizada. No seremos tan rigurosos exigiendo el cumplimiento de las condiciones de Dirichlet.

6.1.1 Transformada Inversa de Fourier La transformada de Fourier nos permite pasar del plano del tiempo al plano de la frecuencia y realizar un an´alisis en frecuencia; pero luego es necesario trasladarnos nuevamente al plano del tiempo. Para esto u ´ ltimo se utiliza la Transformada Inversa de Fourier. Definici´ on 6.2. Sea f (t) una funci´ on con transformada de Fourier F (ω). La transformada inversa de Fourier est´ a definida como Z ∞ 1 −1 f (t) = F {F (ω)} = F (ω) ejωt dω, 2π −∞ para todo t ∈ R. Ejemplo 6.2. Determine la transformada inversa de Fourier de F (ω) = δ(ω). Soluci´ on. Por definici´ on de la transformada inversa de Fourier podemos escribir: Z ∞ Z ∞ 1 1 jωt f (t) = F (ω)e dω = δ(ω)ejωt dω 2π −∞ 2π −∞ Ahora, como ejωt = cos ωt + j sen ωt, entonces se obtiene que Z ∞ 1 f (t) = δ(ω)(cos ωt + j sen ωt) dω 2π −∞ Z ∞ Z ∞ 1 1 δ(ω) cos ωt dω + j δ(ω)sen ωt dω = 2π −∞ 2π −∞ 1 1 (cos(0) + jsen (0)) = . = 2π 2π 

187

CAP´ITULO 6. TRANSFORMADA DE FOURIER

Observaci´ on 6.1. En general y con mucha frecuencia, para el c´ alculo de la transformada inversa de Fourier se utiliza el procedimiento Inversi´ on por Tablas, que se estudiar´a en el Cap´ıtulo 7. En el resto del cap´ıtulo usaremos la siguiente notaci´ on. El s´ımbolo F

f (t) ←→ F (ω), que se lee par de transformadas, denota que F (ω) es la transformada de Fourier de f (t) y que f (t) es la transformada inversa de Fourier de F (ω).

6.2 Propiedades de la Transformada de Fourier A continuaci´ on damos las propiedades de la transformada de Fourier. Cada una estas propiedades la enunciaremos como un teorema. Teorema 6.1 (Linealidad). Si F (ω) y G(ω) son las transformadas de Fourier de f (t) y g(t), respectivamente, entonces F

af (t) + bg(t) ←→ aF (ω) + bG(ω), donde a y b son constantes reales. Demostraci´ on. Por la definici´ on de la transformada de Fourier se tiene que Z



(af (t) + bg(t)) e−jωt dt −∞ Z ∞ Z ∞ −jωt g(t) e−jωt dt f (t) e dt + b = a

F{af (t) + bg(t)} =

−∞

−∞

= aF{f (t)} + bF{g(t)} = aF (ω) + bG(ω), que era lo que dese´ abamos demostrar.

Teorema 6.2 (Desplazamiento en tiempo). Si F (ω) es la transformada de Fourier de f (t), entonces F

f (t − t0 ) ←→ e−jωt0 F (ω), para todo t0 ∈ R. En otras palabras, la propiedad de desplazamiento de tiempo de la transformada de Fourier establece que si desplazamos la funci´on f (t) por una constante t0 , la magnitud de la transformada de Fourier no cambia, pero el t´ermino ωt0 se a˜ nade a su a ´ngulo de fase. Demostraci´ on. Por la definici´ on de la transformada de Fourier se tiene que F{f (t − t0 )} =

Z

∞ −∞

f (t − t0 ) e−jωt dt.

188

6.2. PROPIEDADES DE LA TRANSFORMADA DE FOURIER

Haciendo el cambio de variable r = t − t0 , la integral anterior adquiere la forma: Z ∞ f (r) e−jω(r+t0 ) dr F{f (t − t0 )} = −∞ Z ∞ −jωt0 = e f (r) e−jωr dr −∞

= e−jωt0 F{f (t)} = e−jωt0 F (ω),

con lo cual queda demostrado el teorema. Teorema 6.3 (Desplazamiento en frecuencia). Si F (ω) es la transformada de Fourier de f (t), entonces F

ejω0 t f (t) ←→ F (ω − ω0 ),

para todo ω0 ∈ R. En otras palabras, la multiplicaci´on de la funci´on f (t) por ejω0 t , resulta en un desplazamiento en la transformada de Fourier por ω0 . Demostraci´ on. Se tiene que Z Z ∞ jω0 t −jωt jω0 t e f (t) e dt = F{e f (t)} =



−∞

−∞

f (t) e−j(ω−ω0 )t dt = F (ω − ω0 ).

Teorema 6.4 (Escalamiento en tiempo). Si F (ω) es la transformada de Fourier de f (t), entonces 1 F F (ω/a), f (at) ←→ |a|

para todo a ∈ R distinto de cero. En otras palabras, la propiedad de escalamiento en tiempo establece que si se reemplaza la variable t por at, se debe reemplazar la variable ω en el dominio de la frecuencia por ω/a y dividir a F (ω/a) por el valor absoluto de a.

Demostraci´ on. Consideremos los casos a > 0 y a < 0. Para a > 0, Z F{f (at)} =



f (at) e−jωt dt.

(6.1)

−∞

Haciendo el cambio de variable r = at, la integral de (6.1) adquiere la forma: Z ω 1 ∞ F{f (at)} = f (r) e−j( a )r dr. a −∞ Cambiando r por t en la integral se tiene: Z ω 1 ∞ 1 1 F{f (at)} = F (ω/a). f (t) e−j( a )t dt = F (ω/a) = a −∞ a |a| Para a < 0, haciendo el cambio de variable r = at, la integral de (6.1) adquiere la forma: Z Z ω ω 1 −∞ 1 ∞ F{f (at)} = f (r) e−j( a )r dr = − f (r) e−j( a )r dr. a ∞ a −∞

189

CAP´ITULO 6. TRANSFORMADA DE FOURIER

Cambiando r por t en la integral y considerando que a < 0, se tiene: Z ∞ ω 1 1 F (ω/a). f (t) e−j( a )t dt = F{f (at)} = (−a) −∞ |a|

Teorema 6.5 (Dualidad). Si F (ω) es la transformada de Fourier de f (t), entonces F

F (t) ←→ 2π f (−ω). Demostraci´ on. Como

1 2π

Z

2πf (−t) =

Z

f (t) = entonces

F (ω) ejωt dω,

−∞ ∞

F (ω) e−jωt dω.

−∞

Intercambiando t y ω, se obtiene 2πf (−ω) =



Z



−∞

F (t) e−jωt dt = F{F (t)}.

Teorema 6.6 (Conjugaci´ on). Si F (ω) es la transformada de Fourier de f (t), entonces F

f (t) ←→ F (−ω). Demostraci´ on. Se tiene que Z o Z ∞ n −jωt f (t) e dt = F f (t) = −∞

∞ −∞

f (t) e−jωt dt

=

Z



f (t) ejωt dt = F (−ω).

−∞

Teorema 6.7 (Convoluci´ on). Si F (ω) y G(ω) son las transformadas de Fourier de f (t) y g(t), respectivamente, entonces F

f ∗ g (t) ←→ F (ω) G(ω). En otras palabras, la convoluci´ on en el dominio del tiempo corresponde a la multiplicaci´on en el dominio de la frecuencia. Demostraci´ on. Se tiene que Z



f ∗ g (t) e−jωt dt −∞  Z ∞ Z ∞ f (τ )g(t − τ ) dτ e−jωt dt = −∞ −∞  Z ∞ Z ∞ g(t − τ ) e−jωt dt dτ. f (τ ) =

F{f ∗ g (t)} =

−∞

−∞

190

6.2. PROPIEDADES DE LA TRANSFORMADA DE FOURIER

Haciendo el cambio de variable σ = t − τ , la ecuaci´ on anterior adquiere la forma:  Z ∞ Z ∞ −jω(σ+τ ) g(σ) e dσ dτ f (τ ) F{f ∗ g (t)} = −∞ −∞   Z ∞ Z ∞ −jωσ −jωτ g(σ) e dσ = F (ω) G(ω). f (τ ) e dτ = −∞

−∞

Teorema 6.8 (Multiplicaci´ on). Si F (ω) y G(ω) son las transformadas de Fourier de f (t) y g(t), respectivamente, entonces F

f (t) g(t) ←→

1 F ∗ G (ω). 2π

Es decir, la multiplicaci´on en el dominio del tiempo, corresponde a la convoluci´ on en el dominio de la frecuencia multiplicada por la constante 1/2π. Demostraci´ on. Se tiene que F{f (t) g(t)} = = = = =

Z



(f (t) g(t)) e−jωt dt −∞  Z ∞ Z ∞ 1 jσt F (σ) e dσ g(t) e−jωt dt 2π −∞ −∞  Z ∞ Z ∞ 1 −j(ω−σ)t g(t) e dt dσ F (σ) 2π −∞ −∞ Z ∞ 1 F (σ)G(ω − σ) dσ 2π −∞ 1 F ∗ G (ω). 2π

Teorema 6.9 (Diferenciaci´ on en tiempo). Si F (ω) es la transformada de Fourier de f (t), entonces d F f (t) ←→ jω F (ω). dt Demostraci´ on. Se tiene que 1 f (t) = 2π

Z



F (ω) ejωt dω.

−∞

Derivando con respecto a t en ambos lados de la ecuaci´ on anterior, se obtiene:   Z ∞ d 1 d f (t) = F (ω) ejωt dω dt dt 2π −∞ Z ∞  d  1 F (ω) ejωt dω = 2π −∞ dt Z ∞ 1 = [jω F (ω)] ejωt dω = F −1 {jω F (ω)}, 2π −∞

191

CAP´ITULO 6. TRANSFORMADA DE FOURIER

de donde se deduce que d F f (t) ←→ jω F (ω). dt

Teorema 6.10 (Diferenciaci´ on en frecuencia). Si F (ω) es la transformada de Fourier de f (t), entonces d F t f (t) ←→ j F (ω). dω Demostraci´ on. Se tiene que F (ω) =

Z



f (t) e−jωt dt.

−∞

Derivando con respecto a ω en ambos lados de la ecuaci´ on anterior, se obtiene:  Z ∞ d d f (t) e−jωt dt F (ω) = dω dω −∞ Z ∞  d  = f (t) e−jωt dt −∞ dω  Z ∞ −jωt [t f (t)] e dt = F{t f (t)}, = (−j) −∞

de donde se deduce que F

t f (t) ←→ j

d F (ω). dω

Teorema 6.11 (Integraci´ on). Si F (ω) es la transformada de Fourier de f (t), entonces Z

t

F

−∞

f (τ ) dτ ←→

1 F (ω) + πF (0)δ(ω). jω

Demostraci´ on. Para la prueba usaremos la transformada de Fourier del escal´ on unitario, a saber: 1 + πδ(ω), F{u(t)} = jω la cual la hallaremos m´ as adelante. Ahora, se tiene que Z t Z ∞ f (τ ) dτ, (6.2) f (t)u(t − τ )dτ = f (t) ∗ u(t) = −∞

−∞

para toda funci´ on f (t) de dominio continuo. De esta forma, usando la propiedad de convoluci´ on en la ecuaci´ on (6.2), obtenemos: F{f (t) ∗ u(t)} = F{f (t)} F{u(t)}. Ahora, usando la expresi´ on de la transformada de Fourier del escal´ on unitario y la propiedad de muestreo del impulso unitario, se obtiene   Z t  1 1 f (τ ) dτ = F (ω) F + πδ(ω) = F (ω) + πF (0)δ(ω). jω jω −∞

Se deja al lector la prueba de la ecuaci´ on (6.2)

192

6.2. PROPIEDADES DE LA TRANSFORMADA DE FOURIER

Teorema 6.12 (Modulaci´ on). Si F (ω) es la transformada de Fourier de f (t), entonces F

1 [F (ω + ω0 ) + F (ω − ω0 )]. 2

F

j [F (ω + ω0 ) − F (ω − ω0 )]. 2

cos(ω0 t) f (t) ←→ y sen (ω0 t) f (t) ←→

Demostraci´ on. Para la prueba usaremos las transformadas de Fourier de cos(ω0 t) y sen (ω0 t), a saber: F{cos(ω0 t)} = π[δ(ω − ω0 ) + δ(ω + ω0 )] y F{sen (ω0 t)} =

π [δ(ω − ω0 ) − δ(ω + ω0 )], j

las cuales la hallaremos m´ as adelante. Aplicando la propiedad de multiplicaci´ on F{cos(ω0 t) f (t)} =

1 F{cos(ω0 t)} ∗ F{f (t)}. 2π

Ahora, usando la expresi´ on de la transformada de Fourier de cos(ω0 t) y la propiedad distributiva de la convoluci´ on, se obtiene F{cos(ω0 t) f (t)} = =

1 [δ(ω − ω0 ) ∗ F (ω) + δ(ω + ω0 ) ∗ F (ω)] 2 1 [F (ω − ω0 ) + F (ω + ω0 )] . 2

Por un razonamiento similar se demuestra que j F{sen (ω0 t) f (t)} = [F (ω + ω0 ) − F (ω − ω0 )]. 2 (Se deja al lector la prueba.) Teorema 6.13 (Teorema de Parseval). Si F (ω) es la transformada de Fourier de f (t), entonces Z ∞ Z ∞ 1 |F (ω)|2 dω. |f (t)|2 dt = 2π −∞ −∞ Demostraci´ on. Aplicando la propiedad de multiplicaci´ on, se puede escribir: Z ∞ (f1 (t)f2 (t)) e−jσt dt = F{f1 (t)f2 (t)} −∞

= =

1 F{F1 } ∗ F{F2 } 2π Z ∞ 1 F1 (ω)F2 (σ − ω) dω. 2π −∞

(6.3)

en es cierta para σ = 0, y bajo esta Como (6.2) se cumple para todo σ ∈ R, ella tambi´ condici´ on (6.2) se reduce a Z ∞ Z ∞ 1 F1 (ω)F2 (−ω) dω. (6.4) f1 (t)f2 (t) dt = 2π −∞ −∞

193

CAP´ITULO 6. TRANSFORMADA DE FOURIER

Ahora, tomando f1 (t) = f (t) y f2 (t) = f (t), y luego aplicando la propiedad de conjugaci´ on F

f (t) ←→ F (−ω), de la ecuaci´ on (6.4) se deduce Z ∞ Z ∞ Z ∞ 1 1 2 |f (t)| dt = F (ω)F (ω) dω = |F (ω)|2 dω, 2π 2π −∞ −∞ −∞ que era lo que dese´ abamos demostrar. Las propiedades de la transformada de Fourier se resumen en la Tabla 6.1. Tabla 6.1. Propiedades de la Transformada de Fourier Propiedad

Descripci´ on Matem´ atica

Linealidad

F {a1 f (t) + a2 g(t)} = a1 F (ω) + a2 G(ω)

Desplazamiento en tiempo

F {f (t − t0 )} = e−jωt0 F (ω)

Desplazamiento en frecuencia

 F ejω0 t f (t) = F (ω − ω0 )

Escalamiento en tiempo

F {f (at)} =

Dualidad

F {F (t)} = 2πf (−ω)

Conjugaci´ on

n o F f (t) = F (−ω)

Convoluci´ on

F {f ∗ g (t)} = F (ω)G(ω)

Multiplicaci´ on

F {f (t)g(t)} =

Diferenciaci´ on en tiempo Diferenciaci´ on en frecuencia

Integraci´ on

F



Teorema de Parseval

1 F ∗ G (ω) 2π

 dn f (t) = (jω)n F (ω) dtn

dn F (ω) dω n  t  Z  1 F f (λ) dλ = F (ω) + πF (0)δ(ω)   jω F {tn f (t)} = j n

−∞

Modulaci´ on

1 F (ω/a) |a|

1 F {cos(ω0 t) f (t)} = [F (ω + ω0 ) + F (ω − ω0 )] 2 j F {sen (ω0 t) f (t)} = [F (ω + ω0 ) − F (ω − ω0 )] 2 Z ∞ Z ∞ 1 |f (t)|2 dt = |F (ω)|2 dω 2π −∞ −∞

194

6.3. ALGUNOS PARES DE TRANSFORMADAS

6.3 Algunos Pares de Transformadas En esta secci´ on se calcula la transformada de Fourier de funciones comunes. 1. F

δ(t) ←→ 1 Se tiene que Z

F{δ(t)} =



Z−∞ ∞

=

Z−∞ ∞

=

−∞

δ(t) e−jωt dt δ(t)(cos(−ωt) + jsen (−ωt)) dt Z ∞ δ(t) sin(ωt) dt δ(t) cos(ωt) dt − j

= cos(0) − jsen (0) = 1.

−∞

F

En la Figura 6.1 se aprecia gr´aficamente el par de transformadas δ(t) ←→ 1. δ(t)

F (ω) 1

1

F

←→ ω

t Figura 6.1. Transformada de Fourier de δ(t)

2. F

1 ←→ 2π δ(ω) Aplicando la propiedad de dualidad y considerando que δ(−t) = δ(t), se tiene F{1} = 2πδ(−ω) = 2πδ(ω). F

En la Figura 6.2 se aprecia gr´aficamente el par de transformadas 1 ←→ 2πδ(ω). f (t) = 1

F (ω) 2πδ(ω)

1 F

←→ t Figura 6.2. Transformada de Fourier de 1

ω

195

CAP´ITULO 6. TRANSFORMADA DE FOURIER

3.

F

ejω0 t ←→ 2π δ(ω − ω0 )

Aplicando la propiedad de desplazamiento en frecuencia con f (t) = 1, se tiene F{ejω0 t } = F{1}(ω − ω0 ) = 2πδ(ω − ω0 ). 4.

2 jω Para obtener la transformada de Fourier de la funci´ on signo, es necesario expresar a sgn(t) como el siguiente l´ımite de exponenciales (la prueba de esta ecuaci´ on se deja como ejercicio para el lector):   sgn(t) = l´ım e−at u(t) − eat u(−t) . F

sgn(t) ←→

a→0

De esta forma, por la continuidad de la transformada de Fourier, podemos escribir: n  o F{sgn(t)} = F l´ım e−at u(t) − eat u(−t) a→0   = l´ım F{e−at u(t)} − F{eat u(−t)} a→0  Z ∞ Z ∞ eat u(−t) e−jωt dt = l´ım e−at u(t) e−jωt dt − a→0 −∞ −∞  Z ∞ Z 0 = l´ım e−(a+jω)t dt − e(a−jω)t dt a→0  −∞  0 1 1 2 1 1 − − = . = = l´ım a→0 a + jω a − jω jω −jω jω

5.

1 + πδ(ω) jω Para establecer la transformada de Fourier del escal´ on unitario, emplearemos la siguiente relaci´ on (se deja al lector la verificaci´ on de la relaci´ on): F

u(t) ←→

sgn(t) = 2u(t) − 1, de donde se deduce:

sgn(t) 1 + . 2 2 As´ı, empleando la u ´ ltima ecuaci´ on, se obtiene: u(t) =

F{u(t)} = =

1 1 F{sgn(t)} + F{1} 2  2 1 1 1 2 + πδ(ω). + (2πδ(ω)) = 2 jω 2 jω

6. F

e−αt u(t) ←→

1 α + jω

Se tiene que F{e−αt u(t)} =

Z

∞ −∞

e−αt u(t) e−jωt dt =

Z

∞ 0

e−(α+jω)t dt =

1 . α + jω

196

6.3. ALGUNOS PARES DE TRANSFORMADAS

7. F

sen (ω0 t) ←→

π [δ(ω − ω0 ) − δ(ω + ω0 )] j

Aplicando la propiedad de desplazamiento en frecuencia, se obtiene:

= =



 ejω0 t − e−jω0 t 2j     1 F ejω0 t − F e−jω0 t 2j π 1 [2πδ(ω − ω0 ) − 2πδ(ω + ω0 )] = [δ(ω − ω0 ) − δ(ω + ω0 )]. 2j j

F{sen (ω0 t)} = F

En la Figura 6.3 se aprecia gr´aficamente la parte imaginaria de la transformada de Fourier de sen (ω0 t). Im {F (ω)}

sen (ω0 t) π F

t

←→

ω0 −ω0

ω −π

Figura 6.3. Parte imaginaria de la transformada de Fourier de sen (ω0 t)

8. F

cos(ω0 t) ←→ π [δ(ω − ω0 ) + δ(ω + ω0 )] Aplicando la propiedad de desplazamiento en frecuencia, se obtiene: 

 ejω0 t + e−jω0 t F{sen (ω0 t)} = F 2   1   jω0 t F e + F e−jω0 t = 2 1 [2πδ(ω − ω0 ) + 2πδ(ω + ω0 )] = π [δ(ω − ω0 ) + δ(ω + ω0 )]. = 2 En la Figura 6.4 se aprecia gr´aficamente la transformada de Fourier de cos(ω0 t).

197

CAP´ITULO 6. TRANSFORMADA DE FOURIER

cos(ω0 t)

F (ω) π

π

F

t

←→

ω0

−ω0

ω

Figura 6.4. Transformada de Fourier de cos(ω0 t)

9. F

pT (t) ←→ 2T

sen (ωT ) ωT

Como pT (t) = u(t + T ) − u(t − T ), entonces F{pT (t)} = F{u(t + T )} − F{u(t − T )}

= ejωT F{u(t)} − e−jωT F{u(t)}    jωT  1 −jωT + πδ(ω) = e −e jω   1 + πδ(ω) = 2jsen (ωT ) jω sen (ωT ) = 2 + 2πj sen (ωT )δ(ω) ω sen (ωT ) = 2T . ωT

En la Figura 6.7 se aprecia gr´aficamente la transformada de Fourier de pT (t). pT (t)

F (ω)

1

1 F

←→ −T

T

ω

t

Figura 6.5. Transformada de Fourier de pT (t)

10. F

e−a|t| ←→

ω2

2a , + a2

para a > 0

Como e−a|t| = eat u(−t) + e−at u(t),

198

6.3. ALGUNOS PARES DE TRANSFORMADAS

entonces  F{e−a|t| } = F eat u(−t) + e−at u(t) Z Z ∞ at −jωt e u(−t) e dt + = = =

−∞ Z 0

−∞ Z 0

at −jωt

e e

dt +

e(a−jωt) dt +



0 Z ∞

e−at u(t) e−jωt dt

−∞

e−at e−jωt dt e−(a+jωt) dt

0

−∞

=

Z



1 1 2a + = 2 . a − jω a + jω ω + a2

En la Figura 6.6 se aprecia gr´aficamente la transformada de Fourier de e−a|t| . F (ω)

e−a|t| 1

2/a

F

←→ ω

t Figura 6.6. Transformada de Fourier de e−a|t|

11.

1 π −a|ω| F e , ←→ a2 + t2 a

a>0

Usando las propiedades de linealidad y dualidad, se tiene       π 1 1 2a 1  −a|ω| = e−a|ω| . 2πe F = F = a2 + t 2 2a a2 + t2 2a a En la Figura 6.7 se aprecia gr´aficamente la transformada de Fourier de

1 . + t2

F (ω)

1 a2 +t2

1/a

a2

π/a

F

←→

2

t Figura 6.7. Transformada de Fourier de

ω

a2

1 + t2

En la Tabla 6.1 se muestra un resumen de los pares de transformadas de Fourier de funciones comunes.

199

CAP´ITULO 6. TRANSFORMADA DE FOURIER

Tabla 6.2. Algunos Pares de Transformadas de Fourier Funci´ on

Transformada de Fourier

1

2πδ(ω)

δ(t)

1

u(t)

πδ(ω) +

δ(t − t0 )

e−jωt0

sgn(t)

2 jω

ejω0 t

2πδ(ω − ω0 )

cos (ω0 t)

π[δ(ω − ω0 ) + δ(ω + ω0 )]

sen (ω0 t)

π [δ(ω − ω0 ) − δ(ω + ω0 )] j

cos (ω0 t)u(t) sen (ω0 t)u(t)

1 jω

jω π [δ(ω − ω0 ) + δ(ω + ω0 )] + 2 2 ω0 − ω 2 π ω0 [δ(ω − ω0 ) − δ(ω + ω0 )] + 2 2j ω0 − ω 2

e−at u(t), Re {a} > 0

1 a + jω

te−at u(t), Re {a} > 0

1 (a + jω)2

tn−1 −at e u(t), Re {a} > 0 (n − 1)!

1 (a + jω)n

e−a|t| , a > 0 |t|e−a|t| , Re {a} > 0 a2

1 , Re {a} > 0 + t2

t , Re {a} > 0 2 a + t2 2

e−at , a > 0

a2

2a + ω2

4ajω + ω2

a2

π −a|ω| e a −jπωe−a|ω| 2a r π −ω2 /4a e a

200

6.4. MAGNITUD Y FASE

6.4 Magnitud y Fase Sea f (t) una funci´ on de dominio continuo cuya transformada de Fourier es F (ω). Recordemos que F (ω) es un n´ umero complejo para cada ω ∈ R, por lo que puede expresarse en coordenadas polares, F (ω) = |F (ω)| ejarg (F (ω)) , donde |F (ω)| y arg (F (ω)) son el valor absoluto y el argumento de la transformada de Fourier de f (t), respectivamente. De esta representaci´ on en coordenadas polares surgen las definiciones de magnitud y fase. Definici´ on 6.3 (Magnitud). Sea F (ω) la transformada de Fourier de f (t). La magnitud o amplitud de f (t) se define como el valor absoluto de su transformada de Fourier, esto es, A(ω) = |F (ω)|, para cada ω ∈ R. La funci´ on A(ω) se denomina espectro de magnitud de f (t). Definici´ on 6.4 (Fase). Sea F (ω) la transformada de Fourier de f (t). La fase de f (t) se define como el argumento de su transformada de Fourier, esto es, para cada ω ∈ R.

φ(ω) = arg (F (ω)),

La funci´ on φ(ω) se denomina espectro de fase de f (t). Para calcular la fase es necesario fijar una determinaci´ on de arg (F (ω)). En este escrito la fase se calcular´a como el argumento principal, es decir, φ(ω) = Arg (F (ω)). La siguiente proposici´ on es un resultado de mucha utilidad para el c´ alculo de la magnitud y la fase de una funci´ on. Esta proposici´ on establece que si f (t) es una funci´ on real, entonces la magnitud es una funci´ on par y la fase es una funci´ on impar. Proposici´ on 6.1. Si f (t) es una funci´on real, entonces el espectro de magnitud A(ω) es una funci´on par, y el espectro de fase φ(ω) es una funci´on impar. Demostraci´ on. Por la definici´ on de F (ω) se tiene que F (ω) = Re {F (ω)} + jIm {F (ω)} , donde Re {F (ω)} =

Z

e Im {F (ω)} = −



f (t) cos(ωt) dt

(6.5)

−∞

Z



f (t) sen (ωt) dt.

(6.6)

−∞

As´ı, los espectros de magnitud y fase de f (t) est´ an dados respectivamente por: A(ω) = |F (ω)| =

p

(Re {F (ω)})2 + (Im {F (ω)})2

(6.7)

201

CAP´ITULO 6. TRANSFORMADA DE FOURIER

y   0,      Im   arctan    Re     π/2,      Im   arctan Re φ(ω) =  π,      Im   arctan    Re     −π/2,      Im   arctan Re

Re {F (ω)} > 0, Im {F (ω)} = 0,

{F (ω)} {F (ω)}



,

{F (ω)} {F (ω)}



+ π, Re {F (ω)} < 0, Im {F (ω)} > 0,

{F (ω)} {F (ω)}



− π, Re {F (ω)} < 0, Im {F (ω)} < 0,

{F (ω)} {F (ω)}



,

Re {F (ω)} > 0, Im {F (ω)} > 0, Re {F (ω)} = 0, Im {F (ω)} > 0, Re {F (ω)} < 0, Im {F (ω)} = 0,

(6.8)

Re {F (ω)} = 0, Im {F (ω)} < 0, Re {F (ω)} > 0, Im {F (ω)} < 0.

Veamos que A(ω) es par. Como la funci´ on real | · | es una funci´ on par y, por (6.5) y (6.6), Re {F (ω)} y Im {F (ω)} son funciones reales (por ser f (t) una funci´ on real), entonces por on A(ω) es par. (6.7) la funci´ Veamos ahora que φ(ω) es impar. Por (6.5) y (6.6) se tiene que Re {F (−ω)} = Re {F (ω)}

Im {F (−ω)} = −Im {F (ω)} .

e

De esta forma, por (6.8), se obtiene   0,      Im   − arctan    Re     π/2,      Im   − arctan Re φ(−ω) =  π,      Im   − arctan    Re     −π/2,      Im   − arctan Re

Re {F (ω)} > 0, Im {F (ω)} = 0,

{F (ω)} {F (ω)}



,

{F (ω)} {F (ω)}



+ π,

{F (ω)} {F (ω)}



− π,

{F (ω)} {F (ω)}



,

Re {F (ω)} > 0, Im {F (ω)} < 0, Re {F (ω)} = 0, Im {F (ω)} < 0, Re {F (ω)} < 0, Im {F (ω)} < 0, Re {F (ω)} < 0, Im {F (ω)} = 0, Re {F (ω)} < 0, Im {F (ω)} > 0, Re {F (ω)} = 0, Im {F (ω)} > 0, Re {F (ω)} > 0, Im {F (ω)} > 0.

= −φ(ω),

en otras palabras, φ(ω) es una funci´ on impar.

En los siguientes ejemplos se calculan los espectros de magnitud y fase de diferentes funciones comunes.

202

6.4. MAGNITUD Y FASE

Ejemplo 6.3. Determine los espectros de magnitud y fase de la funci´ on f (t) = e−2t u(t). Soluci´ on. Se tiene que la transformada de Fourier de f (t) es F (ω) =

1 . 2 + jω

Luego, A(ω) A(ω) =

1 1 =√ |2 + jω| 4 + ω2

ω

y φ(ω) π/2

φ(ω) = Arg



1 2 + jω



= − arctan(ω/2).

ω

−π/2



Ejemplo 6.4. Determine los espectros de magnitud y fase de la funci´ on f (t) = u(t). Soluci´ on. Se tiene que la transformada de Fourier de f (t) es F (ω) =

1 + π δ(ω). jω

As´ı, A(ω) 1 1 A(ω) = + π δ(ω) = , jω |ω| y

para ω 6= 0 ω

203

CAP´ITULO 6. TRANSFORMADA DE FOURIER

φ(ω) 

 1 φ(ω) = Arg + π δ(ω) jω   1 = Arg , ω 6= 0 jω ( π/2, ω < 0, = −π/2, ω > 0.

π/2

ω

−π/2

 Ejemplo 6.5. Determine los espectros de magnitud y fase de la funci´ on f (t) = sgn(t). Soluci´ on. Se tiene que la transformada de Fourier de f (t) es F (ω) =

2 . jω

As´ı,

A(ω) 2 2 , A(ω) = = jω |ω|

para ω 6= 0 ω

y φ(ω) π/2





2 φ(ω) = Arg , ω 6= 0 jω ( π/2, ω < 0, = −π/2, ω > 0.

ω

−π/2

 Ejemplo 6.6. Determine los espectros de magnitud y fase de la funci´ on f (t) = δ(t). Soluci´ on. Se tiene que la transformada de Fourier de f (t) es F (ω) = 1. As´ı,

204

6.4. MAGNITUD Y FASE

A(ω) 1

A(ω) = 1,

para ω ∈ R ω

y φ(ω) φ(ω) = Arg (1) = 0,

para ω ∈ R. ω 

Ejemplo 6.7. Determine los espectros de magnitud y fase de la funci´ on

f (t) = e−|t| .

Soluci´ on. Se tiene que la transformada de Fourier de f (t) es

F (ω) =

2 1 + ω2

As´ı, A(ω) A(ω) =

2 , 1 + ω2

1

para ω ∈ R ω

y φ(ω) φ(ω) = Arg



2 1 + ω2



= 0,

para ω ∈ R. ω 

205

CAP´ITULO 6. TRANSFORMADA DE FOURIER

6.5 Transformada de Fourier con M ATLAB La herramientas de matem´ atica simb´ olica de M ATLAB cuenta con el comando fourier que permite calcular la transformada de Fourier de f (t). Observe el siguiente ejemplo donde se calcula la transformada de Fourier de las siguientes funciones: f1 (t) = e−2t u(t),

f2 (t) = e−|t| ,

f3 (t) = p1 (t).

✄ >> syms t w >> f 1 ( t )=exp(−2∗ t )∗ h e a v i s i d e ( t ) ; F1 (w)= f o u r i e r ( f 1 ) F1 (w) = 1/(w∗ i + 2) >> f 2 ( t )=exp(−abs ( t ) ) ; F2 (w)= f o u r i e r ( f 1 ) F2 (w) = 1/(w∗ i + 2) >> f 3 ( t )= h e a v i s i d e ( t +1)−h e a v i s i d e ( t −1); F2 (w)= f o u r i e r ( f 3 ) F2 (w) = − ( c o s (w)∗ i − s i n (w) ) /w + ( c o s (w)∗ i + s i n (w) ) /w





Las transformadas de Fourier obtenidas son: F1 (ω) =

1 , jω

F2 (ω) =

ω2

2 , +1

F3 (ω) =

2 sin(ω) ω

Programa 6.1. Funci´ on magnitudfase.m f u n c t i o n [A , p h i ] = magnitu dfase ( f , w0) %magnitu dfase H a l l a e l v a l o r de l a magnitud A(w) y de l a % f a s e p h i (w) , en w = w0 % p = A(w0) , p h i = p h i (w0) syms t w; F (w) = f o u r i e r ( f ) ; A = abs ( dou ble ( F (w0 ) ) ) ; p h i = a n g l e ( dou ble ( F (w0 ) ) ) ; end

El Programa 6.1 muestra la funci´ on magnitudfase que calcula la magnitud y la fase de f (t) en ω = ω0 , es decir, calcula los valores A(ω0 ) y φ(ω0 ). Calculemos el valor de la magnitud y la fase en ω = 1/2, de cada una de las funciones f1 (t), f2 (t) y f3 (t), dadas arriba, en otras palabras, hallemos los valores de A(1/2) y φ(1/2) para cada una de estas funciones. ✄ syms t w0 = 1/2; f 1 ( t )=exp(−2∗ t )∗ h e a v i s i d e ( t ) ; [A1 , phi1 ] = magnitu dfase ( f1 , w0) = 0.4851 phi1 = −0.2450 >> f 2 ( t )=exp(−abs ( t ) ) ; [ A2 , phi2 ] = magnitu dfase ( f2 , w0) A2 = 1.6000 phi2 = 0 >> f 3 ( t )= h e a v i s i d e ( t +1)−h e a v i s i d e ( t −1); [A3 , phi3 ] = magnitu dfase ( f3 , w0) >> >> >> A1

206

6.5. TRANSFORMADA DE FOURIER CON MATLAB

A3 = 1.9177 phi3 = 0





La funci´ on magnitudfase no s´ olo permite hallar el valor de la magnitud y la fase en ω = ω0 , sino que tambi´ en podemos usarla para obtener una gr´afica de los espectros de magnitud y fase de una funci´ on. A continuaci´ on, con las funciones f1 (t), f2 (t) y f3 (t) creadas previamente, se hallan los espectros de magnitud y fase, para w ∈ (0, 10]. φ (ω)

A (ω) 1

1

0.8

2

0.6

1

0.4

0

0.2

−1

0 −10

−5

0

5

10

−2 −10

−5

2

1

1.5

0.5

1

0

0.5

−0.5 −5

0

5

10

−1 −10

−5

3

4

1.5

2

1

0

0.5

−2 0

0

5

10

5

10

3

2

−5

10

φ (ω)

A (ω)

0 −10

5

2

2

0 −10

0 φ (ω)

A (ω)

5

10

−4 −10

−5

0

Para la construcci´ on de las gr´aficas anteriores, tomamos en cuenta que las funciones consideradas todas eran reales. Por ello, el espectro de magnitud es una funci´ on par y el espectro de fase es impar. Seguidamente mostramos el c´ odigo usado para la generaci´ on de las gr´aficas. ✄ >> >> >> >> >> >> >> >> >> >> >> >> >> >> >> >> >>



syms t w0 = 0 . 0 1 : 0 . 1 : 1 0 ; f 1 ( t )=exp(−2∗ t )∗ h e a v i s i d e ( t ) ; [A1 , phi1]=magnitu dfase ( f1 , w0) ; f 2 ( t )=exp(−abs ( t ) ) ; [ A2 , phi2]=magnitu dfase ( f2 , w0 ) ; f 3 ( t )= h e a v i s i d e ( t +1)−h e a v i s i d e ( t −1); [A3 , phi3 ] = magnitu dfase ( f3 , w0) ; s u b p l o t ( 3 , 2 , 1 ) , p l o t (−w0, A1 , ’ b ’ , ’ LineWidth ’ , 2 ) , hold on p l o t (w0, A1 , ’ b ’ , ’ LineWidth ’ , 2 ) , t i t l e ( ’ A 1 (\ omega ) ’ ) , g r i d on s u b p l o t ( 3 , 2 , 2 ) , p l o t (−w0,−phi1 , ’ b ’ , ’ LineWidth ’ , 2 ) , hold on p l o t (w0, phi1 , ’ b ’ , ’ LineWidth ’ , 2 ) , t i t l e ( ’ \ p h i 1 (\ omega ) ’ ) , g r i d on s u b p l o t ( 3 , 2 , 3 ) , p l o t (−w0, A2 , ’ b ’ , ’ LineWidth ’ , 2 ) , hold on p l o t (w0, A2 , ’ b ’ , ’ LineWidth ’ , 2 ) , t i t l e ( ’ A 2 (\ omega ) ’ ) , g r i d on s u b p l o t ( 3 , 2 , 4 ) , p l o t (−w0,−phi2 , ’ b ’ , ’ LineWidth ’ , 2 ) , hold on p l o t (w0, phi2 , ’ b ’ , ’ LineWidth ’ , 2 ) , t i t l e ( ’ \ p h i 2 (\ omega ) ’ ) , g r i d on s u b p l o t ( 3 , 2 , 5 ) , p l o t (−w0, A3 , ’ b ’ , ’ LineWidth ’ , 2 ) , hold on p l o t (w0, A3 , ’ b ’ , ’ LineWidth ’ , 2 ) , t i t l e ( ’ A 3 (\ omega ) ’ ) , g r i d on s u b p l o t ( 3 , 2 , 6 ) , p l o t (−w0,−phi3 , ’ b ’ , ’ LineWidth ’ , 2 ) , hold on p l o t (w0, phi3 , ’ b ’ , ’ LineWidth ’ , 2 ) , t i t l e ( ’ \ p h i 3 (\ omega ) ’ ) , g r i d on



207

CAP´ITULO 6. TRANSFORMADA DE FOURIER

M ATLAB tambi´ en cuenta con el comando ifourier(F) que permite calcular la transformada inversa de Fourier de F (ω). Observe el siguiente ejemplo donde se calcula la transformada inversa de Fourier de las siguientes transformadas: F1 (ω) = q1 (10ω),

F2 (ω) = δ(ω) + 2 p1 (−ω),

F3 (ω) = j sgn(2ω).

✄ >> syms w t >> u (w) = h e a v i s i d e (w) ; >> F1=(1+10∗w) ∗ ( u(10∗w+1)−u(10∗w))+(1−10∗w) ∗ ( u(10∗w)−u(10∗w−1)); >> f 1 = s i m p l e ( i f o u r i e r ( F1 ) ) f1 = (20∗ s i n ( x / 2 0 ) ˆ 2 ) / ( p i ∗ x ˆ2) >> F2=d i r a c (w)+2∗(u(−w+1)−u(−w−1)); >> f 2 = s i m p l e ( i f o u r i e r ( F2 ) ) f2 = ((2∗ s i n ( x ) ) / x + 1/2)/ p i >> F3=i ∗( u (2∗w)−u(−2∗w) ) ; >> f 3 = s i m p l e ( i f o u r i e r ( F3 ) ) f3 = −1/( p i ∗x )





Las transformadas inversas de Fourier obtenidas son: f1 (t) =

20 sin2 (t/20) , π t2

f2 (t) =

2 sin(t) 1 + , πt 2π

f3 (t) = −

1 πt

6.6 Problemas Resueltos Problema 6.1. Determinar la transformada de Fourier del pulso rectangular p1 (t). Soluci´ on. Hallaremos la transformada de Fourier por dos procedimientos: i) por definici´ on, y ii) por propiedades. i) Por definici´ on, la transformada de Fourier de p1 (t) est´ a dada por:  −jωt 1 Z 1− Z ∞ e e−jωt dt = − p1 (t)e−jωt dt = F (ω) = jω −1 + −1 −∞ =

2 sen ω ejω − e−jω = jω ω

ii) Como p1 (t) = u(t + 1) − u(t − 1), entonces usando convenientemente las propiedades de linealidad y desplazamiento en tiempo, obtenemos: F (ω) = F {p1 (t)} = F {u(t + 1) − u(t − 1)} = F {u(t + 1)} − F {u(t − 1)}

= ejω F {u(t)} − e−jω F {u(t)}     1 1 −jω jω + πδ(ω) − e + πδ(ω) = e jω jω ejω − e−jω 2 sen ω = = jω ω 

208

6.6. PROBLEMAS RESUELTOS

Problema 6.2. Determinar la transformada de Fourier del pulso triangular q1 (t). Soluci´ on. Hallaremos la transformada de Fourier por dos procedimientos: i) por definici´ on, y ii) por propiedades. i) Por definici´ on, la transformada de Fourier de q1 (t) est´ a dada por: Z ∞ q1 (t)e−jωt dt F (ω) = = = =

Z

−∞ 0

−jωt

(t + 1)e

dt +

−1

Z

0

1

(1 − t)e−jωt dt

1 − ejω + jω e−jω − 1 + jω − ω2 ω2 2(1 − cos ω) ω2

ii) Como q1 (t) = (t + 1) [u(t + 1) − u(t)] + (1 − t) [u(t) − u(t − 1)] = (t + 1)u(t + 1) − 2t u(t) + (t − 1)u(t − 1),

entonces usando convenientemente las propiedades de linealidad, desplazamiento en tiempo y diferenciaci´ on en frecuencia, obtenemos: F (ω) = F {q1 (t)} = F {(t + 1)u(t + 1) − 2t u(t) − (1 − t)u(1 − t)} = F {(t + 1)u(t + 1)} − 2F {tu(t)} + F {(t − 1)u(t − 1)}

= ejω F {tu(t)} − 2F {tu(t)} + e−jω F {tu(t)}    1 ′ −jω −jω = e −2+e − 2 + πjδ (ω) ω 2(1 − cos ω) = ω2

 Problema 6.3. Determinar la transformada de Fourier de la funci´ on f (t) = u(t) − u(t − 5). Soluci´ on. Usando convenientemente las propiedades de linealidad y desplazamiento en tiempo, obtenemos: F (ω) = F {u(t) − u(t − 5)}

= F {u(t)} − F {u(t − 5)}   1 1 −5jω + πδ(ω) − e + πδ(ω) = jω jω e−5jω 1 + πδ(ω) − − e−5jω πδ(ω) = jω jω 1 − e−5jω = . jω 

209

CAP´ITULO 6. TRANSFORMADA DE FOURIER

Problema 6.4. Determinar la transformada de Fourier de la funci´ on f (t) = q1/2 (t − 1). Soluci´ on. Primero calculemos por definici´ on la transformada de Fourier de q1/2 (t). Aplicando la definici´ on de la trasformada de Fourier y considerando la identidad trigonom´etrica 1 − cos(2α) = 2 sen 2 α, podemos escribir: Z ∞  q1/2 (t)e−jωt dt F q1/2 (t) = = = = =

Z

−∞ 0

−jωt

(2t + 1) e −1/2

dt + − jω 2

jω 2

Z

1/2

0

(1 − 2t) e−jωt dt

− 2 + jω 2 − 2e + jω 2e − 2 ω ω2  jω jω 1  4 − 2e 2 − 2e− 2 ω2 4 8 sen 2 (ω/4) (1 − cos(ω/2)) = . ω2 ω2

Ahora, usando la propiedad de desplazamiento en tiempo obtenemos:   8 e−jω sen 2 (ω/4) . F (ω) = F q1/2 (t − 1) = e−jω F q1/2 (t) = ω2



Problema 6.5. Determinar la transformada de Fourier de la funci´ on f (t) = p1/2 ((t − 2)/2). Soluci´ on. tiene que

Primero calculemos por definici´ on la transformada de Fourier de p1/2 (t). Se



F p1/2 (t) =

Z



−∞

−jωt

p1/2 (t)e

dt =

Z

1/2−

e−jωt dt =

−1/2+

2 sen (ω/2) . ω

Ahora, usando convenientemente las propiedades de desplazamiento en tiempo y escalamiento en tiempo, obtenemos:       1 t −2jω F (ω) = F p1/2 (t − 2) =e F p1/2 2 2 −2jω  2e sen (ω) = e−2jω 2 F p1/2 (t) (2ω) = . ω 

Problema 6.6. Determinar la transformada de Fourier de la funci´ on f (t) = e2t u(−t) + u(t). Soluci´ on. Usando la propiedad de linealidad obtenemos:   F (ω) = F e2t u(−t) + u(t) = F e2t u(−t) + F {u(t)}  1 + πδ(ω). = F e2t u(−t) + jω

(6.9)

210

6.6. PROBLEMAS RESUELTOS

 Ahora, calculemos la transformada F e2t u(−t) . Sea g(t) la funci´ on definida como g(t) = e−2t u(t).

Por definici´ on, la transformada de Fourier de g(t) est´ a dada por: #r " Z ∞ Z ∞ −(2+jω)t 1 e = . e−(2+jω)t dt = l´ım − e−2t u(t)e−jωt dt = G(ω) = r→∞ 2 + jω 2 + jω 0+ −∞ 0

As´ı, usando la propiedad de escalamiento en tiempo, podemos escribir:  F e2t u(−t) = F {g(−t)} = G(−ω) =

1 . 2 − jω

(6.10)

Usando convenientemente las ecuaciones (6.9) y (6.10), obtenemos: F (ω) =

1 2 1 + + πδ(ω) = + πδ(ω). 2 − jω jω jω(2 − jω) 

Problema 6.7. Sea f (t) una funci´ on de dominio continuo cuya transformada de Fourier est´ a dada por: F (ω) = p1 ((ω − 1)/2).

Determine la transformada de Fourier de g(t) = f (2t − 1) e−2jt . Soluci´ on. Se tiene que la funci´ on g(t) se puede escribir como   1 e−2jt , g(t) = h t − 2

donde h(t) = f (2t). As´ı, aplicando la propiedad de desplazamiento en frecuencia, se obtiene    1 G(ω) = F {g(t)} = F h t − (ω + 2). 2

Ahora, aplicando convenientemente las propiedades de desplazamiento en tiempo y escalamiento en tiempo, podemos escribir:    1 F h t− = e−jω/2 F {h(t)} 2 = e−jω/2 F {f (2t)} −jω/2

= e

=



 1 F (ω/2) 2

1 −jω/2 e p1 ((ω − 2)/4). 2

De esta forma, la transformada de Fourier de g(t) es G(ω) =

1 −j(ω+2)/2 e p1 (ω/4) 2 

211

CAP´ITULO 6. TRANSFORMADA DE FOURIER

Problema 6.8. Determine la transformada inversa de Fourier de F (ω) = u(ω) − δ(ω/2). Soluci´ on. Aplicando la propiedad de dualidad podemos escribir: F {F (t)} = 2πf (−ω).

(6.11)

Por otra parte, aplicando convenientemente las propiedades de linealidad y escalamiento en tiempo, tenemos: F {F (t)} = F {u(t) − δ(t/2)} = F {u(t)} − F {δ(t/2)} 1 + πδ(ω) − 2. = jω

(6.12)

Ahora, usando las ecuaciones (6.11) y (6.12), obtenemos: 2πf (−ω) =

1 + πδ(ω) − 2. jω

Evaluando la expresi´ on anterior en ω = −t y luego despejando f (t), tenemos que la transformada inversa de Fourier de F (ω) est´ a dada por:   1 1 πδ(t) − 2 − . f (t) = 2π jt  Problema 6.9. Determine la transformada inversa de Fourier de F (ω) = q1 (10ω). Soluci´ on. Aplicando convenientemente la propiedad de escalamiento en tiempo, tenemos: F {F (t)} = F {q1 (10t)} =

1 20(1 − cos(ω/10)) F {q1 (t)} (ω/10) = . 10 ω2

(6.13)

Ahora, usando las ecuaciones (6.11) y (6.13), obtenemos: 2πf (−ω) =

20(1 − cos(ω/10)) . ω2

Evaluando la expresi´ on anterior en ω = −t y luego despejando f (t), tenemos que la transformada inversa de Fourier de F (ω) est´ a dada por: f (t) =

10(1 − cos(t/10)) . πt2 

Problema 6.10. Determine la transformada inversa de Fourier de F (ω) = δ(ω) + 2p1 (−ω). Soluci´ on. Aplicando convenientemente las propiedades de linealidad y escalamiento en tiempo, tenemos: F {F (t)} = F {δ(t) + 2p1 (−t)} = F {δ(t)} + 2F {p1 (−t)} = 1 + 2F {p1 (t)} (−ω) 4 sen ω . = 1+ ω

(6.14)

212

6.6. PROBLEMAS RESUELTOS

Ahora, usando las ecuaciones (6.11) y (6.14), obtenemos: 2πf (−ω) = 1 +

4 sen ω . ω

Evaluando la expresi´ on anterior en ω = −t y luego despejando f (t), tenemos que la transformada inversa de Fourier de F (ω) est´ a dada por: f (t) =

2 sen t 1 + 2π πt 

Problema 6.11. Determine la transformada inversa de Fourier de F (ω) = j sgn(2ω). Soluci´ on. Aplicando convenientemente las propiedades de linealidad y escalamiento en tiempo, tenemos: F {F (t)} = F {j sgn(2t)} = jF {sgn(2t)} 2 j F {sgn(t)} (ω/2) = . = 2 ω

(6.15)

Ahora, usando las ecuaciones (6.11) y (6.15), obtenemos: 2πf (−ω) =

2 . ω

Evaluando la expresi´ on anterior en ω = −t y luego despejando f (t), tenemos que la transformada inversa de Fourier de F (ω) est´ a dada por: 1 f (t) = − t−1 . π  Problema 6.12. Determine los espectros de magnitud y fase del pulso rectangular p1 (t). Soluci´ on. La transformada de Fourier de p1 (t) (ver Problema 6.1) es: F (ω) =

2 sen ω . ω

Luego, el espectro de magnitud de p1 (t) es: A(ω) =

2 |sen ω| , |ω|

para ω 6= 0.

A(ω) 2

1

ω −6π

−5π

−4π

−3π

−2π

−π

π











213

CAP´ITULO 6. TRANSFORMADA DE FOURIER

Ahora, se tiene que arg (F (ω)) = arg (2 sen ω) − arg (ω), Luego, el espectro de fase de p1 (t) es:   0,    −π, φ(ω) =  π,    0,

ω ω ω ω

< 0, < 0, > 0, > 0,

para ω 6= 0.

sen ω sen ω sen ω sen ω

< 0, > 0, < 0, > 0,

φ(ω) π

··· −6π

···

−5π

−4π

−3π

−2π

−π

π









ω



−π

 Problema 6.13. Determine los espectros de magnitud y fase del pulso triangular q1 (t). Soluci´ on. La transformada de Fourier de q1 (t) (ver Problema 6.2) es: F (ω) =

2(1 − cos ω) . ω2

Luego, el espectro de magnitud de q1 (t) es: A(ω) =

2 (1 − cos ω) , ω2

para ω 6= 0.

A(ω) 1

ω −6π

−5π

−4π

−3π

−2π

−π

π











Como 2(1 − cos ω) > 0 y ω 2 > 0, para todo ω ∈ R, entonces el espectro de fase de q1 (t) es:   2(1 − cos ω) = 0, para ω 6= 0. φ(ω) = Arg (F (ω)) = Arg ω2 

214

6.6. PROBLEMAS RESUELTOS

Problema 6.14. Determine los espectros de magnitud y fase de f (t) = u(t) − u(t − 5). Soluci´ on. La transformada de Fourier de f (t) = u(t) − u(t − 5) (ver Problema 6.3) es: F (ω) =

1 − cos(5ω) + j sen (5ω) 1 − e−5jω = . jω jω

Luego, el espectro de magnitud de f (t) es: p p (1 − cos(5ω))2 + sen 2 (5ω) 2 − 2 cos(5ω) A(ω) = = , |ω| |ω|

para ω 6= 0.

A(ω)

5

ω −6π

−5π

−4π

−3π

−2π

−π

π











Ahora, se tiene que arg (F (ω)) = arg (1 − e5jω ) − Arg (jω), as´ı, arg (F (ω)) =

(

para ω 6= 0.

arg (1 − cos(5ω) + j sen (5ω)) + π2 , ω < 0, arg (1 − cos(5ω) + j sen (5ω)) − π2 , ω > 0.

Adem´ as, 1 − cos(5ω) > 0, para todo ω ∈ R. De esta forma, el espectro de fase de f (t) es:    π sen (5ω)   + , ω < 0, arctan  1 − cos(5ω) 2 φ(ω) =    sen (5ω) π   − , ω > 0. arctan 1 − cos(5ω) 2 π

φ(ω)

···

ω ··· −π

 Problema 6.15. Determine los espectros de magnitud y fase de f (t) = q1/2 (t − 1). Soluci´ on. La transformada de Fourier de f (t) = q1/2 (t − 1) (ver Problema 6.4) es: 8 e−jω sen 2 (ω/4) . ω2 Luego, el espectro de magnitud de f (t) es: F (ω) =

A(ω) =

8 sen 2 (ω/4) , ω2

para ω 6= 0.

215

CAP´ITULO 6. TRANSFORMADA DE FOURIER

A(ω) 1/2

ω −3π

Como

−2π

−π

π





8 sen 2 (ω/4) ω2

> 0, para ω 6= 0, podemos escribir:   8 sen 2 (ω/4) arg (F (ω)) = arg (e−jω ) + Arg = −ω, ω2

para ω 6= 0.

as´ı, el espectro de fase de f (t) es, para ω 6= 0: φ(ω) =

∞ ∞ X X (2kπ − ω)pπ (ω − 2kπ) − (ω + 2kπ)pπ (ω + 2kπ). k=0

k=1

π

φ(ω)

··· −5π

··· −4π

−3π

−2π

−π

π







ω



−π

 Problema 6.16. Determine los espectros de magnitud y fase de f (t) = e2t u(−t) + u(t). Soluci´ on. La transformada de Fourier de f (t) = e2t u(−t) + u(t) (ver Problema 6.6) es: F (ω) =

2 + πδ(ω). jω(2 − jω)

Luego, el espectro de magnitud de f (t) es: A(ω) =

2 , |ω| 4 + ω 2 √

para ω 6= 0.

A(ω)

ω −3π

−2π

−π

π





216

6.7. PROBLEMAS PROPUESTOS

Ahora, se tiene que arg (F (ω)) = Arg (2) − Arg (jw) − Arg (2 − jω) = −Arg (jw) − Arg (2 − jω),

Como

( −π/2, Arg (jw) = π/2,

ω < 0, ω > 0,

y

para ω 6= 0.

Arg (2 − jω) = − arctan(ω/2),

entonces el espectro de fase de f (t) es, para ω 6= 0: ( π/2 + arctan(ω/2), φ(ω) = −π/2 + arctan(ω/2),

ω < 0, ω > 0,

φ(ω) π 2

ω − π2



6.7 Problemas Propuestos 6.1. Obtener la transformada de Fourier de las siguientes funciones: a) f (t) = et/2 u(−t) b) f (t) = p1 (t + 1) c) f (t) = q1/4 (t) ( t + 4 sgn(t), si |t| > 0; d) f (t) = 0, si t = 0 e) f (t) = |t|u(t) f) f (t) = 2πδ(t − t0 ), donde t0 ∈ R

g) f (t) = π [δ(t − π) + δ(t + π)]

h) f (t) = u(−t − 1/2) q1 (t) i) f (t) = e−a|t| , a > 0 1 j) f (t) = 2 , a > 0. a + t2 k) f (t) = 1 + j sgn(t)

l) f (t) = p1 (t) + jq1/2 (t)

6.2. Sea f (t) una funci´ on de dominio continuo cuya transformada de Fourier es F (ω). Pruebe que la transformada de Fourier de la funci´ on g(t) = f (t) cos ω0 t, con ω0 ∈ R, est´ a dada por 1 F{g(t)} = {f (t) cos ω0 t} = [F (ω − ω0 ) + F (ω + ω0 )]. 2

6.3. Sea f (t) una funci´ on de dominio continuo cuya transformada de Fourier es F (ω) = p1 ((ω − 1)/2). Determine la transformada de Fourier de las siguientes funciones de dominio continuo, utilizando las propiedades de la transformada de Fourier:

217

CAP´ITULO 6. TRANSFORMADA DE FOURIER

h) g8 (t) = f (2t − 1)e−2jt

a) g1 (t) = f (−t)

i) g9 (t) = f (t)e−2jt

b) g2 (t) = t f (t) c) g3 (t) = f (t + 1)

j) g10 (t) = tf (t)e−2jt

d) g4 (t) = f (3 − 5t)

k) g11 (t) = (t − 1)f (t − 1)e−2jt Z t f (τ ) dτ l) g12 (t) =

e) g5 (t) = (t − 1) f (t + 1) d f) g6 (t) = f (t) dt d g) g7 (t) = t f (t) dt

6.4. Utilizando la relaci´ on Z



−∞

m) g13 (t) = f (t) sen (πt)

n) g14 (t) = f (t) ∗ δ(t − 1)

1 f (t)g(t) dt = 2π −∞

Z



F (ω)G(ω) dω

−∞

y un par de transformadas conocida demostrar que Z ∞ 1 π dt = 3 , a > 0. 2 2 2 (a + t ) 4a 0 6.5. Hallar la transformada inversa de Fourier de las siguientes transformadas: a) F (ω) = u(ω) − δ(−ω/2).

b) F (ω) = q1 (ω/2).

c) F (ω) = δ(−ω) + 2p1 (−ω). d) F (ω) = j sgn(−2ω). 6.6. Calcular los espectros de magnitud y fase de las siguientes funciones de dominio continuo: a) f (t) = et/2 u(−t) b) f (t) = p1 (t + 1) c) f (t) = q1/4 (t) d) f (t) = r(t)

e) f (t) = p1/2 ((t − 2)/2)

f) f (t) = |t|u(t) ( t + 4 sgn(t), g) f (t) = 0,

si |t| > 0; si t = 0

7 Transformada de Laplace En este cap´ıtulo se presenta la transformada de Laplace. El nombre de Laplace es en honor a Pierre Simnon Laplace (1749-1827), matem´atico y astr´ onomo franc´es. Se incluyen su definici´ on, sus propiedades y los teoremas de valor inicial y final. Se derivan las transformadas de Laplace de funciones comunes. Finalmente, se calcula la transformada inversa de Laplace a trav´ es de dos m´ etodos: integraci´ on de contornos e inversi´ on por tablas.

7.1 Definici´ on La transformada de Laplace puede considerarse como una generalizaci´ on de la transformada de Fourier; en forma m´ as precisa, la adici´ on de un factor exponencial al integrando de la definici´ on de la transformada de Fourier da como resultado la transformada de Laplace bilateral o de dos lados. Definici´ on 7.1 (Transformada de Laplace Bilateral). Sea f (t) una funci´ on de dominio continuo definida de R en C. La transformada de Laplace bilateral de f (t) se define como Z ∞ f (t) e−st dt, F (s) = L[f (t)] = −∞

para todo s ∈ C tal que |F (s)| < ∞. Cambiando los l´ımites de integraci´ on en la definici´ on de la transformada de Laplace bilateral obtenemos las transformadas de Laplace unilaterales. Definici´ on 7.2 (Transformadas de Laplace Unilaterales). Sea f (t) una funci´ on de dominio continuo definida de R en C. La transformada de Laplace unilateral derecha de f (t) se define como Z ∞ f (t) e−st dt, LD (s) = 0

para todo s ∈ C tal que |LD (s)| < ∞. La transformada de Laplace unilateral izquierda de f (t) se define como LI (s) =

Z

0

f (t) e−st dt,

−∞

para todo s ∈ C tal que |LI (s)| < ∞. Observaci´ on 7.1. En general, la expresi´ on matem´atica de la transformada de Laplace se calcula utilizando la teor´ıa de integraci´ on compleja. 218

219

CAP´ITULO 7. TRANSFORMADA DE LAPLACE

A lo largo del cap´ıtulo nos referiremos con transformada de Laplace a la transformada de Laplace bilateral, en caso contrario lo indicaremos expl´ıcitamente. En la siguiente proposici´ on se establece que si la transformada de Laplace existe, entonces ella es una funci´ on anal´ıtica en su regi´ on de convergencia, de la cual hablaremos m´ as adelante. Para la demostraci´ on de este resultado, simplemente verificaremos que F (s) satisface las condiciones necesarias y suficientes para ser derivable, esto es, las funciones componentes de F (s) poseen derivadas parciales continuas, que satisfacen las ecuaciones de Cauchy-Riemann. Proposici´ on 7.1. Si F (s) es la transformada de Laplace de la funci´on f (t), entonces F (s) es anal´ıtica en todo n´ umero complejo s tal que |F (s)| < ∞. Demostraci´ on. Tomemos s = x+jy. Entonces, por definici´ on de la transformada de Laplace se tiene que Z ∞ Z ∞ Z ∞ −xt −st f (t) e−xt sen (yt) dt. f (t) e cos(yt) dt − j f (t) e dt = F (s) = −∞

−∞

−∞

Esta ecuaci´ on nos indica que las funciones componentes de F (s) = u(x, y) + jv(x, y), son: Z

u(x, y) =



−xt

f (t) e

cos(yt) dt

y

−∞

v(x, y) = −

Z



f (t) e−xt sen (yt) dt.

−∞

Como u(x, y) y v(x, y) envuelven en su definici´ on la integral de funciones exponenciales y trigonom´ etricas, entonces u(x, y) y v(x, y) poseen primeras derivadas parciales, con respecto a x e y, continuas en todo s tal que |F (s)| < ∞. Demostremos que u(x, y) y v(x, y) satisfacen las ecuaciones de Cauchy-Riemann. Se tiene que Z ∞ Z ∞ ∂u ∂u −xt [tf (t)] e cos(yt) dt, [tf (t)] e−xt sen (yt) dt (x, y) = − (x, y) = − ∂x ∂y −∞ −∞ ∂v (x, y) = ∂x

Z



−xt

[tf (t)] e

sen (yt) dt,

−∞

∂v (x, y) = − ∂y

Z



[tf (t)] e−xt cos(yt) dt

−∞

Es claro que ∂u ∂v (x, y) = (x, y) y ∂x ∂y

∂u ∂v (x, y) = − (x, y), ∂y ∂x

es decir, u(x, y) y v(x, y) satisfacen las ecuaciones de Cauchy-Riemann en todo s tal que |F (s)| < ∞. Todo lo anterior nos indica que F (s) es derivable en el dominio {s ∈ C : |F (s)| < ∞}. Por lo tanto, F (s) anal´ıtica en todo s ∈ C tal que |F (s)| < ∞.

7.1.1 Regi´ on de Convergencia Definici´ on 7.3. La regi´ on de convergencia de la transformada de Laplace de f (t), es el conjunto de n´ umeros complejos s donde F (s) existe, en otras palabras, son todos los s ∈ C tales que Z ∞ −st |F (s)| = f (t) e dt < ∞. (7.1) −∞

La derivada de una integral es igual a la integral de la derivada del integrando, por supuesto, cuando el integrando es continuamente diferenciable

220

7.2. PROPIEDADES DE LA TRANSFORMADA DE LAPLACE

Para hallar la regi´ on de convergencia de la transformada de Laplace se realiza el siguiente procedimiento. Si tomamos s = σ + jω, entonces la ecuaci´ on (7.1) adquiere la forma Z ∞ Z ∞ −(σ+jω) |f (t)| e−σt dt < ∞. (7.2) |F (s)| = f (t) e dt ≤ −∞

−∞

Por lo tanto, los valores de σ = Re s que satisfacen la ecuaci´ on (7.2), determinan expl´ıcitamente la regi´ on de convergencia de F (s). Note que, por la Proposici´ on 7.1, la transformada de Laplace es anal´ıtica en su regi´ on de convergencia.

7.1.2 Transformada Inversa de Laplace La transformada de Laplace nos permite pasar del plano complejo. Ahora para retornar al plano del tiempo se utiliza la Transformada Inversa de Laplace. Definici´ on 7.4. Sea F (s) = L[f (t)], para todo s ∈ D ⊂ C, la transformada de Laplace de f (t). La transformada inversa de Laplace es el proceso de obtener f (t) a trav´ es de F (s) y se define como Z σ+j∞ 1 f (t) = F (s) est ds. (7.3) 2πj σ−j∞ ua en la recta del plano complejo σ + jω, desde La integral de la ecuaci´ on (8.1) se eval´ σ − j∞ hasta σ + j∞, siendo σ un n´ umero real fijo tal que la recta Re s = σ est´ e en el interior de D. En este trabajo, se utilizar´ an dos m´ etodos para calcular la transformada inversa de Laplace: Integraci´on de Contornos e Inversi´on por Tablas, los cuales trataremos m´ as adelante. En el resto del cap´ıtulo usaremos la siguiente notaci´ on. El s´ımbolo L

f (t) ←→ F (s),

s∈D

que se lee par de transformadas, denota que F (s) es la transformada de Laplace de f (t) con regi´ on de convergencia el conjunto D ⊂ C, y que f (t) es la transformada inversa de Laplace de F (s).

7.2 Propiedades de la Transformada de Laplace A continuaci´ on damos las propiedades de la transformada de Laplace. Teorema 7.1 (Linealidad). Si las transformadas de Laplace de las funciones f (t) y g(t) son respectivamente F (s), para todo s ∈ Df ⊂ C y G(s), para todo s ∈ Dg ⊂ C, entonces L

af (t) + bg(t) ←→ aF (s) + bG(s), donde a y b son constantes reales.

s ∈ Df ∩ Dg ,

221

CAP´ITULO 7. TRANSFORMADA DE LAPLACE

Demostraci´ on. Por la definici´ on de la transformada de Laplace se tiene que Z ∞ (af (t) + bg(t)) e−st dt L{af (t) + bg(t)} = −∞ Z ∞ Z ∞ −st g(t) e−st dt f (t) e dt + b = a −∞

−∞

= aL{f (t)} + bL{g(t)} = aF (s) + bG(s).

Ahora, la regi´ on de convergencia de L{af (t) + bg(t)} son todos los s = σ + jω tales que Z ∞ |af (t) + bg(t)| e−σt dt < ∞, −∞

pero Z

∞ −∞

|af (t) + bg(t)| e−σt dt, ≤ |a|

Z

∞ −∞

|f (t)| e−σt dt, +|b|

Z

∞ −∞

|g(t)| e−σt dt.

(7.4)

Como Df y Dg son las regiones de convergencia de f y g, respectivamente, entonces se tiene que Z ∞ |f (t)| e−σt dt < ∞, para todo s = σ + jω ∈ Df (7.5) −∞

y

Z

∞ −∞

|g(t)| e−σt dt,

para todo s = σ + jω ∈ Dg

(7.6)

on de convergencia de L{af (t) + bg(t)} son todos los As´ı, por (7.4), (7.5) y (7.6), la regi´ s = σ + jω tales que s ∈ Df ∩ Dg . Teorema 7.2 (Desplazamiento en tiempo). Si la transformada de Laplace de la funci´on f (t) es F (s) para todo s ∈ D ⊂ C, entonces L

f (t − t0 ) ←→ e−t0 s F (s),

s ∈ D,

para todo t0 ∈ R. Demostraci´ on. Por la definici´ on de la transformada de Laplace se tiene que Z ∞ f (t − t0 ) e−st dt. L{f (t − t0 )} = −∞

Haciendo el cambio de variable r = t − t0 , la integral anterior adquiere la forma: Z ∞ f (r) e−s(r+t0 ) dr L{f (t − t0 )} = −∞ Z ∞ −t0 s = e f (r) e−sr dr −t0 s

= e

−t0 s

= e

−∞

L{f (t)} F (s).

222

7.2. PROPIEDADES DE LA TRANSFORMADA DE LAPLACE

Ahora, la regi´ on de convergencia de L{f (t − t0 )} son todos los s = σ + jω tales que Z ∞ |f (t − t0 )| e−σt dt < ∞. −∞

Haciendo el cambio de variable r = t − t0 , la ecuaci´ on anterior adquiere la forma: Z ∞ Z ∞ −t σ e 0 |f (r)| e−rt dt < ∞; |f (r)| e−rt dt = −∞

−∞

en otras palabras, la regi´ on de convergencia de L{f (t − t0 )} es D.

Teorema 7.3 (Desplazamiento en el dominio de s). Si la transformada de Laplace de la funci´on f (t) es F (s) para todo s ∈ D ⊂ C, entonces L

es0 t f (t) ←→ F (s − s0 ),

(s − s0 ) ∈ D,

para todo s0 ∈ C. Demostraci´ on. Se tiene que Z Z ∞ s0 t −st s0 t e f (t) e dt = L{e f (t)} =



−∞

−∞

f (t) e−(s−s0 )t dt = F (s − s0 ).

Ahora, la regi´ on de convergencia de L{es0 t f (t)} son todos los s = σ + jω tales que Z ∞ st e 0 f (t) e−σt dt < ∞. −∞

Tomando s0 = σ0 + jω0 , la ecuaci´ on anterior adquiere la forma Z ∞ Z ∞ jω t −(σ−σ )t 0 0 |f (t)| e−(σ−σ0 )t dt < ∞, |f (t)| e e dt = −∞

−∞

de lo cual se deduce que la regi´ on de convergencia de L{es0 t f (t)} son todos los s = σ + jω tales que (s − s0 ) ∈ D. Teorema 7.4 (Escalamiento en tiempo). Si la transformada de Laplace de la funci´on f (t) es F (s) para todo s ∈ D ⊂ C, entonces L

f (at) ←→

1 F (s/a), |a|

(s/a) ∈ D,

para todo a ∈ R. Demostraci´ on. Por la definici´ on de la transformada de Laplace, Z ∞ f (at) e−st dt. L{f (at)} = −∞

Haciendo el cambio de variable r = at, la integral de (7.7) adquiere la forma: Z ∞ s 1 f (r) e−( a )r dr. L{f (at)} = |a| −∞

(7.7)

223

CAP´ITULO 7. TRANSFORMADA DE LAPLACE

Cambiando r por t en la integral se tiene: Z ∞ s 1 1 1 F (s/a). f (t) e−( a )t dt = F (s/a) = L{f (at)} = |a| −∞ a |a| Ahora, la regi´ on de convergencia de L{f (at)} son todos los s = σ + jω tales que Z ∞ |f (at)| e−σt dt < ∞. −∞

Haciendo el cambio de variable r = at, la ecuaci´ on anterior adquiere la forma: Z ∞ σ 1 f (t) e−( a )r dt < ∞, |a| −∞ de lo cual se deduce que la regi´ on de convergencia de L{f (at)} son todos los s = σ + jω tales que (s/a) ∈ D. Teorema 7.5 (Conjugaci´ on). Si la transformada de Laplace de la funci´on f (t) es F (s) para todo s ∈ D ⊂ C, entonces L

f (t) ←→ F (s),

s ∈ D.

Demostraci´ on. Como es = es , se tiene que Z ∞ Z o Z ∞ n −st −st f (t) e dt = f (t) e dt = L f (t) = −∞

−∞

∞ −∞

f (t) e−st dt = F (s).

n o Ahora, la regi´ on de convergencia de L f (t) son todos los s = σ + jω tales que Z



−∞

Z −σt dt = f (t) e



−∞

|f (t)| e−σt dt < ∞,

n o en otras palabras, la regi´ on de convergencia de L f (t) son todos los s = σ + jω tales que s ∈ D. Teorema 7.6 (Convoluci´ on). Si las transformadas de Laplace de las funciones f (t) y g(t) son respectivamente F (s), para s ∈ Df ⊂ C y G(s), para s ∈ Dg ⊂ C, entonces L

f ∗ g (t) ←→ F (s) G(s),

s ∈ Df ∩ Dg .

Demostraci´ on. Se tiene que Z



f ∗ g (t) e−st dt −∞  Z ∞ Z ∞ f (τ )g(t − τ ) dτ e−st dt = −∞ −∞  Z ∞ Z ∞ g(t − τ ) e−st dt dτ. f (τ ) =

L{f ∗ g (t)} =

−∞

−∞

224

7.2. PROPIEDADES DE LA TRANSFORMADA DE LAPLACE

Haciendo el cambio de variable r = t − τ , la ecuaci´ on anterior adquiere la forma:  Z ∞ Z ∞ −s(r+τ ) g(r) e dr dτ f (τ ) L{f ∗ g (t)} = −∞ −∞   Z ∞ Z ∞ g(σ) e−sr dr = F (s) G(s). f (τ ) e−sτ dτ = −∞

−∞

Ahora, la regi´ on de convergencia de L{f ∗ g (t)} son todos los s = σ + jω tales que Z ∞ Z ∞ Z ∞ −σt f (τ )g(t − τ ) dτ e−σt dt < ∞. |f ∗ g (t)| e dt = −∞

−∞

−∞

Haciendo el cambio de variable r = t − τ , la ecuaci´ on anterior adquiere la forma:   Z ∞ Z ∞ −σr −στ |g(σ)| e dr < ∞, |f (τ )| e dτ −∞

−∞

en otras palabras, la regi´ on de convergencia de L{f ∗ g (t)} son todos los s = σ + jω tales que s ∈ Df ∩ Dg . Teorema 7.7 (Diferenciaci´ on en el dominio del tiempo). Si la transformada de Laplace de una funci´on f (t) derivable es F (s) para todo s ∈ D ⊂ C, entonces d L f (t) ←→ s F (s), dt cuya regi´on de convergencia contiene a D. Demostraci´ on. Asumamos que f (t) es de orden exponencial, esto es, existen constantes K > 0 y a ∈ R tales que |f (t)| ≤ K eat ,

para todo t ∈ R.

Adem´ as, si f (t) es de orden exponencial, entonces (se deja como ejercicio para el lector la prueba de esta ecuaci´ on):   α l´ım f (t)e−st −α = 0, para todo s ∈ D. (7.8) α→∞

Se tiene

L



 Z ∞ d d f (t) = f (t) e−st dt. dt −∞ dt

As´ı, usando la ecuaci´ on (7.8) y la integraci´ on por partes con u = e−st y dv = f ′ (t), se obtiene   Z ∞   d −st α L f (t) e−st dt = sF (s). f (t) = l´ım f (t)e +s −α α→∞ dt −∞

Adem´ as, de esta u ´ltima on se deduce que si s ∈ D, entonces s pertenece a la regi´ on  ecuaci´ d f (t) . de convergencia de L dt d Observaci´ on 7.2. Generalmente, la regi´ on de convergencia de L dt f (t) coincide con la regi´ on de convergencia on de  d de L {f (t)}, pero existen funciones f (t) tales que la regi´ f (t) contiene a la regi´ on de convergencia de L {f (t)}. convergencia de L dt

225

CAP´ITULO 7. TRANSFORMADA DE LAPLACE

Teorema 7.8 (Diferenciaci´ on en el dominio de s). Si la transformada de Laplace de la funci´on f (t) es F (s) para todo s ∈ D ⊂ C, entonces L

t f (t) ←→ − Demostraci´ on. Se tiene que F (s) =

Z

d F (s), ds



s ∈ D.

f (t) e−st dt.

−∞

Derivando con respecto a s en ambos lados de la ecuaci´ on anterior, se obtiene:  Z ∞ d d F (s) = f (t) e−st dt ds ds Z ∞ −∞  d  f (t) e−st dt = ds Z−∞ ∞ [−tf (t)] e−st dt = −L {t f (t)} , = −∞

d as, como F (s) es anal´ıtica en todo de donde se deduce que L {t f (t)} = − F (s). Adem´ ds d s ∈ D, entonces F (s) tambi´ en es anal´ıtica en todo s ∈ D, en otras palabras, D es la ds d regi´ on de convergencia de F (s). ds Teorema 7.9 (Integraci´ on). Si la transformada de Laplace de la funci´on f (t) es F (s) para todo s ∈ D ⊂ C, entonces Z t 1 L f (τ ) dτ ←→ F (s), s −∞ cuya regi´on de convergencia es igual al conjunto D o est´ a contenida en ´el. Demostraci´ on. Para la prueba usaremos la transformada de Laplace del escal´ on unitario, a saber: 1 L{u(t)} = , Re s > 0, s la cual la hallaremos m´ as adelante. Recordemos que Z t Z ∞ f (τ ) dτ, (7.9) f (t)u(t − τ )dτ = f (t) ∗ u(t) = −∞

−∞

para toda funci´ on f (t) de dominio continuo. De esta forma, usando la propiedad de convoluci´ on en la ecuaci´ on (7.9) L{f (t) ∗ u(t)} = L{f (t)} L{u(t)},

s ∈ {s : Re s > 0} ∩ D.

Ahora, usando la expresi´ on de la transformada de Laplace del escal´ on unitario, se obtiene    Z t 1 1 = F (s), s ∈ {s : Re s > 0} ∩ D. f (τ ) dτ = F (s) L s s −∞

226

7.2. PROPIEDADES DE LA TRANSFORMADA DE LAPLACE

Teorema 7.10 (Teorema del Valor Inicial). Sea f (t) una funci´on continua a trozos en R tal que f (t) = 0 para t < 0. Si la transformada de Laplace de la funci´on f (t) es F (s) para todo s ∈ D ⊂ C, entonces f (0+ ) = l´ım s F (s). s→∞

Demostraci´ on. Como f (t) = 0 para t < 0, entonces   Z ∞ d d + f (t) = s F (s) − f (0 ) = f (t) e−st dt. L dt dt 0

(7.10)

Tomando l´ımite en ambos lados de (7.10), cuando s → ∞, se obtiene " #  Z ∞ Z α d d −st −st + f (t) e dt = l´ım l´ım f (t) e dt . l´ım [s F (s) − f (0 )] = l´ım s→∞ α→∞ ε dt s→∞ s→∞ dt 0 ε→0 Intercambiando el orden de los l´ımites, se tiene Z + l´ım [s F (s) − f (0 )] = l´ım s→∞

α

α→∞ ε ε→0

h i d f (t) l´ım e−st dt s→∞ dt

y como l´ım e−st = 0,

s→∞

la expresi´ on de arriba se reduce a l´ım [s F (s) − f (0+ )] = 0

s→∞

o, equivalentemente, f (0+ ) = l´ım s F (s). s→∞

Teorema 7.11 (Teorema del Valor Final). Sea f (t) una funci´on continua a trozos en R tal que f (t) = 0 para t < 0. Si la transformada de Laplace de la funci´on f (t) es F (s) para todo s ∈ D ⊂ C, entonces l´ım f (t) = l´ım s F (s). t→∞

s→0

Demostraci´ on. Tomando l´ımite en ambos lados de (7.10), cuando s → 0, se obtiene " #  Z ∞ Z α d d f (t) e−st dt = l´ım l´ım f (t) e−st dt . l´ım [s F (s) − f (0+ )] = l´ım s→0 α→∞ ε dt s→0 s→0 dt 0 ε→0 Intercambiando el orden de los l´ımites, se tiene Z + l´ım [s F (s) − f (0 )] = l´ım s→0

α→∞ ε ε→0

α

i h d f (t) l´ım e−st dt s→0 dt

y como l´ım e−st = 1,

s→0

227

CAP´ITULO 7. TRANSFORMADA DE LAPLACE

la expresi´ on de arriba se reduce a Z α Z α d f (t) dt = l´ım f (t) dt = f (∞) − f (0− ) = f (∞) l´ım [s F (s) − f (0+ )] = l´ım α→∞ ε α→∞ ε dt s→0 ε→0 ε→0 o, equivalentemente, l´ım f (t) = l´ım s F (s). s→∞

t→∞

Las propiedades de la transformada de Laplace se resumen en la Tabla 7.1.

Tabla 7.1. Propiedades de la Transformada de Laplace Sean f (t) y g(t) dos funciones de dominio continuo con transformada de Laplace F (s), para s ∈ Df , y G(s), para s ∈ Dg , respectivamente. Propiedad

Descripci´ on Matem´ atica

Linealidad

L {a1 f (t) + a2 g(t)} = a1 F (s) + a2 G(s),

Desplazamiento en tiempo

L {f (t − t0 )} = e−st0 F (s),

Desplazamiento en s

 L es0 t f (t) = F (s − s0 ),

Escalamiento en tiempo

L {f (at)} =

Conjugaci´ on

n o L f (t) = F (s),

Convoluci´ on

L {f ∗ g (t)} = F (s)G(s), 

1 F (s/a), |a|

Diferenciaci´ on en tiempo

L

Diferenciaci´ on en s

L {t f (t)} = −

Integraci´ on

Teorema del valor final

{s ∈ C : (s − s0 ) ∈ Df } s ∈ {s ∈ C : (s/a) ∈ Df }

s ∈ Df ∩ Dg

s ∈ Df

d F (s), s ∈ Df ds   t  1 Z f (λ) dλ = F (s), s ∈ D ⊇ Df L  s  −∞

Teorema del valor inicial

s ∈ Df

s ∈ Df

 d f (t) = s F (s), dt

l´ım sF (s) = f (0+ )

s→∞

l´ım f (t) = l´ım sF (s)

t→∞

s→0

s ∈ Df ∩ Dg

228

7.3. ALGUNOS PARES DE TRANSFORMADAS

7.3 Algunos Pares de Transformadas En esta secci´ on se calcula la transformada de Laplace de funciones comunes. 1.

L

δ(t) ←→ 1,

s∈C

Se tiene que Z



Z



δ(t) e−σt e−jωt dt −∞ Z−∞ Z ∞ ∞ −σt = δ(t) e cos(ωt) dt − j δ(t) e−σt sen (ωt) dt

L{δ(t)} =

δ(t) e−st dt =

−∞ −σ·0

= e

−∞

−σ·0

cos(ω · 0) − je

sen (ω · 0) = 1.

La regi´ on de convergencia de L{δ(t)} son todos los s = σ + jω tales que Z ∞ δ(t)(t) e−σt dt < ∞, −∞

pero

Z



δ(t)(t) e−σt dt = 1;

−∞

por lo tanto, la regi´ on de convergencia de L{δ(t)} es todo C. 2.

1 , s

L

u(t) ←→

Re s > 0

Se tiene que L{u(t)} =

Z



−st

u(t) e

dt =

−∞

Z



−st

e

0+

Ahora, como Z

∞ −∞

−σt

|u(t)| e

Z

dt =

∞ 0+

 ∞ e−st 1 dt = − = . s s 0+ 

e−σt dt < ∞



σ > 0,

entonces la regi´ on de convergencia de L{u(t)} son todos los s = σ + jω ∈ C tales que Re s = σ > 0. 3. L

−u(−t) ←→

1 , s

Re s < 0

Hallemos la transformada de Laplace L{−u(−t)} mediante dos procedimientos: (a) por definici´ on, y (b) usando las propiedades de la transformada de Laplace. (a) Se tiene que L{−u(−t)} =

Z

∞ −∞

−st

[−u(−t)] e

dt = −

Z

0−



−st

e

dt =



 0− 1 e−st = . s ∞ s

229

CAP´ITULO 7. TRANSFORMADA DE LAPLACE

Como

Z



−σt

−∞

| − u(−t)| e

dt =

Z

0−



e−σt dt < ∞



σ < 0,

entonces la regi´ on de convergencia de L{−u(−t)} son todos los s = σ + jω ∈ C tales que Re s = σ < 0. (b) Usando las propiedades de linealidad y escalamiento en tiempo, se obtiene   1 1 L{u(t)}(−s) = , Re (−s) > 0, L{−u(−t)} = −L{u(−t)} = − − | − 1| s o, equivalentemente,

1 L{−u(−t)} = , Re s < 0. s Observaci´ on 7.3. Note que las transformadas de Laplace de u(t) y −u(−t) son algebraicamente iguales, pero tienen regiones de convergencia diferentes, m´ as a´ un, son complementarias.

4.

1 , Re s > −Re α s+α Usando la propiedad de desplazamiento en el dominio de s, se obtiene L

e−αt u(t) ←→

L{e−αt u(t)} = L{u(t)}(s − (−α)) =

1 , s+α

Re (s + α) > 0,

o, equivalentemente, 1 , s+α

L{e−αt u(t)} =

Re s > −α.

La transformada de Laplace de e−αt u(t) tambi´ en se puede hallar por definici´ on, lo cual se deja como ejercicio para el lector. 5.

1 , s2

L

t u(t) ←→ Se tiene que L{t u(t)} =

Z



Re s > 0

−st

[t u(t)] e

dt =

Z



te−st dt.

0+

−∞

Usando integraci´ on por partes, se obtiene  ∞  Z 1 ∞ −st te−st e dt + L{t u(t)} = − s 0+ s 0+  −st  ∞ 1 e = 1. = 0+ − s s 0+ s2 Como

Z



−∞

−σt

|t u(t)| e

dt =

Z



t e−σt dt

0+

=

1 σ

Z



0+

e−σt dt < ∞



σ > 0,

230

7.3. ALGUNOS PARES DE TRANSFORMADAS

entonces la regi´ on de convergencia de L{t u(t)} son todos los s = σ + jω ∈ C tales que Re s = σ > 0. La transformada de Laplace de t u(t) tambi´ en se puede hallar usando la propiedad de diferenciaci´ on en s, lo cual se deja como ejercicio para el lector. 6. L

sen t u(t) ←→

1 , +1

s2

Re s > 0

Se tiene que L{sen t u(t)} = = = = = Como

Z



−∞

Z



[sen t u(t)] e−st dt −∞  Z ∞  jt e − e−jt e−st dt 2j + 0  Z ∞ Z ∞ 1 −(j+s)t (j−s)t e dt e dt − 2j 0+ + "" # ∞ # ∞ # "0 1 e−(j+s)t e(j−s)t − − 2j j−s + j+s + 0    0 1 2j 1 1 1 1 − . = = 2 2j s − j s+j 2j s2 + 1 s +1 −σt

|sen t u(t)| e

dt ≤

Z

∞ 0+

e−σt dt < ∞



σ > 0,

entonces la regi´ on de convergencia de L{sen t u(t)} son todos los s = σ + jω ∈ C tales que Re s = σ > 0. 7.

L

cos t u(t) ←→

s , s2 + 1

Re s > 0

Usando las propiedades de linealidad y desplazamiento en el dominio de s, se obtiene  jt   e + e−jt L{cos t u(t)} = L u(t) 2  1 L{ejt u(t)} + L{e−jt u(t)} = 2 1 = [L{u(t)}(s + j) + L{u(t)}(s − j)] 2  1 1 1 = + 2 s+j s−j s , {s ∈ C : Re (s + j) > 0} ∩ {s ∈ C : Re (s − j) > 0} = 2 s +1 o, equivalentemente, L{cos t u(t)} =

s2

s , +1

Re s > 0.

231

CAP´ITULO 7. TRANSFORMADA DE LAPLACE

8. L

pT (t) ←→

2jT senh(T s) , Ts

Re s 6= 0

Se tiene que L{pT (t)} = = = = =

Z

Z



pT (t) e−st dt

−∞ T−

e−st dt

−T +



 T − e−st − s −T +

esT − e−sT s 2jT sen (sT ) , sT

s 6= 0.

Como Z



−∞

|pT (t)| e−σt dt =

Z

T− −T +

e−σt dt =

eσT − e−σT <∞ σ



σ 6= 0,

entonces la regi´ on de convergencia de la transformada de Laplace L{pT (t)} son todos los s ∈ C tales que Re s 6= 0. En la Tabla 7.2 se muestra un resumen de los pares de transformadas de Laplace de funciones comunes.

232

7.3. ALGUNOS PARES DE TRANSFORMADAS

Tabla 7.2. Algunos Pares de Transformadas de Laplace Funci´ on

Transformada de Laplace

δ(t)

1,

δ(t − a)

e−as ,

u(t) −u(−t) u(t) − u(t − a) tn u(t) −tn u(−t) e−at u(t) −e−at u(−t) tn e−at u(t) −tn e−at u(−t) cos(ω0 t)u(t) sen (ω0 t)u(t) cos2 (ω0 t)u(t) sen 2 (ω0 t)u(t) e−at cos(ω0 t)u(t) e−at sen (ω0 t)u(t) t cos(ω0 t)u(t) tsen (ω0 t)u(t) pT (t)

s∈C s∈C

1 , Re s > 0 s 1 , Re s < 0 s 1 − e−as , Re s > 0 s n! , n = 1, 2, . . . , Re s > 0 n+1 s n! , n = 1, 2, . . . , Re s < 0 n+1 s 1 , Re s > −a s+a 1 , Re s < −a s+a n! , Re s > −a (s + a)n+1 n! , Re s < −a (s + a)n+1 s , Re s > 0 2 s + ω02 ω0 , Re s > 0 2 s + ω02 s2 + 2ω02 , Re s > 0 s(s2 + 4ω02 ) 2ω02 , Re s > 0 s(s2 + 4ω02 ) s+a , Re s > −a (s + a)2 + ω02 ω0 , Re s > −a (s + a)2 + ω02 s2 − ω02 , Re s > 0 (s2 + ω02 )2 2ω0 s , Re s > 0 2 (s + ω02 )2 2jT senh(T s) , Re s 6= 0 Ts

233

CAP´ITULO 7. TRANSFORMADA DE LAPLACE

7.4 C´ alculo de la Transformada Inversa de Laplace Se utilizar´ an dos m´ etodos para calcular la transformada inversa de Laplace: Integraci´on de Contornos e Inversi´ on por Tablas.

7.4.1 Integraci´ on de Contornos El m´ etodo de Integraci´ on de Contornos utiliza el Teorema de los Residuos para calcular la transformada inversa de Laplace. Expliquemos el fundamento de este m´ etodo mediante un ejemplo. La siguiente proposici´ on se utiliza en el ejemplo. Proposici´ on 7.2. Sea F (s) la transformada de Laplace de f (t), con regi´on de convergencia dada por Re s > −a, donde a > 0. Si |F (s)| < b|s|−m para b > 0, un entero m > 0, y R0 > 0 tal que |s| > R0 , entonces f (t) = 0, para t < 0. Demostraci´ on. Se deja como ejercicio para el lector. Ayuda: utilice un contorno cerrado simple C apropiado y el hecho que |F (s)| < 1/|s|. Ejemplo 7.1. Determine la transformada inversa de Laplace de F (s) =

1 , s+2

Re s > −2.

Soluci´ on. Consideremos el contorno cerrado simple C formado por el contorno C1 , el arco de la circunferencia z(t) = R ejt , π/2 ≤ t ≤ 3π/2, y el contorno C2 dado por el segmento de recta −R ≤ t ≤ R,

z(t) = j t,

con R > 0. En la Figura 7.1 se aprecian los contornos C1 y C2 . ω

Re s > −2

C1 C2 −2

σ

Figura 7.1. Contornos C1 y C2

234

´ 7.4. CALCULO DE LA TRANSFORMADA INVERSA DE LAPLACE

Se tiene que 1 2πj

Z

st

e F (s) ds =

C

=

Z 1 e F (s) ds + est F (s) ds 2πj C2 C1 Z Z jR st 1 e 1 est ds + ds. 2πj C1 s + 2 2πj −jR s + 2 1 2πj

Z

st

(7.11)

Se deja como ejercicio para el lector verificar que l´ım

R→∞

Z

C1

est ds = 0, s+2

para t ≥ 0.

Luego, tomando l´ımite en el lado derecho de (7.11), cuando R → ∞, se obtiene 1 2πj

Z

j∞ −j∞

est 1 ds = s+2 2πj

Z

C

 st  est e ds = Res = e−2t , s=−2 s + 2 s+2

de lo cual se deduce que f (t) = e−2t ,

para t ≥ 0.

Ahora, como |F (s)| = 1/|s + 2| < |s|−1 para |s| > 2, entonces por la Proposici´ on 7.2, f (t) = 0 para t < 0. Por lo tanto, todo lo anterior nos indica que la transformada inversa de Laplace de F (s) es f (t) = e−2t u(t), cuya gr´afica se muestra en la siguiente figura. e−2t u(t) 1

t 

El ejemplo anterior es un caso particular, pero el procedimiento de resoluci´ on empleado en el c´ alculo de la transformada inversa de Laplace, puede usarse en casos m´ as generales. M´as a´ un, este mismo procedimiento constituye la base para la demostraci´ on de la proposici´ on que damos a continuaci´ on. Tal proposici´ on plantea un procedimiento para hallar la transformada inversa de Laplace, cuando F (s) es una funci´ on racional propia o, a lo sumo, una funci´ on racional multiplicada por una funci´ on exponencial.

235

CAP´ITULO 7. TRANSFORMADA DE LAPLACE

Proposici´ on 7.3. Sea F (s) la transformada de Laplace de f (t) con regi´on de convergencia dada por r1 < Re s < r2 , donde r1 , r2 ∈ R. Sean pIk los polos ubicados a la izquierda de la regi´on de convergencia, para k = 1, . . . , nI , y sean pD k los polos ubicados a la derecha de la regi´on de convergencia, para k = 1, . . . , nD (ver, por ejemplo, la Figura 7.2). Entonces, la transformada inversa de Laplace est´ a dada por f (t) =

nI X k=1

n

D X    Res e F (s) u(t) − Res est F (s) u(−t).

s=pIk



st

k=1

s=pD k

ω b

pI1

r1 < Re s < r2

b

b

r1

pI3

r2

σ

b

pI2 b

pD 1

pD 2

Figura 7.2. Ubicaci´ on de los polos con respecto a la regi´ on de convergencia Ejemplo 7.2. Determinar la transformada inversa de Laplace de F (s) =

5s − 1 , s3 − 3s − 2

−1 < Re s < 2.

Soluci´ on. Los polos de F (s) son p1 = −1 y p2 = 2, adem´as, p1 est´ a ubicado a la izquierda de la regi´ on de convergencia y p2 est´ a ubicada a la derecha. As´ı, la transformada inversa de Laplace de F (s) est´ a dada por:     f (t) = Res est F (s) u(t) − Res est F (s) u(−t). (7.12) s=−1

s=2

Ahora,

y

st



 est (5s − 1) Res s=−1 s3 − 3s − 2   d est (5s − 1) = (2t − 1) e−t = dz (s − 2) s=−1

 Res e F (s) =

s=−1



 st    e (5s − 1) Res est F (s) = Res 3 s=2 s − 3s − 2 s=2   st e (5s − 1) = e2t . = (s + 1)2 s=2

(7.13)

(7.14)

236

´ 7.4. CALCULO DE LA TRANSFORMADA INVERSA DE LAPLACE

Sustituyendo convenientemente (7.13) y (7.14) en (7.12), se tiene que la transformada inversa de Laplace de F (s) es f (t) = (2t − 1) e−t u(t) − e2t u(−t). 

7.4.2 Inversi´ on por Tablas El m´ etodo de inversi´on por tablas consiste en expresar a F (s) como la suma F (s) = F1 (s) + F2 (s) + · · · + FR (s),

s ∈ D,

(7.15)

donde F1 (s), F2 (s), . . . , FR (s) son funciones tales que se les conoce su transformada inversa de Laplace f1 (t), f2 (t), . . . , fR (t). Entonces, la transformada inversa de Laplace de F (s) est´ a dada por f (t) = f1 (t) + f2 (t) + · · · + fR (t). Con mucha frecuencia en aplicaciones importantes de Ingenier´ıa se consigue con transformadas de Laplace F (s) racionales. A este tipo de transformadas se le prestar´ a una mayor atenci´ on. Particularmente, es de mucho inter´ es calcular la transformada inversa de Laplace de b0 + b1 s + · · · + bm sm , F (s) = a0 + a1 s + · · · + an sn donde an 6= 0 y m < n; es decir, cuando F (s) es una funci´ on racional propia. Cuando F (s) es racional propia, la expansi´ on (7.15) se denomina expansi´on en fracciones parciales, que etodo inversi´ on por tablas. se estudi´ o en el Cap´ıtulo 4. Expliquemos con un ejemplo el m´ Ejemplo 7.3. Determine la transformada inversa de Laplace de F (s) =

144s2 + 144s + 144 , (s − 3)2 (s − 2)2 (s + 1)

2 < Re s < 3.

Soluci´ on. La expansi´ on en fracciones parciales de F (s) es de la forma F (s) =

A2,1 A2,2 A3,2 A3,1 A1 + + + + , 2 s + 1 s − 2 (s − 2) s − 3 (s − 3)2

donde A1 = A2,2 = A2,1 = A3,2 = A3,1 =

Res [F (s)] = 1,   (s − 2)2 F (s) s=2 = 336,   d [(s − 2)2 F (s)] = 800, ds s=2   (s − 3)2 F (s) s=3 = 468,   d 2 [(s − 3) F (s)] = −801. ds s=3 s=−1

237

CAP´ITULO 7. TRANSFORMADA DE LAPLACE

As´ı, f (t) = A1 L

−1



1 s+1 

+A3,1 L−1



−1



1 s−2 

+ A2,1 L  1 + A3,2 L−1 s−3



+ A2,2 L  1 . (s − 3)2

−1



1 (s − 2)2



Ahora, considerando la regi´ on de convergencia, 2 < Re s < 3, se halla la transformada inversa de Laplace de cada uno de los sumandos de la expansi´ on en fracciones parciales de F (s):     1 1 = e−t u(t), L−1 = e2t u(t), L−1 s+1 s−2     1 1 −1 2t −1 L = te u(t), L = −e3t u(−t), (s − 2)2 s−3   1 −1 L = −te3t u(−t). (s − 3)2 Por lo tanto, la transformada inversa de Laplace de F (s) es:   f (t) = e−t + 800 e2t + 336 te2t u(t) + 801 e3t − 468 te3t u(−t).



7.5 Transformada de Laplace con M ATLAB La herramienta de matem´ atica simb´ olica de M ATLAB cuenta con el comando laplace que calcula la transformada unilateral de Laplace de una funci´ on f (t). Observe el siguiente ejemplo donde se calcula la transformada unilateral de Laplace de las siguientes funciones: f1 (t) = e−2t u(t),

f2 (t) = (2t − 1) e−t u(t) − e−2t u(−t).

✄ >> syms t >> f 1 ( t ) = exp(−2∗ t )∗ h e a v i s i d e ( t ) ; >> F1 = l a p l a c e ( f 1 ) F1 = 1/( s + 2) >> f 2 ( t ) = (2∗ t −1)∗exp(− t )∗ h e a v i s i d e ( t )−exp (2∗ t )∗ h e a v i s i d e (− t ) ; >> F2 = l a p l a c e ( f 2 ) F2 = 2/( s + 1)ˆ2 − 1/( s + 1)





Las transformadas unilateral de Laplace obtenidas son: F1 (s) =

1 , s+2

F2 (s) =

2 1 2 − s + 1. (s + 1)

Note que el comando laplace suministra la expresi´ on matem´atica de la transformada unilateral de Laplace, pero no nos dice nada acerca de su regi´ on de convergencia; adem´as, para utilizar laplace se debe asumir que f (t) es una funci´ on causal. Cuando f (t) no es causal, entonces laplace halla la transformada unilateral de Laplace de la parte causal de

238

7.5. TRANSFORMADA DE LAPLACE CON MATLAB

f (t). En el Programa 7.1 se muestra la funci´ on TransLaplace, la cual calcula la transformada bilateral de Laplace de una funci´ on f (t) cualquiera. La funci´ on TransLaplace se basa en el hecho que toda funci´ on f (t) se puede expresar como: f (t) = fn (t) + fc (t), donde fn (t) es una funci´ on no causal y fc (t) es una funci´ on causal. Recuerde que una funci´ on g(t) se dice causal si g(t) = 0 para t < 0, y ella es no causal, si g(t) = 0 para t > 0. Programa 7.1. Funci´ on TransLaplace.m f u n c t i o n [ F ] = T r a n s L a p l a c e ( fn , f c ) %T r a n s L a p l a c e H a l l a l a t r a n s f o r m a d a b i l a t e r a l de L a p l a c e % de una f u n c i o ´ n f ( t ) = f n ( t )+ f c ( t ) syms t s ; Fc ( s ) = l a p l a c e ( f c ) ; Fn ( s ) = l a p l a c e ( f n (− t ) ) ; F ( s ) = Fn(− s ) + Fc ( s ) ; end

Calculemos la transformada bilateral de Laplace de las funciones f1 (t) y f2 (t) dadas arriba. Observe que f1 (t) es una funci´ on causal y f2 (t) = f2n (t) + f2c (t), donde f2n (t) = −e−2t u(−t) y

f2c (t) = (2t − 1) e−t u(t).

Seguidamente se emplea la funci´ on TransLaplace para hallar las transformada bilateral de Laplace de f1 (t) y la de f2 (t). ✄ >> syms t >> f 1 c ( t )=exp(−2∗ t )∗ h e a v i s i d e ( t ) ; f1n ( t ) = sym ( 0 ) ; >> F1 = T r a n s L a p l a c e ( f1n , f 1 c ) F1 ( s ) = 1/( s + 2) >> f 2 c ( t )=(2∗ t −1)∗exp(− t )∗ h e a v i s i d e ( t ) ; >> f2n ( t)=−exp (2∗ t )∗ h e a v i s i d e (− t ) ; >> F2 = T r a n s L a p l a c e ( f2n , f 2 c ) F2 ( s ) = 2/( s + 1)ˆ2 − 1/( s + 1) + 1/( s − 2)





Note que la expresi´ on f1n(t) = sym(0) indica que f1n (t) es la funci´ on 0. De esta forma, las expresiones matem´ aticas de las transformadas bilaterales de Laplace obtenidas son: F1 (s) =

1 , s+2

F2 (s) =

2 1 1 2 − s + 1 + s − 2. (s + 1)

Por otra parte, M ATLAB tambi´ en cuenta con el comando ilaplace que halla la transformada inversa de Laplace. Observe el siguiente ejemplo donde se halla la transformada inversa de Laplace de las transformadas F1 (s) y F2 (s) dadas arriba. ✄ >> syms s >> F1 ( s ) = f1 ( t ) = exp(−2∗ t ) >> F2 ( s ) = >> f 2 ( t ) = f2 ( t ) = exp (2∗ t ) −



1/( s + 2 ) ; f 1 ( t ) = i l a p l a c e ( F1 )

2/( s + 1)ˆ2 − 1/( s + 1) + 1/( s − 2 ) ; i l a p l a c e ( F2 ) exp(− t ) + 2∗ t ∗ exp(− t )



239

CAP´ITULO 7. TRANSFORMADA DE LAPLACE

Las transformadas inversas de Laplace obtenidas son: f1 (t) = e−2t ,

f2 (t) = e2t + (2 t − 1) e−t .

Note que ilaplace halla la transformada inversa de Laplace de F (s), asumiendo que F (s) es la transformada unilateral de Laplace de f (t).

7.6 Problemas Resueltos Problema 7.1. Determinar la transformada de Laplace del pulso rectangular p1 (t). Soluci´ on. Por definici´ on, la transformada de Laplace de p1 (t) est´ a dada por: Z 1− Z ∞ 2 senh s es − e−s = . p1 (t)e−st dt = e−st dt = F (s) = s s + −∞ −1 Ahora bien, la regi´ on de convergencia de F (s) son todos los n´ umeros complejos s = σ + jω tales que Z 1− Z ∞ −σt |f (t)| e dt = e−σt dt < ∞. −1+

−∞

Es claro que la desigualdad anterior se satisface para todo σ ∈ R. En consecuencia, la transformada de Laplace de p1 (t) es: F (s) = Recuerde que la funci´ on anal´ıtica.

2 senh s , s

para s ∈ C.

2 senh s se puede redefinir en s = 0 de manera que all´ı sea s 

Problema 7.2. Determinar la transformada de Laplace del pulso triangular q1 (t). Soluci´ on. Por definici´ on, la transformada de Laplace de q1 (t) est´ a dada por: Z 1 Z 0 Z ∞ (1 − t)e−st dt (t + 1)e−st dt + q1 (t)e−st dt = F (s) = −∞

s−

0

−1 e−s

es

2

+1 s+ −1 4 senh (s/2) + = . s2 s2 s2 Ahora bien, la regi´ on de convergencia de F (s) son todos los n´ umeros complejos s = σ + jω tales que Z ∞ Z 0 Z 1 |f (t)| e−σt dt = (t + 1)e−σt dt + (1 − t)e−σt dt = −

−∞

−1 1



Z

−1

0

e−σt dt < ∞.

Es claro que la desigualdad anterior se satisface para todo σ ∈ R. En consecuencia, la transformada de Laplace de q1 (t) es: F (s) =

4 senh2 (s/2) , s2

para s ∈ C. 

240

7.6. PROBLEMAS RESUELTOS

Problema 7.3. Determinar la transformada de Laplace de f (t) = e−3t (u(t) − u(t − 4)). Soluci´ on. La funci´ on f (t) se puede expresar como f (t) = f1 (t) − f2 (t − 4), donde f1 (t) = e−3t u(t) y

f2 (t) = e−3(t+4) u(t).

As´ı, aplicando la propiedad de linealidad tenemos: F (s) = L {f (t)} = L {f1 (t)} − L {f2 (t − 4)} ,

para s ∈ D1 ∩ D2 ,

(7.16)

donde D1 , D2 ⊂ C son las regiones de convergencia de L {f1 (t)} y L {f2 (t − 4)}, respectivamente. Calculemos las transformadas L {f1 (t)} y L {f2 (t − 4)}. • Aplicando la propiedad de desplazamiento en s, tenemos que la transformada de Laplace de f1 (t) est´ a dada por:  L {f1 (t)} = L e−3t u(t) =

1 , s+3

Re s > −3.

• Aplicando convenientemente las propiedades de desplazamiento en tiempo y desplazamiento en s, tenemos que la transformada de Laplace de f2 (t − t) est´ a dada por: n o  L {f2 (t − 4)} = e−4s L e−3(t+4) u(t) = e−4s e−12 L e−3t u(t) =

e−4(s+3) , s+3

Re s > −3.

De esta forma, sustituyendo convenientemente las expresiones de L {f1 (t)} y L {f2 (t − 4)} en (7.16), se tiene que la transformada de Laplace de f (t) es: F (s) =

1 e−4(s+3) 1 − e−4(s+3) − = , s+3 s+3 s+3

Re s > −3.

Seguidamente se muestra el diagrama de polos (rojo) y ceros (azul) de F (s). ω

.. . b

Re s > −3

b

b

b

σ

−3 b

b

.. . b

Diagrama de polos y ceros



241

CAP´ITULO 7. TRANSFORMADA DE LAPLACE

Problema 7.4. Determinar la transformada de Laplace de f (t) = e2t sen (4t)u(t). Soluci´ on. Se tiene que 2t

f (t) = e =



e4tj − e−4tj 2j



u(t)

1 1 (4j+2)t e u(t) − e−(4j−2)t u(t). 2j 2j

As´ı, aplicando convenientemente las propiedades de linealidad y desplazamiento en s, obtenemos: F (s) = L



 1 (4j+2)t 1 −(4j−2)t e u(t) − e u(t) 2j 2j

=

o o 1 n 1 n (4j+2)t L e u(t) − L e−(4j−2)t u(t) 2j 2j

=

1 1 L {u(t)} (s − (4j + 2)) − L {u(t)} (s + (4j − 2)) 2j 2j

=

1 1 − . 2j(s − (4j + 2)) 2j(s + (4j − 2)) {z } | {z } | Re s>2

Re s>2

Por lo tanto, la transformada de Laplace de f (t) es: F (s) =

4 , (s − (4j + 2))(s + (4j − 2))

Re s > 2.

Seguidamente se muestra el diagrama de polos y ceros de F (s).

ω

Re s > 2 2 + 4j b

σ

2

2 − 4j b

Diagrama de polos y ceros



242

7.6. PROBLEMAS RESUELTOS

Problema 7.5. Determinar la transformada de Laplace de la funci´ on f (t) cuya gr´afica se muestra en la siguiente figura. f (t) 1

−1

1

t

−1

Soluci´ on. Se tiene que la funci´ on f (t) se puede expresar equivalentemente como: f (t) = (u(t + 1) − u(t)) − δ(t + 1) − (u(t) − u(t − 1)) + δ(t − 1) = u(t + 1) − 2u(t) + u(t − 1) − δ(t + 1) + δ(t − 1).

Luego, aplicando la propiedad de linealidad obtenemos: F (s) = L {f (t)} = L {u(t + 1) − 2u(t) + u(t − 1) − δ(t + 1) + δ(t − 1)} = L {u(t + 1)} − 2L {u(t)} + L {u(t − 1)} − L {δ(t + 1)} + L {δ(t − 1)} . Ahora, aplicando la propiedad de desplazamiento en tiempo tenemos: F (s) = es L {u(t)} − 2L {u(t)} + e−s L {u(t)} − es L {δ(t)} + e−s L {δ(t)} es 2 e−s = − + − |{z} es + |{z} e−s . s s s |{z} |{z} |{z} C

Re s>0

Re s>0

C

Re s>0

Por lo tanto, la transformada de Laplace de f (t) es: F (s) = =

es + e−s − 2 − (es − e−s ) s 2 cosh s − 2 − 2 senh s, Re s > 0. s 

Problema 7.6. Sea f (t) una funci´ on de dominio continuo tal que f (t) = 0 para t < 0, cuya transformada de Laplace es: F (s) =

s e−2s , (s + 1)(s − 2)

Re s > 2.

Determinar la transformada de Laplace de la funci´ on   d 1 2 + t f (t + 1). g(t) = t f t − 2 dt

243

CAP´ITULO 7. TRANSFORMADA DE LAPLACE

Soluci´ on. Aplicando convenientemente las propiedades de linealidad y diferenciaci´ on en s, podemos escribir:     d 1 2 + t f (t + 1) G(s) = L {g(t)} = L t f t − 2 dt      1 d 2 = L t f t− + L t f (t + 1) 2 dt =

     d 1 d d2 L f t− f (t + 1) . + L ds2 2 ds dt

(7.17)

  d Ahora, calculemos las transformadas L f t − 21 y L dt f (t + 1) . Aplicando la propiedad de desplazamiento en tiempo y considerando la expresi´ on de F (s) dada en el enunciado del problema, tenemos:    s e−5s/2 1 , = e−s/2 F (s) = L f t− 2 (s + 1)(s − 2)

Re s > 2

(7.18)

Aplicando convenientemente las propiedades de diferenciaci´ on en tiempo y desplazamiento en tiempo, y considerando la expresi´ on de F (s) dada en el enunciado del problema, obtenemos:   d f (t + 1) = s L {f (t + 1)} = s es F (s) L dt s2 e−s , Re s > 2 (7.19) = (s + 1)(s − 2) De esta forma, usando (7.17), (7.18) y (7.19), la transformada de Laplace de g(t) es: # "   d2 d s2 e−s s e−5s/2 G(s) = + ds2 (s + 1)(s − 2) ds (s + 1)(s − 2)  5s e− 2 25 s5 − 30 s4 − 87 s3 + 100 s2 + 108 s − 96 = − 4 (−s2 + s + 2)3  s e−s −s3 + s2 + s − 4 + , Re s > 2. (−s2 + s + 2)2  Problema 7.7. Determinar la transformada inversa de Laplace de F (s) =

s5 − 2s − 1 , s2 (s − 1)3

0 < Re s < 1.

Soluci´ on. Se tiene que F (s) es una funci´ on racional no propia. Por ello, es necesario expresar a F (s) de la siguiente forma equivalente: F (s) = 1 + F1 (s),

0 < Re s < 1,

244

7.6. PROBLEMAS RESUELTOS

donde F1 (s) =

3s4 − 3s3 + s2 − 2s − 1 , s2 (s − 1)3

0 < Re s < 1.

Luego, la transformada inversa de Laplace de F (s) es: f (t) = L−1 {F (s)} = L−1 {1} + L−1 {F1 (s)} = δ(t) + L−1 {F1 (s)} .

(7.20)

Debemos determinar la transformada inversa de Laplace de F1 (s). Se tiene que los polos a la izquierda y la derecha de la regi´ on de convergencia de F1 (s) son z0 = 0 y z1 = 1, a respectivamente. Por la Proposici´ on 7.3, la transformada inversa de Laplace de F1 (s) est´ dada por:     f1 (t) = Res est F1 (s) u(t) − Res est F1 (s) u(−t). s=0

s=1

Ahora,

  st  e (3s4 − 3s3 + s2 − 2s − 1) Res e F1 (s) = Res s=0 s=0 s2 (s − 1)3   st d e (3s4 − 3s3 + s2 − 2s − 1) = = t + 5, ds (s − 1)3 s=0  st   st  e (3s4 − 3s3 + s2 − 2s − 1) Res e F1 (s) = Res s=1 s=1 s2 (s − 1)3    1 d2 est (3s4 − 3s3 + s2 − 2s − 1) t 2 = −e t − 7t + 2 . = 2 ds2 s2 s=1 

st

As´ı, la transformada inversa de Laplace de F1 (s) es:

 f1 (t) = (t + 5) u(t) + et t2 − 7t + 2 u(−t).

Luego, sustituyendo f1 (t) en (7.20) tenemos que la transformada inversa de Laplace de F (s) es:  f (t) = δ(t) + (t + 5) u(t) + et t2 − 7t + 2 u(−t). 

Problema 7.8. Determinar la transformada inversa de Laplace de F (s) =

2s3 + 3s2 + 6s + 4 , (s − 2j)(s + 2j)(s + 1 − j)(s + 1 + j)

−1 < Re s < 0.

Soluci´ on. Se tiene que los polos a la izquierda de la regi´ on de convergencia son: z1 = −1 + j y z2 = −1 − j, y los polos a la derecha de la regi´ on de convergencia son: z3 = 2j y z4 = −2j. Por la Proposici´ on 7.3, la transformada inversa de Laplace de F1 (s) est´ a dada por:    st   st  f (t) = Res e F1 (s) + Res e F1 (s) u(t) s=−1−j s=−1+j    st   st  − Res e F1 (s) + Res e F1 (s) u(−t). s=2j

s=−2j

245

CAP´ITULO 7. TRANSFORMADA DE LAPLACE

Ahora, se tiene que Res

s=−1+j



 1 est F (s) = e−(1−j)t , 2

Res

s=−1−j



 1 est F (s) = e−(1+j)t 2

  1 Res est F1 (s) = e−2jt . s=−2j 2

  1 Res est F1 (s) = e2jt , s=2j 2

Luego, la transformada inversa de Laplace de F (s) es: 

  2jt  ejt + e−jt e + e−2jt f (t) = e u(t) − u(−t) 2 2 = e−t cos(t)u(t) − cos(2t)u(−t). −t



Problema 7.9. Sea f (t) una funci´ on de dominio continuo cuya transformada de Laplace F (s), −1 < Re s < 0, satisface: • s1 = 2 y s2 = 3, son los u ´ nicos ceros de F (s); • p1 = −1, p2 = 0 y p3 = 1, son los u ´ nicos puntos singulares de F (s), adem´as, p1 y p3 son polos simples y p2 es un polo de orden 2; • F (4) =

1 . 24

Determinar la transformada inversa de F (s). Soluci´ on. Por las condiciones dadas en el enunciado del problema, una expresi´ on matem´ atica de F (s) puede ser (existen otras considerando ceros con multiplicidad mayor que 1): k(s − 2)(s − 3) F (s) = , (s + 1)s2 (s − 1) donde k ∈ C. Ahora bien, como F (4) = 1/24, entonces podemos escribir: 1 k(4 − 2)(4 − 3) = 2 (4 + 1)4 (4 − 1) 24



k = 5.

As´ı, F (s) =

5(s − 2)(s − 3) , (s + 1)s2 (s − 1)

−1 < Re s < 0.

Se tiene que el polo a la izquierda de la regi´ on de convergencia es: p1 = −1 y los polos a la derecha de la regi´ on de convergencia son: p2 = 0 y p3 = 1. Por la Proposici´ on 7.3, la transformada inversa de Laplace de F (s) est´ a dada por:        f (t) = Res est F (s) u(t) − Res est F (s) + Res est F (s) u(−t). s=−1

s=0

s=1

246

7.7. PROBLEMAS PROPUESTOS

Ahora,   Res est F (s) =

s=−1

  Res est F (s) = s=0

  Res est F (s) = s=1

 5est (s − 2)(s − 3) = −30 e−t , s2 (s − 1) s=−1  st  d 5e (s − 2)(s − 3) = 25 − 30t, ds (s + 1)(s − 1) s=0   st 5e (s − 2)(s − 3) = 5 et . (s + 1)s2 

s=1

Luego, la transformada inversa de Laplace de F (s) es:

f (t) = −30 e−t u(t) − (25 − 30t + 5 et )u(−t). 

7.7 Problemas Propuestos 7.1. Calcular la transformada de Laplace de las siguientes funciones de dominio continuo. a) f (t) = δ(t − 1) + δ(t) + e−t u(−t) b) f (t) = eat (u(t) − u(t − 5)), para a < 0 c) f (t) = e4t sen (4t) u(t) d) f (t) = e−3t u(t) − e5t u(−t) e) f (t) = |t| f) f (t) = (1 − |t|) g) f (t) = (1 − |t|)u(t) h) f (t) = tn e−t u(−t), n ∈ Z+ i) f (t) = cos(at)u(t), a > 0 j) f (t) = senh(at)u(−t), a > 0 7.2. Obtener la transformada unilateral de Laplace de las siguientes funciones de dominio continuo: a) f1 (t) = tp2 (t − 1)

c) f3 (t) = p1 (t/2)

b) f2 (t) = f1 (t) + 12 δ(t)

d) f4 (t) = q1/2 (1 − t)

247

CAP´ITULO 7. TRANSFORMADA DE LAPLACE

7.3. La transformada de Laplace de una funci´ on causal f (t) (f (t) = 0 para t < 0) es: F (s) =

s e−2s , s2 + 2s + 1

Re s > −1.

Determinar la transformada de Laplace de las siguientes funciones:   d t e) g(t) = t f (t) a) g(t) = 3f dt 3 b) g(t) = t f (t)

f) g(t) = δ(t − a) f (t), a ∈ R

c) g(t) = t f (t − 1)

g) g(t) = (t − 1) f (t − 1) +

d d) g(t) = f (t) dt

h) g(t) =

Z

d f (t) dt

t

f (τ ) τ 0

7.4. Determinar los valores inicial y final de las funciones causales f (t), cuya transformada de Laplace se indica a continuaci´ on, sin calcular la transformada inversa de Laplace. Si no existe valor final, indicar el motivo. a) F (s) =

1 , Re s > −2 s+2

b) F (s) =

1 , Re s > −2 (s + 2)25

c) F (s) = d) F (s) =

s(s2

6 , Re s > 0 + 25)

s+2 , Re s > −3 s+3

s2 + s + 3 , Re s > 2 s4 − 9s2 + 4s + 12 s , Re s > 3 f) F (s) = 2 s − 2s − 3

e) F (s) =

7.5. Determinar la transformada inversa de Laplace de cada una de las siguientes transformadas, mediante el procedimiento por residuos. a) F (s) = b) F (s) =

s2

1 , Re s < −1 +1

s2

1 , Re s > −a, con a < 0 + a2

c) F (s) =

s−2 , −1 < Re s < 3 (s + 1)(s − 3)

d) F (s) =

s2 + 2s − 2 , Re s > 1 s(s − 1)

e) F (s) =

s3 + 4s2 + s − 1 , −1 < Re s < 0 s3 (s − 1)(s + 1)2

248

7.7. PROBLEMAS PROPUESTOS

f) F (s) =

2s3 + 3s2 + 6s + 4 , −1 < Re s < 0 (s2 + 4)(s2 + 2s + 2)

7.6. Sea f (t) una funci´ on de dominio continuo cuya transformada de Laplace F (s) satisface: • D = {s ∈ C : −1 < Re s < 1} es la regi´ on de convergencia de F (s); • s1 = 2 y s2 = 3, son los u ´nicos ceros de F (s);

• p1 = −2, p2 = −1 y p3 = 1, son los u ´ nicos puntos singulares de F (s), adem´as, todos son polos simples;

• F (0) = α, donde α > 0. Utilizar el procedimiento por residuos para determinar la transformada inversa de F (s).

8 Transformada z La transformada z es una herramienta tan poderosa para el an´alisis de sistemas de tiempo discreto, como lo es la transformada de Laplace para sistemas de tiempo continuo. En este cap´ıtulo se presenta la transformada z, su definici´ on y propiedades. Se calculan las transformadas z de funciones comunes de dominio discreto. Finalmente, se halla la transformada z inversa mediante tres m´ etodos: integraci´ on compleja, expansi´on en serie de potencias e inversi´on por tablas.

8.1 Definici´ on La transformada z es la contraparte en el dominio discreto de la transformada de Laplace. La transformada z opera sobre funciones de dominio discreto, a diferencia de la transformada de Laplace que opera sobre una funci´ on de dominio continuo. Definici´ on 8.1 (Transformada z bilateral). Sea f (n) una funci´ on de dominio discreto definida de Z en C. La transformada z bilateral de f (n) se define como F (z) = Z[f (n)] =

∞ X

f (n)z −n ,

n=−∞

para todo z ∈ C tal que |F (z)| < ∞. Cambiando los l´ımites de la sumatoria en la definici´ on de la transformada z bilateral, se obtienen las transformadas z unilaterales. Definici´ on 8.2. Sea f (n) una funci´ on de dominio discreto definida de Z en C. Se define la transformada z unilateral derecha de f (n) como ZD {f (n)} =

∞ X

f (n)z −n ,

n=0

para todo s ∈ C tal que |ZD [f (n)](z)| < ∞. La transformada z unilateral izquierda de f (n) se define como ZI {f (n)} =

0 X

n=−∞

para todo s ∈ C tal que |ZI [f (n)](z)| < ∞.

249

f (n)z −n ,

250

´ 8.1. DEFINICION

Observaci´ on 8.1. De la definici´ on de la transformada z se infiere que f (n) son los coeficientes del desarrollo en serie de Laurent de F (z) centrado en z0 = 0. Adem´ as, F (z) es la P∞ −n suma de la serie n=−∞ f (n)z . A lo largo del cap´ıtulo nos referiremos con transformada z a la transformada z bilateral, en caso contrario lo indicaremos expl´ıcitamente. En la siguiente proposici´ on se establece que si la transformada z existe, entonces ella es una funci´ on anal´ıtica. Proposici´ on 8.1. Si F (z) es la transformada z de la funci´on f (n), entonces F (z) es anal´ıtica en todo n´ umero complejo z tal que |F (z)| < ∞. Demostraci´ on. Consideremos la transformada z bilateral. Por definici´ on de la transformada z se tiene que ∞ X f (n)z −n , F (z) = n=−∞

P −n converge a adem´as, |F (z)| < ∞; en otras palabras, la serie de potencia ∞ n=−∞ f (n)z F (z) para cada z tal que |F (z)| < ∞. Por lo tanto, F (z) es una funci´ on anal´ıtica para z tal que |F (z)| < ∞.

on de Convergencia 8.1.1 Regi´ Definici´ on 8.3. La regi´ on de convergencia de la transformada z de f (n), es el conjunto de n´ umeros complejos z donde F (z) existe, esto es, son todos los z ∈ C tales que |F (z)| < ∞. En otras palabras, la regi´ on de convergencia de F (z) es un anillo centrado en el origen donde la serie ∞ X f (n)z −n n=−∞

es convergente.

En el siguiente ejemplo se calculan las transformadas z unilateral derecha y bilateral de una funci´ on f (n), con sus correspondientes regiones de convergencia. Ejemplo 8.1. Calcular las transformadas z bilateral y unilateral derecha de la funci´ on f (n) = an u(n + 1),

a ∈ R, a 6= 0.

Soluci´ on. Se tiene que la transformada z bilateral de f (n) es: F (z) =

∞ X

f (n)z −n =

an u(n + 1)z −n =

n=−∞

n=−∞

= a−1 z +

∞ X

∞ X

(az −1 )n =

n=0

∞ X

an z −n

n=−1

z 1 + , a 1 − az −1

|az −1 | < 1

o, equivalentemente, F (z) =

z2 , a(z − a)

|z| > |a|.

=

∞ X

(az −1 )n

n=−1

251

CAP´ITULO 8. TRANSFORMADA Z

Ahora, la transformada z unilateral derecha de f (n) es ZD {f (n)} = =

∞ X

an u(n + 1) z −n =

∞ X

(az −1 )n

n=0

n=0

1 z = , 1 − az −1 z−a

|z| > |a|.

En la siguiente figura se aprecia la regi´ on de convergencia de las transformadas z halladas. y

|z| > |a|

|a|

x



8.1.2 Transformada z Inversa La transformada z inversa es la contraparte en el dominio discreto de la transformada inversa de Laplace en el dominio continuo. En s´ı, la transformada z inversa es el proceso de obtener f (n) a partir de la transformada z, F (z). Definici´ on 8.4. Sea F (z), para r1 < |z| < r2 , la transformada z de f (n). La transformada z inversa es el proceso de obtener f (n) a trav´ es de F (z) y se define como: Z 1 f (n) = f (n) z n−1 dz, para cada n ∈ Z, (8.1) 2πj C donde C es un contorno cerrado simple contenido en la regi´ on de convergencia de F (z) y que confina la circunferencia |z| = r1 . Para el c´ alculo de la integral de la ecuaci´ on (8.1) se utiliza, generalmente, el Teorema de los Residuos o la Extensi´ on de la F´ ormula Integral de Cauchy. Nos centraremos en tres m´ etodos alternativos para calcular la transformada z inversa: Integraci´on Compleja, Inversi´on por Tablas y Expansi´on en Serie de Potencias, los cuales se describir´ an en la Secci´ on 8.4. En el resto del cap´ıtulo usaremos la siguiente notaci´ on. El s´ımbolo Z

f (n) ←→ F (z),

z∈D

que se lee par de transformadas, denota que F (z) es la transformada z de f (n) con regi´ on de convergencia el conjunto D ⊂ C, y que f (n) es la transformada z inversa de F (z).

252

8.2. PROPIEDADES DE LA TRANSFORMADA Z

8.2 Propiedades de la Transformada z A continuaci´ on damos las propiedades de la transformada z. Teorema 8.1 (Linealidad). Si las transformadas z de las funciones f (n) y g(n) son respectivamente F (z), para todo z ∈ Df ⊂ C y G(z), para todo z ∈ Dg ⊂ C, entonces Z

af (n) + bg(n) ←→ aF (z) + bG(z),

z ∈ Df ∩ Dg ,

donde a y b son constantes reales. Demostraci´ on. Por la definici´ on de la transformada z se tiene que Z{af (n) + bg(n)} =

∞ X

(af (n) + bg(n))z −n

n=−∞ ∞ X

f (n)z −n + b

= a

∞ X

g(n)z −n

n=−∞

n=−∞

= aZ{f (n)} + bZ{g(n)} = aF (z) + bG(z), adem´as, como Df y Df son las regiones de convergencia de la series ∞ X

f (n)z −n

y

∞ X

g(n)z −n ,

n=−∞

n=−∞

respectivamente, entonces se tiene que el conjunto Df ∩ Dg es la regi´ on de convergencia de Z{af (n) + bg(n)}. Teorema 8.2 (Desplazamiento en tiempo). Si la transformada z de la funci´on f (z) es F (z) para todo z ∈ D ⊂ C, entonces Z

f (n − n0 ) ←→ z −n0 F (z),

z ∈ D,

para todo n0 ∈ Z. Demostraci´ on. Por la definici´ on de la transformada z se tiene que Z{f (n − n0 )} =

∞ X

n=−∞

f (n − n0 )z −n .

Haciendo el cambio de variable k = n − n0 , la serie anterior adquiere la forma: Z{f (n − n0 )} =

∞ X

f (k) z −n0 z −k

k=−∞

= z −n0

∞ X

n=−∞

f (n)z −n = z −n0 F (z),

z ∈ D.

253

CAP´ITULO 8. TRANSFORMADA Z

Teorema 8.3 (Inversi´ on en el tiempo). Si la transformada z de la funci´on f (z) es F (z) para todo z ∈ D ⊂ C, entonces Z

f (−n) ←→ F (z −1 ),

z −1 ∈ D.

Demostraci´ on. Por la definici´ on de la transformada z se tiene que Z{f (−n)} =

∞ X

f (−n)z −n .

n=−∞

Haciendo el cambio de variable k = −n, la serie anterior adquiere la forma: Z{f (−n)} =

−∞ X

f (k) z k =

k=∞

−∞ X

f (k) (z −1 )−k .

k=∞

Cambiando k por n en la serie y considerando la conmutatividad de la suma, se tiene: Z{f (−n)} =

∞ X

f (n) (z −1 )−n = F (z −1 ),

n=−∞

z −1 ∈ D.

Teorema 8.4 (Escalado en el dominio de z). Si la transformada z de la funci´on f (z) es F (z) para todo z ∈ D ⊂ C, entonces: Z

z0n f (n) ←→ F (z/z0 ) ,

z ∈ D,

 Z ejω0 n f (n) ←→ F e−jω0 z , z ∈ D,  Z an f (n) ←→ F a−1 z , z ∈ D,

para z0 ∈ C, ω0 ∈ R y a ∈ R.

Demostraci´ on. Como ejω0 n f (n) y an f (n) son casos particulares de z0n f (n) (tome respectivamente z0 = ejω0 y z0 = a), entonces solo demostraremos que Z{z0n f (n)} = F (z/z0 ), z ∈ D. Se tiene que Z{z0n f (n)} =

∞ X

z0n f (n)z −n =

∞ X

f (n) (z/z0 )−n = F (z/z0 ),

n=−∞

n=−∞

z ∈ D.

Teorema 8.5 (Conjugaci´ on). Si la transformada z de la funci´on f (z) es F (z) para todo z ∈ D ⊂ C, entonces Z f (n) ←→ F (z), z ∈ D. Demostraci´ on. Se tiene que Z{f (n)} =

∞ X

n=−∞

f (n)z

−n

=

∞ X

n=−∞

f (n) z −n

=

∞ X

n=−∞

f (n) (z)−n = F (z),

z ∈ D.

254

8.2. PROPIEDADES DE LA TRANSFORMADA Z

Teorema 8.6 (Convoluci´ on). Si las transformadas z de las funciones f (n) y g(n) son respectivamente F (z), para todo z ∈ Df ⊂ C y G(z), para todo z ∈ Dg ⊂ C, entonces Z

f ∗ g (n) ←→ F (z) G(z),

z ∈ Df ∩ Dg .

Demostraci´ on. Se tiene que Z{f ∗ g (n)} =

∞ X

n=−∞

f ∗ g (n)z −n =

∞ X

n=−∞

"

∞ X

k=−∞

#

f (k) g(n − k) z −n

Haciendo el cambio de variable ℓ = n − k, la ecuaci´ on anterior adquiere la forma: # " ∞ ∞ X X f (k) g(ℓ) z −(ℓ+k) , Z{f ∗ g (n)} = ℓ=−∞

k=−∞

de lo cual se deduce que Z{f ∗ g (n)} =

∞ X

k=−∞

f (k) z

−k

!

∞ X

g(ℓ) z

−ℓ

ℓ=−∞

!

= F (z) G(z),

z ∈ Df ∩ Dg .

Teorema 8.7 (Primera diferencia). Si la transformada z de la funci´on f (z) es F (z) para todo z ∈ D ⊂ C, entonces   z−1 Z f (n) − f (n − 1) ←→ F (z), z ∈ D ∩ {z ∈ C : |z| > 0}. z Demostraci´ on. Se tiene que Z{f (n) − f (n − 1)} = =

∞ X

[f (n) − f (n − 1)]z −n

n=−∞ ∞ X

f (n)z

−n

n=−∞

= F (z) −

∞ X

n=−∞



∞ X

n=−∞

f (n − 1)z −n

f (n − 1)z −n .

Haciendo el cambio de variable k = n − 1, de la ecuaci´ on anterior se deduce que Z{f (n) − f (n − 1)} = F (z) −

∞ X

f (k)z −(n+1)

k=−∞

= F (z) − z −1 −1

∞ X

f (k)z −n

k=−∞

= F (z) − z F (z)   z−1 F (z), z ∈ D ∩ {z ∈ C : |z| > 0}. = z

255

CAP´ITULO 8. TRANSFORMADA Z

Teorema 8.8 (Acumulaci´ on). Si la transformada z de la funci´on f (z) es F (z) para todo z ∈ D ⊂ C, entonces n X

k=−∞

Z

f (k) ←→



z z−1



F (z),

z ∈ D ∩ {z ∈ C : |z| > 1}.

Demostraci´ on. Para la prueba usaremos la transformada z del escal´ on unitario discreto, a saber: z , |z| > 1. Z{u(n)} = z−1

la cual la hallaremos m´ as adelante. Ahora, se tiene que f (n) ∗ u(n) =

∞ X

k=−∞

f (k)u(n − k) =

n X

f (k),

(8.2)

k=−∞

para toda funci´ on f (n) de dominio discreto. . De esta forma, usando la propiedad de convoluci´ on en la ecuaci´ on (8.2), se tiene Z{f (n) ∗ u(n)} = Z{f (n)} Z{u(n)},

z ∈ D ∩ {z ∈ C : |z| > 1}.

(8.3)

Ahora, usando la expresi´ on de la transformada z del escal´ on unitario discreto, entonces por (8.2) y (8.3), se obtiene ( n )   X z F (z), z ∈ D ∩ {z ∈ C : |z| > 1}. Z f (k) = z−1 k=−∞

Teorema 8.9 (Diferenciaci´ on en el dominio de z). Si la transformada z de la funci´on f (z) es F (z) para todo z ∈ D ⊂ C, entonces Z

n f (n) ←→ −z

d F (z), dz

z ∈ D.

Demostraci´ on. Se tiene que, para todo z ∈ D, " ∞ # X d d −z F (z) = −z f (n)z −n dz dz n=−∞ = −z = z

∞ X

f (n)

n=−∞ ∞ X

d  −n  z dz

(n f (n)) z −(n+1) =

(n f (n))z −n ,

n=−∞

n=−∞

de donde se deduce que Z{n f (n)} = −z

∞ X

d F (z), dz

z ∈ D.

Se deja al lector la prueba de la ecuaci´ on (8.2)

256

8.2. PROPIEDADES DE LA TRANSFORMADA Z

Teorema 8.10 (Teorema del Valor Inicial). Sea f (n) una funci´on tal que f (n) = 0 para n < 0. Si la transformada z de f (n) es F (z) para todo z ∈ C tal que |z| > a con a > 0, entonces f (0) = l´ım F (z). z→∞

Demostraci´ on. Se tiene que F (z) =

∞ X

f (z)z −n

n=−∞

= f (0) + f (1)z −1 + f (2)z −2 + · · · + f (n)z −n + · · · Tomando l´ımite en ambos lados de la ecuaci´ on anterior, cuando z → ∞, se obtiene f (0) = l´ım F (z). z→∞

Teorema 8.11 (Teorema del Valor Final). Sea f (n) una funci´on tal que f (n) = 0 para n < 0. Si la transformada z de f (n) es F (z) para todo z ∈ C tal que |z| > a con a > 0, entonces   z−1 l´ım f (n) = l´ım F (z). n→∞ z→1 z Demostraci´ on. Aplicando la propiedad de primera diferencia se tiene   z−1 F (z), z ∈ {|z| > a} ∩ {|z| > 1}. Z{f (n) − f (n − 1)} = z

(8.4)

Como f (n) = 0 para n < 0, entonces se obtiene: Z{f (n) − f (n − 1)} =

∞ X

[f (n) − f (n − 1)] z −n

n=0

= l´ım

N →∞

N X

[f (n) − f (n − 1)] z −n .

(8.5)

n=0

De (8.4) y (8.5), se deduce: l´ım

N →∞

N X

[f (n) − f (n − 1)] z

n=0

−n

=



z−1 z



F (z),

z ∈ {|z| > a} ∩ {|z| > 1}.

De esta forma, tomando l´ımite en ambos lados de la ecuaci´ on anterior, cuando z → 1, se tiene:   z−1 l´ım f (N ) = l´ım F (z). z→1 N →∞ z Las propiedades de la transformada z se resumen en la Tabla 8.1.

257

CAP´ITULO 8. TRANSFORMADA Z

Tabla 8.1. Propiedades de la Transformada z Sean f (n), f1 (n) y f2 (n), funciones de dominio discreto con transformadas z dadas por: F (z) para z ∈ D, F1 (z) para z ∈ D1 y F2 (z) para z ∈ D2 , respectivamente. Propiedad

Descripci´ on Matem´ atica

Linealidad

Z {a1 f (n) + a2 f2 (n)} = a1 F (z) + a2 F2 (z), z ∈ D1 ∩ D2 ( D, z∈ D − {0},

Desplazamiento en tiempo

Z {f (n − n0 )} = z −n0 F (z),

Escalado en el dominio de z

Z {an f (n)} = F (a−1 z),

Inversi´ on en el tiempo

Z {f (−n)} = F (z −1 ),

Conjugaci´ on

n o Z f (n) = F (z),

Convoluci´ on

Z {f1 ∗ f2 (n)} = F1 (z)F2 (z),

Primera diferencia

Z {f (n) − f (n − 1)} =

Acumulaci´ on

Z

(

n X

)

f (k)

k=−∞

Z {n f (n)} = −z

Teorema del valor inicial

f (0) = l´ım F (z)

Teorema del valor final

a−1 z ∈ D z −1 ∈ D

z∈D

=

Diferenciaci´ on en z



z−1 z

z z−1



d F (z), dz



z ∈ D ⊇ D1 ∩ D2 F (z),

F (z),

z∈D

z→∞

l´ım f (n) = l´ım

n→∞

z→1



si n0 ≤ 0, si n0 > 0

z−1 z



F (z)

z ∈ D ∩ {|z| > 0}

z ∈ D ∩ {|z| > 1}

258

8.3. ALGUNOS PARES DE TRANSFORMADAS

8.3 Algunos Pares de Transformadas En esta secci´ on se calcula la transformada z de funciones comunes. 1.

Z

δ(n) ←→ 1,

Se tiene que Z{δ(n)} =

∞ X

z∈C

δ(n)z −n = δ(0)z 0 = 1,

z ∈ C.

n=−∞

2.

z , z−1

Z

u(n) ←→ Se tiene que Z{u(n)} = =

∞ X

|z| > 1

u(n)z

−n

=

n=−∞

∞ X

(z −1 )n

n=0

1 z = , −1 1−z z−1

|z| > 1.

en La regi´ on de convergencia de Z{u(n)} se muestra en la Figura 8.1. Aqu´ı tambi´ se aprecia la ubicaci´ on de los polos y ceros de Z{u(n)}. Esta gr´afica se denomina diagrama de polos y ceros. y

|z| > 1

1 b

b

0

x

1

Figura 8.1. Diagrama de polos y ceros de Z{u(n)} 3.

Z

−u(−n − 1) ←→ Se tiene que Z{−u(−n − 1)} =

∞ X

z , z−1

[−u(−n − 1)]z

|z| < 1

−n

=−

n=−∞

Haciendo el cambio de variable k = −n, se obtiene Z{−u(−n − 1)} = − = −

1 X

k=∞



zk = −

1 1−z

"

 −1 ,

∞ X

n=0

−1 X

z −n .

n=−∞

#

zk − 1

|z| < 1

259

CAP´ITULO 8. TRANSFORMADA Z

o, equivalentemente, Z{−u(−n − 1)} =

z , z−1

|z| < 1.

En la Figura 8.2 se muestra el diagrama de polos y ceros de Z{−u(−n − 1)}. y |z| < 1 1 b

b

0

x

1

Figura 8.2. Diagrama de polos y ceros de Z{−u(−n − 1)} 4.

z , z−α

Z

αn u(n) ←→ Se tiene que Z{αn u(n)} = = o, equivalentemente,

∞ X

[αn u(n)]z −n =

n=−∞

5.

z , z−α Z

−αn u(−n − 1) ←→ Se tiene que ∞ X

∞ X

(αz −1 )n

n=0

1 , 1 − αz −1

Z{αn u(n)} =

Z{−αn u(−n − 1)} =

|z| > |α|

|αz −1 | < 1

|z| > |α|.

z , z−α

|z| < |α|

[−αn u(−n − 1)]z −n = −

n=−∞

−1 X

(α−1 z)−n .

n=−∞

Haciendo el cambio de variable k = −n, se obtiene Z{−αn u(−n − 1)} = −

1 X

"∞ # X (α−1 z)k = − (α−1 z)k − 1

k=∞



 1 = − −1 , 1 − α−1 z

k=0

|α−1 z| < 1

o, equivalentemente, Z{−αn u(−n − 1)} =

z , z−α

|z| < |α|.

260

8.3. ALGUNOS PARES DE TRANSFORMADAS

6.

αz , (z − α)2

Z

n αn u(n) ←→

|z| > |α|

Se tiene que Z{αn u(n)} =

z , z−α

|z| > |α|.

Luego, aplicando la propiedad de diferenciaci´ on en z, se obtiene d [Z{αn u(n)}] dz   d z = −z , |z| > |α| dz z − α

Z{n αn u(n)} = −z

o, equivalentemente, Z{n αn u(n)} =

αz , (z − α)2

7. Z

−n αn u(−n − 1) ←→

|z| > |α|.

αz , (z − α)2

|z| < |α|

Se tiene que Z{u(−n − 1)} = −

z , z−1

|z| < 1.

(8.6)

Aplicando las propiedades de linealidad y diferenciaci´ on en z, se tiene Z{−n αn u(−n − 1)} = z

d Z{αn u(−n − 1)} dz

(8.7)

Ahora, aplicando la propiedad escalado en el dominio de z, entonces por (8.6) se tiene Z{αn u(−n − 1)} = Z{u(−n − 1)}(α−1 z), z = − , |z| < |α|. z−α

|α−1 z| < 1 (8.8)

Luego, por (8.7) y (8.8), se obtiene   d z Z{−n α u(−n − 1)} = −z , dz z − α n

|z| < |α|

o, equivalentemente, Z{−n αn u(−n − 1)} =

αz , (z − α)2

|z| < |α|.

261

CAP´ITULO 8. TRANSFORMADA Z

8. z , z − e−aT

Z

e−aT n u(n) ←→

|z| > e−aT

Se tiene que

Z{e−aT n u(n)} = =

∞ X

e−aT n u(n)z −n

n=−∞ ∞ X

e−aT z −1

n=0

n

=

1 1−

,

e−aT z −1

−aT −1 e z <1

o, equivalentemente, z , z − e−aT

Z{e−aT n } =

|z| > e−aT .

9. Z

cos(ωn)u(n) ←→

z2

z(z − cos(ω)) , − 2z cos(ω) + 1

|z| > 1

Se tiene que

Z{cos(ωn)u(n)} = =

∞ X

n=−∞ ∞  jωn X e

n=0

= =

cos(ωn)u(n)z −n + e−jωn 2



z −n

"∞ # ∞ 1 X jω −1 n X −jω −1 n e z + e z 2 n=0 n=0   1 1 1 + , 2 1 − ejω z −1 1 − e−jω z −1

para z ∈ {|ejω z −1 | < 1} ∩ {|e−jω z −1 | < 1} o, equivalentemente, Z{cos(ωn)u(n)} =

z2

z(z − cos(ω)) , − 2z cos(ω) + 1

|z| > 1.

En la Tabla 8.2 se muestra un resumen de los pares de transformadas z de funciones comunes.

262

8.3. ALGUNOS PARES DE TRANSFORMADAS

Tabla 8.2. Algunos Pares de Transformadas z Funci´ on

Transformada z

δ(n)

1,

u(n)

z , z−1

z∈C |z| > 1

nu(n)

z , (z − 1)2

|z| > 1

n2 u(n)

z(z + 1) , (z − 1)3

|z| > 1

−u(−n − 1) αn u(n) −αn u(−n − 1) n αn u(n) −n αn u(−n − 1)

z , z−1

z , z−α z , z−α

|z| < 1 |z| > |α| |z| < |α|

αz , (z − α)2 αz , (z − α)2

|z| > |α| |z| < |α|

e−aT n u(n)

z , z − e−aT

cos(ωn)u(n)

z(z − cos(ω)) , z 2 − 2z cos(ω) + 1

|z| > 1

sen (ωn)u(n)

z sen (ω) , z 2 − 2z cos(ω) + 1

|z| > 1

αn cos(ωn)u(n) αn sen (ωn)u(n)

|z| > e−aT

z2

z(z − α cos(ω)) , − 2zα cos(ω) + α2

|z| > |α|

z2

zα sen (ω) , − 2zα cos(ω) + α2

|z| > 1

263

CAP´ITULO 8. TRANSFORMADA Z

8.4 C´ alculo de la Transformada z Inversa Sea f (n) una funci´ on de dominio discreto cuya transformada z es F (z), r1 < |z| < r2 , donde r1 y r2 son n´ umeros reales positivos. Se utilizar´ an tres m´ etodos para calcular la transformada z inversa, a saber: Integraci´on Compleja, Expansi´on en Serie de Potencias e Inversi´on con Tablas.

8.4.1 Integraci´ on Compleja El m´ etodo de Integraci´on Compleja utiliza la definici´ on de la transformada z inversa. En otras palabras, f (n) se calcula como f (n) =

1 2πj

Z

f (n) z n−1 dz,

para cada n ∈ Z,

C

donde C es un contorno cerrado simple contenido en la regi´ on de convergencia de F (z) y que confina la circunferencia |z| = r1 .Para el c´ alculo de la integral se utiliza, generalmente, el Teorema de los Residuos. Expliquemos el m´ etodo integraci´ on compleja mediante un ejemplo. Ejemplo 8.2. Calcular la transformada z inversa de F (z) =

z , z−a

|z| > |a|.

Soluci´ on. Sea C la circunferencia |z| = r > |a|. Ahora, considerando la expresi´ on de F (z) se tiene Z 1 f (n) = F (z) z n−1 dz 2πj C Z zn 1 dz, para cada n ∈ Z. (8.9) = 2πj C z − a Supongamos que n ≥ 0. Aplicando el Teorema de los Residuos en la integral de (8.9), se obtiene:  n  z f (n) = Res = an . z=a z − a Cuando n < 0, entonces aplicando nuevamente el Teorema de los Residuos en la integral de (8.9), se consigue   n  z zn f (n) = Res + Res = −an + an = 0. z=a z − a z=0 z − a 

Por lo tanto, la transformada z inversa de F (z) es f (n) = an u(n). 

Sin p´ erdida de generalidad, C puede ser la circunferencia |z| = r, con r1 < r < r2

264

´ 8.4. CALCULO DE LA TRANSFORMADA Z INVERSA

8.4.2 Expansi´ on en Serie de Potencias El procedimiento expansi´on en serie de potencias consiste en encontrar el desarrollo de Laurent de F (z) centrado en z0 = 0, que es v´alido en el anillo r1 < |z| < r2 , y luego definir a f (n) como los coeficientes de esta serie de potencias. En el siguiente ejemplo se explica esta metodolog´ıa. Ejemplo 8.3. Determinar la transformada z inversa de F (z) =

1 , (1 + z −1 )(1 − z −1 )2

|z| > 1. 

Soluci´ on. Escribimos a F (z) en potencias positivas: F (z) = Recuerde que el m´ etodo expansi´ on en fracciones parciales, solo se aplica a funciones racionales propias.

z3 . (z + 1)(z − 1)2

Luego, expresamos a F (z)/z en fracciones parciales: F (z) 1 1 1 1 3 1 = + + . z 4 (z + 1) 4 (z − 1) 2 (z − 1)2 De esta forma, obtenemos: F (z) =

z z z 3 1 1 + + , 4 (z + 1) 4 (z − 1) 2 (z − 1)2

|z| > 1.

(8.10)

Hallamos los desarrollos de Laurent centrados en z0 = 0 y v´alidos en |z| > 1, de cada uno de los sumandos de (8.10). Se tiene que ∞ ∞ X X z z n −n (−1)n u(n) z −n , (−1) z = = = (z + 1) z(1 + z −1 ) n=−∞ n=0

∞ ∞ X X z z −n u(n) z −n , z = = = (z − 1) z(1 − z −1 ) n=0 n=−∞

|z| > 1, |z| > 1;

(8.11)

(8.12)

ahora, derivando en ambos lados de (8.12) se obtiene

∞ ∞ X X −1 −(n+1) −1 nu(n) z −n , (−n)z = −z = (z − 1)2 n=0 n=−∞

de donde se deduce

∞ X z nu(n) z −n , = (z − 1)2 n=−∞

|z| > 1,

|z| > 1.

Sustituyendo convenientemente (8.11), (8.12) y (8.13) en (8.10), se obtiene: ∞ ∞ ∞ 3 X 1 X 1 X (−1)n u(n) z −n + u(n) z −n + nu(n) z −n 4 n=−∞ 4 n=−∞ 2 n=−∞  ∞  X 1 3 1 n (−1) u(n) + u(n) + nu(n) z −n , |z| > 1. = 4 4 2 n=−∞

F (z) =

(8.13)

265

CAP´ITULO 8. TRANSFORMADA Z

Por lo tanto, la transformada z inversa de F (z) es f (n) =

3 1 1 (−1)n u(n) + u(n) + n u(n). 4 4 2 

8.4.3 Inversi´ on por Tablas El m´ etodo inversi´on por tablas consiste en expresar a F (z) como una suma de la forma F (z) = F1 (z) + F2 (z) + · · · + FR (z),

(8.14)

donde F1 (z), F2 (z), . . . , FR (z), son funciones tales que se les conoce su transformada z inversa f1 (n), f2 (n), . . . , fR (n). De esta forma, la transformada z inversa de F (z) est´ a dada por: f (n) = f1 (n) + f2 (n) + · · · + fR (n). Observaci´ on 8.2. Cuando F (z) es una funci´ on racional propia, se emplea la expansi´ on en fracciones parciales para obtener la descomposici´ on (8.14). Ejemplo 8.4. Determinar la transformada z inversa de F (z) =

1 (1 +

z −1 )(1

− z −1 )2

,

|z| > 1.

Soluci´ on. Escribimos a F (z) en potencias positivas: F (z) =

z3 . (z + 1)(z − 1)2

Ahora, aplicando la expansi´ on en fracciones parciales a F (z) z2 = , z (z + 1)(z − 1)2 se tiene que F (z) A1 A2,1 A2,2 = + + , z (z + 1) (z − 1) (z − 1)2

donde A1 = A2,1 = A2,2 = As´ı, F (z) =



   z2 1 F (z) = , (z + 1) = 2 z (z − 1) z=−1 4 z=−1    2  d z d 3 F (z) = = , (z − 1)2 dz z dz z + 1 4 z=1 z=1   2   z 1 2 F (z) (z − 1) = = . z z=1 z + 1 z=1 2 z z z 3 1 1 + + , 4 (z + 1) 4 (z − 1) 2 (z − 1)2

|z| > 1.

266

8.5. TRANSFORMADA Z CON MATLAB

Como Z

(−1)n u(n) ←→

z , z+1

|z| > 1,

z , |z| > 1, z−1 z Z , |z| > 1, n u(n) ←→ (z − 1)2 Z

u(n) ←→

entonces la transformada z inversa de F (z) es f (n) =

3 1 1 (−1)n u(n) + u(n) + n u(n). 4 4 2 

8.5 Transformada z con M ATLAB La herramienta de matem´ atica simb´ olica de M ATLAB cuenta con el comando ztrans que calcula la transformada z unilateral derecha de una funci´ on f (n). Observe el siguiente ejemplo donde se calcula la transformada z unilateral derecha de las siguientes funciones: f1 (n) = 2−n u(n),

f2 (t) = n u(n).

✄ >> syms n z >> f 1 ( n ) = 2ˆ(−n ) ; F1 ( z ) = z t r a n s ( f 1 ) F1 ( z ) = z / ( z − 1/2) >> f 2 ( n ) = n ; F2 ( z ) = z t r a n s ( f 2 ) F2 ( z ) = z / ( z − 1)ˆ2





Las transformadas z unilaterales derechas obtenidas son: F1 (z) =

z , z − 12

F2 (z) =

z . (z − 1)2

Note que el comando ztrans suministra la expresi´ on matem´atica de la transformada z unilateral derecha, pero no nos dice nada acerca de su regi´ on de convergencia; adem´as, para utilizar ztrans se debe asumir que f (n) es una funci´ on causal. Cuando f (n) no es causal, entonces ztrans halla la transformada z de la parte causal de f (n) . Observe el siguiente ejemplo. ✄ >> syms n z >> f 3 ( n ) = n∗ h e a v i s i d e ( n)+3ˆn∗ h e a v i s i d e (−n ) ; >> F3 ( z ) = z t r a n s ( f 3 ) F3 ( z ) = z / ( z − 1)ˆ2





En este ejemplo se calcula la transformada z de f3 (n) = n u(n) + 3n u(−n), obteniendo como transformada z unilateral derecha a F3 (z) = z/(z−1)2 , es decir, ztrans no consider´ o on TransZ, la cual calcula la parte no causal de f (n). En el Programa 8.1 se muestra la funci´ la transformada z bilateral de una funci´ on f (n) cualquiera.

267

CAP´ITULO 8. TRANSFORMADA Z

Programa 8.1. Funci´ on TransZ.m f u n c t i o n [ F ] = TransZ ( f l , f r ) %TransZ H a l l a l a t r a n s f o r m a d a z b i l a t e r a l de una % funcio ´n f (n) syms n z ; F l ( z ) = z t r a n s ( f l (−n ) ) ; Fr ( z ) = z t r a n s ( f r ( n ) ) ; F ( z ) = F l ( z ˆ( −1)) + Fr ( z ) ; end

La funci´ on TransZ se basa en el hecho que toda funci´ on f (n) se puede expresar como: f (n) = fl (n) + fr (n), donde fl (t) es una funci´ on no causal y fr (n) es una funci´ on causal. A continuaci´ on se calcula la transformada z bilateral de la funci´ on f3 (n) dada arriba. ✄ >> syms n z >> f 3 l ( n ) = 3ˆn∗ h e a v i s i d e (−n ) ; f 3 r ( n ) = n∗ h e a v i s i d e ( n ) ; >> F3 ( z ) = TransZ ( f 3 l , f 3 r ) F3 ( z ) = z / ( z − 1)ˆ2 + 1/(3/ z − 1) + 1/2





La expresi´ on matem´ atica de la transformada z bilateral de f3 (n) obtenida es: F3 =

z + (z − 1)2

3 z

z 1 1 1 z + = 2 − z − 3 + 2. 2 −1 (z − 1)

Por otra parte, M ATLAB tambi´ en cuenta con el comando iztrans que halla la transformada z inversa. Observe el siguiente ejemplo donde se halla la expresi´ on matem´atica de la transformada z inversa de las transformadas F1 (z), F2 (z) y F3 (z) dadas arriba. ✄ >> syms z >> F1 ( z ) = z / ( z − 1 / 2 ) ; f 1 ( n ) = i z t r a n s ( F1 ) f1 (n) = (1/2)ˆ n >> F2 ( z ) = z / ( z − 1 ) ˆ 2 ; f 2 ( n ) = i z t r a n s ( F2 ) f2 (n) = n >> F3 ( z ) = z / ( z − 1)ˆ2 + 1/(3/ z − 1) + 1/2; >> f 3 ( n ) = i z t r a n s ( F3 ) f3 (n) = n − 3ˆn + k r o n e c k e r D e l t a ( n , 0)/2





Tenga presente que M ATLAB utiliza el s´ımbolo kroneckerDelta(n, 0) para denotar δ(n). De esta forma, las transformadas z inversas obtenidas son: f1 (n) = (1/2)n ,

f2 (n) = n,

1 f3 (n) = n − 3n + δ(n). 2

Note que iztrans halla la transformada z inversa de F (z), asumiendo que F (z) es la transformada z unilateral derecha de f (n).

268

8.6. PROBLEMAS RESUELTOS

8.6 Problemas Resueltos Problema 8.1. Determinar la transformada z de f (n) = (1 + n)u(n). Soluci´ on. Por definici´ on, la transformada z de f (n) est´ a dada por: F (z) = = =

∞ X

n=−∞ ∞ X −n

z

n=0 ∞ X

n=0

∞ X

(1 + n)u(n) z −n =

z

(1 + n) z −n

n=0

+

∞ X

n z −n

n=0

 −1 n

+

∞ X

n z −1

n=0

n

.

(8.15)

Ahora bien, se tiene que z z−1 z (z − 1)2

∞ X n 1 = z −1 , |z| > 1, −1 1−z n=0 "∞ #   z d d X −1 n = (−z) = (−z) z dz z − 1 dz n=0 # " ∞ ∞ X X n  −1 n −1 n z −1 , n z = = (−z) (−z )

=

n=0

n=0

|z| > 1.

Usando convenientemente los desarrollos en series de potencias anteriores en la ecuaci´ on (8.15), obtenemos que la transformada z de f (n) es: F (z) =

z z2 z + = , z − 1 (z − 1)2 (z − 1)2

|z| > 1.

Seguidamente se muestra el diagrama de polos (rojo) y ceros (azul) de F (z). y |z| > 1

b

b

1

x

Diagrama de polos y ceros



269

CAP´ITULO 8. TRANSFORMADA Z

Problema 8.2. Determinar la transformada z de f (n) = n 2n cos(2n) u(−n). Soluci´ on. Por definici´ on, la transformada z de f (n) est´ a dada por: F (z) =

∞ X

n

n 2 cos(2n) u(−n) z

−n

=

=

n 2n

n=−∞

=

n 2n cos(2n) z −n

n=−∞

n=−∞ 0 X

0 X



e2nj

+ e−2nj 2



z −n

0 0 1 X 1 X n 2nj −n n2 e z + n 2n e−2nj z −n . 2 n=−∞ 2 n=−∞

Haciendo un cambio de ´ındices k = −n y luego colocando n en lugar de k, la expresi´ on anterior adquiere la forma: F (z) =





1X 1X (−n) 2−n e−2nj z n + (−n) 2−n e2nj z n 2 2

= −

n=0 ∞ X

n=0

1 n 2−1 e−2j z 2 n=0

n



∞ 1X

2 n=0

n 2−1 e2j z

n

.

(8.16)

Sea el siguiente desarrollo de Laurent: ∞

X w n wn , = 2 (1 − w) n=0

|w| < 1.

(8.17)

Tomando w = 2−1 e−2j z en el desarrollo de Laurent (8.17), obtenemos: ∞

X  2−1 e−2j z −1 −2j n n 2 e z , = (1 − 2−1 e−2j z)2 n=0

|z| < 2.

Ahora, tomando w = 2−1 e−2j z en el desarrollo de Laurent (8.17), obtenemos: ∞

X n 2−1 e2j z n 2−1 e2j z , = −1 2j 2 (1 − 2 e z)

|z| < 2.

n=0

Usando convenientemente los desarrollos en serie de potencias anteriores en la ecuaci´ on (8.16), obtenemos que la transformada z de f (n) es: F (z) = − = −

1 2−1 e−2j z 2−1 e2j z 1 − 2 (1 − 2−1 e−2j z)2 2 (1 − 2−1 e2j z)2 z e2j (z − 2 e2j )2



z e2j (z e2j − 2)2

,

|z| < 2. 

270

8.6. PROBLEMAS RESUELTOS

Problema 8.3. Determinar la transformada z de ( 4n − 2n , n ≤ −1, f (n) = n n (1/4) − (1/2) , n ≥ 0. Soluci´ on. La funci´ on f (n) se puede expresar equivalentemente como: f (n) = (4n − 2n )u(−1 − n) + (4−n − 2−n )u(n). As´ı, aplicando la propiedad de linealidad de la transformada z, podemos escribir: F (z) = = =

∞ X

n=−∞ ∞ X

n=−∞ ∞ X

n=−∞

=

−1 X

n=−∞

= =

−1 X

(4n − 2n )u(−1 − n) + (4−n − 2−n )u(n)z −n (4n − 2n )u(−1 − n)z −n + (4n − 2n )u(−1 − n)z −n + (4n − 2n )z −n + n −n

4 z

n=−∞ ∞ X −1

(4

n=1



z)n −

−1 X

∞ X

(4−n − 2−n )u(n)z −n

n=−∞ ∞ X

(4−n − 2−n )u(n)z −n

n=−∞

(4−n − 2−n )z −n

n=0

n −n

2 z

+

n=−∞

∞ X

∞ X

∞ X

−n −n

4

z

n=0 ∞ X

(2−1 z)n +

(4−1 z −1 )n −

n=0

n=1



∞ X

n=0 ∞ X

2−n z −n (2−1 z −1 )n .

n=0

Luego, calculando a que converge cada una de las series de potencias anteriores, obtenemos:         1 1 1 1 F (z) = −1 + −1 + − 1 − 4−1 z 1 − 2−1 z 1 − 4−1 z −1 1 − 2−1 z −1 | {z } | {z } | {z } | {z } |z|<4

|z|<2

|z|>1/4

|z|>1/2

De esta forma, la transformada z de f (n) es:

 3 z −3z 2 + 4z − 3 F (z) = , 4(z − 4)(z − 2)(z − 14 )(z − 21 )

1 < |z| < 2. 2

Seguidamente se muestra el diagrama de polos (rojo) y ceros (azul) de F (z). y 1 2

< |z| < 2

b

b

b

b

1/2

b

b

2

x

b

Diagrama de polos y ceros



271

CAP´ITULO 8. TRANSFORMADA Z

Problema 8.4. Determinar la transformada z inversa de F (z) =

1+z , z − 12

|z| >

1 . 2

Soluci´ on. Inicialmente, hallemos el desarrollo de Laurent de F (z) centrado en z0 = 0 y on F (z) se puede expresar como: v´alido en |z| > 12 . La funci´ F (z) =

1 z−

Hallemos los desarrollos de Laurent de |z| > 12 . Se tiene que: 1 z− z z−

1 2

1 2

= z −1 =

1 2

+

1 z−

z , z − 21 1 2

y

1 |z| > . 2

z , centrados en z0 = 0 y v´alidos en z − 21

∞ ∞ X X 1 −n −n −1 2−n u(n)z −(n+1) , 2 z = = z 1 − 2−1 z −1 n=−∞ n=0

∞ ∞ X X 1 −n −n 2−n u(n)z −n , 2 z = = 1 − 2−1 z −1 n=−∞

1 |z| > , 2

1 |z| > . 2

n=0

As´ı, el desarrollo de Laurent de F (z) centrado en z0 = 0 y v´alido en |z| > 12 , es: ∞ X

F (z) = = =

n=−∞ ∞ X

n=−∞ ∞ X

n=−∞

−n

2

u(n)z

−(n+1)

+

∞ X

2−n u(n)z −n

n=−∞ ∞ X

2(1−n) u(n − 1)z −n +

2−n u(n)z −n

n=−∞

2−n (2 u(n − 1) + u(n))z −n ,

1 |z| > . 2

De esta forma, la transformada z inversa de F (z) es: f (n) = 2−n (2 u(n − 1) + u(n)).  Problema 8.5. Determinar la transformada z inversa de  z z − 21 1  < |z| < 4. F (z) = 1 , 4 (z − 4) z − 4

Soluci´ on. Inicialmente, hallemos el desarrollo de Laurent de F (z) centrado en z0 = 0 y v´alido en |z| > 21 . La expansi´ on en fracciones parciales de F (z)/z est´ a dada por: 14 1 F (z) 15 15  , = + z (z − 4) z − 41

as´ı, F (z) se puede expresar como: F (z) =

z z 1 14 , + 15 (z − 4) 15 z − 41

1 < |z| < 4. 4

272

8.6. PROBLEMAS RESUELTOS

Hallemos los desarrollos de Laurent de 1 4

< |z| < 4. Se tiene que: z z−4



= z = −

z z−

1 4

=

−4−1 1 − 4−1 z

−1 X

n=−∞



z z y , centrados en z0 = 0 y v´alidos en z − 4 z − 14

=−

4n z −n = −

∞ X

4−(n+1) z (n+1)

n=0 ∞ X

n=−∞

4n u(−1 − n)z −n ,

|z| < 4,

∞ ∞ X X 1 −n −n 4−n u(n)z −n , 4 z = = 1 − 4−1 z −1 n=−∞ n=0

1 |z| > . 4

De esta forma, el desarrollo de Laurent de F (z) centrado en z0 = 0 y v´alido en es:

1 4

< |z| < 4,

∞ ∞ 14 X n 1 X −n 4 u(−1 − n)z −n + 4 u(n)z −n 15 n=−∞ 15 n=−∞  ∞  X 1 −n 14 n 1 = 4 u(n) − 4 u(−1 − n) z −n , < |z| < 4. 15 15 4 n=−∞

F (z) = −

Por lo tanto, la transformada z inversa de F (z) es: f (n) =

14 n 1 −n 4 u(n) − 4 u(−1 − n). 15 15 

Problema 8.6. Determinar la transformada z inversa de z 3 + 4 z 2 − 11 F (z) = 3 7 2 3 6 1 , z +6z −2z+3

1 1 < |z| < . 3 2

Soluci´ on. La funci´ on F (z) se puede expresar como F (z) = 1 + F1 (z), donde F1 (z) =

z

17 2 6 z − 31

+ 32 z − 13  6 , z − 12 (z + 2)

1 1 < |z| < . 3 2

As´ı, la transformada z inversa de F (z) est´ a dada por

f (n) = Z −1 {1 + F1 (z)} = Z −1 {1} + Z −1 {F1 (z)} = δ(n) + Z −1 {F1 (z)} .

(8.18)

Determinemos Z −1 {F1 (z)}. Para ello, hallemos primero el desarrollo de Laurent de F1 (z) centrado en z0 = 0 y v´ alido en 13 < |z| < 21 . La expansi´ on en fracciones parciales de F1 (z) est´ a dada por: F1 (z) =

73 21

z−

 1 +

3

73 − 17 10  15 , + (z + 2) z − 12

1 1 < |z| < . 3 2

273

CAP´ITULO 8. TRANSFORMADA Z

Ahora, se tiene que ∞ ∞ X X z −1 1 −n −(n+1) = 3(1−n) u(n)z −n , 3 z = = 1 − 3−1 z −1 n=0 z − 31 n=−∞

1  z − 21

1 (z + 2)

= =

∞ ∞ X X −2 2(1−n) u(−n)z −n , 2(n+1) z n = − =− 1 − 2z n=−∞ n=0 ∞ ∞ X X 2−1 −(n+1) n 2(n−1) u(−n)z −n , 2 z = = 1 + 2−1 z n=0 n=−∞

1 |z| > , 3 1 |z| < , 2 |z| < 2.

As´ı, usando los series de potencias anteriores obtenemos que el desarrollo de Laurent de F1 (z) centrado en z0 = 0 y v´ alido en 13 < |z| < 12 es: F1 (z) =

∞ ∞ 73 X (1−n) 17 X (1−n) 3 u(n)z −n + 2 u(−n)z −n 21 n=−∞ 10 n=−∞

+

∞ 73 X (n−1) 2 u(−n)z −n . 15 n=−∞

Luego, sustituyendo este desarrollo de Laurent en (8.18), tenemos que el desarrollo de Laurent de F (z) centrado en z0 = 0 y v´alido en 13 < |z| < 12 es: F (z) =

∞  X

n=−∞

 17 (1−n) 73 (n−1) 73 (1−n) u(n) + 2 u(−n) + 2 u(−n) z −n . δ(n) + 3 21 10 15

Por lo tanto, la transformada z inversa de F (z) es: f (n) = δ(n) +

73 (1−n) 17 73 3 u(n) + 2(1−n) u(−n) + 2(n−1) u(−n). 21 10 15 

8.7 Problemas Propuestos 8.1. Determinar la transformada z y su regi´ on de convergencia de las siguientes funciones de dominio discreto. a) f (n) = (−1)n 2−n u(n) b) f (n) = (an + a−n )u(n), para |a| > 1 c) f (n) = n 3n sen (n) u(n)

d) f (n) = (−3)n u(−n − 1) ( 1, −5 ≤ n ≤ 5; e) f (n) = 0, |n| > 5 ( (1/3)n , n ≥ 0; f) f (n) = −n (1/2) , n < 0

274

8.7. PROBLEMAS PROPUESTOS

( (1/4)n − 2n , n ≥ 0; g) f (n) = 0, n<0 ( (1/5)n sen (πn/2), n ≤ 0; h) f (n) = 0, n>0 i) f (n) = eαn senh(βn)u(n), α > β > 0. 8.2. La transformada z de una funci´ on causal f (n) (f (n) = 0 para n < 0) es: F (z) =

1 , (z − 21 )3

|z| >

1 . 2

Determinar la transformada z y su regi´ on de convergencia de las siguientes funciones. a) g(n) = 2n f (n − 2)

b) g(n) = nf (n)

c) g(n) = f (−n − 1) − f (−n)

d) g(n) = (n + 1)(1/3)n f (n − 2) 8.3. Calcular la transformada inversa de las siguientes transformadas z mediante el desarrollo en serie de potencias. z+1 1 1 , |z| < 3 z+3 z , |z| > 1 b) F (z) = (z − 1)(z + 1) a) F (z) =

c) F (z) = d) F (z) =

z 3 − 13 z 2 , 1 < |z| < 2 (z − 1)2 (z + 2)

z 3 + 4 z 2 − 11 6 , |z| < 1/3 z 3 + 67 z 2 − 32 z + 31

8.4. Calcular la transformada z inversa de   1 −1 F (z) = Log 1 − z , 3

|z| > 1.

mediante los siguientes m´ etodos: a) Utilizando el desarrollo en serie de potencias Log (1 − w) = −

∞ X wk k=1

k

,

|w| < 1.

b) Derivando F (z) y utilizando las propiedades de la transformada z.

Respuesta a los Problemas Propuestos Cap´ıtulo 1

1.5 a) −165 + i 721/32 b) (−3353 + i 2879)/256 c) (6 + i 43)/5 d) (−323 − i 36)/2500

1.1 a) 6 + i 4 b) (11 + i 10)/17 c) (3 − i 5)/2 d) 5 + i 14 e) −352 + i 936 f) 4 − i 15/2 g) −2/5 h) −4

1.6 a) 5329/32 √ b) √ 3125 32768/32768 c) 1885/5 d) 13/100 1.7 4

1.2 a) z1 = i − 1, z2 = 2 − i b) z1 = 1 + i, z2 = −3 − i

1.8 a) π4 + 2nπ, n = 0, ±1, ±2, . . . b) π3 + 2nπ, n = 0, ±1, ±2, . . . c) − π6 + 2nπ, n = 0, ±1, ±2, . . .

1.3 a) i b) (1 + i)/2 c) i d) −i, o ´ 2+i e) (2 + i)/5 o ´ −i

1.9 a) Cerrado, no acotado b) Conexo, no acotado c) Abierto, conexo, no acotado d) Cerrado, conexo, acotado e) Acotado f) Abierto, conexo, acotado g) Abierto, conexo, no acotado h) Abierto, conexo, no acotado i) Cerrado, no acotado j) Cerrado, conexo, acotado

1

1.4 a) z0 = 2 5 ,  1

z1 = 2 5

1

z2 = −2 5 z3 = −2

1 5

1 5



5 4



√

5 4

√

5 4



1 4

1 4

+

+

1 4

+

1 4

 √ √√ 2 5+5 i , 4

− +



5 4



2





2





2

√√

z4 = −2 − + √ √ √√ b) z0 = − 22+2 + 2−2 2 i , √ √ √√ 2+2 2− 2 i − , z1 = √2 √ √2√ 2− 2 2+2 i z2 = − 2 − , √√ 2 √ √ 2 2+2 i + z3 = 2− 2 2 √ √

√  5− 5 i , 4 √  5− 5 i , 4 5+5 i

4



Cap´ıtulo 2 2.1 a) {z ∈ C : |z| 6= 1} b) {z ∈ C : z 6= ±i} c) {z ∈ C : Re z 6= 0} √ d) {z ∈ C : z 6= i, (−1 ± 2)i} e) {z ∈ C : z 6= −i}√ f) {z ∈ C : z 6= i ± 2} g) {z ∈ C : z 6= 0} h) {z ∈ C : z 6= 0, ±i}



= − 22 3 + 22 i , √ √ √ = 22 3 − 22 i , √ =− √ 2 i, = 2 i, √  √ = 2 23 + 2i ,  √ √ z5 = − 2 23 + 2i

c) z0 z1 z2 z3 z4

2.2 a) u(x, y) = v(x, y) = b) u(x, y) = v(x, y) =

2 x2 −3 x+2 y 2 −5 4 x2 −20 x+4 y 2 +25 , −7 y 4 x2 −20 x+4 y 2 +25 3 2

c) u(x, y) = x − 3x(y + 1) + 2, v(x, y) = y(3x2 − y 2 − 3)

2

d) z0 = 2 3 ,  2 z1 = 2 3 − 21 +  2 z2 = −2 3 21 +

x2 −y 2 x4 +2 y 2 x2 +y 4 , −2xy x4 +2 y 2 x2 +y 4

√  3i , 2 √  3i 2

d) u(x, y) = v(x, y) =

275

2x2 +3x+y+2y 2 −2 4 x2 −4 x+4 y 2 +4 y+2 , −x−5y−2 4 x2 −4 x+4 y 2 +4 y+2

276

RESPUESTA A LOS PROBLEMAS PROPUESTOS

2.17 a) (2n ± 31 )πi, n = 0, ±1, ±2, . . .

2.3 a) z = −i √ b) z = (1 ± 5)/2 c) z = 4/3

b) (2n + 12 )πi, n = 0, ±1, ±2, . . . c) (2n + 12 )πi −

2.4 a) −(3/5) − 4i/5 b) √ 12 − 13 i c) 2(1 + i)/2 d) (7 − 4i)/13 e) 1/2 f) 1/3 g) (1/2) − 2i/3 h) 1 i) −π/2 j) −π/2

ln 3 2 ,

d) i

n = 0, ±1, ±2, . . .

e) ln 3 + (2n + 1)πi, n = 0, ±1, ±2, . . . 2.18 a) e−π/2 b) eπ+i2 ln 2 c) e3π

2

/2

2.19 D = C − {z ∈ C : Re z ≤ 0, Im z = −1} 2.20 D = C − {z ∈ C : Re z = 0, |Im z| ≥ 1}

2.5 1 + i

1

z

+1) 2.21 a) f (z) = e 2 Log (e √ z , ′ z f (z) = e /(2 e + 1)

2.6 a) C − {−3i} b) C c) C − {z : Im z = 0}

b) f (z) = cos(Log (z)), f ′ (z) = −sen (Log (z)) /z

2.7 C 2.9 a) D = {z = x + i y : y = x}, f ′ (z) = 2x

b) D = {z = x + i y : x = −3/2}, f ′ (z) = 2x + 2 c) D = {− 21 , − 21 + i}, f ′ (z) = 2y + i

2.11 a) D = C, f ′ (z) = 2(z + 1) b) D = C − {0}, f ′ (z) = 1 − c) D = C − {3}, f ′ (z) =

8 (z−3)2 3

2

f) D = C \ {z√∈ C : Im z = 0 y Re z ≤ −4 o ´ z = ± 22 (1 − i)}, (z+4) 1 f ′ (z) = (z2 +i)(z+4) − 2zLog (z 2 +i)2 h) D = C, f ′ (z) = ez cos(ez ) 2.13 D = {z = x + iy : xy < 1} 2.14 a) D = R2 , v(x, y) = y 2 − x2 − 3x + c b) D = R2 − {(1, 0)}, −y v(x, y) = x2 −2x+y 2 +1 + c c) D = R2 − {(0, 0)}, 2 2 +1) u(x, y) = x(xx2+y +c +y 2

d) D = R2 − {(−1, −2)}, u(x, y) = (x+1)x+1 2 +(y+2)2 + c

e) f (z) = ecos z Log (i) , f ′ (z) = −Log (i) eLog (i) cos(z) sen (z)

z

+3 z +1 d) D = C − {0, ±i}, f ′ (z) = − 4zz2 (z 2 +1)2  √ 1 1  e) D = C − ± 2 2 ± 2 i , z3 f ′ (z) = (z44+1) 2

z

2

d) f (z) = e(Log (z)) , 2 f ′ (z) = 2 eLog(z) Log (z) /z

f) f (z) = esen z Log (z) ,  f ′ (z) = eLog(z) sin(z) cos(z) Log (z) +

1 z2

g) D = C, f ′ (z) = ez+e

c) f (z) = Log (ez + 1), f ′ (z) = ez /(ez + 1)

2.22 f ′ (z) = log(i) ez ee log(i) , f ′ (πi/2) = 3πe3π/2 /2 ( v ≤ −1/2 2.27 f (D) : u+v ≥1

2.28 a) f (D) = M1√ − M2 , donde M1 : ( |w| ≥ 2/2, u−v ≥0 M2 : u+v ≤0 b) f (D) = M1 − M2 , donde M1 : |w ( − 1 + 2i| ≥ 2, |w| ≤ 1 M2 : |w + 1 + i| ≤ 1 c) f (D) = M1 − M √2 , donde 8, M1 : |w + 3| ≥ ( √ |w − 1| ≤ 2 √ M2 : |w + 1| ≤ 2 d) f (D) = M1 − M2 , donde √ M1 : ( |w + 1 − 2i| ≤ 2, |w| ≥ 1 M2 : |w − 1 − i| ≥ 1

sen(z) z



277

RESPUESTA A LOS PROBLEMAS PROPUESTOS

Cap´ıtulo 3 3.1 a) |z| < 1

g) |z| < e

b) |z| < ∞

h) |z| < e−1

d) |z| < 2

j) |z − 1| > 2

i) |z| < |a|

c) z = 0

e) |z + i| < 1

k) |z + 2| < 2

f) |z| < 1

3.2 a)

∞ P

n=0 ∞ P

b) −

c)

l) |z| < 1

(−1)n (z−2i)n , (2i)n+1 (−1)n (2i)n (z−2i)n+1 ,

n=0 ∞ P

−1

|z − 2i| > 2

(z − 2)

n=0 ∞ P

, |z − 2| > 1

d)

n+1

(−1)

n=1 ∞ P

nz

2n−2

h) z0 = 0, polo de orden 2

Cap´ıtulo 4 4.1 a) − πi 2 b) πi

, |z| < 1

(−1)n (n + 1)z −2(n+2) , |z| > 1

e)

n=1 ∞  P

n=0 ∞ P

f)

n=0 ∞ P

n=0 ∞ P

n=0 ∞ P

n=0

g) h)

∞ P

n=0 ∞ P

n=0 ∞ P

n=0

i)

∞ P

n=0

j)

∞ P

n=0 ∞ P

n=0

1

1

3n+1



(z+1)n 3n+1

+

2n+1 ∞ P

n=0

3n −2n (z+1)n+1 , (−1)n 4n+1 (z



n

(z + 1) , |z + 1| < 2

2n (z+1)n+1 ,

2 < |z+1| < 3

1 8(z−1)

+

1 2(z−1)2 ,

0 < |z − 1| < 4 (−1)n 4n (z−1)n+1

+

1 8(z−1)

+

1 2(z−1)2 ,

|z − 1| > 4 n

(−1) (2n)!

+

n=0 ∞ P

6(−1)n (2n)! (z−2)2n−2

+





e

−2i

n 2i

−(−1) e 2i

3 n! (z−1)n

+

1 n! (z−1)n−2 ,

∞ P

∞ P

12(−1)n (2n)! (z−2)2n−1 +

n=0

n=0

4.3 2 − πi   √ 4.4 8 32+4 i

4.5 a) C no debe contener ni confinar el punto z = −2

(−1)n (2n)! (z−2)2n−3 ,

|z − 2| > 0 n

(2i) (z+1)n−3 , n!

|z + 1| > 0

3 n! (z−1)n−1 +

|z − 1| > 0

c) C no debe contener ni confinar los ceros de la funci´ on f (z) = 1 + ez d) C no debe contener ni confinar los ceros de la funci´ on f (z) = 1 − ez

1 4.6 a) F (z) = − z−i +c

1 b) F (z) = − z−i +1

z 2(n−1) , |z| > 0

8(−1)n (2n)! (z−2)2n

4.2 0

b) C no debe intersectar con el eje real negativo ni el cero

|z + 1| > 3

− 1)n +

= −1, polo simple = 1, polo simple √ 2 = 2 (1 + i), polo simple √ = − 22 (1 + i), polo simple

g) z0 = 0, punto singular removible

(−1)n z −2(n+1) , |z| > 1

n=0 ∞ P

= −1, polo simple = 1, punto singular removible √ 2 = 2 (1 + i), polo simple √ = − 22 (1 + i), polo simple

f) z0 = 0, punto singular esencial

(−1)n z 2n , |z| < 1

n=0 ∞ P

c) z0 z1 z2 z3

e) z0 = 0, polo de orden 2

(z − 2)n , |z − 2| < 1 −(n+1)

b) z0 = −2, polo de orden 2 z1 = 3, polo simple

d) z0 z1 z2 z3

|z − 2i| < 2

n=0

∞ P

3.3 a) z0 = 0, polo de orden 2 z1 = 2 + i, polo simple z2 = 2 − i, polo simple

4.7 a) 0 b) 2 e−2 πi c)

sen (1)(e−sen (1)) 4 sen (1)(cos(1)+e) 16 21

d) 20! 2

πi

e) 2πi sen (a)/a  f) ea 1 + a2

4.8 a) 2π

b) π/3 c) 4π

+

cos(1)(cos(1)+e) 4



278

RESPUESTA A LOS PROBLEMAS PROPUESTOS

√ d) 6 5

d) f4 (t)

4.9 a) Res [f (z)] = 1/3, Res [f (z)] = 5/3 z=−1

z=2

1

b) Res [f (z)] = 1, Res [f (z)] = −1 z=0

z=1

c) Res [f (z)] = 1/2, Res [f (z)] = 54 (1− 3i ), z=1

Res [f (z)] =

z=−3i

t

a

z=3i 5 (1 + 3i ) 4

d) Res [f (z)] = 1/4, Res [f (z)] = 83/4 z=0

z=−4

e) Res [f (z)] = −1/8, Res [f (z)] = 0 z=−3

e)

z=0

f5 (t)

f) Res [f (z)] = 1 z=0

g) h)

Res [f (z)] =

z=eπi/4

1 √ 2 4 (1

Res [f (z)] = −sen

z=eπi/3

i) Res [f (z)] = −π

+ i)  √  1+ 3i 2

··· √  3+ 3 12

··· −3π 2

−π 2



π 2

3π 2

π

t

z=π

4.10 a) i) 0, ii) 2πi b) i) π3 (−2 − 4i 3 ), ii) 2πi 6π √ √ c) i) 0, ii) (2 6−1)( , iii) 0 6+1)

f) f6 (t) 1

d) i) 0, ii) 0 e) i) 2πi, ii) 6πi π (−14 + 2i) f) i) 0, ii) 25 g) i)

19πi 108 ,

−(

2 2

+ 1)

(



2 2

t

− 1)

ii) − 19πi 108 g)

Cap´ıtulo 5 5.1 a)



f7 (t) 4

f1 (t)

3

4

2

3

1 1/2

2 1

1 2

1

2

3

1

3 2

2

t

t

b) f2 (t)

h)

f8 (t)

1 4 1

2

3

t

3

c)

2

f3 (t)

1

1

a

t

-5 - 9 -4 2

-3

-2 - 3 -1 2

1

3 2

2

3 t

279

RESPUESTA A LOS PROBLEMAS PROPUESTOS

5.2 a) 0

b)

g(n)

b) 12

2

c) 1/2

1

b

b

d) −9

1

5.3 a) h(t) = pa (t − (a + b))

c)

b) No existe c) h(t) =

1 2 2t

u(t) −

1 2

(t − 1) u(t − 1)

f) h(t) = (1 − (t + 1)e−t ) u(t)  g) h(t) = 12 3 − e−2t u(t)

n

-4 -1

b

b

-2

g(n) b

1

2

3

b

-2

e)

g(n) 2 b

b

b

b

b

···

b

1

b

b

b

b

-6 -5 -4 -3 -2 -1

1

f)

b

b

··· b

2

3

4

5

6

b

b

b

g(n) b

b

b

b

···

1 b

b

b

-6 -5 -4 -3 -2 -1 5 P

k=1

u(n − k) −

b

b

b

1

2

3

4 P

··· 4

5

2

3

4

5 b

-1 b

n

6

n

u(n + k)

k=0

b) h(n) = (4(1/2)n − 2(1/3)n ) u(n) + (1/2)n−1 u(n − 1)

b

n

2

5.6 a) h(n) = b

n

b

-1

g(n)

b

1

1

c) h(t) = 2(t + 2)u(t + 2) − 2(t + 1) u(t + 1) − 2t u(t)+ 2(t − 1)u(t − 1) + p1 (t − 2)  0, t ≤ −1,      2  − (t+1) (t−2) , −1 < t ≤ 0,   6   2 d) h(t) = (t−1) (t+2)−3t(t−2) , 0 < t ≤ 1, 6    2  (t−2)   , 1 < t < 2,  2     0, t≥2

-2

-12

-16

2

b) h(t) = −(t + 1)u(t + 2) + 2(t + 1) u(t + 1) − 2 u(t)− 2(t − 1)u(t − 1) + (t − 1) u(t − 2)

1

b

-20

5.4 a) h(t) = −(t + 2)u(t + 2) + 2(t + 1) u(t + 1) − 2(t − 1)u(t − 1) + (t − 2) u(t − 2)

1

2

b

d)

h) h(t) = e−2(t−1) u(t − 1) + e−2t (et − 1) u(t)

2

g(n) b

d) h(t) = −(t + 1)u(t + 1) + 2t u(t) −(t − 1)u(t − 1)  e) h(t) = u(−t) + 2e−t − e−2t u(t)

5.5 a)

n

2

c) h(n) = (n + 3) (u(n + 2) − u(n − 1)) + (3 − n) (u(n − 1) − u(n − 3)) d) No existe 3 P (n − k)u(n − k) e) h(n) = k=−2

280

RESPUESTA A LOS PROBLEMAS PROPUESTOS

f) h(n) = (n − 1) (u(n + 1) − u(n − 4)) + (n − 2) (u(n) − u(n − 5)) + 2 P k u(k − n) k=−2 2 P

g) h(n) =

k=−2

k

(−1) 2k+2

[δ(n − k)−

δ(n − (k + 1)) + δ(n − (k + 2)) −δ(n − (k + 3))]

h) h(n) =

5 4 P P

k=0 ℓ=0

(−1)ℓ ℓ+1

δ(n − (k + ℓ))

m) n) 6.5 a) b) c) d)

Cap´ıtulo 6 6.1 a) F (ω) =

1 2

6.6 a)

1 − jω

2 ejω sen ω b) F (ω) = ω 16 sen 2 (ω/8) c) F (ω) = ω2 8 d) F (ω) = + 8j πδ ′ (ω) jω e) F (ω) = πj δ ′ (ω) −

1 ω2

f) F (ω) = 2π e−jωt0

g) F (ω) = 2π cos(πω)   jω jω e 2 2 − 2 e 2 + jω h) F (ω) = 2 ω2 2a i) F (ω) = 2 a + ω2 j) F (ω) = 2π e−a|ω| 1 k) F (ω) = + 2πδ(ω) ω 2 sen ω 8j e−jω sen 2 (ω/4) + l) F (ω) = ω ω2 6.3 a) G1 (ω) = p1 ((−ω − 1)/2) b) G2 (ω) = j [δ(ω + 1) − δ(ω − 3)] c) G3 (ω) = ejω p1 ((ω − 1)/2)

d) G4 (ω) =

1 5

e−3jω/5 p1 (−(ω + 5)/10)

e) G5 (ω) = −2 ejω p1 ((ω − 1)/2) + j(δ(ω + 1) − δ(ω − 3)) f) G6 (ω) = jω p1 ((ω − 1)/2)

g) G7 (ω) = −p1 ((ω − 1)/2) + δ(ω + 1) + 3δ(ω − 3) h) G8 (ω) =

1 2

e−j(ω+2)/2 p1 (ω/4)

p1 ((ω − 1)/2) + πδ(ω) jω   − p1 ω−π−1 G13 (ω) = 2j p1 ω+π−1 2 2  G14 (ω) = e−jω p1 ω−1 2   1 1 f (t) = 2π πδ(t) − 2 − jt  1 −2 f (t) = 4π t (1 − cos(2t))   4 sen t 1 f (t) = 2π 1 + t 1 f (t) = πt √ A(ω) = 2/ 1 + 4ω 2 , para ω ∈ R φ(ω) = arctan(2ω), para ω ∈ R

l) G12 (ω) =

b) A(ω) = 2|sen ω|/|ω|, para ω 6= 0   ∞ P , (ω−kπ)pπ/2 ω − (2k+1)π φ(ω) = 2 k=0

para ω > 0 16 sen 2 (ω/8) c) A(ω) = , para ω 6= 0 ω2 φ(ω) = 0 1 d) A(ω) = 2 , para ω 6= 0 ω φ(ω) = π, para ω 6= 0

e) A(ω) =  2|sen ω|/|ω|, para ω 6= 0 −2ω, 0 < ω ≤ π2 ,  φ(ω) = 2π − 2ω, π2 < ω ≤ π   3π − 2ω, π < ω ≤ 2π (peri´ odica, con periodo 2π, para ω > 0) 1 f) A(ω) = 2 , para ω 6= 0 ω φ(ω) = π, para ω 6= 0 8 g) A(ω) = , para ω 6= 0 |ω| π φ(ω) = − 2 u(−ω) + π2 u(ω)

Cap´ıtulo 7 (s + 1)e−s + s , Re s < −1 s+1 1 − e−5(s−a) , Re s > a F (s) = s−a 4 F (s) = 2 , Re s > 4 s − 8s + 32 2(s − 1) , −3 < Re s < 5 F (s) = (s + 3)(s − 3) No existe

7.1 a) F (s) = b) c) d)

i) G9 (ω) = p1 ((ω + 1)/2)

e)

j) G10 (ω) = j(δ(ω + 3) − δ(ω − 1))

f) No existe s−1 g) F (s) = 2 , Re s > 0 s

k) G11 (ω) = j(ej δ(ω + 3) − e−3j δ(ω − 1))

281

RESPUESTA A LOS PROBLEMAS PROPUESTOS

n! , Re s < −1 (s + 1)n+1 s i) F (s) = 2 , Re s > 0 s + a2 a , Re s < −a j) F (s) = 2 a − s2

h) F (s) = −

1 − (1 + 3s)e−3s , Re s > 0 s2 1 − (1 + 3s)e−3s 1 + , Re s > 0 b) F2 (s) = s2 2 1 − e−2s c) F3 (s) = , Re s > 0 s 2 2e−3s/2 es/2 − 1 , Re s > 0 d) F4 (s) = s2

7.2 a) F1 (s) =

7.3 a) G(s) =

18e−6s , Re s > − 13 (s + 31 )2

b) G(s) =

4e−2s (s + 2) , Re s > −1 (s + 1)3

c) G(s) =

2e−3s (3s + 5) , Re s > −1 (s + 1)3

d) G(s) =

2s e−2s , Re s > −1 (s + 1)2

e−2s (4s2 + 6s − 2) , Re s > −1 (s + 1)3 ( 0, a < 0, f) G(s) = s∈C −as f (a)e , a ≥ 0,  2e−3s s2 es + s(2 + es ) + 4 g) G(s) = , (s + 1)3 Re s > −1 2e−2s h) G(s) = , Re s > 0 s(s + 1)3 e) G(s) =

7.4 a) f (0+ ) = 1, l´ım f (t) = 0 t→∞

b) f (0+ ) = 0, l´ım f (t) = 0 t→∞

+

c) f (0 ) = 0, l´ım f (t) = t→∞

6 25

+

d) f (0 ) = ∞, l´ım f (t) = 0 t→∞

e) f (0+ ) = 0, l´ım f (t) = 0 t→∞

+

1 e) f (t) = − 24 1 4 −t

f) f (t) = e cos(t) u(t) − cos(2t) u(−t)    7.6 f (t) = α9 et u(−t) − 20e−2t − 18e−t u(t)

Cap´ıtulo 8

8.1 a) F (z) =

b) F (z) =

d) f (t) = δ(t) + (2 + et ) u(t)

(z − a)(z − a1 )

, |z| > |a|

3 senh(1) z (z 2 − 9) , |z| > 1 (z 2 − 6 cos(1) z + 9)2 3 , |z| < 3 d) F (z) = z+3 z 11 − 1 , |z| > 1 e) F (z) = 5 z (z − 1) f) F (z) =

− 35 z , (z − 13 )(z − 2)

1 3

< |z| < 2

− 47 z , |z| > 2 (z − 14 )(z − 2) −5 z , |z| < 15 h) F (z) = 25z 2 + 1 eα senh(β) z , i) F (z) = 2 z − 2eα cosh(β)z + e2α |z| > eα+β g) F (z) =

16 , |z| > 1 z 2 (z − 1) 2z b) G(z) = , |z| > 12 (z − 21 )3

8.2 a) G(z) =

4z 2 (z − 1) , |z| < 2 (z − 2)2 10z − 1 , |z| > d) G(z) = 162 z 2(z − 61 ) c) G(z) =

1 6

8.3 a) f (n) = δ(n) + 2(−1)n 3n u(n) b) f (n) = c) f (n) =

t→∞

b) f (t) = sen (at) u(t)   c) f (t) = 41 3e−t u(t) − e3t u(−t)

z , |z| > 21 z + 21   2  +1 2z z − a 2a

c) F (z) =

f) f (0 ) = 1, l´ım f (t) = 0 7.5 a) f (t) = sen (−t) u(−t)

 + 5t e−2t u(t) −  5 t 13 1 2 2 t − t − 4 + 9 e u(−t) 37 6

1 2

(1 − (−1)n ) u(n) u(n) +

14 27

2n u(−n − 1)

d) f (n) = δ(n) + 17 5

8.4 f (n) = −

2 9

13 27

37 70

n u(n) −

(−1)n 2n u(−n) +

2−n u(−n) −

3−n u(n − 1) n

73 7

3−n u(−n)

A Introducci´ on a M ATLAB M ATLAB es un sistema interactivo para el c´ alculo num´ erico. A finales de 1970, el analista num´ erico Cleve Moler escribi´ o la versi´ on inicial de M ATLAB en Fortran como un material did´ actico. Se hizo popular para la ense˜ nanza y la investigaci´ on y se convirti´ o en un paquete de software comercial escrito en C. Desde hace muchos a˜ nos, M ATLAB ha sido ampliamente utilizado en las universidades y la industria. Presentamos brevemente la sintaxis y los procedimientos b´asicos de programaci´ on en M ATLAB . Para un estudio m´ as extenso de M ATLAB se recomienda la referencia [4].

A.1 Escritorio de M ATLAB Por ser M ATLAB un sistema iterativo, al escribir comandos en el prompt (>>) en la ventana de comandos (Command Window) del escritorio de trabajo de M ATLAB (ver Fialculos se realizan a cabo cuando se pulsa la tecla Intro o Retorno . gura A.1), los c´ 

Figura A.1. Escritorio de trabajo de M ATLAB En su nivel m´ as simple, M ATLAB se puede utilizar como una calculadora de bolsillo: ✄ >> (1+ s q r t ( 2 ) ) / 2 ans = 1.2071 >> 2ˆ(−87) ans = 6.4623e−27







El primer ejemplo calcula (1 + 2)/2 y el segundo 2−87 . Observe que el segundo resultado se muestra en notaci´ on exponencial: esto representa 6.4623 × 10−27 . La variable ans se crea autom´aticamente (o se reescribe, si ya existe) cuando una expresi´ on no se asigna a 282

283

´ A MATLAB ´NDICE A. INTRODUCCION APE

una variable, y se puede hacer referencia de ella m´ as adelante, al igual que cualquier otra variable. En la ventana de comandos de M ATLAB tambi´ en se pueden escribir distintas operaciones: suma de n´ umeros, vectores o matrices, multiplicaci´ on de n´ umeros, vectores o matrices, entre otras. Al escribir en M ATLAB se debe tener presente lo siguiente: • M ATLAB distingue entre may´ usculas y min´ usculas, por ejemplo, A y a son variables distintas. • Es posible escribir varias instrucciones en la misma l´ınea, separ´ andolas por una coma o por punto y coma. • Se pueden recuperar comandos anteriores, usando las teclas de flechas: arriba ↑ y abajo ↓ .

• Con las flechas izquierda ← y derecha → nos podemos desplazar sobre la l´ınea de comando y modificarla. En M ATLAB tambi´ en podemos crear secuencias de comandos relacionados. Para ello se utiliza un M-archivo (M-file). M´as detalles de la creaci´ on y uso de los M-archivos se encuentra en la Secci´ on A.4. M ATLAB almacena n´ umeros y calcula con una precisi´ on relativa de aproximadamente 16 d´ıgitos decimales. Por defecto se muestran los n´ umeros en un formato de 5-d´ıgitos fijos. Si bien este formato es conciso, ´este no siempre es el m´ as u ´ til. El comando format se puede utilizar para establecer un formato de punto flotante de 5 d´ıgitos (tambi´en conocido como notaci´ on cient´ıfica o exponencial): ✄ >> 1/3 ans = 0.3333 >> format s h o r t e >> ans ans = 3.3333e−01





El formato por defecto puede ser reintegrado escribiendo format short o simplemente format. El comando format tiene muchas opciones, que se pueden ver escribiendo help format. Consulte la Tabla A.1 para algunos ejemplos. Tabla A.1. 10π representado en varios formatos de salida format format format format format format format format format

short long short e long e short g long g hex bank rat

31.4159 31.415926535897931 3.1416e+01 3.141592653589793e+01 31.416 31.4159265358979 403f6a7a2955385e 31.42 3550/113

Todas las salidas de M ATLAB que se muestran en estas notas se han generado con format compact, que suprime las l´ıneas en blanco. En general, help foi muestra informaci´ on sobre el comando foi.

Pruebe con help help

284

A.2. OBJETOS Y SINTAXIS

A.2 Objetos y Sintaxis El objeto b´asico de trabajo de M ATLAB es una matriz bidimensional cuyos elementos son n´ umeros reales o complejos. Los escalares y vectores son matrices particulares, por ejemplo, un n´ umero se considera una matriz 1 × 1 y un vector de n componentes, dependiendo de la definici´ on dada por el usuario, se puede considerar una matriz n × 1 (matriz columna de n filas) o una matriz 1 × n (matriz fila de n columnas). Tambi´ en se pueden construir cadenas de caracteres, booleanas, enteras y de polinomios. Como M ATLAB considera a todos estas estructuras de datos una matriz, define sus operaciones aritm´eticas a trav´ es del as detalles de la manipulaci´ on de los algebra de las matrices. En la Secci´ on A.3 daremos m´ objetos de M ATLAB . Constantes Algunas constantes num´ ericas muy particulares y de gran importancia en el c´ alculo cient´ıfico tambi´ en tienen una representaci´ on espec´ıfica. A continuaci´ on damos una lista de algunas constantes predefinidas: • • • •

pi es el n´ umero π, e es el n´ umero e, i es la unidad imaginaria, eps es la precisi´ on de la m´ aquina (mayor n´ umero real en doble precisi´ on para el que 1+eps/2 es indistinguible de 1; aproximadamente se tiene que eps ≈ 2.220×10−16 ), • inf es el “infinito m´ aquina” (overflow: cualquier n´ umero que supere al mayor n´ umero real representable en doble precisi´ on), • nan es el s´ımbolo NaN (Not a Number) para una operaci´ on inv´ alida (por ejemplo, 0/0 es nan).

Operadores Aritm´ eticos, L´ ogicos y de Comparaci´ on

Ejecute (5-3i)+i en el promtp y observe el resultado.

Adem´ as de la aritm´etica real, M ATLAB tambi´ en puede trabajar con aritm´etica compleja. Un n´ umero complejo en M ATLAB est´ a representado por a+bi, donde a y b son n´ umeros reales. La aritm´etica compleja tiene la misma sintaxis que la aritm´etica real. Las siguientes tablas muestran una lista de los s´ımbolos que denotan los operadores aritm´eticos, l´ ogicos y de comparaci´ on usados en la sintaxis de M ATLAB . Tabla A.2. Operaciones aritm´eticas Operaci´ on Suma Resta Multiplicaci´ on Divisi´ on Potenciaci´ on Producto punto de vectores o matrices (producto de Hadamard) Divisi´ on punto de vectores o matrices Elevar a una potencia un vector o matriz

S´ımbolo + * / ^ .* ./ .^

285

´ A MATLAB ´NDICE A. INTRODUCCION APE

Tabla A.3. Operadores l´ ogicos Operador y o no

S´ımbolo & | ∼

Tabla A.4. Operadores de comparaci´ on Operador igual que no igual que menor que mayor que menor o igual que mayor o igual que

S´ımbolo == ∼= < > <= >=

Variables En M ATLAB no hace falta declarar las variables tal como se hace en C++ o Visual Basic, entre otros lenguajes de programaci´ on. El tipo de variable y su tama˜ no cambian de forma din´amica de acuerdo con los valores que le son asignados. Por ello, una misma variable puede ser utilizada, por ejemplo, para almacenar un n´ umero, despu´es puede ser una matriz de n´ umeros enteros con 10 filas y 4 columnas, y luego puede ser una cadena de caracteres. Las variables se crean autom´aticamente al asignarles un valor. Tambi´ en existe la posibilidad de eliminar una variable de la memoria si ya no se utiliza. Para asignar un valor a una variable se utiliza el s´ımbolo “=”, observe el siguiente ejemplo.pPrimero creamos una variable a asign´ andole el valor de 5, luego realizamos la operaci´ on log(a3.3 + 4). ✄

>> a=5 a = 5 >> s q r t ( l o g ( a ˆ 3 . 3 + 4 ) ) ans = 2.3088





El resultado de la operaci´ on sqrt(log(aˆ3.3 + 4)) se guarda en la variable ans. Ahora, si colocamos b=sqrt(log(aˆ3.3 + 4)), M ATLAB crea la variable b con el valor obtenido en la operaci´ on, como se puede apreciar seguidamente. ✄ >> b=s q r t ( l o g ( a ˆ 3 . 3 + 4 ) ) b = 2.3088





Para conocer en cualquier instante el valor almacenado en una variable basta con escribir su nombre. Tambi´ en M ATLAB cuenta con algunos comandos que permiten listar o eliminar las variables definidas. Tales comandos se muestran en la Tabla A.5.

286

A.2. OBJETOS Y SINTAXIS

Tabla A.5. Comandos para listar y eliminar variables Comando who whos clear clear a b c

Descripci´ on lista las variables actuales lista en forma detallada las variables actuales elimina todas las variables que existan en ese momento elimina las variables a, b y c (las variables no se separan con comas)

Seguidamente se muestra el uso de los comandos de la Tabla A.5, con las variables a y b creadas en los ejemplos anteriores. ✄ >> who Your v a r i a b l e s a r e : a b >> whos Name Size a 1x1 b 1x1 >> c l e a r b >> whos Name Size a 1x1



Bytes 8 8

Class dou ble dou ble

Attributes

Bytes 8

Class dou ble

Attributes



Funciones Elementales M ATLAB est´ a provisto de un gran n´ umero de funciones elementales. Algunas de estas funciones se pueden apreciar en la Tabla A.6. Los argumentos de las funciones elementales pueden ser (siempre que tenga sentido) reales o complejos y el resultado se devuelve en el mismo tipo del argumento. En los siguientes ejemplos mostramos como se utilizan las funciones elementales en M ATLAB . La sintaxis de M ATLAB es muy sencilla. Observe el siguiente ejemplo: ✄ >> c o s ( p i ) ans = −1 >> c o s (1+ p i ∗ i ) ans = 6.2632 − 9.7179 i





Con el comando cos(pi) se calcula cos(π) = −1 y con cos(1+pi*i) se calcula cos(1 + πi) que es n´ umero complejo. Ahora calculemos exp(i*pi). ✄ >> exp ( i ∗ p i ) ans = −1.0000 + 0.0000 i

✂ ¿Qu´e observa?



287

´ A MATLAB ´NDICE A. INTRODUCCION APE

Tabla A.6. Funciones elementales Comando exp(x) log(x) log10(x) sqrt(x) abs(x) conj(z) real(z) imag(x) rat(x) mod(x,y) floor(x) ceil(x) round(x) sin(x) cos(x) tan(x) cotg(x) asin(x) acos(x) atan(x) sinh(x) cosh(x) tanh(x) asinh(x) acosh(x) atanh(x)

Nombre de la funci´ on exponencial logaritmo natural logaritmo en base 10 ra´ız cuadrada valor absoluto o m´ odulo conjugado parte real parte imaginaria aproximaci´ on racional resto de dividir x entre y entero n tal que (n-1) <= x <= n entero n tal que n <= x < (n+1) redondeo (hacia el entero m´as cercano) seno (radianes) coseno (radianes) tangente (radianes) cotangente (radianes) arcoseno arcocoseno arcotangente seno hiperb´ olico coseno hiperb´ olico tangente hiperb´ olica arcoseno hiperb´ olico arcocoseno hiperb´ olico arcotangente hiperb´ olica

Utilidades En la Tabla A.7 se muestran algunos comandos utilitarios disponibles.

Tabla A.7. Comandos utilitarios Comando ls dir pwd cd clc date cputime

Descripci´ on lista los archivos del directorio actual lista los archivos del directorio (formato distinto a ls) devuelve el path (direcci´ on) del directorio actual permite cambiar el directorio actual limpia la ventana de trabajo muestra la fecha actual tiempo de CPU en segundos

288

A.3. MATRICES

A.3 Matrices Como mencionamos al comienzo de este anexo, M ATLAB tiene la propiedad que sus objetos b´asicos son matrices, lo cual permite una manera muy f´acil e intuitiva de trabajar con vectores y matrices. Generaci´ on de Matrices Muchas matrices elementales pueden ser construidas directamente con funciones de M ATLAB . La matriz de ceros, la matriz de unos y la matriz identidad, se generan utilizando las funciones zeros, ones y eye, respectivamente. Por ejemplo, ones(m,n) produce una matriz m × n de unos, mientras que ones(n) produce una matriz n × n de unos. Ejemplos: ✄

>> z e r o s ( 2 ) ans = 0 0 0 0 >> ones ( 3 , 2 ) ans = 1 1 1 1 1 1 >> eye ( 3 , 3 ) ans = 1 0 0 1 0 0

0 0 1





Otras dos funciones muy importantes para crear matrices son rand y randn, que generan matrices de n´ umeros aleatorios con la misma sintaxis que eye. La funci´ on rand produce matrices con elementos distribuidos uniformemente en el intervalo (0, 1). La funci´ on randn produce matrices cuyos elementos siguen una distribuci´ on normal con media 0 y varianza 1 o simplemente distribuci´ on normal est´ andar. Cuando ambas funciones se utilizan sin argumentos, producen un u ´ nico n´ umero aleatorio. ✄ >> rand ans = 0.8147 >> rand ( 4 ) ans = 0.9058 0.1270 0.9134 0.6324

0.0975 0.2785 0.5469 0.9575



0.9649 0.1576 0.9706 0.9572

0.4854 0.8003 0.1419 0.4218



En M ATLAB tambi´ en se pueden construir matrices escribiendo uno a uno sus elementos, por filas, entre corchetes rectos ([ ]). Por ejemplo, una matriz 3 × 3 con los 9 primeros n´ umeros primos puede construirse con el siguiente comando: ✄ >> A = [2 3 5 7 11 13 17 19 23] A = 2 3 7 11 17 19



5 13 23



289

´ A MATLAB ´NDICE A. INTRODUCCION APE

El final de una fila puede ser especificado por un punto y coma en lugar de un retorno de carro, por lo que un comando m´ as compacto con el mismo efecto es ✄ >> A = [2 3 5 ; 7 11 13; 17 19 23] ✂



Dentro de una fila los elementos pueden estar separados por espacios o por comas. En el primer caso, si los n´ umeros se especifican con un signo m´ as o menos, se debe tener cuidado de no dejar un espacio despu´es del signo, de lo contrario M ATLAB interpretar´a el signo como una adici´ on u operador de resta. Como ilustraci´ on veamos esto con vectores: ✄ >> v = [0 9 −6 5 −7] v = 0 9 −6 5 >> u = [0 ,9 , −6 ,5 , −7] u = 0 9 −6 5 >> w = [0 9 − 6 5 −7] w= 0 3 5 −7

−7

−7





Las matrices tambi´ en pueden construirse por bloques. Dada una matriz B=[1,4;3,5], se puede crear ✄ >> A = [ eye ( 2 ) ones ( 2 ) z e r o s ( 2 ) B] A = 1 0 1 0 1 1 0 0 1 0 0 3

1 1 4 5





Esto tambi´ en se puede utilizar para generar nuevos vectores, utilizando otros previamente creados: ✄ >> u1 = [1 2 3 4] u1 = 1 2 3 4 >> u2 = [ u1 , 5 , 0 , 0 , 7 , 8 , 9 ] u2 = 1 2 3 4

5

0



0

7

8

9



Sub´ındices y la Notaci´ on de Dos Puntos Para permitir el acceso y la asignaci´ on de submatrices, M ATLAB posee una poderosa notaci´ on basada en el car´ acter dos puntos (:). Los dos puntos se utilizan para definir vectores que pueden actuar como sub´ındices. Para enteros i y j, i:j denota el vector fila de n´ umeros enteros de i hasta j (en pasos de 1). Un paso no unitario (o paso) s se especifica como i:s:j. Esta notaci´ on es v´alida incluso para n´ umeros i, j y s no enteros. Veamos unos ejemplos: ✄ >> 1:6 ans = 1 2 >> 5: −1:1 ans = 5 4 >> 0 : 0 . 2 5 : 1 . 2 5

3

4

5

3

2

1

6

290

A.3. MATRICES

ans = 0

0.2500

0.5000

0.7500



1.0000

1.2500



Un elemento simple de una matriz A se accede con A(i,j), donde i≥1 y j≥1 (M ATLAB no soporta sub´ındices negativos o cero). La submatriz que comprende la intersecci´ on de las filas p hasta q y las columnas r hasta s, se denota por A(p:q,r:s). Ahora, dos puntos solos indicando la fila o columna, comprende todos los elementos de esa fila o columna, as´ı A(:,j) es la j-´esima columna de A y A(i,:) es la i-´esima fila. He aqu´ı algunos ejemplos: ✄ >> A = [1 2 3;4 5 6;7 8 9] A = 1 2 3 4 5 6 7 8 9 >> A( 2 , 2 ) ans = 5 >> A ( 2 : 3 , 2 : 3 ) ans = 5 6 8 9 >> A ( : , 2 ) ans = 2 5 8





Un caso particular es A(:), que denota un vector que contiene todos los elementos de A tomados de la primera columna hasta la u ´ ltima: ✄ >> B=A ( : ) B = 1 4 7 2 5 8 3 6 9





Cuando A(:) se coloca en el lado izquierdo de una instrucci´ on de asignaci´ on, ´este llena la matriz A preservando su forma. Usando este hecho la matriz 3 × 3 de los primeros 9 primos se puede construir como: ✄ >> A=z e r o s ( 3 ) ; A(:)= primes ( 2 3 ) ; A=A ’ A = 2 3 5 7 11 13 17 19 23

✂ ¿Qu´e uso tiene el punto y coma? Utilice help primes para saber acerca del comando primes



Con el comando A’ se obtiene la matriz traspuesta de A, de la cual daremos detalles m´ as adelante. En ciertas circunstancias, cuando el lado derecho es un n´ umero escalar, el n´ umero de elementos en una asignaci´ on de sub´ındices puede ser de diferente medida en los dos lados

291

´ A MATLAB ´NDICE A. INTRODUCCION APE

de la asignaci´ on. En este caso el escalar es expandido para que coincida con el n´ umero de elementos de la izquierda: ✄ >> A = ones ( 4 ) ; >> A ( 1 : 2 , 1 : 2 ) = 0 A = 0 0 1 0 0 1 1 1 1 1 1 1

1 1 1 1





La notaci´ on [] denota una matriz vac´ıa. Asignando [] a una fila o columna es una manera de eliminar esa fila o columna de una matriz: ✄ >> A = [1 2 3;4 5 6;7 8 9 ] ; >> A ( 1 , : ) = [ ] A = 4 5 6 7 8 9





Operaciones de Matrices Para las operaciones aritm´eticas de matrices se utilizan los operadores aritm´eticos ya definidos, siempre que est´ e bien definida la operaci´ on. Cuando los operadores aritm´eticos van precedidos de un punto significa que la operaci´ on se efect´ ua “elemento a elemento”. En la Tabla A.8 se resumen las operaciones aritm´eticas de matrices. Aqu´ı, A y B son matrices de elementos aij y bij , respectivamente, y k es un escalar. Tabla A.8. Operaciones aritm´eticas de matrices Operaci´ on A±B A*B Aˆk A±k k*A A/k = (1/k)*A A.ˆk k.ˆA k./A A.*B A./B A.ˆB

Descripci´ on matriz de elementos aij ± bij producto matricial de A y B matriz A elevada a la potencia k matriz de elementos aij ±k matriz de elementos k*aij matriz de elementos aij /k matriz de elementos aijˆk matriz de elementos kˆaij matriz de elementos k/aij matriz de elementos aij *bij matriz de elementos aij /bij matriz de elementos aijˆbij

La suma y resta est´ an definidas para matrices de igual dimensi´ on. El producto A*B es el resultado del producto matricial, definido solamente cuando el n´ umero de columnas de A es igual al n´ umero de filas de B. Ejemplos: ✄ >> A = [1 2;3 4 ] ; B = ones ( 2 ) ; >> A+B ans = 2 3 4 5 >> A∗B ans =

292

A.3. MATRICES

3 7

3 7





La operaci´ on Aˆk, est´ a definida para matrices cuadradas, en cambio las operaciones A±k y k*A est´ an definidas para matrices de cualquier dimensi´ on. Con la matriz A del ejemplo anterior obtenemos: ✄ >> Aˆ3 ans = 37 81 >> 5∗A ans = 5 15 >> A−4 ans = −3 −1

54 118

10 20

−2 0





Los operaciones A.*B, A./B, y A.ˆk, se efect´ uan elemento a elemento. If A y B son matrices de igual dimensi´ on, entonces los elementos de la matriz C = A.*B est´ an dados por C(i,j) = A(i,j)*B(i,j), y los elementos de la matriz C = A./B son C(i,j) = A(i,j)/B(i,j). Para una matriz A cualquiera, los elementos de la matriz C = A.ˆk est´ an dados por C(i,j) = A(i,j)ˆk. Con las matrices A y B de los ejemplos anteriores obtenemos: ✄ >> A . ∗ B ans = 1 2 3 4 >> B . / A ans = 1.0000 0.3333 >> A. ˆ 3 ans = 1 8 27 64

0.5000 0.2500



Utilice help para conocer m´ as de las funciones ctranspose y transpose



Otra operaci´ on importante de las matrices es la trasposici´ on. La traspuesta conjugada de la matriz A se obtiene con A’. Si A es real, esto no es m´ as que la matriz traspuesta de A. La transpuesta sin conjugaci´ on se obtiene con A.’. Las funciones alternativas ctranspose(A) y transpose(A) a veces son m´ as convenientes. Para el caso especial de los vectores columna x e y, x’*y es el producto escalar o producto punto, que tambi´ en se puede obtener usando la funci´ on dot(x,y). El producto vectorial o producto cruz de dos vectores 3 × 1 o 1 × 3 (tal como se utiliza en mec´anica) es producido por cross. Ejemplo: ✄

>> x = [1 −1 0 ] ’ ; y = [3 2 1 ] ’ ; >> x ’ ∗ y ans = 1 >> dot ( x , y ) ans = 1 >> c r o s s ( x , y ) ans =

´ A MATLAB ´NDICE A. INTRODUCCION APE

293

−1 −1 5





La mayor´ıa de las funciones predefinidas en M ATLAB est´ an hechas de forma que admiten matrices como argumentos. Esto se cumple particularmente para las funciones matem´ aticas elementales y cuando se aplican a una matriz se obtiene una nueva matriz, esto es, si A = (aij ), entonces funcion(A) es otra matriz cuyos elementos son funcion(aij ), donde funcion es cualquier funci´ on matem´atica elemental. Veamos un ejemplo. Para la matriz A = [2 0;1 4] se tiene: ✄ >> s q r t (A) ans = 1.4142 1.0000 >> exp (A) ans = 7.3891 2.7183 >> s i n (A) ans = 0.9093 0.8415

0 2.0000

Funci´ on de una matriz en el sentido de Algebra Lineal, es un t´ opico avanzado que no se considera en estas notas

1.0000 54.5982

0 −0.7568





Manipulaci´ on de Matrices M ATLAB tiene disponible comandos que permiten la manipulaci´ on de matrices, algunos de ellos se dan en la Tabla A.9. Tabla A.9. Comandos para la manipulaci´ on de matrices Comando reshape diag tril triu

Descripci´ on cambia la dimensi´ on de la matriz lista los archivos del directorio (formato distinto a ls) devuelve el path (direcci´ on) del directorio actual permite cambiar el directorio actual

La funci´ on reshape cambia la dimensi´ on de la matriz, estos es, reshape(A,m,n) produce una matriz m × n cuyos elementos se toman uno a uno de las columnas de A. Por ejemplo: ✄ >> A = [1 3 5;2 4 6 ] , B = r e s h a p e (A, 3 , 2 ) A = 1 3 5 2 4 6 B = 1 4 2 5 3 6





La funci´ on diag trabaja con las diagonales de una matriz y puede tomar un vector o una matriz como argumento. Para un vector v, diag(v) es la matriz diagonal con v su diagonal principal:

En una matriz diagonal todo los elementos fuera de su diagonal principal son ceros

294

A.3. MATRICES

✄ >> d i a g ( [ 1 ans = 1 0 0

2 3]) 0 2 0

0 0 3





M´as generalmente, diag(v,k) coloca a v en la k-´esima diagonal, donde k > 0 especifica diagonales por encima de la diagonal principal y k < 0 diagonales por debajo (k = 0 indica la diagonal principal): ✄ >> d i a g ( [ 1 ans = 0 0 0 >> d i a g ( [ 1 ans = 0 0 1 0

1 ] , 1) 1 0 0 1 0 0 1 ] , −2) 0 0 0 1

0 0 0 0

✂ Observe que diag([1 1], 1) coloca el vector [1 1] en la diagonal 1 de una matriz 3 × 3 de ceros

0 0 0 0



Para una matriz A, diag(A) es el vector columna conformado por los elementos de la diagonal principal de A. Entonces, para producir una matriz diagonal cuya diagonal principal sea igual a la diagonal principal de la matriz A, se escribe diag(diag(A)). An´alogamente al caso del vector, diag(A,k) produce un vector columna con los elementos de la k-´esima diagonal de A. Por ejemplo, si A = [1 2 3;4 5 6;7 8 9], entonces: ✄ >> d i a g (A) ans = 1 5 9 >> d i a g (A, −1) ans = 4 8



Observe que triu(A) y tril(A) son matrices triangular superior y triangular inferior, respectivamente



Con las funciones tril y triu se pueden extraer las partes triangulares de una matriz. La funci´ on tril(A) genera la parte triangular inferior de A (los elementos sobre y por debajo de la diagonal principal) y triu(A) genera la parte triangular superior (los elementos sobre y por encima de la diagonal principal). M´as generalmente, tril(A,k) genera los elementos sobre y por debajo de la k-´ esima diagonal de A, mientras que triu(A,k) genera los elementos sobre y por encima de la k-´ esima diagonal de A. Considerando nuevamente A = [1 2 3;4 5 6;7 8 9], se tiene: ✄ >> t r i u (A) ans = 1 0 0 >> t r i l (A) ans = 1 4 7

2 5 0

3 6 9

0 5 8

0 0 9

´ A MATLAB ´NDICE A. INTRODUCCION APE

>> t r i l (A , 1 ) ans = 1 2 4 5 7 8

295

0 6 9





A.4 Elementos de Programaci´ on En esta secci´ on se dan algunas herramientas de programaci´ on necesarias para la implementaci´ on del algoritmos. Instrucciones de Control de Flujo M ATLAB posee cuatro instrucciones de control de flujo: la instrucci´ on if, el bucle for, el bucle while y la instrucci´ on switch. Instrucci´ on if La estructura simple de la instrucci´ on if es: i f expresio ´n instrucciones end

donde las instrucciones se ejecutan si expresi´ on es verdadera. Por ejemplo, el siguiente c´ odigo intercambia el valor de a y b si a es m´ as grande que b: if a > b temp = a ; a = b; b = temp ; end

En caso de ser necesario la ejecuci´ on de instrucciones cuando expresi´ on es falsa, a la instrucci´ on if se le puede agregar las instrucciones else y elseif. He aqu´ı un caso general con tres condiciones: i f expresio ´n 1 instrucciones e l s e i f expresio ´n 2 instrucciones e l s e i f expresio ´n 3 instrucciones else instrucciones end

Las instrucciones de cada grupo se ejecutan si es verdadera la condici´ on que las encabeza; si ninguna de ellas es verdadera, se ejecutan las instrucciones que est´ an encabezadas por la instrucci´ on else. Bucle for La estructura del bucle for es:

296

´ A.4. ELEMENTOS DE PROGRAMACION

for variable = expresio ´n instrucciones end

Normalmente, expresi´ on es un vector de la forma i:s:j. Las instrucciones se ejecutan con el valor de variable igual a cada elemento de expresi´ on. Por ejemplo, la suma de 30 t´erminos de la serie geom´etrica (1/2)i se puede calcular como: ✄ >> s = 0 ; >> f o r i =0:1:29 , s = s +(1/2)ˆ i ; end , s s = 2.0000



Una matriz A es sim´ etrica si AT = A



Varios bucles for pueden anidarse, en cuyo caso es recomendable utilizar sangr´ıa para mejorar la legibilidad. El siguiente c´ odigo produce una matriz sim´etrica A 5 × 5 con elementos A(i,j) = i*j para j≥i: n = 5 ; A = eye ( n ) ; for j = 2:n f o r i = 1 : j −1 A( i , j ) = i ∗ j ; A( j , i ) = A( i , j ) ; end end

Bucle while La estructura del bucle while es: while expresio ´n instrucciones end

Las instrucciones se ejecutan mientras expresi´ on es verdadera. El siguiente ejemplo aproxima el n´ umero positivo m´ as peque˜ no en punto flotante: ✄ >> num = 1 ; w h i l e num > 0 , nmin = num; num = num/ 2 ; end , nmin nmin = 4.9407e−324

✂ Ejecute los comandos realmin y realmax. ¿Qu´e observa?



A veces es necesario salirse de un bucle while de manera abrupta, es decir, detener repentinamente la ejecuci´ on de las instrucciones dentro del bucle. Esto se hace empleando el comando break (comando que permite finalizar forzosamente un bucle). Normalmente, break se usa conjuntamente con una instrucci´ on condicional dentro de un bucle infinito. Un bucle infinito puede construirse utilizando, while 1, ..., end, que es u ´ til cuando no es conveniente poner expresi´ on en la parte superior del bucle. Con estas consideraciones podemos reescribir el ejemplo previo: num = 1 ; while 1 nmin = num; num = num/ 2 ; i f num==0 break end end

´ A MATLAB ´NDICE A. INTRODUCCION APE

297

La instrucci´ on break tambi´ en puede usarse para salir de un bucle for. En un bucle anidado un break sale al bucle del siguiente nivel superior. Instrucci´ on switch La u ´ ltima instrucci´ on de control de flujo que mostramos es switch, cuya estructura est´ a dada por: switch expresio ´n case caso 1 instrucciones . . . case caso n instrucciones otherwise instrucciones end

La instrucci´ on switch compara el valor de expresi´ on con cada uno de los casos, si el valor de expresi´ on coincide con alguno de los casos, se ejecutan las instrucciones que el caso coincidente encabeza. Si no hay coincidencia con ninguno de los casos, entonces se ejecutan las instrucciones que est´ an encabezadas por otherwise. Vea el siguiente ejemplo donde se eval´ ua la norma-p de un vector v (esto es, norm(v,p)) para tres valores de p. switch p case 1 norma = sum( abs ( v ) ) ; case 2 norma = s q r t ( dot ( v , v ) ) ; case i n f norma = max( abs ( v ) ) ; otherwise e r r o r ( ’ p debe s e r 1 , 2 o i n f . ’ ) end

Para un vector u, sum(u) arroja la suma de todos los elementos de u, max(u) arroja el m´ aximo valor de los elementos de u y abs(u) es un vector cuyos elementos son |u(i)|. El comando error muestra un mensaje para indicar que se ha producido un error, tal mensaje es la cadena de caracteres dentro de los ap´ ostrofes. Scripts y Funciones No todo se puede realizar en la ventana de comandos de M ATLAB , generalmente en la mayor´ıa de las aplicaciones se necesitan hacer c´ alculos que requieren de instrucciones muy elaboradas que, inevitablemente, deben ser programadas. As´ı, para llevar a cabo una tarea, en vez de escribir las instrucciones una por una en la ventana de comandos de M ATLAB , se pueden escribir una detr´ as de otra en un M-archivo, que es un archivo de texto con una extensi´ on *.m. Estos son los equivalentes de los programas, funciones, subrutinas y procedimientos en otros lenguajes de programaci´ on. Distinguiremos dos tipos de M-archivos: Scripts y Funciones. Scripts Un script es un conjunto de instrucciones (de cualquier lenguaje) guardadas en un archivo de texto que son ejecutadas normalmente mediante un int´ erprete, adem´as, no

Utilice help para conocer m´ as del comando norm

298

´ A.4. ELEMENTOS DE PROGRAMACION

tienen argumentos de entrada ni de salida y operan sobre las variables del espacio de trabajo (workspace). Es u ´ til para automatizar peque˜ nas tareas. Tambi´ en puede hacer las veces de un programa principal para ejecutar una aplicaci´ on. Un simple ejemplo de un M-archivo script, numero.m, se muestra en el siguiente programa. Programa A.1. Script numero.m %NUMERO % % %

V e r i f i c a s i un n´ u mero r e a l i n t r o d u c i d o por t e c l a d o e s cero , m´as i n f i n i t o , menos i n f i n i t o o d i s t i n t o de c e r o y f i n i t o

x = i n p u t ( ’ I n t r o d u z c a un n´ u mero r e a l : ’ ) ; switch x case i n f d i s p ( ’ M´ as i n f i n i t o ’ ) c a s e −i n f d i s p ( ’ Menos i n f i n i t o ’ ) case 0 d i s p ( ’ Cero ’ ) otherwise d i s p ( ’ N´ umero d i s t i n t o de c e r o y f i n i t o ’ ) end

Este corto script le pide al usuario que introduzca por teclado un n´ umero real, luego muestra un mensaje indicando que tipo de n´ umero es: +∞, −∞, 0 o distinto de 0 y finito. Las primeras cuatro l´ıneas comienzan con el s´ımbolo % lo cual indica que es un comentario. Cada vez que M ATLAB encuentra un % ignora el resto de la l´ınea. Esto permite introducir texto que hace mejor la comprensi´ on del c´ odigo. En el script numero.m tambi´ en se observa el comando disp(’string’) que imprime el string de caracteres en el escritorio de M ATLAB . Asumiendo que este script existe como un archivo numero.m, escribir numero es equivalente a ejecutar todas l´ıneas en la ventana de comandos de M ATLAB . M´as adelante trataremos la edici´ on y el manejo de los M-archivos. Funciones En muchas aplicaciones es necesario definir funciones, es decir, procedimientos que permiten introducir valores y devuelven, despu´es de ejecutar las instrucciones programadas, los valores de salida. En M ATLAB es posible definir funciones M ATLAB , las cuales son generalmente archivos muy parecidos a los scripts, pero se diferencian en que una funci´ on tiene una “interfase” de comunicaci´ on con el exterior mediante argumentos de entrada y de salida. Las funciones escritas en M-archivos permiten ampliar el lenguaje de M ATLAB , escribiendo nuestras propias funciones que aceptan y devuelven argumentos. Se pueden utilizar exactamente de la misma manera que las funciones de M ATLAB existentes tales como sin, cos, eye, etc. Las funciones de M ATLAB tienen la siguiente estructura: f u n c t i o n [ s a l i d a ] = name( e n t r a d a ) %name Breve d e s c r i p c i o ´n . % f u n c t i o n [ s a l i d a ] = name( e n t r a d a ) % Descripcio ´n instrucciones end

´ A MATLAB ´NDICE A. INTRODUCCION APE

299

Aqu´ı se ilustran algunas caracter´ısticas. La primera l´ınea comienza con la palabra clave function seguida del argumento de salida, salida, y el s´ımbolo =. A la derecha del s´ımbolo = viene el nombre de la funci´ on, name, seguido por el argumento de entrada, entrada, dentro de par´ entesis. (En general, puede haber cualquier n´ umero de argumentos de entrada y de salida.) El nombre de la funci´ on debe ser el mismo que el nombre del Marchivo donde ella es almacenada. La segunda l´ınea de un archivo de funci´ on se llama l´ınea H1 (ayuda 1). Debe ser una l´ınea de comentario con una forma especial: comienza con un car´ acter %, seguido sin espacio por el nombre de la funci´ on, seguido de uno o m´ as espacios, y luego una breve descripci´ on. La descripci´ on debe comenzar con una letra may´ uscula, termina con un punto y se deben omitir las palabras “the” y “a”. Todas las l´ıneas de comentario que est´ an encima de las l´ıneas no comentadas (normalmente una l´ınea en blanco, para facilitar la lectura del c´ odigo fuente) se muestran cuando se escribe help name. De esta forma, estas l´ıneas deben describir la funci´ on y sus argumentos. Veamos algunos ejemplos de funciones. Programa A.2. Funci´ on minelement f u n c t i o n y = minelement (A) %minelement Elemento m´ as peque˜ n o de una m a t r i z . % minelement (A) e s e l m´ınimo de l o s elementos % de A en v a l o r a b s o l u t o . y = min ( min ( abs (A ) ) ) ; end

El Programa A.2 muestra una simple funci´ on, minelement, que halla el menor elemento en valor absoluto de una matriz. Para esta funci´ on tenemos que ✄ >> h e l p minelement minelement Elemento m´ as peque˜ n o de una m a t r i z . minelement (A) e s e l m´ınimo de l o s elementos de A en v a l o r a b s o l u t o .





Adem´ as, minelement se llama igual que cualquier otra funci´ on de M ATLAB : ✄ >> minelement ( −1:1:10) ans = 0 >> minelement ( magic ( 6 ) ) ans = 1





Programa A.3. Funci´ on fpoli function %f p o l i % % %

[ f , df ] = f p o l i (x , a ) Polinomio y su d e r i v a d a . [ f , df ] = f p o l i (x , a ) evalu ´ a l a funcio ´ n f ( x ) = x . ∗ ( a−x ) y su d e r i v a d a d f en l a m a t r i z x , donde a e s un par´ a metro e s c a l a r .

f = x . ∗ ( a−x ) ; d f = a−2∗x ; end

300

´ A.4. ELEMENTOS DE PROGRAMACION

La funci´ on fpoli mostrada en el Programa A.3 eval´ ua la funci´ on escalar f (x) = x(a−x) y su derivada f ′ (x) = a − 2x con respecto a x. Los dos argumentos de salida f y df est´ an encerrados entre corchetes. Cuando se llama a una funci´ on con m´ ultiples entradas o argumentos de salida no es necesario pedir todos los argumentos de salida, sino aquellos argumentos que se deseen mostrar comenzando de izquierda a derecha, sin omitir ning´ un argumento intermedio. Si se requieren m´ as de un argumento de salida, ´estos deben aparecer entre corchetes. He aqu´ı algunos ejemplos del uso de fpoli: ✄ >> f = f p o l i ( −3 ,0.1) f = −9.3000 >> [ f , d f ] = f p o l i ( −3 ,0.1) f = −9.3000 df = 6.1000





Note que en la funci´ on fpoli aparece la operaci´ on matricial .∗, que se usa en la instrucci´ on f = x.*(a-x). De esta forma, si un vector o una matriz es suministrada a trav´ es de x, la funci´ on se eval´ ua en cada elemento de forma simult´ anea: ✄ >> [ f , d f ] = f p o l i ( 1 : 4 , 2 ) f = 1 0 −3 −8 df = 0 −2 −4 −6





Programa A.4. Funci´ on estcal f u n c t i o n [ x ord , x prom , x med , x d e s ] = e s t c a l ( x ) %e s t c a l An´ a l i s i s e s t a d´ı s t i c o de c a l i f i c a c i o n e s . % Dado un v e c t o r de c a l i f i c a c i o n e s x , % [ x ord , x prom , x med , x d e s ] = e s t c a l ( x ) c a l c u l a % una l i s t a ordenada de l a s c a l i f i c a c i o n e s , y e l % promedio , l a mediana y l a d e s v i a c i o ´ n e s t ´a n d a r % de l a s c a l i f i c a c i o n e s . x ord = s o r t (x ) ; i f nargou t > 1 x prom = mean( x ) ; end i f nargou t > 2 x med = median ( x ) ; end i f nargou t > 3 x des = std ( x ) ; end end

Una funci´ on m´ as complicada se muestra en el Programa A.4, la cual realiza un peque˜ no an´alisis estad´ıstico a un vector de clasificaciones. Para un vector v, el comando sort(v) ordena en forma creciente los elementos de v, y los comandos mean(v), median(v) y std(v), calculan el promedio, la mediana y la desviaci´ on est´ andar de v, respectivamente. Seguidamente se ilustra el uso de la funci´ on estcal.

301

´ A MATLAB ´NDICE A. INTRODUCCION APE

✄ >> c a l i f i c a c i o n e s = [4 12 15 20 18 2 1 3 4 1 4 ] ; >> x o r d = e s t c a l ( c a l i f i c a c i o n e s ) x ord = 1 2 3 4 4 12 14 15 >> [ x ord , x prom , x med ] = e s t c a l ( c a l i f i c a c i o n e s ) x ord = 1 2 3 4 4 12 14 15 x prom = 9.3000 x med = 8



18

20

18

20



En el Programa A.4 tambi´ en se observa el comando nargout, que retorna el n´ umero de argumentos de salida. Si el comando nargout se escribe en el interior del cuerpo de una funci´ on, el devuelve el n´ umero de argumentos de salida que se emplearon para llamar a la funci´ on. Otra comando an´alogo a nargout es nargin que retorna el n´ umero de argumentos de entrada; adem´as, si se escribe en el interior del cuerpo de una funci´ on, nargin devuelve el n´ umero de argumentos de entrada que se emplearon para llamar a la funci´ on. Creaci´ on y Edici´ on de M-Archivos Para crear y editar M-archivos se tienen dos opciones. Se puede usar cualquier editor de archivos ASCII (si se trata de un procesador de textos es necesario asegurarse de que guarda los archivos en formato ASCII est´ andar, no en el propio formato del procesador de textos). O bien, se puede utilizar el editor de M ATLAB (Editor/Debugger), que se muestra ´ ste se invoca tecleando edit en la l´ınea de comandos o escogiendo Fileen la Figura A.2. E New o File-Open del men´ u. El editor de M ATLAB tiene varias caracter´ısticas que ayudan a la edici´ on de los M-archivos, incluyendo sangr´ıa autom´atica de bucles e instrucciones if, resaltado de sintaxis en color, etc. Estas y otras caracter´ısticas se pueden desactivar o personalizar a trav´ es de File-Preferences del men´ u del editor.

Figura A.2. Editor de M ATLAB

302

´ A.4. ELEMENTOS DE PROGRAMACION

Lectura y Escritura de archivos Utilizaremos tres maneras de pasar datos a y de M ATLAB , a saber: ⊲ Introducci´ on de datos a trav´ es del teclado o el rat´ on ⊲ Escritura en un archivo ⊲ Uso de los comandos save o load Introducci´ on de datos a trav´ es del teclado La instrucci´ on input le permite a M ATLAB aceptar datos de entrada a trav´ es del teclado. Con la siguiente instrucci´ on se introduce un n´ umero: x = input ( ’ s t r i n g ’ )

La cadena string se muestra como un mensaje en el escritorio de M ATLAB . Tambi´ en se pueden introducir cadenas desde el teclado, por ejemplo: z = input ( ’ s t r i n g ’ , ’ s ’ )

El s´ımbolo ’s’ indica que la entrada del teclado es una cadena. Se puede introducir una cadena con input, sin ’s’ si la cadena se teclea encerrada entre ap´ ostrofes; por ejemplo, z = i n p u t ( ’ I n t r o d u z c a su a p e l l i d o ( e n c e r r a d o en a p o ´strofes ): ’ )

Escritura en un archivo Daremos un breve vistazo a los formatos de salida de M ATLAB . La funci´ on disp muestra el valor de una variable, de acuerdo con el formato actual, sin necesidad de imprimir primero el nombre de la variable y el s´ımbolo =. Si el argumento es una cadena, disp muestra la cadena. Ejemplo: ✄ >> d i s p ( ’ Se muestra l a m a t r i z magic 3x3 ’ ) , d i s p ( magic ( 3 ) ) Se muestra l a m a t r i z magic 3x3 8 1 6 3 5 7 4 9 2





El comando fprintf permite imprimir mensajes y n´ umeros con un formato espec´ıfico; por ejemplo: ✄ >> f p r i n t f ( ’ E l t e r c e r n´ u mero primo e s : %d\n ’ , 5 ) E l t e r c e r n´ u mero primo e s : 5





Aqu´ı se observa el s´ımbolo \n que es el operador de l´ınea nueva que avanza en una l´ınea la posici´ on de la pantalla. Este operador se puede colocar en cualquier lugar de la cadena. El s´ımbolo %d indica que el n´ umero que se imprime es entero. Ahora, en el ejemplo ✄ >> f p r i n t f ( ’ E l v a l o r de p i e s : %6.3 f \n ’ , p i ) E l v a l o r de p i e s : 3.142





el s´ımbolo %6.3f especifica un formato con ancho de campo de 6 y 3 d´ıgitos decimales. Si f se reemplaza por e, entonces el n´ umero se muestra en notaci´ on exponencial o cient´ıfica: ✄ >> f p r i n t f ( ’ E l v a l o r de p i e s : %6.3e\n ’ , p i ) E l v a l o r de p i e s : 3.142 e+00





´ A MATLAB ´NDICE A. INTRODUCCION APE

303

Al elegir el ancho del campo recuerde que para un n´ umero negativo el signo menos ocupa una posici´ on: ✄ >> f p r i n t f ( ’ %5.2 f \n%5.2 f \n ’ , s i n ( p i /6) , − s i n ( p i / 6 ) ) 0.50 −0.50





Para imprimir % se usa \% y para imprimir \ se usa \\, en la cadena de formato. Otro especificador de formato u ´ til es %g, que utiliza a %e o %f, seg´ un sea el caso: ✄ >> f p r i n t f ( ’%g %g\n ’ , exp ( 1 ) , exp ( 2 0 ) ) 2.71828 4.85165 e+08





Si se suministran los elementos de una matriz para imprimir, entonces las especificaciones del formato de fprintf, se aplican a los elementos de la matriz, comenzando por los elementos de la primera columna, luego se toman los de la segunda columna, y as´ı sucesivamente. Esta caracter´ıstica se puede utilizar para evitar un bucle. Ejemplo: ✄ >> A = [40 60 80 100 120]; >> f p r i n t f ( ’%g mi/h = %g km/h\n ’ , [A ; 8∗A/ 5 ] ) 40 mi/h = 64 km/h 60 mi/h = 96 km/h 80 mi/h = 128 km/h 100 mi/h = 160 km/h 120 mi/h = 192 km/h





Para imprimir una variable de cadena se utiliza el operador %s. Ejemplo: ✄ >> t e x t o = s p r i n t f ( ’ C ´ a l c u l o Num´e rico , A˜ no %d ’ , 2013); >> f p r i n t f ( ’ A s i g n a t u r a : %s \n ’ , t e x t o ) Asignatura : C´ a l c u l o Num´ e rico , A˜ no 2013





La funci´ on sprintf es an´aloga a fprintf pero devuelve su salida como una cadena. Esta funci´ on es u ´til para la producci´ on de etiquetas de gr´aficos. Pasemos ahora a describir como se escriben datos en un archivo de texto con un formato espec´ıfico. Inicialmente se debe abrir o crear el archivo; para ello se utiliza la funci´ on fopen como se indica a continuaci´ on: f i d = fopen (name , permiso )

donde name es una cadena que contiene el nombre del archivo que se abrir´a, fid es un escalar entero denominado identificador del archivo y permiso indica el modo de apertura del archivo. Los modos de apertura de archivos son: ’r’ ’w’ ’a’ ’r+’ ’w+’ ’a+’ ’W’ ’A’

abre el archivo para leer abre el archivo para escribir; descarta el contenido existente abre o crea un archivo para escribir; agrega la data al final del archivo abre (no crea) el archivo para leer y escribir abre o crea el archivo para leer y escribir; descarta el contenido existente abre o crea el archivo para leer y escribir; agrega la data en al final del archivo abre el archivo para escribir sin limpieza autom´atica abre el archivo para agregar sin limpieza autom´atica

Despu´ es de abrir o crear el archivo, los datos se escriben utilizando la funci´ on fprintf, que toma como primer argumento el identificador de archivo, fid. Finalmente se cierra el archivo con la funci´ on fclose(fid). As´ı, el c´ odigo

304

´ A.4. ELEMENTOS DE PROGRAMACION

A = [40 60 80 100 120]; ind = fopen ( ’ m i s a l i d a ’ , ’w ’ ) ; f p r i n t f ( ind , ’%g mi/h = %g km/h\n ’ , [A ; 8∗A / 5 ] ) ; f c l o s e ( ind ) ;

crea el archivo misalida que contiene: 40 mi/h = 64 km/h 60 mi/h = 96 km/h 80 mi/h = 128 km/h 100 mi/h = 160 km/h 120 mi/h = 192 km/h

Uso de los comandos save y load La funci´ on save(name) guarda todas las variables actuales en un archivo tipo binario de extensi´ on .mat de nombre name. En la funci´ on save se puede especificar las variables que desean guardarse: el comando s a v e (name , v a r i a b l e s )

guarda solo las variables que se indiquen en variables. Tambi´ en se puede especificar el formato de escritura: .MAT o ASCII. Por defecto save(name,variables) utiliza el formato .MAT; en cambio, s a v e (name , v a r i a b l e s , ’− a s c i i ’ ) Use help para conocer m´ as de save

utiliza el formato ASCII. El siguiente c´ odigo a = [1 2 3 4 ] ; b = [9 8 7 6 ] ’ ; save ( ’ misdatos ’ , ’ a ’ , ’ b ’ ) ;

crea el archivo binario misdatos.mat que contiene los vectores a y b. Si en el c´ odigo anterior se cambia la instrucci´ on save(’misdatos’,’a’,’b’); por s a v e ( ’ m i s d a t o s . t x x ’ , ’ a ’ , ’ b ’ , ’− a s c i i ’ ) ;

el c´ odigo resultante crea un archivo llamado misdatos.txx en formato ASCII de 8 d´ıgitos. Si abre el archivo misdatos.txx, se ver´a as´ı: 1.0000000e+00 9.0000000e+00 8.0000000e+00 7.0000000e+00 6.0000000e+00

2.0000000e+00

3.0000000e+00

4.0000000e+00

La funci´ on load(name) carga todas las variables de un archivo .MAT con nombre name. El comando l o a d (name , v a r i a b l e s )

carga solo las variables indicadas en variable. Veamos el uso de load con el siguiente ejemplo: ✄ >> v = primes ( 3 7 ) ; A = r e s h a p e ( primes ( 3 7 ) , 4 , 3 ) ’ ; >> s a v e ( ’ m a t r i z p r i m o s ’ , ’ A ’ , ’ v ’ ) ; >> whos Name Size Bytes Class Attributes A 3x4 96 dou ble v 1x12 96 dou ble

305

´ A MATLAB ´NDICE A. INTRODUCCION APE

clear whos load ( ’ matrizprimos ’ ) whos Name Size Bytes A 3x4 96 v 1x12 96 >> c l e a r >> l o a d ( ’ m a t r i z p r i m o s ’ , ’ v ’ ) >> whos Name Size Bytes v 1x12 96 >> >> >> >>

Class dou ble dou ble

Attributes

Class dou ble

Attributes





aficos A.5 Gr´ M ATLAB tiene potentes y vers´ atiles capacidades gr´aficas. Muchos tipos de figuras se pueden generar con relativa facilidad y su aspecto es altamente personalizable. En esta secci´ on cubriremos el uso b´asico de las herramientas b´asicas de M ATLAB para la representaci´ on gr´afica de datos de dos dimensiones. Nos centraremos en la generaci´ on de gr´aficos a trav´ es de la l´ınea de comandos o de M-archivos, pero existen figuras que tambi´ en pueden ser modificadas interactivamente utilizando el editor de gr´aficos (Plot Editor). Para usar el editor de gr´aficos ver help plotedit. Gr´ aficos simples en 2D La funci´ on plot se puede usar para generar gr´aficos uniendo puntos x−y. Por ejemplo, escribiendo ✄ >> x = [ 0 . 5 1 . 5 3 . 1 3 . 2 4 . 6 5 . 5 8 . 2 ] ; >> y = [ 1 . 1 3 . 3 6 . 1 7 . 2 8 . 8 9 . 1 2 . 6 ] ; >> p l o t ( x , y )





se produce la imagen de la izquierda de la Figura A.3, donde los puntos (x(i),y(i)) est´ an unidos en secuencia. 10

10

9

9

8

8

7

7

6

6

5

5

4

4

3

3

2

2

1

0

1

2

3

4

5

6

7

8

9

1

0

1

2

3

4

5

6

7

8

9

Figura A.3. Simple gr´afico x − y. Izquierda: por defecto. Derecha: personalizado M´as generalmente, se puede reemplazar plot(x,y) con plot(x,y,’string ’), donde string combina hasta tres elementos que controlan el color, marcador y estilo de l´ınea. Por ejemplo, plot (x, y, ’r*--’) especifica que un asterisco rojo se va a colocar en cada

306

´ A.5. GRAFICOS

punto (x(i),y(i)) y que los puntos se van a unir por una l´ınea discontinua de color rojo. La Tabla A.10 muestra una lista de las opciones disponibles. La imagen de la derecha de la Figura A.3 fue generada con plot(x,y,’bd:’), que produce una l´ınea de puntos azul y un marcador en forma de rombo tambi´ en azul. Tabla A.10. Opciones para el comando plot

r g b c m y k w

Marcador C´ırculo Asterisco Punto M´ as Cruz Cuadrado Rombo Tri´ angulo ascendente Tri´ angulo descendente Tri´ angulo a la derecha Tri´ angulo a la izquierda Estrella de cinco puntas Estrella de seis puntas

o * . + x s d ˆ v > < p h

Color Rojo Verde Azul Cyan Magenta Amarillo Negro Blanco

-: -.

Estilo de L´ınea L´ınea s´ olida L´ınea de trazos L´ınea de puntos L´ınea de trazo-puntos

M´as de un conjunto de datos pueden aparecer en un gr´afico; por ejemplo, p l o t ( x , y , ’ g− ’ , b , c , ’ r−− ’ )

superpone los gr´aficos x − y y b − c, con l´ınea s´ olida verde y l´ınea roja discontinua, respectivamente. Se puede ejercer aun m´ as control sobre el estilo de las l´ıneas y de los marcadores agregando m´ as argumentos a plot. Las propiedades LineWidth (por defecto 0.5 puntos) y MarkerSize (por defecto 6 puntos) se pueden definir en puntos. Por ejemplo, los comandos plot(x,y,’LineWidth’,4) y plot(x,y,’p’,’MarkerSize’,12) producen gr´aficos con un ancho de l´ınea de 4 puntos y un marcador tipo estrella de cinco puntas de tama˜ no 12 puntos, respectivamente. Para los marcadores que tienen un interior bien definido, las propiedades MarkerEdgeColor y MarkerFaceColor se pueden ajustar a uno de los colores de la Tabla A.10; por ejemplo, p l o t ( x , y , ’ o ’ , ’ MarkerEdgeColor ’ , ’m ’ )

hace que los bordes del c´ırculo sean magenta. 10

10

9

9

8

8

7

7

6

6

5

5

4

4

3

3

2

2

1

1 0

1

2

3

4

5

6

7

8

9

0

1

2

3

4

5

6

7

8

9

Figura A.4. Dos gr´aficos x − y con estilos personalizados

307

´ A MATLAB ´NDICE A. INTRODUCCION APE

El gr´afico de la izquierda de la Figura A.4 fue producido con p l o t ( x , y , ’ r−−ˆ ’ , ’ LineWidth ’ , 2 , ’ MarkerFaceColor ’ , ’ y ’ , ’ MarkerSize ’ , 9 )

y el gr´afico de la derecha con p l o t ( x , y , ’−−ms ’ , ’ MarkerSize ’ , 2 0 , ’ MarkerFaceColor ’ , ’ g ’ )

Para ciertas aplicaciones es necesario utilizar escala logar´ıtmica. Esto se puede lograr con la funci´ on loglog. El siguiente c´ odigo genera la gr´afica en escala logar´ıtmica de 2 |1 + h + h /2 − exp(h)| versus h, para h = 10−4 , 10−3 , 10−2 , 10−1 , 1, la cual se aprecia en la Figura A.5. Est´a gr´afica representa el error al aproximar exp(h) con los tres primeros t´erminos de su desarrollo de Taylor. Tal error es del orden h3 . h = 10.ˆ( −4:1:0); e r r o r = abs ((1+h+h . ˆ 2 / 2 ) − exp ( h ) ) ; l o g l o g (h , e r r o r , ’− ’ , h , h . ˆ 3 , ’−− ’ ) xlabel ( ’h ’ ) y l a b e l ({ ’ V a l o r a b s o l u t o ’ , ’ d e l e r r o r ’ }) t i t l e ( ’ E r r o r de aproximaci o ´ n de l a s e r i e c u ´ b i c a de T a y l o r de exp ( h ) ’ ) box o f f

Se utilizaron title(’string ’), xlabel(’string ’) y ylabel(’string ’). Estas funciones muestran string encima del gr´afico, en el eje x y en el eje y, respectivamente. Tambi´ en se emple´ o el comando box off, que quita el cuadro del gr´afico, dejando s´ olo los ejes x e y. Error de aproximación de la serie cúbica de Taylor de exp(h)

0

10

−2

10

−4

Valor absoluto del error

10

−6

10

−8

10

−10

10

−12

10

−14

10

−4

10

−3

10

−2

10 h

−1

10

0

10

Figura A.5. Ejemplo de escala logar´ıtmica Por supuesto, loglog no acepta valores negativos como argumento; en este caso M A muestra una advertencia y grafica solo los valores positivos. Otras funciones relacionadas son semilogx y semilogy, que colocan escala logar´ıtmica solamente en el eje x o en el eje y, respectivamente. Si uno genera un gr´afico usando plot e inmediatamente genera otro gr´afico con un nuevo llamado de plot, la nueva imagen reemplaza el gr´afico anterior. Escribiendo hold on causa que gr´aficos subsiguientes se superpongan al actual, mientras que hold off especifica que cada gr´afico nuevo sustituye el actual. El estado por defecto corresponde a hold off. El comando clf borra la figura actual, mientras que close cierra la ventana de figura. Es posible tener varias ventanas de figura en la pantalla. La forma m´ as sencilla de crear TLAB

Use help para conocer m´ as detalles de title, xlabel y ylabel

308

´ A.5. GRAFICOS

una nueva ventana de figura es escribir figure. La n-´esima ventana de figura (donde se muestra n en la barra de t´ıtulo) se puede crear escribiendo figure(n). El comando close all cierra todas las ventanas de figuras. Cuando la imagen se muestra en la ventana de figura, muchos aspectos de la misma se pueden cambiar. Para ello, se pueden utilizar los elementos de la barra de herramientas de la ventana de figura o en su men´ u desplegable Tools. Ejes y Anotaciones Varios aspectos de los ejes de un gr´afico pueden controlarse con el comando axis. Algunas de estas opciones se muestran en la Tabla A.11. Tabla A.11. Algunos comandos para controlar los ejes axis([xmin xmax ymin ymax]) axis auto axis equal axis off axis square xlim([xmin xmax]) ylim([ymin ymax])

Especifica los l´ımites de los ejes x e y Regresa al tipo de ejes por defecto Iguala las unidades de los datos x e y Remueve los ejes Hace que los ejes sean cuadrados Especifica los l´ımites del eje x Especifica los l´ımites del eje y

Los ejes se pueden remover del gr´afico con el comando axis off. La funci´ on axis equal establece una sola unidad de proporci´ on para que los incrementos de marcas en los ejes x e y sean del mismo tama˜ no. La opci´ on axis square hace que la caja actual sea cuadrada. Para ilustrar, la imagen de la izquierda de la Figura A.6 fue producida por p l o t ( f f t ( eye ( 1 7 ) ) ) , a x i s equ al , a x i s s q u a r e Use help fft para mayor informaci´ on

y la imagen de la derecha fue producida por p l o t ( f f t ( eye ( 1 7 ) ) ) , a x i s equ al , a x i s o f f

1

0.8

0.6

0.4

0.2

0

−0.2

−0.4

−0.6

−0.8

−1 −1

−0.8

−0.6

−0.4

−0.2

0

0.2

0.4

0.6

0.8

1

Figura A.6. Uso de axis La instrucci´ on axis[xmin xmax ymin ymax] ajusta los l´ımites del gr´afico en el eje x de xmin a xmax y en eje y de ymin a ymax. Para volver a la escala de los ejes por defecto, que M ATLAB elige autom´aticamente en base a los datos que est´ an siendo representados,

309

´ A MATLAB ´NDICE A. INTRODUCCION APE

se escribe axis auto. Los l´ımites de los ejes x e y se pueden ajustar individualmente con xlim([xmin xmax]) y ylim([ymin ymax]). 2 1 en el intervalo [−1, 2]: En el siguiente ejemplo se grafica la funci´ on f (x) = 2 + x (x − 1)2 x = l i n s p a c e ( −1 ,2 ,500); plot (x , 1 . / x .ˆ2 + 2 . / ( x −1).ˆ2); g r i d on

El comando linspace(x1,x2,n) genera un vector fila de n puntos igualmente espaciados desde x1 hasta x2. Con grid on se a˜ naden las l´ıneas de cuadr´ıcula a los ejes. El resultado obtenido se muestra a la izquierda de la Figura A.7. Debido a las singularidades en x = 0 y x = 1, el gr´afico es poco informativo. Ahora, ejecutando el comando adicional ylim ([−1 50])

se produce la gr´afica de la derecha de la Figura A.7, en el cual se percibe mejor el comportamiento de la funci´ on. 5

5

x 10

50

4.5

45

4

40

3.5

35

3

30

2.5

25

2

20 15

1.5

10

1

5

0.5

0 −1

0 −0.5

0

0.5

1

1.5

−1

2

−0.5

0

0.5

1

1.5

2

Figura A.7. Uso de ylim (derecha) para cambiar los l´ımites autom´aticos del eje y (izquierda)

Epicicloide: a=12, b=5 25

20

15

10

y(t)

5

0

−5

−10

−15

−20

−25 −25

−20

−15

−10

−5

0 x(t)

5

10

15

20

25

Figura A.8. Gr´afica del epicicloide En la Figura A.8 se muestra la gr´afica del epicicloide ( x(t) = (a + b) cos(t) − b cos((a/b + 1)t)

y(t) = (a + b)sen (t) − bsen ((a/b + 1)t)

0 ≤ t ≤ 10π,

310

´ A.5. GRAFICOS

para a = 12 y b = 5. Las instrucciones utilizadas para generar esta gr´afica son: a = 12; b = 5 ; t = 0:0.05:10∗ pi ; x = ( a+b )∗ c o s ( t ) − b∗ c o s ( ( a /b+1)∗ t ) ; y = ( a+b )∗ s i n ( t ) − b∗ s i n ( ( a /b+1)∗ t ) ; plot (x , y) a x i s equal a x i s ([−25 25 −25 25]) g r i d on t i t l e ( ’ E p i c i c l o i d e : a=12, b=5 ’ ) xlabel ( ’ x( t ) ’ ) , ylabel ( ’ y( t ) ’ ) Ejecute el c´ odigo sin la instrucci´ on axis([-25 25 -25 25]). ¿Qu´e observa?

Los l´ımites de los ejes fueron elegidos para agregar m´ as espacio alrededor del epicicloide. A continuaci´ on se construye la gr´afica los polinomios de Chebyshev de grados 1 a 4 y usamos la funci´ on legend para agregar una leyenda. El resultado se muestra en la Figura A.9. x = −1:.01:1; p1 = x ; p2 = 2∗x . ˆ 2 − 1 ; p3 = 4∗x . ˆ 3 − 3∗ x ; p4 = 8∗x . ˆ 4 − 8∗ x . ˆ 2 + 1 ; p l o t ( x , p1 , ’ r : ’ , x , p2 , ’ g−− ’ , x , p3 , ’ b−. ’ , x , p4 , ’m− ’ ) a x i s square ylim ([ −1.5 1 . 5 ] ) box o f f legend ( ’ \ i t n=l ’ , ’ \ i t n=2 ’ , ’ \ i t n=3 ’ , ’ \ i t n=4 ’ , ’ L o c a t i o n ’ , ’ S o u t h E a s t ’ ) x l a b e l ( ’ x ’ , ’ F o n t S i z e ’ , 1 2 , ’ FontAngle ’ , ’ i t a l i c ’ ) y l a b e l ( ’ P n ’ , ’ F o n t S i z e ’ , 1 2 , ’ FontAngle ’ , ’ i t a l i c ’ , ’ R o t a t i o n ’ , 0 ) t i t l e ( ’ P o l i n o m i o s de Chebyshev ’ , ’ F o n t S i z e ’ ,14) t e x t ( − . 3 5 , 1 . 3 , ’ P {n+1}(x ) = 2xP n ( x ) + P {n−1}(x ) ’ , ’ F o n t S i z e ’ , 1 2 , . . . ’ FontAngle ’ , ’ i t a l i c ’ )

Polinomios de Chebyshev 1.5

P

(x) = 2xP (x) − P

n+1

n

(x)

n−1

1

0.5

P

n

0

−0.5

−1

−1.5 −1

n=l n=2 n=3 n=4 −0.8

−0.6

−0.4

−0.2

0

0.2

0.4

0.6

0.8

1

x

Figura A.9. Ejemplo de los polinomios de Chebyshev, usando legend

311

´ A MATLAB ´NDICE A. INTRODUCCION APE

Por lo general, legend(’string1 ’,’string2 ’,...,’stringn ’) crear´ a un cuadrado con la leyenda que contiene a stringi justo al lado de informaci´ on del color, marcador y estilo de l´ınea del gr´afico correspondiente. Por defecto, el cuadrado aparece en la esquina superior derecha (noreste) del ´ area de los ejes. La ubicaci´ on de la caja se puede especificar con la sintaxis legend(’,...,’Location’,ubicaci´ on ), donde ubicaci´ on es una cadena que puede incluir los valores que se indican en la Tabla A.12. Tabla A.12. Opciones para la ubicaci´ on de la leyenda ’North’ ’South’ ’East’ ’West’ ’NorthEast’ ’NorthWest’ ’SouthEast’ ’SouthWest’ ’NorthOutside’ ’SouthOutside’ ’EastOutside’ ’WestOutside’ ’NorthEastOutside’ ’NorthWestOutside’ ’SouthEastOutside’ ’SouthWestOutside’ ’Best’

dentro de la caja del gr´afico en la parte superior dentro de la caja del gr´afico en la parte inferior dentro de la caja del gr´afico a la derecha dentro de la caja del gr´afico a la izquierda dentro de la caja del gr´afico arriba a la derecha dentro de la caja del gr´afico arriba a la izquierda dentro de la caja del gr´afico abajo a la derecha dentro de la caja del gr´afico abajo a la izquierda fuera de la caja del gr´afico en la parte superior fuera de la caja del gr´afico en la parte inferior fuera de la caja del gr´afico a la derecha fuera de la caja del gr´afico a la izquierda fuera de la caja del gr´afico arriba a la derecha fuera de la caja del gr´afico arriba a la izquierda fuera de la caja del gr´afico abajo a la derecha fuera de la caja del gr´afico abajo a la izquierda menos conflictos con los datos dentro del gr´afico

En este ejemplo tambi´ en se usa el comando text(x,y,’string ’), que coloca a string en la posici´ on cuyas coordenadas est´ an dadas por x e y. Observe que las cadenas en los comandos ylabel y text usan la notaci´ on del sistema TEX para especificar las letras griegas, los s´ımbolos matem´ aticos y los super´ındices y sub´ındices. La Tabla A.13 lista algunos comandos TEX que pueden emplearse. Tabla A.13. Algunos comandos TEX que se admiten en cadenas de texto Letras Griegas Min´ usculas α \alpha β \beta γ \gamma .. .. . . ω \omega May´ usculas Γ \Gamma ∆ \Delta Θ \Theta .. .. . . Ω

\Omega

S´ımbolos seleccionados ≈ \approx ◦ \circ ≥ \geq Im \Im ∈ \in ∞ \infty R \int ≤ \leq 6= \neq ⊕ \oplus ∂ \partial ± \pm ℜ \Re ∼ \sim √ \surd

Fuentes Normal \rm Negrita \bf Cursiva \it

TEX es un sistema de tipograf´ıa escrito por Donald E. Knuth en el a˜ no 1985

312

´ A.5. GRAFICOS

Una nota final acerca de este u ´ ltimo ejemplo es que hemos utilizado las propiedades FontSize y FontAngle para ajustar el tama˜ no y el ´angulo del texto producido por los comandos title, xlabel, ylabel y text (el valor predeterminado de FontSize es 10 y de FontAngle es normal). Sin embargo, la leyenda no acepta estos argumentos, as´ı que utilizamos notaci´ on TEX para producir fuentes cursivas en la leyenda. Guardar Figuras Con regularidad es necesario introducir figuras en reportes, informes, etc., con un formato gr´afico espec´ıfico (BMP, TIFF, JPEG, EPS, PDF, etc.). Una alternativa es utilizar el comando print para guardar la figura como un archivo. Por ejemplo, p r i n t −deps2 m i f i g u r a . eps

Use help print para ver m´ as opciones de print

guarda la figura actual en el archivo mifigura.eps de tipo PostScript encapsulado de nivel 2 en blanco y negro, que posteriormente se puede imprimir en una impresora PostScript o incluirlo en un documento. Algunas opciones del comando print pueden apreciarse en la Tabla A.14. Tabla A.14. Algunas opciones de formato gr´afico de print Formato Gr´afico BMP monocrom´ atico BMP 24-bit EPS blanco y negro EPS a color EPS nivel 2 blanco y negro EPS nivel 2 a color JPEG 24-bit PDF archivo pdf a color PNG 24-bit TIFF 24-bit

Opci´ on -dbmpmono -dbmp -deps -depsc -deps2 -depsc2 -djpeg -dpdf -dpng -dtiff

El comando print tambi´ en tiene una forma funcional, a saber: p r i n t ( ’−deps2 ’ , ’ m i f i g u r a . eps ’ )

Para ilustrar la utilidad de esta forma funcional, el pr´ oximo ejemplo genera una secuencia de cinco figuras y guarda las mismas en los archivos figura1.eps, ..., figura5.eps: x = l i n s p a c e (0 ,2∗ pi , 5 0 ) ; f o r i =1:5 p l o t ( x , c o s ( i ∗x ) ) name = s p r i n t f ( ’ f i g u r a%d . eps ’ , i ) ; p r i n t ( ’−deps2 ’ ,name) end

El segundo argumento de print es una cadena que contiene el nombre del archivo y su extensi´ on, la cual la creamos con el comando sprintf. As´ı cuando i=1, por ejemplo, la instrucci´ on print es equivalente a print(’-deps2’,’figura1.eps’). Es importante tener en cuenta que los gr´aficos que se ven bien en la pantalla pueden no ser tan buenos para la impresi´ on. En particular, los valores por defecto como el tama˜ no de la fuente, el ancho de las l´ıneas, entre otros, pueden hacer que sea dif´ıcil leer el texto o no permitan apreciar el estilo de las l´ıneas en las figuras impresas. Para producir figuras

´ A MATLAB ´NDICE A. INTRODUCCION APE

313

visualmente atractivas, con impresiones legibles, por lo general es necesario aumentar el valor de los par´ ametros que controlan estos efectos. Esto se puede hacer de dos maneras: 1. A˜ nadiendo modificadores como FontSize,12 a los comandos pertinentes. 2. Estableciendo los valores predeterminados de las propiedades, antes de la creaci´ on de la figura de M ATLAB . Esto se realiza con los comandos get y set. El comando saveas guarda una figura en un archivo en una forma que puede ser recargada en M ATLAB . Por ejemplo, saveas ( gcf , ’ mifigura ’ , ’ f i g ’ )

guarda la figura actual como un archivo-FIG binario, que puede ser recargado en M ATLAB con open(’mifigura.fig’). Tambi´ en es posible guardar, imprimir o exportar figuras desde el men´ u desplegable File de la ventana de figura de M ATLAB .

Use help para conocer m´ as de get y set.

Bibliograf´ıa [1] Churchil, R. y Ward, J. (2004). Variable Compleja y Aplicaciones 7ma. ed. McGrawHill, Madrid. [2] David, A. (1997). Variable Compleja con Aplicaciones 2da. ed. Addison-Wesley Iberoamericana, Wilmington, Delaware. [3] Derrick, W. (1987). Variable Compleja con Aplicaciones. Grupo Editorial Iberoam´erica, M´exico. [4] Higham, D.J. and Higham, N.J. (2005). Matlab Guide. SIAM, Philadelphia. [5] James, G. (2002). Matem´ aticas Avanzadas para Ingenier´ıa 2da. ed. Prentice Hall, M´exico. [6] Kamen, E.W. y Heck, B.S. (2008). Fundamentos de Se˜ nales y Sistemas Usando la Web y Matlab 3era. ed. Prentice Hall, M´exico. [7] Lindner, D.K. (1999). Introduction to Signals and Systems. McGraw-Hill, Boston. [8] Markush´ evich, A.I. (1967). Teor´ıa de las Funciones Anal´ıticas, vol. I. Editorial Mir, Mosc´ u. [9] Soliman, S.S. y Srinath, M.D. (1999). Se˜ nales y Sistemas Continuos y Discretos 2da. ed. Prentice Hall, M´exico.

314

´Indice A

Fase, 200 definici´ on, 200 Frontera de un conjunto, 15 Funci´ on arm´ onica, 36 arm´ onica conjugada, 36 Funci´ on de variable compleja, 24–66 anal´ıtica, 34 continua, 28 definici´ on, 24 derivada de, 31 dominio de, 24 entera, 35 funciones componentes, 26 l´ımite de, 26 monovaluada, 24 multivaluada, 24 punto singular de, 35 racional, 25 rango de, 24 Funciones de dominio continuo, 140–155 convoluci´ on, 151 definici´ on, 140 escal´ on unitario, 143 funci´ on de muestro o Sa , 147 funci´ on rampa, 147 funci´ on seno cardinal o sinc , 148 funci´ on signo, 145 impulso unitario, 141 propiedades, 142 pulso exponencial, 146 pulso rectangular, 144 pulso triangular, 145 Funciones de dominio discreto, 155–161 convoluci´ on, 159 definici´ on, 155 escal´ on unitario discreto, 156 funci´ on rampa discreta, 156 funci´ on signo discreto, 157 impulso unitario discreto, 155

Argumento, 9 principal, 10 Aritm´etica compleja con M ATLAB , 17

C Cero de una funci´ on, 36 Circunferencia unitaria, 13 Conjunto abierto, 14 Conjunto acotado, 16 Conjunto cerrado, 15 Conjunto conexo, 16 Conjunto no acotado, 16 Continuidad, 28–30 teoremas de, 29 Contorno, 106, 107 cerrado simple, 107 curva, 106 curva suave, 107 Convoluci´ on en el dominio continuo, 151 definici´ on, 151 propiedades, 151 Convoluci´ on en el dominio discreto, 159 definici´ on, 159 propiedades, 159

D Derivada generalizada, 149 Desarollo de MacLaurin, 82 Diferenciaci´ on, 31–34 ecuaciones de Cauchy-Riemann, 32 reglas de, 31 Disco unitario, 13 Dominio, 16

E Euler, Leonhard, 12 Expansi´ on en fracciones parciales, 121

F F´ ormula integral de Cauchy, 115 extensi´ on, 116 315

316

Funciones elementales, 39–50 exponencial, 40 exponente complejo, 46–47 exponencial de base c, 47 funciones hiperb´ olicas inversas, 49–50 funciones trigonom´ etricas inversas, 48–49 inversa de la tangente, 48 inversa del coseno, 48 inversa del seno, 48 hiperb´ olicas, 43–44 cosecante hiperb´ olica, 44 coseno hiperb´ olico, 43 cotangente hiperb´ olica, 44 secante hiperb´ olica, 44 seno hiperb´ olico, 43 tangente hiperb´ olica, 44 logaritmo, 44–46 corte ramal, 46 rama principal del, 45 ramas del, 45 valor principal del, 45 trigonom´ etricas, 41–42 cosecante, 42 coseno, 41 cotangente, 42 secante, 42 seno, 41 tangente, 42 Funciones, L´ımite, Derivada y Mapeos con Matlab, 61

I Integraci´ on y residuos con Matlab, 124 Integral, 105–116 contorno, 107 cerrado simple, 107 curva, 106 suave, 107 de l´ınea, 106–109 propiedades, 108 definida, 105 propiedades, 106 f´ ormula integral de Cauchy, 115 extensi´ on de la, 116 indefinida, 113 primitiva, 114

´INDICE

principio de independencia de la trayectoria, 113 teorema de Cauchy-Goursat, 110 extendei´ on del, 111 Introducci´ on al c´ alculo operacional con Matlab, 162

M Magnitud, 200 definici´ on, 200 Mapeo, 50–60 bilineal, 59–60 inversi´ on, 56–59 inyectivo, 51 lineal, 51–56 w = bz, 53 w = bz + c, 55 w = z + c, 51 Matlab, 282–313 aritm´etica compleja con, 17 constantes, 284 definici´ on de variables, 285 editor de gr´aficos, 305 escritorio, 282 funciones, 298–301 funciones, l´ımite, derivada y mapeos con, 61 gr´aficos, 305–311 FontAngle, 312 FontSize, 312 LineWidth, 306 MarkerEdgeColor, 306 MarkerFaceColor, 306 MarkerSize, 306 axis, 308 clf, 307 close, 307 figure, 308 hold off, 307 hold on, 307 linspace, 309 loglog, 307 plot, 305 semilogx, 307 semilogy, 307 title, 307 xlabel, 307 ylabel, 307

317

´INDICE

guardar figuras, 312–313 print, 312 saveas, 313 instrucciones de control de flujo, 295–297 for, 295 if, 295 switch, 297 while, 296 introducci´ on al c´ alculo operacional con, 162 introducci´ on de datos, 302 input, 302 lectura y escritura de archivos, 302–305 disp, 302 fclose, 303 fopen, 303 fprintf, 302 load, 304 save, 304 sprintf, 303 m-archivos, 298, 301 creaci´ on y edici´ on, 301 matrices, 288–295 manipulaci´ on de , 293 notaci´ on de dos puntos, 289 operaciones, 291 scripts, 297 series de potencias y singularidades con, 94 tabla de comandos TEX, 311 tabla de comandos para controlar los ejes, 308 tabla de comandos para la manipulaci´ on de matrices, 293 tabla de comandos para lsitar y eliminar variables, 286 tabla de formatos de salida, 283 tabla de funciones elementales, 287 tabla de opciones de print, 312 tabla de opciones para ubicar la leyenda, 311 tabla de operaciones aritm´eticas, 284 tabla de operaciones aritm´eticas de matrices, 291 tabla de operadores de comparaci´ on,

285 tabla de operadores l´ ogicos, 285 tabla de utilizades, 287 tablas de opciones de plot, 306 transformada z con, 266 transformada de Fourier con, 205 transformada de Laplace con, 237 Matlab:integraci´ on y residuos con, 124

N N´ umero complejo, 2–20 argumento, 9 principal, 10 conjugado, 6 coordenadas polares, 9 definici´ on, 2 desigualdad triangular, 7 f´ ormula de Euler, 12 inverso aditivo, 4 inverso multiplicativo, 4 operaciones algebraicas, 3 divisi´ on, 3 multiplicaci´ on, 3 resta, 3 suma, 3 parte imaginaria, 3 parte real, 3 plano complejo, 5 potencia n-´esima, 13 ra´ız n-´esima, 13 representaci´ on geom´etrica, 5 unidad imaginaria, 2 valor absoluto, 6

P Parte principal, 89 Polinomio complejo, 25 Punto de acumulaci´ on, 15

R Residuo, 117 c´ alculo del, 118 en un polo, 118 en un punto singular esencial, 118 en un punto singular removible, 118 teorema de los residuos, 120

S Serie, 77–88

318

de Laurent, 85 de n´ umeros complejos, 78 convergente, 78 de potencias, 79 de Taylor, 81 desarollo de MacLaurin, 82 propiedades adicionales de, 87 Teorema de Cauchy-Hadamard, 80 Teorema de Laurent, 86 Teorema de Taylor, 82 Serie de Laurent, 85 Serie de n´ umeros complejos, 77 Series de potencias y singularidades con Matlab, 94 Singularidades aisladas, 88–94 polo de orden m, 90 punto singular aislado, 88 punto singular esencial, 93 punto singular removible, 94 Sucesi´ on, 77 de n´ umeros complejos, 77 convergente, 77

T Teorema de Cauchy-Goursat, 110 Teorema de Cauchy-Hadamard, 80 Teorema de Laurent, 86 Teorema de los Residuos, 120 Teorema de Taylor, 82 Transformada z, 249–273 bilateral, 249 propiedades de la, 252 regi´ on de convergencia, 250 tabla de pares de transformadas, 262 tabla de propiedades, 257

´INDICE

unilateral derecha, 249 unilateral izquierda, 249 Transformada z con Matlab, 266 Transformada z inversa, 251 definici´ on, 251 expansi´ on en serie de potencias, 264 integraci´ on compleja, 263 inversi´ on por tablas, 265 Transformada de Fourier, 185–204 definici´ on, 185 propiedades de la, 187 tabla de pares de transformadas, 199 tabla de propiedades, 193 Transformada de Fourier con Matlab, 205 Transformada de Laplace, 218–246 bilateral, 218 propiedades de la, 220 regi´ on de convergencia, 219 tabla de pares de transformadas, 232 tabla de propiedades, 227 unilateral derecha, 218 unilateral izquierda, 218 Transformada de Laplace con Matlab, 237 Transformada inversa de Fourier, 186 definici´ on, 186 Transformada inversa de Laplace, 220 definici´ on, 220 integraci´ on de contornos, 233 inversi´ on por tablas, 236

V Vecindad, 13

Related Documents


More Documents from "Graciela Severini"